Download as pdf or txt
Download as pdf or txt
You are on page 1of 288

AIIMS and All India

PG Medical Entrance
Examinations PGMEE
Review Questions 2002–2015
Over
6000 Qs

PGMEE
PGMEE
PGMEE
PGMEE
AIIMS and All India
PG Medical Entrance
Examinations PGMEE
Review Questions 2002–2015
Over
6000 Qs

Arun Kumar MBBS DNB (s)

CBS Publishers & Distributors Pvt Ltd


New Delhi • Bengaluru • Chennai • Kochi • Kolkata • Mumbai • Pune
Hyderabad • Nagpur • Patna • Vijayawada
Disclaimer
Science and technology are constantly changing
fields. New research and experience broaden the
scope of information and knowledge. The author
has tried his best in giving information available to
him while preparing the material for this book.
Although all efforts have been made to ensure
optimum accuracy of the material, yet it is quite
possible some errors might have been left
uncorrected. The publisher, the printer and the
author will not be held responsible for any
inadvertent errors or inaccuracies.

ISBN: 978-81-239-2861-6

Copyright © Author and Publisher

First Edition: 2016

All rights reserved. No part of this book may be reproduced or transmitted in any form or by any means, electronic or mechanical, including
photocopying, recording, or any information storage and retrieval system without permission, in writing, from the author and the publisher.

Published by Satish Kumar Jain and produced by Varun Jain for


CBS Publishers & Distributors Pvt Ltd
4819/XI Prahlad Street, 24 Ansari Road, Daryaganj, New Delhi 110 002, India.
Ph: 23289259, 23266861, 23266867 Website: www.cbspd.com
Fax: 011-23243014 e-mail: [email protected]; [email protected].
Corporate Office: 204 FIE, Industrial Area, Patparganj, Delhi 110 092
Ph: 4934 4934 Fax: 4934 4935 e-mail: [email protected]; [email protected]

Branches
• Bengaluru: Seema House 2975, 17th Cross, K.R. Road,
Banasankari 2nd Stage, Bengaluru 560 070, Karnataka
Ph: +91-80-26771678/79 Fax: +91-80-26771680 e-mail: [email protected]

• Chennai: 7, Subbaraya Street, Shenoy Nagar, Chennai 600 030, Tamil Nadu
Ph: +91-44-26680620, 26681266 Fax: +91-44-42032115 e-mail: [email protected]

• Kochi: Ashana House, No. 39/1904, AM Thomas Road, Valanjambalam, Ernakulam 682 018, Kochi, Kerala
Ph: +91-484-4059061-65 Fax: +91-484-4059065 e-mail: [email protected]

• Kolkata: 6/B, Ground Floor, Rameswar Shaw Road, Kolkata-700 014, West Bengal
Ph: +91-33-22891126, 22891127, 22891128 e-mail: [email protected]

• Mumbai: 83-C, Dr E Moses Road, Worli, Mumbai-400018, Maharashtra


Ph: +91-22-24902340/41 Fax: +91-22-24902342 e-mail: [email protected]

• Pune: Bhuruk Prestige, Sr. No. 52/12/2+1+3/2 Narhe, Haveli


(Near Katraj-Dehu Road Bypass), Pune 411 041, Maharashtra
Ph: +91-20-64704058, 64704059, 32392277 Fax: +91-20-24300160 e-mail: [email protected]

Representatives
• Hyderabad 0-9885175004 • Nagpur 0-9021734563
• Patna 0-9334159340 • Vijayawada 0-9000660880

Printed at: Goyal Offset, Delhi


to
my parents and
grandmother
who gave me values
Preface

T his book has been written keeping in view that many questions are repeatedly asked in subsequent
examinations. The book contains questions in the form of statements from 2002 onwards. Questions from
AIIMS and All India PGMEEs have been compiled. Question–Answers have been written in the form such that
the students remember the facts clearly and no confusion remains in their minds. This book is extremely
valuable for those starting their preparation for postgraduate entrance examinations during internship or
before and for quick revision just before appearing in the PGMEEs. Questions asked repeatedly have been put
only once in the book to make it concise. Around 6000 questions have been included in this book.
While every effort has been made to make this book error-free, readers are requested to point out if they come
across any; in addition, their valuable comments and suggestions for further improvement will be greatly
appreciated and acknowledged.

Arun Kumar
Mob. 09718161947
e-mail id: [email protected]
Acknowledgments

F irst I would like to thank all my teachers who have encouraged and inspired me for hard work. I also want
to thank Dr Nitish, Dr Aporva, Dr Rajdeep and Dr Puneet who have been very supportive of me to write
this book. My inspiration for writing this book is my teacher Dr Arup Kumar Kundu who has been a leading
writer in this field.
My heartfelt thanks to my wife who contributed significantly in writing this book. I would also like to thank
all my family members who have stood by me through thick and thin.
I would also like to thank CBS Publishers & Distributors, Mr YN Arjuna (Senior Vice President—Publishing, Editorial
and Publicity), Mrs Ritu Chawla (Production Manager) and Mr Vikrant Sharma (DTP Operator) to help me realize my
dream of writing the book and publishing the book so beautifully.
I would also like to thank my friend Pawan, Bipulji, Dr Pradeep and others for their valuable support. I would
also like to thank Dr Rahul, Dr Abhishek, Dr Anant, Dr Rajeev, Dr Dharvind, Dr Lokesh and others for helping
me recall questions.
I would also like to thank my teacher Dr MP Sharma for his guidance. Finally, with all my humbleness and
sincerity, I thank one and all who have helped me directly or indirectly in writing this book.

Arun Kumar
Contents
Preface v

1. Anatomy 1

2. Biochemistry 9

3. Physiology 17

4. Pharmacology 25

5. Microbiology 44

6. Pathology 59

7. Forensic Medicine and Toxicology 76

8. Preventive and Social Medicine/Community Medicine 86

9. Ophthalmology 110

10. Ear, Nose, and Throat 123

11. Medicine 130

12. Surgery 160

13. Obstetrics and Gynecology 186

14. Pediatrics 206

15. Dermatology 225

16. Anesthesia 233

17. Radiology 242

18. Psychiatry 251

19. Orthopedics 260


1

Anatomy

AIIMS NOVEMBER 2015 C Nerves of general visceral efferents are


Oculomotor, facial and glossopharyngeal.
C Structures piercing the buccinator muscle are
(Note: Also vagus)
Parotid duct, ducts of molar glands of cheek and
buccal branch of facial nerve. C Reticular fibres of connective tissue are present in
C Anal canal is not supplied by Bone marrow, lymph node and spleen.
Superior rectal artery. (Note: Absent in thymus)
C Diaphragm develops from C What are the contents of mesorectal fascia
Septum transversum, pleuroperitoneal membranes Superior rectal vein, pararectal nodes and inferior
and lateral body wall. mesenteric plexus.
(Note: and dorsal mesentery of esophagus) (Note: Inferior rectal vein and artery is not present
C Bones articulating with maxilla are in mesorectal fascia)
Lacrimal bone, ethmoid bone and zygomatic bone. C Oil red O stain is used in
C Nerve invoved in pronator teres syndrome Frozen section
Median nerve (Note: For fat and lipids)
C True about location of otic ganglia- C Mesonephric tubule and duct give rise to
Anterior to middle meningeal artery
Vas deferens, epididymis and para-oophoron.
Inferior to foramen ovale
C Microvilli is present in
Lateral to tensor veli palate.
Duodenum, jejunum, ileum, gallbladder and and
(Note: and medial to mandibular nerve)
PCT.
(Note: Collecting duct has numerous P cells and
AIIMS MAY 2015 few I cells. I cells have more microvilli. Relative to
C The nucleus that lies deep to facial colliculus other in the options it has lesser microvilli.)
Abducens nucleus C Most abundant collagen present in basement
membrane is
C Following are the branches of external carotid artery
Lingual artery, posterior auricular artery and Type 4 collagen.
superior thyroid, facial, occipital, superficial C At birth, in girls oogenesis is arrested at
temporal, maxillary, ascending pharyngeal Prophase of first meiotic division.
artery. (Note: Diplotene stage)
C Muscles attached to the greater tuberosity C Independent assortment of chromosome occurs at
Supraspinatus, infraspinatus and teres minor. which stage of spermatogenesis
C Card test is done for Primary spermatocyte to secondary spermato-
Palmar interossei cyte.

1
2 AIIMS and All India PGMEE—Review Questions

AIIMS NOVEMBER 2014 C Structure seen in the cavernous sinus


Internal carotid artery.
C Made of dense collagen C The medulla receives its blood supply from
Ligament, tendon and aponeurosis. Anterior spinal artery, vertebral artery and PICA.
C Mucosa associated lymphoid tissue (MALT) is C Extraocular muscle supplied by contralateral
maximally associated at nucleus
Ileum
Superior oblique.
C Regarding sphincter urethrae, true are
C Trigeminal nerve supplies
It is under voluntary control, it lies within urethra
Medial pterygoid, lateral pterygoid, tensor veli
and it is supplied by the pudendal nerve.
palatine, masseter, temporalis, tensor tympani,
C Structure not passing through foramen ovale mylohyoid and anterior belly digastric.
Maxillary nerve. C Facial nerve gives secretomotor fibers to
C Membrane tectoria is a continuation of Lacrimal gland, submandibular gland, sublingual,
Posterior longitudinal ligament. palatal gland and nasal gland.
C The Turkish saddle named space in the cranium is
known to contain which cranial structure
AIIMS NOVEMBER 2013
Pituitary gland/hypophysis cerebri
C The nucleus that lies deep to facial colliculus is C Vaginal epithelium is derived from
Abducens nucleus. Endoderm of urogenital sinus
C Part of the ethmoid bone
AIIMS MAY 2014 Agar nasi, crista galli, labyrinth and uncinate
process.
Structures derived from ectoderm are
Part of fallopian tube acting as a functional
C
C
Hair follicles, sebaceous glands and mammary anatomical sphincter
glands, epidermis.
Isthmus
Muscles derived from the ectoderm are
Most important blood supply to stomach
C
C
Sphincter pupillae and dilator pupillae.
Left gastric artery.
Buccopharyngeal membrane develops from
Dynamic stabilizer of shoulder joint term is used for
C
C
Ectoderm and endoderm. Rotator cuff.
C Epitheloid cells include C Derivatives of neural ectoderm
Interstitial cells of Leydig. Sphincter pupillae, retina and dilator pupillae.
C Venous drainage of esophagus are C Muscle that is anatomically a back muscle, but
Hemiazygos, inferior thyroid and left gastric functionally related to thorax is
veins. Lattisimus dorsi.
C The most common variant in the blood supply of C Artery supplying medulla
colon is Anterior spinal artery, vertebral artery and PICA.
Absence of middle colic artery. C External anal sphincter is innervated by
C Part of mesorectal fascia are S2, S3, S4.
Superior rectal vein, pararectal nodes and inferior C Nerve root for dermatome supplying thumb and
mesenteric plexus. index finger
C True about blood testis barrier C6, C7.
Formed by Sertoli cells, germ cells are not necessary C A patient woke-up with difficulty in extending
for formation of the barrier and any rupture of fingers, can make a grip and hold a pen. Extension
barrier can cause immune response to germ cells. was normal. Nerve most commonly involved
C True about dura mater Posterior interosseous nerve.
Outermost layer of the cranial meninges, it has 2 C Regular nerve, artery, vein arrangement not
layers—endosteal and meningeal and it is supplied present in intercostal space
by a branch of fifth cranial nerve. First
Anatomy 3

C Enclosed in mesorectal fascia C Organs developing in the mesentery of stomach


Pararectal nodes, superior rectal vein and inferior Liver, spleen and pancreas.
mesenteric plexus. C Root value of long thoracic nerve are
C In spermatogenesis, independent assortment of C5, C6, C7
maternal and paternal chromosomes occur during C Vertebral segment numerically most constant in the
Primary to secondary spermatocytes. body
Cervical
AIIMS MAY 2013 C Sternocleidomastoid muscle is supplied by
C Vaginal epithelium develops from Occipital artery, suprascapular and superior
Endoderm of urogenital sinus thyroid artery
C Anorectal ring is formed by C Structures in the spermatic cord at risk of injury
during vasectomy
Puborectalis, internal sphincter and external sphincter
Testicular artery, cremasteric artery and genital
C Part of the inferior wall of the orbit
branch of genitofemoral nerve.
Maxilla, palatine and zygomatic
C Nerves supplying gluteal region
C Lacrimation is lost in the lesion of
Superior and inferior gluteal nerves, nerve to
Greater petrosal nerve
quadratus femoris and nerve to obturator internus.
Floor of third ventricle is formed by
Ligaments of the knee joint connecting menisci to
C
C
Optic chiasma, infundibulum and mammillary body the tibia
C In larynx the only cartilage making complete ring is Coronary ligaments
Cricoid
C The role of one of the rotator cuff muscles had been AIIMS MAY 2012
ignored and given less importance to its role. It is
also known as forgotten muscle. Which muscle is C Unilateral injury to hypoglossal nerve leads to
that Hemiatrophy of involved side, deviation of tongue
Subscapularis towards same side and fasciculation of the tongue.
C Urethral crest is an elevation seen in urethra due to C Muscle spared by complete transaction of cranial
Trigone muscle part of accessory nerve
Stylopharyngeus
AIIMS NOVEMBER 2012 C A 50 years old female patient has undergone
mastectomy for carcinoma breast. After mastec-
C True about testes tomy, patient is not being able to extend, adduct
It descends into the scrotum with its inferior pole and internally rotate the arm. Nerve supply to
attached to the gubernaculums testis. which muscle damaged
C Epiglottis is derived from Latissimus dorsi
4th branchial arch. C True statement is
C Loss of extension of little and ring finger, along with Occipital lobe is part of cerebrum
hypothenar atrophy is found in injury of C Difference between cervical and thoracic vertebra
Ulnar nerve Foramen transversarium
C A patient presents with numbness in little and ring
fingers along with atrophy of the hypothenar
AIIMS NOVEMBER 2011
eminence. Nerve injured is
Ulnar nerve before it divides into superficial and C Structure passing through esophageal hiatus
deep branches. Left gastric artery, left vagus nerve and right vagus
C A patient with fall of bookshelf on his arm laterally nerve, esophagus.
presents with inability to extend his wrist, unable to C Component of superficial perineal pouch in male
make a strong hand grip and loss of sensation on Ducts of bulbourethral glands, bulbospongiosus
dorsum of hand and fingers. Cause is and posterior scrotal nerve, ischiocavernosus
Radial nerve injury perineal nerve, dorsal and deep arteries of penis.
4 AIIMS and All India PGMEE—Review Questions

AIIMS MAY 2011 C True about scalenus anterior muscle


It separates the subclavian vein from the subclavian
C Vertical crest in fundus of the internal auditory artery.
canal is known as C Lacrimation is lost in the lesion of
Bill’s bar Greater petrosal nerve.
C Upper limb weight is transmitted to axial skeleton by C Sphincter of lower genitourinary tract of female are
Interclavicular ligament, costoclavicular ligament Pubovaginalis, bulbospongiosus and external
and acromioclavicular ligament. urethral sphincter or sphincter urethrae.
C Trochlear nerve supplies
Contralateral superior oblique muscle. AIIMS MAY 2010
C Sphincter of Oddi consists of
C Extrinsic membranes of larynx are
3 sphincters
Cricotracheal, hyoepiglottic and thyrohyoid.
C Carpal tunnel syndrome is caused by
C Paneth cell, true are
Amyloidosis, hypothyroidism and diabetes mellitus.
Rich in RER, high zinc content and numerous
Horizontal part of duodenum is crossed by
lysozyme granules.
C
Root of mesentry C Prostatic urethra is
C Neurovascular bundle in axilla is surrounded by Semilunar (transversely arched) in cross section.
sheath derived from C Cranial nerves carrying parasympathetic fibers
Prevertebral fascia 3, 7, 9 and 10.
C On contrast radiography, part of duodenum has a C Middle superior alveolar nerve is a branch of
cap Infraorbital division of maxillary nerve.
Proximal (Note: Anterior-superior alveolar nerve is also a
C Spleen projects into space of peritoneal cavity branch of infraorbital nerve)
Greater sac C Healthy young athlete sitting at the edge of table
C Muscles which retract the scapula are with knee at 90° flexion. He fully extends it
Trapezius, rhomboid major and rhomboid minor. Movement of tibial tuberosity towards lateral
C Boundary of triangle of auscultation is formed by border of patella.
Scapula, latissimus dorsi and trapezius. C Celiac plexus is located
C Sternocleidomastoid is supplied by Anteromedial to lumbar sympathetic chain
Occipital artery, suprascapular artery and superior C Anterior ethmoidal nerve supplies
thyroid artery. Interior of nasal cavity, dural sheath of anterior
C Axillary abscess is safely drained by approach cranial fossa and ethmoidal air cells.
Floor C Appendices epiploicae absent in
Cecum, Appendix and Rectum (Mn: CAR)
AIIMS NOVEMBER 2010 C Function of 8th cranial nerve
Balance and hearing.
C Tensor tympani is supplied by
C Pain insensitive structure in brain is
Trigeminal nerve.
Choroid plexus
C Muscles supplied by superior gluteal nerve are
C Supplied by pelvic splanchnic nerves
Gluteus medius, gluteus minimus and tensor fascia
Urinary bladder, rectum and uterus
lata.
C Derivatives of septum transversum are
AIIMS NOVEMBER 2009
Mesentery of lesser sac, falciform ligament and
coronary ligament. C Structures felt on per rectal examination
C Organs developing in mesentery of stomach are Bulb of penis, anorectal ring and urogenital diaphragm
Liver, spleen and pancreas. C Thyroidea ima artery is an
C Structures attaining adult size before birth Inconsistent artery which may arise from arch of
Ear ossicles. aorta or other arteries.
Anatomy 5

C Structures passing through flexor retinaculum are C Fascia around nerve bundle of brachial plexus is
FDP, FDS and median nerve Prevertebral fascia
C Phrenic nerve lies C Brainstem nuclei derived from the alar plate
Anterior to scalenus anterior muscle Inferior olivary, substantia nigra and dentate
C Nerves involved in injury to head of fibula are
C In subclavian artery block at outer border of 1st rib,
Common peroneal nerve, superficial peroneal
following arteries help in maintaining the
nerve and deep peroneal nerve
circulation to upper limb
C Deep peroneal nerve supplies
Subscapular artery, suprascapular artery and
1st web space of foot
thyrocervical trunk
C Meissner’s and Aurbach’s plexus are derived from
Trendelenburg test is positive due to injury to
Neural crest
C

C In post-ductal coarctation of aorta blood flow to Superior gluteal nerve


lower limb is maintained through C Permanent mucosal folds are
Suprascapular artery, intercostal arteries and Spiral valve of Heister, plica semilunaris and
internal thoracic artery transverse rectal fold.
C Urothelium lines C Terminal group of lymph node for colon
Major calyx, ureters and urinary bladder Preaortic
C Stability of ankle joint is maintained by C Passive movement causing pain in deep posterior
Shape of the bones, tendons of muscle which cross compartment syndrome
the joint and collateral ligaments. Dorsiflexon of foot
C Medial most in femoral triangle C Structures piercing the buccinator muscle are
Lymphatics Parotid duct, ducts of molar glands of the cheek and
buccal branch of facial nerve.
AIIMS MAY 2009
C Deltoid ligament is attached to AIIMS MAY 2008
Medial malleolus, spring ligament and sustentacu- C Arteries involved in collateral formation in
lum tali. coarctation of aorta
C Structures lying deep to posterior belly of diagastric Posterior intercostal arteries, axillary artery and
are subscapular artery
Hypoglossal nerve, hyoglossal muscle and occipital
C Unilateral injury to hypoglossal nerve leads to
artery
Hemiatrophy of involved side, deviation of
C Urothelium lines
tongue towards same side and fasciculation of
Minor calyx, ureter and urinary bladder tongue
C Aortic hiatus contains
C In median nerve injury at wrist, there is loss of
Aorta, azygos vein and thoracic duct function of
C Enophthalmos is due to palsy of Lumbrical muscles to index finger, lumbrical
Orbitalis muscle muscles to middle finger and muscles of thenar
C 8th cranial nerve supplies eminence.
Equilibrium C In standing position, venous return to heart from
C Adult size at birth lower limbs is affected by
Mastoid antrum, ear ossicles and tympanic cavity Competent valves, deep fascia and contraction of
calf muscles
AIIMS NOVEMBER 2008
C The thoracic duct receives tributaries from
C Cells present in cerebellar cortex are Bilateral ascending trunk, bilateral descending
Purkinje, granular and Golgi thoracic duct and left upper intercostals duct left
(Note: also stellate cells and bipolar cells) subclavian, left jugular trunks.
6 AIIMS and All India PGMEE—Review Questions

C True about portal venous system C Intrinsic muscles of the tongue are derived from
The whole system is valveless Occipital somites
C Branches of cavernous part of internal carotid C Regarding sexual differentiation of the fetus
artery Gonadal development begins at 5th week of
Meningeal artery, cavernous branch and inferior intrauterine life.
hypophyseal branch. (Mn: MCH) C In emergency tracheostomy, structures damaged are
Isthmus of thyroid, thyroidea ima and inferior
AIIMS NOVEMBER 2007 thyroid vein.
In fracture of penis involving rupture of tunica
In forceful inversion of foot, fracture of the
C
C
albuginea with intact fascia, noted clinically is
tuberosity of 5th metatarsal is due to pull exerted by
Hematoma involving only the shaft of penis
Peroneus brevis
C Primitive streak is initiated and maintained by C In obstruction of second part of axillary artery, the
anastomosis between the following artery will
Nodal gene
maintain the blood supply of upper limb
C Watershed area between SMA and IMA which
Deep branch of transverse cervical artery and
commonly results in ischemia is
subscapular artery.
Splenic flexure
C The normal gallbladder epithelium is
Simple columnar with brush border AIIMS MAY 2007
C Perforators connecting superficial veins to deep C Exposure of left subclavian artery by supraclavicular
veins approach does not require the cutting of
Ankle, mid calf and lower thigh Scalenus medius.
C In a case of chest pain with pericarditis and C Most common site of ectopic pancreatic tissue is
pericardial effusion, pain is referred by Stomach.
Phrenic nerve C Facial nerve stimulation during testing of nerve is
C Pleural tapping in mid-axillary line, muscles indicated by contraction of
pierced are Orbicularis oris muscle.
Internal intercostal, external intercostal and C Cells which migrate from the base of the crypts to
innermost intercostal. ends of villi are
C In post-ductal coarctation of aorta, blood flow to Enterocyte, endocrine cells and goblet cell.
lower limb is maintained through C Renal angle lies between
Intercostal arteries and superior epigastric artery 12th rib and lateral border of sacrospinalis.
C Most cranial structure in root of left lung C Characteristics differentiating a typical cervical
Pulmonary artery vertebra from a thoracic vertebra except
C Ureteric peristalsis is due to intact supply of Has a large vertebral body.
Intrinsic smooth muscle pacemaker activity of renal C Pudendal nerve supplying motor part of external
calyces. sphincter is derived from
C True about left renal vein S2–S3 roots.
Posterior and inferior to superior mesenteric artery.
C If circumflex artery gives the posterior interven- AIIMS NOVEMBER 2006
trical branch, this circulation is described as C Characteristic of oculomotor nerve
Left dominance Enters orbit through superior orbital fissure
C In ulnar nerve injury in arm, following are seen C Common peroneal nerve is related to
Hypothenar atrophy, loss of sensation of medial 1/ Neck of fibula
3rd of hand and claw hand. C A person showing two cell lines derived from two
C Ureter develops from different zygotes is known as
Mesonephric duct Chimerism
Anatomy 7

C In IVC obstruction collaterals which open up are C Posterior relations of head of pancreas are
Superior epigastric vein and inferior epigastric vein CBD, aorta and IVC
Azygos vein and ascending lumbar vein C Urethral crest is situated in
Lateral thoracic vein and prevertebral vein Prostatic urethra
C Component of hypogastric sheath are
ALL INDIA 2012 Uterosacral ligament, transverse cervical ligament
and lateral ligament of bladder.
C Dilator pupillae is supplied by
The bifurcation of common carotid artery occurs at
Post-ganglionic sympathetic fibers from the
C

cervical sympathetic chain. The upper border of thyroid cartilage


C Structures crossing the right ureter anteriorly are C Muscles supplied by anterior division of mandibular
nerve
Right colic and ileocolic artery, terminal ileum and
vas deferens. Temporalis, masseter and lateral pterygoid
C Structure involved in perforation of ulcer in the C Structures forming floor of third ventricle
posterior wall of 1st part of duodenum Optic chiasma, infundibulum and mammillary body
Gastroduodenal artery (Note: Tuber cinerium also)
C Derivatives of the pharyngeal arches C Pneumatic bones are
Orbicularis oculi, palatine tonsil and tensor tympani. Frontal, ethmoid, sphenoid and maxilla
C True about valves of Houston C Type of joint seen in growth plate is
They disappear after mobilization of the rectum. Primary cartilaginous
C Thoracotomy is done by taking an incision in the C Diaphragm develops from structures
4th/5th intercostal space starting 1 cm away from Septum transversum, pleuroperitoneal membranes
the lateral margin of the sternum and cervical myotomes
Internal thoracic artery. C Muscular component of dorsal aorta develops from
C Structures passing through the inguinal canal in Paraxial mesoderm
females C Vessels not carrying deoxygenated blood in fetal
Ilioinguinal nerve, lymphatics from the fundal part circulation
of the uterus and round ligament of uterus. Umbilical vein
C True about spinal cord
The efferent nerve fibers originate from anterior
horn cells, it has gray matter core with white matter ALL INDIA 2010
covering and the ligamentum denticulum anchors C Gluteus medius is supplied by
the lateral borders of the spinal cord to the Superior gluteal nerve and artery
meninges. C Tendon passing below the sustentaculum tali
C The supraduodenal bile duct is chiefly supplied by FHL
Vessels that run upwards from gastroduodenal and
C Tendon with attachments on sustentaculum tali
retroduodenal arteries.
Tibialis posterior
C Urogenital diaphragm is made-up of the following
ALL INDIA 2011 Deep transverse perinei, perineal membrane and
C Movements affected by involvement of L5 nerve sphincter urethrae.
root are C Hypogastric sheath is a condensation of
Knee flexion, knee extension and toe extension Pelvic sheath
C True about injury to common peroneal nerve C Lymphatics from spongy urethra drain into
Results from injury to neck of fibula, loss of toe following lymph nodes
dorsiflexion and foot drop Deep inguinal nodes
C Branch from trunk of brachial plexus C Location of celiac plexus
Suprascapular nerve, nerve to subclavius. Anterolateral to aorta
8 AIIMS and All India PGMEE—Review Questions

C In post-ductal coarctation of aorta, collaterals may ALL INDIA 2008


be formed by
Suprascapular artery, subscapular artery and C Composite muscles are
posterior intercostal arteries Pectineus, adductor magnus, FDP and biceps
C Artery passing through foramen magnum femoris
Vertebral artery C Digastrics muscles are
C Movement that occurs during abduction of shoulder Muscle fibers in ligament of Treitz, omohyoid and
occipitofrontalis
Lateral rotation of scapula
C In the lungs, bronchial arteries supply the broncho-
C Composite muscles are
pulmonary tree till
Flexor digitorum profundus, pectineus and biceps
Respiratory bronchioles
femoris, adductor magnus
C The skin overlying the region where a venous cut
C Left sided SVC drains into
down is made to access the great saphenous vein is
Coronary sinus supplied by
Saphenous nerve
ALL INDIA 2009 C Correct pathway for a sperm
C Artery in anatomical snuffbox is Straight tubule- Rete testis- Efferent tubules (Mn:
Radial artery SRE)
C Branches of splenic artery are C Veins found in relation to paraduodenal fossa
Short gastric artery, hilar branches and arteria Inferior messentric vein
pancreatica magna C The artery to ductus deferens is a branch of
C Contents of deep perineal pouch Superior vesical artery
Dorsal nerve of penis, bulbourethral gland. It is C Tributary of the cavernous sinus includes
within wall of urethra (internal urethral sphinc- Superior petrosal sinus, inferior petrosal sinus and
ter). superficial middle cerebral vein
C Middle superior alveolar nerve is branch of C Diploic veins, true are
Maxillary nerve Valveless, present in cranial bones and have a thin
(Note: Not a direct branch) wall lined by a single layer of endothelium.
C The parasympathetic secretomotor fibers to parotid C Bones contributing to the nasal septum
traverse through Sphenoid, palatine and ethmoid
Otic ganglion, tympanic plexus and auriculo- C Cranial nerves containing somatic efferents
temporal nerve 3, 4 and 6
C Area that lies immediately lateral to anterior (Note: and 12)
perforating substance is C Facial colliculus is seen in
Limen insulae Pons
C Medulla oblongata is supplied by C Nucleus of Masseteric reflex
Anterior spinal artery, vertebral artery and PICA
Mesencephalic nucleus of trigeminal nerve
True about Sternberg canal
(Note: efferent is motar nucleus of trigeminal
C
Located anterior and medial to foramen rotun- nerve)
dum
C Closure of neural tube begins at
ALL INDIA 2007
Cervical region
C Chief cells are located in part of gastric gland C Primordial germ cells is derived from
Fundus Endoderm
C Gut associated lymphoid tissue (GALT) is primarily C Movements of pronation and supination occur in
located in joints
Lamina propria Superior, middle and inferior radioulnar joints
Anatomy 9

C Posterior cruciate ligament C True about trigone of urinary bladder is


Prevents posterior dislocation of tibia Mucosa is firmly attached to the underlying
C Structures that pass from thorax to abdomen musculature
behind the diaphragm are C Supports of uterus are
Aorta, thoracic duct and azygos vein (Mn: ATA) Uterosacral ligament, Mackenrodt’s ligament and
C Most common site of Morgagni hernia is levator ani
Right anterior C Posterior communicating artery is a branch of
C Structures not crossing the midline is Internal carotid artery
Left gonadal vein C Difference between typical cervical and thoracic
C Portosystemic shunt is seen in vertebrae
Liver, anorectum and gastroesophageal junction Has a foramen transversarium
C Injury to the male urethra below the perineal C Part of vertebral canal showing secondary curves
membrane causes urine to accumulate in with concavity backwards
Superficial perineal pouch Cervical
10 AIIMS and All India PGMEE—Review Questions

Biochemistry

AIIMS NOVEMBER 2015 glycogenesis in cytoplasm of hepatocytes but is not


immediately degraded by the enzyme glucose 6
C Codon for selenocysteine phosphatase because
UGA Glucose 6 phosphatase is present in the endoplasmic
(Note: Stop codons are UAA, UGA and UAG. Mn: reticulum and can not act on glycogen formed in the
You aa, You (gana) ga and You ag (laga) cytoplasm.
C Important in metabolism of but not specific to iron C Thiamine is a cofactor for
metabolism α ketoglutarate dehydrogenase
DMT1 branched chain keto acid dehydrogenase
C Heme is not synthesized in pyruvate dehydrogenase.
RBC (Note: Succinate dehydrogenase is catalysed by FAD)
C Biotin is required as a cofactor in
Acetyl CoA to malonyl CoA AIIMS MAY 2015
Propionyl CoA to methylmalonyl CoA
Pyruvate to oxaloacetate. C A young boy presenting with hypoglycemia,
(Note: Gamma carboxylation of glutamate requires hepatomegaly and accumulation of limit dextrins is
Vitamin K) most probably suffering from
C Electrophoretic mobility of HbS in a patient of sickle Cori’s disease
cell anaemia as compared to HbA (Note: Debranching enzyme)
Decreases C Crumpled tissue appearance is due to accumulation of
C Technique for protein precipitation include Glucocerebrosides
Trichloracetic acid (Note: Gaucher’s disease)
Heat precipitation C Enzyme involved in both glycogenesis and glyco-
Isoelectric point method. genolysis
C Increased in urine in lead poisoning Phosphoglucomutase.
ALA C RNAi
C Cofactors for glycogen phosphorylase include Knock down
Calmodulin, cAMP and protein kinase A C CRE recombinase binds specifically to
C Glucogenic intermediates of metabolism are LoxP site
Pyruvate, lactate and oxaloacetate C Lipid profile of a patient shows
C Vitamin is required in Total cholesterol-300 mg/dl, TG-150 mg/dl and
Methylmalonyl CoA to succinyl CoA conversion. HDL-25 mg/dl. What is the value of serum LDL-245.
C Glycogen synthesis and lysis takes place in the same (Note: Serum LDL = Total cholesterol-HDL-
cell. Glucose 6 phosphate is synthesized during Triglycerides/5 ).

10
Biochemistry 11

C Young boy presents with severe abdominal pain, on C Methylation of cytosine leads to
examination he has xanthoma. Blood sample drawn Decreased expression of gene.
has milky appearance of plasma. Lipoprotein C 2, 3 BPG binds to 1 site of hemoglobin and decreases
increased is the affinity for oxygen.
Chylomicrons C The last receiver of electrons in electron transport
C Acrid smell (pear) is found in system is
Paraldehyde O2
C Lactic acidosis due to thiamine deficiency occurs
due to enzyme affected AIIMS NOVEMBER 2013
Pyruvate dehydrogenase
C Most common metabolite of progesterone excreted
C A 10 years old boy presents with pain abdomen,
in urine
muscle weakness and fatigue and increased lead
levels in the serum. Enzyme activity increased is Pregnanediol
ALA synthase C Known as suicidal enzyme
Cyclo-oxygenase
(Note: Self catalysed destructions)
AIIMS NOVEMBER 2014
C Reasons responsible for ketosis in a patient of von
C A 7 months old child develops vomiting after fruit Gierke’s disease
juice feeds and is diagnosed as hereditary fructose Hypoglycemia, low blood sugar levels and oxalo-
intolerance. Enzyme deficiency acetate is required for gluconeogenesis.
Aldolase B C Exact location of a genetic loci is identified by
C Amino acids found in collagen fiber Fluorescent in situ hybridization
Glycine, lysine and proline C True about ribozyme
C A 9 months old child has pellagra like feature and Peptidyl transferase activity
mild aminoaciduria. Her elder sister has the same C Enzyme activity increased in low insulin/glucagon
complaints while four siblings are normal and level
parents asymptomatic. The most likely diagnosis Glucose-6-phosphatse
Hartnup disease C True about NADPH
C A 6 years old African American boy presents with Required for reductive biosynthesis, stabilizes the
recurrent infections, severe abdominal pain and membrane of RBCs
chronic hemolytic anemia. On evaluation, a non- C Component of fatty acid synthase complex
healing ulcer was found on his left leg. Peripheral
Ketoacyl synthase, enoyl reductase and acetoacetyl
smear showed abnormal shape of erythrocytes.
coa transferase
Basic defect
Single nucleotide change in a codon
AIIMS MAY 2013
C The investigation of choice to detect biomolecules is
X-ray crystallography C Enzymes catalyzing the irreversible steps of glycolysis
Glucokinase, phosphofructokinase and pyruvate
kinase
AIIMS MAY 2014
C Ketone bodies can be utilized by
C Not an intermediate product of citric acid cycle Heart, brain and skeletal muscles
Acyl CoA C A person with von Gierke’s disease has ketosis.
C The enzyme common to both glycogenolysis and True for ketosis in this patient
glycogenesis are Lactic acidosis, hypoglycemia and hypertriglyceri-
Phosphoglucomutase demia
C In the entire human genome, coding DNA consti- C Real time PCR is used for
tutes To know how much amplification of DNA has
0.02 occurred
12 AIIMS and All India PGMEE—Review Questions

C Enzymes participating in oxidation reduction AIIMS MAY 2012


reactions
Oxygenases, peroxidases, hydroperoxidases and C Gluconeogenesis in fasting state can be indicated by
dehydrogenases Pyruvate carboxylase activation by acetyl CoA
C Which of the following is a suicide enzyme C Same amino acid coded by multiple codons
Degeneracy
Cyclo-oxygenase
C Fluorescence means
C Embryonic hemoglobin is mainly composed of
Release of longer wavelength light on absorbing
Zeta and epsilon chains light of shorter wavelength
C Technique that can be used for exact localization of C DNA estimation by
a genetic locus Spectrophotometer
FISH
C Biomineralization is seen in AIIMS NOVEMBER 2011
Molluscs shell
C Chromosomal mutations can be identified from
C Insulin resistance is seen in patients with liver
disease due to Agarose gel electrophoresis, dideoxy nucleotide tail
sequencing and single strand conformational
Hepatic steatosis
polymorphism.
C Required in PCR
AIIMS NOVEMBER 2012 DNA template, deoxyribonucleotides, thermostable
enzyme, Mg2+ (dNTP).
C A child develops blisters on exposure to sunlight.
Irregular dark spots are found on the skin. Likely C Starting material for insulin production in bacteria
defects in the mechanism for therapeutic purpose
mRNA from beta-pancreatic cell of human
Nucleotide excision repair
C If 4 consecutive nucleotides code for an amino acid.
C In glycogen metabolism, some metabolically
No. of amino acids that can be theoretically coded
important enzymes of liver are activated by
by nucleic acid
phosphorylation. True
256
Commonly seen in fasting state than in feeding
C True about LDL receptor
state.
Taken by endocytosis, present on clathrin coated
C Method used to detect chromosomal differences pits on cell membrane and increased cellular
between neoplastic cells and their normal counter- cholesterol downregulates the receptors (Mn: CDE)
parts
C Involved in protein translation in eukaryotes
Comparative genomic hybridization Amino acyl tRNA, ribosomes and peptidyl
C Enzymes found in CSF are transferase
CK+ LDH C Most potent inhibitor of carnitine palmitate acyl
C Found in the association with membrane defect transferase on outer mitochondrial membrane, in
GPI anchored protein the well fed state
C Small scale mutations include Malonyl CoA
Substitution, deletion and insertion
C Advantages of nanotechnology in the diagnosis of AIIMS MAY 2011
cancers C Degraded by colonic flora
Nanocrystals exhibit bright, photostable fluore- Pectin, starch and glucose
scence, nanocrystals have a narrow spectrum C Cap in lac operon is
wavelength and peak spectrum wavelength is Positive regulator
tunable. C Modification in Gs subunit leading to watery
C True about ribozymes diarrhea in cholera
Peptidyl transferase activity ADP ribosylation
Biochemistry 13

C Apo B48 and Apo B100 are synthesized from AIIMS NOVEMBER 2009
mRNA, the difference between them is due to
Deamination of cytidine to uridine C In CRP, the C stands for
C Hydrophilic hormones that act on cytosolic receptors C polysaccharide of Streptococcus pneumoniae
Epinephrine, GH and ACTH C Glucose transporter in myocyte stimulated by
C Gene duplication plays an important role in the GLUT 4
evolution of C Karyotyping under light microscopy is done by
mRNA G banding
C Movement of protein from nucleus to cytoplasm can C Western blot detects
be seen by Protein
FRAP C True statements are
C Histone acetylation causes Chemiluminescence is excited electrons in higher
Increased euchromatin formation orbits comes to lower orbits by emitting energy in
C Triplex DNA is due to form of photon, bioluminescence is a form of
Hoogsteen pairing chemiluminescence and phosphorescence is energy
emitted following absorption of EM radiation.
C Acetyl CoA can be directly converted to
C Zinc is a co-factor for
Fatty acid, cholesterol and ketone bodies
Alcohol dehydrogenase
C After digestion by restriction endonucleases DNA
strands can be joined again by
DNA ligase AIIMS MAY 2009
C True about sickle cell disease C Receptors present in liver for uptake of LDL
Single nucleotide change results in change of ApoE and ApoB 100
glutamate to valine, HbS confers resistance against C Thiamine requirement increases in excessive intake
malaria in heterozygotes and RFLP results from a of
single base change. Carbohydrates
C Amino acid migrating fastest on paper chromato-
AIIMS NOVEMBER 2010 graphy on methylcellulose medium
C In phenylketonuria the main aim of first line of Valine
therapy is C NO is synthesized by
Limiting the substrate for deficient enzyme Arginine
C Which binds to CoA and condenses oxaloacetate to C Protein is purified using ammonium sulphate by
inhibit the TCA cycle Salting out
Fluoroacetate C True about eukaryotic protein synthesis
C A 45 years old woman presents with progressive mRNA reads from 5’ to 3’, EF2 shifts between GDP
palmoplanter pigmentation, calcification of to GTP and capping helps in attachment of mRNA
intervertebral discs in X-ray spine and Benedict’s to 40S ribosome.
reagent to urine gives greenish brown precipitate C A patient has isolated increase in LDL. His father
and blue-black supernatant fluid. Diagnosis is and brother had the same disease with increased
Alkaptonuria cholesterol. Diagnosis is
C After overnight fasting, levels of glucose transporter LDL receptor mutation
reduced in C Test using oligomer with single base pair substitution
Adipocytes RFLP
C Splicing activity is a function of
snRNA AIIMS NOVEMBER 2008
C After digestion by restriction endonucleases DNA C Replacing alanine by which amino acid will increase
strands can be joined again by UV absorbance of protein at 280 nm wavelength
DNA ligase Tryptophan
14 AIIMS and All India PGMEE—Review Questions

C Methods determining the protein structure are AIIMS NOVEMBER 2007


Mass spectroscopy, X-ray crystallography and
NMR spectrometry. C Nephelometry is based on the principle of
C Enzyme responsible for carboxylation reaction Light attenuated in intensity by scattering
Biotin C Involved in fast axonal transport
C Fireflies produce light due to Kinesin, dyenin and lysosomes
Peroxidase enzyme is used in estimating
ATP
C
Glucose
C Ribosome has enzymatic activity
Molecular change in lysosomal storage disorder is
Peptidyl transferase
C
Mutation of genes encoding lysosomal hydrolases
C True about glutathione
C Type of collagen found in hyaline cartilage is
It is a tripeptide, it conjugates xenobiotics and it is a
cofactor of various enzymes Type 2
C Non-functional enzymes are C Different sequence of amino acids having similar
structure of proteins is an example of
Alkaline phosphatase, acid phosphatase and γ
glutamyl transferase. Convergence
Cation used in PCR is
Refsum’s disease is due to deficiency of
C
C
Magnesium
Phytanic α oxidase
Hemoglobin estimation is done by
Intron is found in
C
C
Drabkin’s, Sahli’s and spectrophotometry
Nuclear DNA, B DNA and Z DNA
C In muscle, phosphorylase b is inactivated by
ATP AIIMS MAY 2007
C Action of α subunit G proteinis C For PCR required is
Breakdown of GTP to GDP Taq polymerase, d-NTP, and primer
C Synthesis of an immunoglobulin in membrane C The amino acid producing ammonia in kidney is
bound or secretory form is determined by Glutamine
Differential RNA processing C Hb electrophoresis is based on
Charge
AIIMS MAY 2008 C Enzyme responsible for complete oxidation of
C Steroid hormone receptors have attachment site for glucose to CO2 and water is present in
Steroid hormone, hormone responsive element and Mitochondria
transcription activators C First substrate of Kreb’s cycle is
C Insulin causes lipogenesis by Pyruvate
Increasing acetyl CoA carboxylase activity, C Rapid method of chromosome identification in
increasing the transport of glucose into the cells and intersex is
decreasing intracellular cAMP level. FISH
C True about G protein coupled receptors is C SYBR green dye is used for
G proteins act as excitatory and inhibitory because PCR
of difference in α subunit C Vitamin A intoxication causes injury to
C Poly A tail translates into Lysosomes
Polylysine C Y chromosome is
C True about glutathione Acrocentric
Tripeptide, conjugates xenobiotics and scavenger C End product of purine metabolism in non-primate
free radicals and superoxide ions. mammals is
C In molecular cloning, blue white screening is used for Allantoin
To identify desired chromosomal DNA insert in C Essential amino acids are
plasmid vectors. Lysine, leucine and methionine
Biochemistry 15

(Note: other essential amino acids are threonine C Decreased glucose concentration in hepatic cells
isoleucine, valine, phenylalanine, and tryptophan) triggers
C Increased copper excretion in urine is seen in Increased glucagon level in blood, activation of
Primary sclerosing cholangitis, primary biliary fructose 2, 6 bisphosphatase and inhibition of
cirrhosis and Wilson’s disease phosphofructokinase 2
C Active form of vitamin D is C Amino acids undergoing phosphorylation by
1, 25 (OH)2 vitamin D protein kinases
C Triglycerides are maximum in Serine, threonine and tyrosine
Chylomicrons C Autosomal dominant porphyrias are
C Glycolysis occurs in Acute intermittent porphyria, hereditary copropor-
phyria and porphyria cutanea tarda.
Cytosol
C Enzyme involved in both cholesterol and ketone
Following take place in mitochondria
body synthesis
C
Fatty acid oxidation, electron transport chain and HMG CoA synthase
citric acid cycle
C True regarding cytoplasmic mRNA
Vitamin excreted in urine is
It is chiefly translated from nuclear DNA
C
Vitamin C C Restriction enzymes type 2
Cleaves specific palindromic DNA sequence
AIIMS NOVEMBER 2006 C Amino acid in a protein acting as a potential
O-glycosylation site for attachment of an oligo-
C Increased level of lipoprotein (a) predisposes to
saccharide unit
Atherosclerosis.
Serine
C Same amino acid is coded by multiple codon due to
C Structure of protein involved in immune complex
following
formation
Degeneracy
Quaternary
Cytosolic cytochrome C mediates
Seen post 12–24 hours of starvation
C
C
Apoptosis Decreased liver glycogen, increased plasma FFA
C Chymotrypsin is a and increased ketone bodies
Zymogen
C Anticoagulant used to estimate glucose from a
ALL INDIA 2011
sample sent from PHC is
Potassium oxalate and NaF C Vitamin K is involved in post-translational
C Mucopolysaccharide hyaluronic acid is present in modification of
Vitreous humor Glutamate
C The 40 nm gap between the tropocollagen molecule in C Thiamine deficiency causes decreased energy
collagen serving as site of bone formation is occupied by production because
Ca2+ It is a coenzyme for pyruvic acid dehydrogenase
C Steroids act via nuclear receptors which interact and α ketoglutarate dehydrogenase in tricarboxylic
with DNA through acid pathway.
Zinc finger motif C True statement is
C Which estimates blood creatinine level most accurately Fluoroacetate competitively inhibits aconitase
Enzyme assay C Urea cycle occurs in
Liver
C Transfer of an amino group from an amino acid to
ALL INDIA 2012 an α keto acid is done by
C Methods used for protein estimation Transaminases
Bromocresol green method, Biuret method and C Cardioprotective fatty acid is
Lowry’s method ω3 fatty acid
16 AIIMS and All India PGMEE—Review Questions

C True about HDL C Technique used for detection of variations in DNA


It prevents oxidation of LDL sequence and gene expression
C Associated with increased risk of atherosclerotic Microarray
plaque formation C Test used for detection of specific aneuploidy
Apo E mutation, oxidized LDL and increased FISH, RT-PCR and QF PCR
homocysteine C Pre-natal diagnosis of hemophilia is best done by
C Disorders of protein misfolding are PCR
Alzheimer’s disease, cystic fibrosis and Creutzfeldt
Jakob disease ALL INDIA 2009
C Process in a vector used to increase the yield of
protein produced in recombinant protein synthesis C Enzymes involved in oxidation reduction reactions
Promoter induction are
C Factors responsible for deciding whether an Dehydrogenases, oxygenases and peroxidases
antibody/immunoglobulin will remain membrane C Least polar side chain is
bound or get secreted is Methyl
Differential RNA processing C Statement true about facilitated diffusion
C Methods of fusing two cells in genetic recombina- It requires carrier protein
tion technique include following techniques C True about lipoprotein lipase
Fusion mediated by ethylene glycol, electric current Found in adipose tissue, found in myocytes and
and viral transformation deficiency leads to hypertriglycerolemia
C Two transgenic plants were genetically engineered C The gap between segments of DNA on the lagging
using recombinant DNA technology using a strand produced by restriction enzymes are sealed by
plasmid vector. Which plant will glow spontaneously DNA ligases
in the dark C Normal role of micro RNA is
Plant with luciferase gene Gene regulation
C Sickling in HbS disease is primarily caused by
ALL INDIA 2010 Decreased solubility
C Inulin like fructans are used as prebiotics as they are C True about recommended dietary allowances
non-digestible. Resistance to digestion in upper GI RDA is statistically defined as two standard
tract results from deviations above estimated average requirement
β configuration of anomeric C2 C Plant component not fermented or degraded by
C Branched chain ketoacid decarboxylation is gastrointestinal microorganisms
defective in Lignin
Maple syrup urine disease C A 11 years old child with aggressive behavior, poor
C Sphingomyelinase deficiency is seen in concentration, joint pain, reduced urinary output
Niemann Picks disease and self multilating behavior. Deficient enzyme is
C The lipoproteins which does not move towards HGPRTase
charged end in electrophoresis C False about lambda phage
Chylomicrons Both lytic and lysogenic phase occur together
C Methods of chromatography in which molecules C Initiation of visual impulse is associated with
that are negatively charged are selectively released Photoisomerization and hydrolysis of visual purple
from stationary phase into the positively charged C A patient with molecular weight 100 KD is
molecules in mobile phase is termed as subjected to SDS PAGE electrophoresis. The SDS
Ion exchange chromatography PAGE electrophoresis pattern shows two widely
C Rothera’s test is used for detection of separated bands of 20 KD and 30 KD after addition
Ketones of mercaptoethanol. True is
C Liver enzyme which is predominantly mitochondrial The protein is a dimer of two 20 KD and two 30 KD
SGOT (AST) dimers.
Biochemistry 17

ALL INDIA 2008 C Treatment of multiple carboxylase deficiency is


Biotin
C Metabolic pathway generating ATP C Basement membrane degeneration is mediated by
Glycolysis, TCA cycle and fatty acid oxidation. Metalloproteinase
C In which tissue, glycogen is incapable of contributing
C Enzyme stable at acidic pH
to blood glucose
Pepsin
Muscle
C Proteoglycans, true are
C Phosphorylase is maintained in an inactivated state by
Chondroitin sulphate is a proteoglycan, they are
Insulin
made-up of sugar and amino acids and they carry
C Group of proteins assisting in folding of other proteins charge.
Chaperones C Protein purification and separation can be done by
C Proteins are sorted by Chromatography, centrifugation and electrophoresis
Golgi bodies C Study of multiplication of proteins in disease process
C Trypsin inhibitors are is known as
α1 antitrypsin, β1 antiproteinase and egg white Proteomics
C True about apoproteins C Protein that precipitates on heating to 45° and
Apoprotein A1 activates LCAT, apoprotein C1 redissolves on boiling is
activates lipoprotein lipase and apoprotein C2 Bence-Jones protein
activates lipoprotein lipase. C Northern blot is used for the separation of
C Type of hypertriglyceridemia associated with mRNA
increase VLDL and chylomicron remnants
C C terminal end of androgen receptors is concerned
Type 3 with
C Okazaki fragments are formed during the synthesis of Ligand binding
dsDNA
C HMP shunt, true are
C Water soluble vitamin is
Occurs in cytosol, no ATP produced in cycle and
Folic acid active in adipose tissue, liver and gonads.
C Vitamin required for carboxylation of clotting factors
C Within the RBC, hypoxia stimulates glycolysis by
Vitamin K regulating pathway
C Active form of calcium is
Hypoxia stimulates release of all glycolytic
Ionized calcium enzymes from band 3 on RBC membrane.
C Neutral molecules are transported by
C Dinitrophenol causes
Simple diffusion
Uncoupling of oxidation and phosphorylation
C Synthesis of triacylglycerol in adipose tissue, true
ALL INDIA 2007 are
C True regarding oxygenases Synthesis from dihydroxyacetate phosphate,
Incorporate 2 atoms of oxygen, incorporate 1 atom enzyme glycerol 3 phosphate dehydrogenase play
of oxygen and required for hydroxylation of steroid. important role and phosphatidate is hydrolysed.
18 AIIMS and All India PGMEE—Review Questions

Physiology

AIIMS NOVEMBER 2015 AIIMS MAY 2015


C Interstitial fluid volume can be measured by C Cholera toxin disrupts
Radioactive sodium and radioactive labelled Tight junctions/Zona occludens.
albumin. C If radius of artery is increased by 50%, blood
(Note: Radioactive sodium measures ECF volume velocity is increased by
and radiolabelled albumin measures plasma 5 times
volume. C β-cell of pancreas produces which hormone in
Interstial fluid volume = ECF-plasma volume) addition to insulin
C Anatomical dead space is measured with Amylin
Single breath nitrogen test C Fasting RBC use
C Not affected by immunity (immune privileged) Glucose
Seminiferous tubules
C Flow cytometry forward dispersion signifies
C Applying Nernst equation, equilibirium potential
Cell size
for potassium if extracellular potassium concentra-
tion is 100 meq/mmol and 10 meq/mmol C Hydrostatic pressure in a capillary is 37 mm Hg,
interstitial pressure is 3 mm Hg and net pressure
60 V
is 7 mm Hg, what is the capillary oncotic
C Formula for urea clearance is C = U × V/P. U stands pressure
for
27 mm Hg
Urinary concentration in mg/ml
C Clotting factor that helps in the stabilization of clot
C If the latent period of a muscle twitch is 10ms,
after it has formed by polymerization of fibrinogen
contraction time 40 ms and relaxation time 50 ms.
into fibrin
Tetanizing frequency for this muscle
Factor XIII
25 Hz
(Note: Tetanizing frequency = 1/twitch duration in C Adhesion of platelet to fibrinogen occurs via
seconds) VWF
C Actions of ANP are C 10 gram of mannitol is injected intravenously.
Afferent arteriole dilatation 10% is excreted. Plasma concentration of mannitol
Decreased sodium absorption in PCT measured is 50 mg/100 ml. ECF volume is
Inhibition of sodium reabsorption in medullary 18 L
collecting duct. C During initiation of voluntary movements
(Note: ANP also causes mesangial relaxation Both α and γ motor neurons are activated together.
leading to sodium excretion) (Note: α–γ co-activation)

18
Physiology 19

AIIMS NOVEMBER 2014 C Factor causing a shift of the oxygen-hemoglobin


dissociation curve to the left
C The current flow across a single ion channel can be pH decrease
measured by
C Which defines the pressure in the vascular system
Patch clamp test
in the absence of blood flow
C Sleep spindles and K complexes are characteristic of
Mean circulatory filling pressure
NREM sleep stage 2
C True about measurement of blood pressure
C Patellar tendon reflex is due to stimulation of
The length of the bladder in the blood pressure cuff
Dynamic muscle fiber
should cover more than 80% of the arm area
C The center of satiety is located in part of hypothalamus
C The left atrial ‘v’ wave is larger than ‘a’ wave in right
Ventromedial nucleus
atrium because
C Concentration and attention are the functions of
Left side is high pressure system
Frontal lobe
C The right atrium chronic overload is indicated by a
C The posterior column of the spinal cord carries
P wave of more than
sensations
2.5 mm
Proprioception, touch and vibration
(Note: and pressure) C The cells of the human body most sensitive to
ischemia are
C Parts of countercurrent mechanism
Neurons
Vasa recta, thick ascending loop of Henle and thin
ascending loop of Henle C The most important hormone that increases gall-
bladder contraction after a fatty meal is
CCK
AIIMS MAY 2014 C The most serious complication of prolonged sitting
C In an encephale isole preparation, the transaction is position is
done at Venous air embolism
First cervical spinal ligament
(Note: Brain is deprived of all sensory input from AIIMS NOVEMBER 2013
the spine but all cranial nerves intact)
Sympathetic stimulation increases blood flow to
If a supramaximal stimulus is applied to an
C
C
excitable tissue like a nerve or muscle and it elicits a Skin, coronary and cerebral circulation
response, then the tissue is said to be in C True about excitation and contraction coupling
Relative refractory period Acetylcholine is released at the nerve terminal,
(Note: Tetany) calcium is pumped back into the sarcoplasmic
C Nuclear bag fibers are related to reticulum during relaxation and calcium is released
from sarcoplasmic reticulum during contraction.
Length and velocity
C True about Golgi tendon organ is C Transcutaneous electrical nerve stimulation is
based on
Sense muscle tension
Gate controlled theory of pain
C The mechanostretch receptors in joints and
ligaments are C True about Kluver
Slow adopting Bucy syndrome-Hypersexuality, hypermetamor-
C During hip replacement, loss of joint and ligament phosis and placidity
receptors leads to C Peripheral resistance is best indicated by
No loss of sensation MAP > Diastolic blood pressure
(Note: Knee replacement–Loss of proprioception) C Blood testis barrier is located between
C Fast fatigue fibers are recruited during walking Sertoli cells and Sertoli cells
In the end (Note: Tight junctions)
20 AIIMS and All India PGMEE—Review Questions

AIIMS MAY 2013 AIIMS MAY 2012


C Repair of a cell membrane damaged by insertion of C Ventricular depolarization starts from
microneedle shall occur by Left part of interventricular septum
Resealing by lipid bilayer C During moderate exercise, blood flow to brain has
(Note: Macropuncture is repaired by Ca 2+ ion No effect
mediated vesicle exocytosis) C Statements true for ADH
C Release of synaptic vesicles from the presynaptic Post-operative increase in secretion, neurosecretion
terminals is inhibited by and act on distal tubule and increase permeability.
Prevention of Ca2+ influx C Basis for thermodilution method used in
C Fick’s law of diffusion explains measurement of cardiac output by pulmonary
Passive diffusion along concentration gradient catheter
C A normal person changes his position from Stewart-Hamilton principle
standing to supine. What will occur (Note: Swan-Ganz catheter)
Immediate increase in venous return C Critical closing volume is
C True regarding pulmonary neuroendocrine cells Close to residual volume
Contain serotonin and respond to a decrease in C CSF pressure is mainly regulated by
pO2 Rate of CSF absorption
C Most common end product of progesterone C Left lobe is responsible for
metabolism found in urine is Written and spoken language
Pregnanediol C In Lewis triple response, redness when skin is
scratched with a pointed object occurs due to
Histamine release due to local injury to mast cells
AIIMS NOVEMBER 2012
C Which does not occur in a neonate for heat production
C Clamping of the carotid arteries below the carotid Shivering
sinus is likely to produce C What will happen if the external cues are removed,
Increase in vasomotor center activity the sleep wake cycle in humans
C End diastolic volume increases due to Continue with cycle length of > 24 hours
Increase in negative intrathoracic pressure
C Self stimulation can be induced experimentally
AIIMS NOVEMBER 2011
most effectively from part of brain
Medial forebrain bundle C On exposure to cold, neonate shows
(Note: also ventral tegmentum, nucleus accumbens Crying and flexion of body like fetus position,
and frontal cortex) cutaneous vasocontraction and increased produc-
C A person, shot in the back at the level of D8 tion of noradrenaline for breakdown of fat in
vertebral body, immediately losing all the adipose tissue.
sensations below the level of transaction and is put C Noble prize for discovering structure and amino
on treatment but with no recovery. Possible reasons acid sequence of insulin as well as interspecies
are difference in molecular structure of it was given to
Glial scar formation, absence of growth factors and Fredrick Sanger
absence of endoneural tube C Increased level of alanine in serum on fasting is
C True about increase in blood supply in muscles primarily due to
during exercise Net loss of muscle protein due to increased
Local metabolites, increased arterial pressure and breakdown
cholinergic stimulation C Earliest barrier faced by microorganisms on
C Second messengers are exposed surface of the body
cAMP, IP3 and DAG Acidic pH
Physiology 21

C Receptors stimulated by the load producing reflex C In lungs, true about hyaline membrane disease
in the lungs that maintains the tidal volume FRC is smaller than closing volume
Stretch receptors in the bronchioles (Note: Closing volume > FRC > RV)
C Deleterious effects of hypothermia C Not permeable through blood brain
Cardiac arrhythmias, reversible coagulation and Proteins
renal failure C True about vasopressin receptors
C Primary role of the microbes of the gut flora of humans V1 smooth muscles, V2 collecting duct and V3 anterior
Fermentation of mucous, production of vitamin K pituitary
and formation of short chain fatty acid
(Note: Mnemonics: SCA)
C Gas used to measure the diffusion capacity of lung
AIIMS MAY 2011 CO
C Animals chronically exposed to cold C Hypoxemia is independent of
Increased sympathetic stimulation Hb
C True about shunt vessels C Stimulus for normal/resting ventilation
Play a role in thermoregulation Stretch receptors, pO2 and pCO2
(Note: Finger tips and ear lobule) C CO2 diffuses more easily through the respiratory
C Vasomotor centre of medulla membrane than O2 because it is
Acts with the cardiovagal centre to maintain BP More soluble in plasma
C Energy expenditure in resting state depends on C Pacemaker regulating the rate of respiration
Lean body mass Pre-Botzinger complex
C The primary direct stimulus for excitation of central C Negative intrapleural pressure is due to
chemoreceptor regulating ventilation is Absorption by lymphatics
Increased H+ C Produced by stomach and role in control of food intake
C True regarding myocardial O2 demand Ghrelin
Has constant relation to external cardiac work C True about neuropeptide Y
C Angiotensin 2 causes Mediated through melanocorticotrophin hormone,
Stimulation of thirst, aldosterone secretion and decreases thermogenesis and contains 36 amino
increased ADH secretion. acids.
C Maximum post-prandial motility is seen in C Not absorbed from PCT
H+
Sigmoid colon > Descending colon
C Cyanosis in trauma is interpreted as
C CSF pressure is mainly regulated by
Late sign of hypoxia
Rate of CSF absorption
C A man is given continuous testosterone, it would
True regarding GFR
lead to
C
If clearance of a substance is greater than GFR, then Azoospermia
tubular secretion must be present.
C Nitrogen narcosis is caused due AIIMS MAY 2010
Increased solubility of nitrogen in nerve cell
membrane C Hyperaldosteronism is associated with
Hypernatremia, hypokalemia and hypertension
C Intrinsic factor of Castle is secreted by
AIIMS NOVEMBER 2010
Parietal cells
C Caisson’s disease is C The main cause of increased blood flow to
Gas embolism exercising muscles is
C Vitamin K dependent clotting factor Vasodilatation due to local metabolites
Factor 7 (Note: Adenosine (most important), K+ ions, ATP,
(Note: 2, 7, 9 and 10) lactic acid, CO2)
22 AIIMS and All India PGMEE—Review Questions

C Apetite is stimulated by C Estrogen action on carbohydrate metabolism


Agouti related peptide, melanocyte concentrating Increased uptake of glucose through increase in
hormone and neuropeptide Y insulin sensitivity
C True about spinocerebellar tract is C Maximum potassium ion secretion is seen in
Smoothens and coordinates movement Saliva
C Capacitance of sperm takes place in
Uterine tube > uterus AIIMS NOVEMBER 2008
C Not done by insulin C True about presynaptic inhibition
Ketogenesis It occurs due to inhibition of release of neuro-
C Somatomedin mediates transmitter from presynaptic terminal.
Deposition of chondroitin sulphate C Nucleus controlling the circadian rhythm
Suprachiasmatic nucleus
AIIMS NOVEMBER 2009 C Mean circulatory filling pressure is
Arterial pressure taken at the point when heart
C Highest concentration of potassium is seen in stops beating
Rectum (Note: 7 mm Hg)
C In peripheral tissues which contains substance P C Mineralocorticoid receptors are present at
Nerve terminal Hippocampus, kidney and colon
C Stability of alveoli is maintained by C Structures lying in the renal medulla
Reduced surface tension by surfactant Loop of Henle, collecting duct and vasa recta
C Memory cells does not undergo apoptosis due to C Weber Fechner law is
presence of Magnitude of stimulus strength perceived is
Nerve growth factor approximately proportionate to the log of the
C Transaction at mid-pons level results in intensity of stimulus strength.
Apneusis C Vagal stimulation of the heart causes
C Hormones with cAMP as second messenger Increased R-R interval in ECG
Corticotropin, dopamine and glucagon C The second messengers are
C Events occurring in past one week is an example cAMP , IP3 and DAG
of
Recent memory AIIMS MAY 2008
C Equilibirium potential for an ion is calculated using
AIIMS MAY 2009 Nernst equation
Why fetal cells continue to divide but terminally
Cortical representation of body in the brain is
C
C
differentiated cells cannot divide
(Cortex)
There are many cyclin inhibitors which prevent cell
Vertical
to enter into S phase in adult.
Muscle spindle detects
Cerebral blood flow is regulated by
C
C
Length Blood pressure, arterial pCO2 and cerebral metabolic
C Fetal hemoglobin has higher affinity for O2 due to rate
Decreased 2, 3 DPG concentration C Pulmonary circulation differs from systemic
C Rodes and cones differ in circulation
Light sensitivity, wavelength and acuity Pulmonary vasoconstriction in hypoxia
C Physiological response to smoking are C Regarding Golgi tendon organ true is
Decreased HDL, increased hematocrit and increased Senses muscle tension
heart rate and increased catecholamine release C Increase in threshold level on applying subthres-
C For color vision, true is hold slowly rising stimulus is known as
Involves opponent color cells Accommodation
Physiology 23

C Regarding transport of substances through cell C Liver synthesizes


membrane. True are C3 complement component, haptoglobin and
Glucose is transported by facilitated diffusion, fibrinogen
active transport is an energy driven process and C Juxtaglomerular apparatus lies in relation to
facilitated diffusion requires carrier protein. Glomerulus
C Mechanism of positive nitrogen balance by C Renin is secreted by
addition of amino acids in diet in a seriously ill Juxtaglomerular apparatus
patient is C In isometric exercise, increases
Increased secretion of insulin Heart rate, cardiac output and mean arterial
pressure
AIIMS NOVEMBER 2007 (Note: Muscle tone also increases)
C O2 delivery to tissues depends on
C True about myocardial oxygen demand
Cardiac output, Hb conc. and affinity of Hb for O2
Directly proportional to mean arterial pressure
C True about free water clearance is
C Bicarbonate is maximally absorbed in Regulated by ADH
PCT C Bile acids are synthesized from
C Correct pair Cholesterol
G cell- gastrin C Iron is actively absorbed in
C Patient on low Ca + diet for 8 weeks, following Duodenum and proximal jejunum
increases to maintain serum calcium levels
PTH AIIMS NOVEMBER 2006
C Change noted during exercise
C Adrenaline, noradrenaline and dopamine act
Body temperature increases
through
A normal person changes position from standing to
Seven pass receptors
C
supine, following occurs
C Mean arterial pressure is calculated as
Immediate increase in venous return
(SBP + 2DBP)/3
C Changes in blood passing through the systemic
Factor favouring filtration at the arteriolar end of
capillaries are
C
capillary bed
Increase in hematocrit, pH decreases and increase
Increase in hydrostatic pressure of capillaries
in protein content
Increase in turbulence in blood flow by
Sertoli cells have receptors for
C
C
FSH Increase in diameter of blood vessel
C The mechanism of action of surfactant is
ALL INDIA 2012
Breaks the structure of water in the alveoli
C True about CSF C Intrapleural pressure is negative because
Removal of CSF during dural tap causes intense Chest wall and lungs recoil in opposite directions to
intracranial headache, normally contains no each other
neutrophils and pH less than that of plasma. C Cardiac output decreases during
C True about NO Standing from lying down position
Helps to regulate vascular tone, important role in C Glutamate as a neurotransmitter is synthesized
penile erection and present in low concentration in mainly in part of brain
cigarette smokers. Subthalamic nucleus
C Sex drive is chiefly regulated by part of hypothalamus
AIIMS MAY 2007 Posterior hypothalamic area
C True regarding endothelin 1 C Self stimulation can be induced experimentally
Brochoconstriction, vasoconstrictor, decreased most effectively from
GFR and has inotropic effect. Periaqueductal area (area around aqueduct of Sylvius)
24 AIIMS and All India PGMEE—Review Questions

C The dissolved oxygen transported by 100 ml of C Orthopnea in heart failure is due to


blood in a person breathing 100% oxygen at 4 atm Reservoir function of leg veins
9 ml C Not seen in humans
(Note: Amount of dissolved O 2 = 0.003 ml/dl Estrous cycle
blood/mm Hg PO2) C The nucleus involved in Papez circuit is
C Discharge from baroreceptor causes inhibition of Anterior nucleus of thalamus
Rostral ventrolateral medulla. C Basal metabolic rate depends most closely on
Lean body mass
C Amylin is secreted by cells of islet of Langerhan’s
C True about phagocytosis
β-cells
Ameba and other protozoa make their living out of
C Electromechanically, the systole is the time interval it, used to ingest particles < 0.5 μ in size and
between- Q and S2. digestion occurs within phagolysosomes.
C Ventrolateral cordotomy helps pain relief in the
right leg as it interrupts
ALL INDIA 2010
Left lateral spinothalamic tract.
C Reason of diurnal variation in eosinophil count C Vitamin K dependent coagulation factors include
Variation in cortisol levels Factor 9 and 10
C Possible genotype for blood group A person is (Note: 2, 7, 9 and 10)
During heavy exercise cardiac output increases up
AO
C
to five fold while pulmonary arterial pressure rises
C With no external cues, the sleep wake cycle in very little. This physiological ability of pulmonary
humans circulation is explained by
Continue with cycle length of > 24 hours Increase in number of open capillaries
C The nerve impulse leads to initiation of muscle C Venous return to heart during quite standing is
contraction facilitated by
It causes opening of the calcium channels which Calf muscle contraction during standing, valves in
leads to increase in Ca2+ ion concentration with the perforators and sleeves of deep fascia.
use of sarcomere. C During cardiac imaging the phase of minimum
motion of heart is
ALL INDIA 2011 Mid-diastole
C True about bronchial circulation
C The primary action of NO in the gastrointestinal is
Contributes 2% of systemic circulation, causes
Gastrointestinal smooth muscle relaxation venous admixing of blood and provide nutritive
C Maximum water reabsorption in GI tract occurs in function to lung.
Jejunum C An important non-respiratory function of lungs is
C Small airways have laminar airflow Sodium balance
Because of extremely low velocity (Note: ACE in lung)
C Pulmonary compliance is decreased in C Hot water relieves pain of abdominal spasm
Pulmonary congestion, decreased surfactant and By stimulation of adrenergic fibers
pulmonary fibrosis C Insulin secretion is inhibited by
C A man connected to a body plethysmograph for Epinephrine
estimation of FRC inspires against closed glottis. C Lesions of lateral cerebellum causes
True is Ataxia, incoordination and intention tremor
The pressure in the lung decreases, but that in box C Low CSF protein may be seen in
increases Recurrent lumbar puncture, pseudotumor cerebri
C A 32 years old high altitude mountaineer is obser- and infant
ved to have a hematocrit of 70 percent. Likely cause is C Decreased basal metabolic rate is seen in
Polycythemia with increased red cell mass Obesity
Physiology 25

ALL INDIA 2009 ALL INDIA 2008


C The oxygen-Hb dissociation curve is sigmoid C Lewis triple response is mediated by
because Histamine
Binding of one oxygen molecule increases the (Note: axon reflex mediates only flare)
affinity of binding to other O2 molecules. C True about histamine
C An increase in the concentration of 2, 3 DPG may be Found in mast cells, increases gastric acid secretion
seen in and related to arousal and blood pressure.
Anemia, hypoxia and inosine C True regarding secretin
C Central chemoreceptors are most sensitive to Increases bicarbonate rich secretion, inhibit gastric
change in blood acid secretion and causes contraction of pyloric
sphincter.
Increase in pCO2
C The site where myosin head binds to actin in
C During diastole arterial pressure is maintained by skeletal muscles are covered by
Elastic recoil of aorta Tropomyosin
(Note: Windkessel effect) C Sarcolemmal proteins are
C True about myocardial oxygen demand Dystroglycan, sarcoglycan and dystrophin
Correlate with heart rate (Note: also syntropin)
C Cyanosis does not occur in severe anemia because C The blood within the vessels does not clot normally
because
Critical concentration of Hb required to produce
cyanosis is reduced Vascular endothelium is smooth and coated with
glycocalyx
C Factors involved in intrinsic pathway
C The blood pressure measured by a sphygmomano-
Factor 9 and 11 meter is
C Factors which help in bridging the fibrin in a clot and Leaser than intra-arterial pressure
stabilizes the clot C True statement
Factor 13 Vagal stimulation decreases rate of contraction,
C When a person changes position from standing to noradrenaline increases force of contraction and
lying down, change seen is denervated heart has more heart rate.
Venous return to heart increases immediately C Spermatogenesis occurs at
C Renal physiology Temperature lower than core body temperature
C True regarding fertilization and implantation
Distal tubule always receives hyposmotic solution,
The tubes are lined by ciliated epithelium
GFR is controlled by resistance in afferent and
efferent arterioles and glomerulus receives C True regarding flow of lymph from lower limb
capillaries from the afferent arteriole. Increased by massage of foot
C Effect of moderate exercise on cerebral blood flow
C Sertoli cells play a key role in
Does not change
Spermiogenesis C Sensations transmitted by dorsal tract
C Action potential is initiated at the axon hillock, Fine touch
initial segment of the neuron because
Threshold for excitation is lowest ALL INDIA 2007
C Involved in transmission of regulatory signals C Agranular endoplasmic reticulum is involved in the
through ECF synthesis of
Synaptic signals through neurotransmitters, Lipids
endocrine signals through hormones and G protein C Substance present in both serum and plasma
coupled receptors. Factor 7
C Key regulators of sleep are located in C Site of RBC formation in 20 years old healthy male
Hypothalamus Flat bones
26 AIIMS and All India PGMEE—Review Questions

C True about S4 C Delta waves are seen in


Heard during ventricular filling phase Deep sleep
C Heart muscles, true are C Toxic effects of high oxygen tension include
Acts as a syncytium, has gap junctions and has Pulmonary edema, retinal damage and CNS
branching excitation and convulsion
C Broca’s area is concerned with
C Adaptations apt to increase the work capacity at
Word formation
high altitude
C The mechanism of hearing and memory include
Decreasing workload and increasing duration of
Changes in level of neurotransmitter at synapse,
exercise
increasing protein synthesis and spatial reorganiza-
tion of synapse. C Hormones secreted by kidney
C The processing of short term memory to long term Renin, erythropoietin and 1, 25 DHCC (Mn: RED)
memory is done in C The difference in the age of onset of puberty
Hippocampus amongst males may be explained by
(Note: Explicit/Declarative) Increased inhibin levels
Pharmacology 27

Pharmacology

AIIMS NOVEMBER 2015 (Note: < 2 μg/kg/min- D1


2–10 μg/kg/min- D1 + β1
Aspirin inhibits platelet aggregation by
> 10 μg/kg/min- D1 + β1 + α1)
C
Inhibiting platelet Thromboxane A2 synthesis.
C Antimicrobial drug acting through DNA are
(Note: By inhibiting cyclooxegenase-COX enzyme)
Nitrofurantoin, quinolones and rifampicin
Low volume of distribution of a drug is due to
(Note: Linezolid acts by inhibiting protein synthesis)
C
Low distribution into organs or tissues.
C Safety of drug assessed in
Drug of choice in pregnancy induced hypertension
Phase 1 of drug trial
C
α methy dopa
C Contraction of visual field defect is seen with
C Oral drug approved for the treatment of Hepatitis C anticonvulsant
Ledipsavir Vigabatrin
C Pilocarpine acts on sphincter papillae. Drug acting C Gender dependent side effect of valproate
on dilator papillae analogous to pilocarpine
PCOD
Epinephrine
C Drug of choice for avian flue
A young woman presents with history of unprotected
Oseltamavir
C
intercourse. Emergency contraceptives that can be
C Drugs used in treatment of bird flue
used are
Oseltamivir, Zanamivir and Peramivir
Copper IUCD, levonorgestrel and ulipristal acetate.
(Mnemonics: COLUM-Copper IUCD, OCP, C Absorption of drug increased after a fatty meal
Levonorgestrel, Ulipristal acetate and Mifepristone) Griseofulvin
C Newly approved drug for drug resistant TB C Calcineurin inhibitor
Bedaquiline Cyclosporine
C Antidote of fibrinolytics C Advice given with bisphosphonates
EACA To be taken in empty stomach with full glass of
C Anticraving drugs for alcohol are water.
Naltrexone, ondansetron, topiramate and (Note: To avoid esophagitis)
acamprosate. (Mn: NOTA) C Anticholinergic drug exclusively used in pre
C Mechanism of action of Etanercept anaesthetic medication
Inhibits TNFα Glycopyrrolate
(Note: Etanercept, Adalimumab and Infliximab are C Drug to be sold only on prescription of registered
TNFα inhibitors. Mnemonics: Etalian adalat ka medical practioner belong to schedule- H.
insaf.) C Glucocorticoid can be used in
C Low dose dopamine at 3–5 μg/kg/minute causes Multiple myeloma, Hodgekin’s lymphoma and
β1 stimulation CLL.

27
28 AIIMS and All India PGMEE—Review Questions

AIIMS MAY 2015 C True about NSAIDS


Reduce the effect of antihypertensive medications,
C Heterochromia irides occurs due to local application can be used by topical route of administration and
of cause nephrotoxicity.
Latanoprost C Lithium toxicity indicated by
C Methacholine acts on Plasma level above 2 mEq/L
M2 receptor agonist C Associated with development of edema
C Drugs causing amorphous whorl like opacities in Amlodepine, cyclosporine and estrogen
the cornea are
C Polypeptide with an average molecular weight of
Amiodarone, chloroquine and chlorpromazine. 30,000, exerts oncotic pressure similar to albumin,
C Drug given in the pulmonary arterial hyperten- not antigenic but to be watched for rare hyper-
sion sensitivity reaction
Bosentan Polygeline
C Drug enhancing insulin secretion are C Long term use of PPI use is associated with increased
Sitagliptin, exenatide and glipizide. incidence or risk of
(Note: Pioglitazone reduces blood glucose and Community acquired pneumonia, Clostridium
HbA1c without increasing circulating insulin. difficile infections and hip fracture in women
C Protease inhibitor C Time dependent killing and prolonged post-
Nelfinavir antibiotic effect is seen with
(Note: All-navirs) Clindamycin
C Xenobiotics are metabolized by CYP enzyme C True about penicillin G
To increase water solubility. Used for treatment of rat bite fever
C Adrenergic β receptor causing lipolysis C Best treatment for infection due to Pseudomonas
β3 aeruginosa infection
C Nitroglycerine is given sublingually as it is Piperacilin, Tazobactam
absorbed rapidly below the tongue because of C 3rd generation anti-MRSA cephalosporin among the
Non-ionic and high lipid soluble. following
C Drugs causing hearing loss are Ceftriaxone
Kanamycin, chloroquine and vancomycin C A 35 years old man presents with antibiotic
associated diarrhea caused by Clostridium difficile.
C Diuretic preferred in mild to moderate hypertension
Drug effective
Thiazide diuretics.
Oral vancomycin
C Type of enzyme inhibition in organophosphorus
C Drugs used for the treatment of chronic hepatitis B
poisoning is
Entecavir, lamivudine and telbivudine
Irreversible
C The most effective treatment for the severe falcipa-
rum malaria among the following
AIIMS NOVEMBER 2014
Artesunate
C Drug considered as pregnancy category B-presumed C True about paclitaxel
safety in humans based on animal studies It binds to microtubules and promotes stabilization
Brimonidine of tubulins, used for breast, ovarian and lung
C Therapeutic indications of cholinomimetics cancers and myelosuppression and alopecia are the
Glaucoma, post-operative atony and myasthenia main side effects.
gravis C Drug approved in man for advanced prostate cancer
C Antimuscarinic not tertiary amine Goserlin > abarelix
Glycopyrrolate C True about rituximab
C Dexmedetomidine is a It is a chimeric antibody that targets the CD20 β cell
Centrally acting β2 receptor agonist antigen, it is the first monoclonal antibody that
Pharmacology 29

received FDA approval for relapsed indolent C Drugs useful for the treatment of advanced prostate
lymphomas and toxicity is mainly associated with cancer
infusion reactions. Goserelin
C Abatacept, a T cell co-stimulation inhibitor is used
for the treatment of AIIMS NOVEMBER 2013
Rheumatoid arthritis
C Correct match of drug and mechanism of action
Brimonidine decreases aqueous production
AIIMS MAY 2014 C Rho-kinase inhibitor
C Drugs converted into active metabolite are Fasudil
Fluoxetine, cyclophosphamide and diazepam C True about uses of iodine
C Methacholine is a It inhibits release of thyroid hormone, it causes
M2 receptor agonist acute inhibition of iodotyrosine and iodothyronine
C β2 agonist causes synthesis and it can cause iodism.
Analgesia, sedation and anxiolysis C Duration of action of flumazenil
C Peripheral vasospasm is observed with anti- 40 minutes
Parkinsonian drug C Drug having both α and β agonistic activity
Bromocriptine Epinephrine
C Due to potentiating action of lithium to non- C Drug that can be stopped without any withdrawal
depolarizing muscle relaxants, how many days symptom
before lithium should be stopped before adminis- Fluoxetine
tration of the muscle relaxant C Ability of body to eliminate the drug from body is
2 days called as
C True about methadone Clearance
It is a long acting β receptor agonist, it is rapidly C True about carbamazepine
absorbed from the gastrointestinal tract and is Has been known to cause Stevens-Johnson syndrome.
detected in plasma 30 minutes after oral adminis- C Tricyclic anti-depressents produce side effects like
tration and the primary use of methadone is relief of dry mouth, urinary retention due to action on
chronic pain. Muscarinic receptors.
C The site of action of frusemide is C Cholinomimetic drugs are used in
Thick ascending limb of loop of Henle Glaucoma, myasthenia gravis and post-surgical
C Pair of drug and its indication matched correctly ileus/atony
Octreotide VIPoma associated diarrhea, desmo- C Anticholinergic drug not crossing the blood brain
pressin- diabetes insipidus and hCG-infertility in barrier
man and woman. Glycopyrrolate and hyoscine butyl bromide
C Time dependent killing and post-antibiotic effect is C As compared to unfractionated heparin, low
seen with molecular weight heparin has reliable anti-
Clindamycin coagulant action because
C NNRTI includes It is less protein bound
Etravirine, delavirdine and nevirapine
C Antitubercular drug associated with hypothyroidism AIIMS MAY 2013
Ethionamide > rifampicin C Rho-kinase inhibitor
C Anti-cancer drug least likely to cause nausea and Fasudil
vomiting C Opoid receptor responsible for dysphoric effects
Chlorambucil Kappa
C Essential for ameliorating the toxicity of pemetrexed C Ritonavir inhibits
Folic acid and vitamin B12 Amiodarone, cisapride and midazolam
30 AIIMS and All India PGMEE—Review Questions

C Time dependent killing and prolonged post- C Treatment of choice for extended spectrum beta-
antibiotic effect is seen with lactamase producing enterococci
Clindamycin Piperacilin + tazobactam
C True about treatment with iodine C Rho-kinase inhibitor is
It causes acute inhibition of synthesis of diiodotyro- Fasudil
sine and diiodothyronine C In new drug designing, problem arises in
C Orphan drugs are Increasing drug interaction with nontarget proteins
Drugs for treating rare disease C Unfavorable interaction of drug and substrate in
C Trials performed to determine the maximum human beings are all except
tolerated dose of a drug Barbiturates decreases B12 absorption.
Phase 1 C True statement is
C Lithium potentiates the action of non-depolarizing Dobutamine decreases peripheral vascular resistance.
muscle relaxants. Lithium should be stopped how
many days before administration of the muscle AIIMS MAY 2012
relaxant
2 days C Which of the following statements about anti-
epileptics are
C Injection available for subcutaneous administration
Phenytoin and carbamazepine act by prolonging
Terbutaline
Na+ channel activation, carbamazepine is also used
C Drugs causing SLE like syndrome in trigeminal neuralgias and diazepam is an anti-
INH, hydralazine and sulphonamide epileptic.
C One concern of the drug designing is to C Proton pump inhibitors have short half lives
Increase the number of interactions of the drug with (2 hours) but cause a prolonged suppression of acid
the target protein. secretion (up to 48 hours) because
C One of the many mechanism of adverse events is its They irreversibly inhibit the the proton pump
increased binding to secondary targets, usually molecule and hence, acid secretion requires
proteins. With respect to primary target, the synthesis of new proton pumps.
secondary target C Which of the following antipsychotics is available
Should have similar binding sites in a depot injection
C Vasopressin antagonists act on part of the nephron Fluphenazine
Medullary collecting duct C True about first order kinetics is
The rate of elimination is proportional to the plasma
AIIMS NOVEMBER 2012 concentration
C A glaucoma patient is being given systemic beta- C Drug induced colitis is most frequently associated
blockers. What cannot be given with
Levobunolol Clindamycin
C 2nd generation antihistamines are C A female patient under treatment for depression
took a massive dose amitriptyline for suicide. True
Cetrizine, loratadine and fexofenadine
about management.
Drugs used in treatment of erectile dysfunction
Gastric lavage was done, sodium bicarbonate was
C
Prostaglandin E, Vardenafil and alprostadil administered to treat acidosis and diazepam was
C True regarding penicillin G injected to control seizures.
Used for treatment of rat bite fever C Drug used to treat hormone responsive breast
C Therapeutic indications of cholinomimetics cancer
Glaucoma, post-operative atony and myasthenia Tamoxifen
gravis C Drugs banned for players by the International
C Ocular side effects of HAART therapy Olympic Committee (IOC) are
Uveitis Salbutamol, erythropoietin and spironolactone
Pharmacology 31

C Traditional medicines effective in the treatment of C A patient on insulin and acarbose develops
heart failure hypoglycemia. For treatment to be given is
Terminalia Arjuna Glucose
C Amifostine is C Hypoglycemia caused by
Radiorotector Insulin, glimepiride and nateglinide (Mn: Anti-
diabetic drug with-ide cause hypoglycemia)
C True about exenatide
AIIMS NOVEMBER 2011
Decreases glucagon secretion, GLP1 analogue and
C Bremelanotide is used for given subcutaneously.
Erectile dysfunction C Methetrexate is used in
C Most recent oral DTI used for prevention of acute Psoriasis, rheumatoid arthritis and ankylosing
stroke spondylitis.
Dabigatran C Indications of cotrimoxazole are
C Lid retraction is side effect of Lower UTI, prostatitis, and typhoid (Mn: PLT)
Apraclonidine C True about Ifosfamide
C Brinzolamide is Its is a nitrogen mustard, metabolized by CYP 3A4
Highly specific, reversible and noncompetitive to form active metabolite and chloracetaldehyde is
carbonic anhydrase inhibitor. (Mn: NRS) active form.
C OZURDEX C Angina worsened by
0.7 dexamethasone Sumatriptan, dipyridamole and thyroxine. (Mn: STD)
C Peripheral conversion of T4 to T3 inhibited by C Megaloblastic anemia caused by
Phenytoin, sulfasalazine and alcohol and Methotre-
Amiodarone, propylthiouracil and propranolol
xate.
C True about octreotide
C Octreotide is used in
Useful in controlling secretory diarrhea.
Insulinoma, glucagonoma and carcinoids
Drugs used for the treatment of multiple sclerosis
True about ESBL
C
C
Interferon β Sensitive to carbapenems, Ambler classification is
C Drugs approved for the treatment of relapsing based on molecular structure and classification is
remitting multiple sclerosis based on 3rd generation cephalosporin sensitivity.
Interferon α 1a, interferon β 2b and glatiramer C True about diazoxide
acetate K+ channel opener, can be used as antihypertensive
C Only drug found to be beneficial in halting the pro- agent and used in insulinoma.
gression on EDSS (expanded disability status score) C Topical mitomycin C is used in treatment of
Natalizumab Tracheal stenosis
C Drugs used as a transcranial patch for treatment of C Drug used in sickle cell anemia
Parkinson’s disease Hydroxyurea
Rotigotine (Mn: Patch looks like roti) C Bleomycin toxicity is characterized by destruction of
C Factors predisposing to chloroquine toxicity Type 1 pneumocytes
Cumulative dose > 460 gm, daily dose > 250 mg/ C Regarding ACE inhibitor, true is omission of prior
day or > 3 mg/kgand renal failure. diuretic dose decreases the risk of postural
C Efavirenz inhibits hypotension.
HIV-1 reverse transcriptase C Adverse effects of tamoxifen used for breast cancer are
Thromboembolism, endometrial carcinoma, and
cataract.
AIIMS MAY 2011 C Side effects of naloxone are
C True about Pioglitazone Hypertension, pulmonary oedema and ventricular
Metabolized in liver, not given in case of diastolic dysrhythmia.
dysfunction and PPAR gamma agonist (Mn: G for C Muscle rigidity due to opoids is because of its effect on
gamma, g for glitazone). Mu receptor
32 AIIMS and All India PGMEE—Review Questions

AIIMS NOVEMBER 2010 C Drug causing hypomagnesemia by increased


excretion is
C Hemorrhagic cystitis is caused by Frusemide therapy
Cyclophosphamide C Pharmacovigilance is done for monitoring
C Methorexate resistance occurs by Drug safety/toxicity
Overproduction of DHFR
C Drug that is both antiresorptive and bone formative is
SERM drug used for osteoporosis is
Strontium ranelate
C
Raloxifene
C Thalidomide is used in
C HER-2/neu receptor plays a role in
HIV induced mouth ulcer, Beçhet’s syndrome and
Predicting therapeutic response erythema nodosum leprosum.
True about Remelteon are
Alkylating agents are
C
C
It acts on both MT1 and MT2 receptor, metabolized
Chlorambucil, melphalan and cyclophosphamide
by cyp450 and recently approved for insomnia.
C Drug used in pulmonary hypertension
C Following can be done in amitryptiline poisoning
Calcium channel blocker, endothelin receptor
Sodium bicarbonate infusion, gastric lavage and
antagonist and prostacyclin.
diazepon for seizure control.
C Opioid agonist are
C Abnormality to check if lithium is given to a
Morphine, codeine and methadone.
pregnant female
Cardiac anomaly
C Drug used in estrogen dependent breast cancer AIIMS NOVEMBER 2009
Tamoxifen C Associated with thiazide diuretics
C Large dose of EDTA is used in carbonic anhydrase Impotence
enzyme inactivation. The mechanism by which C Increased prolactin is associated with
EDTA act is Increased estradiol
It chelates with the metal ion of the enzyme C Metabolite of prodrug carisoprodol
C A child has phocomelia. This is due to drug taken by Meprobamate
mother (Hint: Both has-pro)
Thalidomide C Voriconazole is effective against
C Used in overactive bladder is Aspergillosis, Candida albicans and Candida tropicalis
Darifenacin C Buspirone is an
C ICU patient empirically started on ceftriaxone and Anxiolytic
amikacin with blood culture report showing C Flumazenil, true are
Klebsiella with ESBL, next step is Specific antagonist of BZD, given intravenously
Change ceftriaxone to imipenem and acts on GABA A receptor.
C Erythromycin is given in decreased bowel motility
AIIMS MAY 2010 because
It binds with motilin receptor
C Used in treatment of heroin dependence
C Drug causing nephrotoxicity
Buprernorphine, clonidine and lofexidine
Tacrolimus
C Cyp 3A inhibitors are
True about phenytoin
Erythromycin, itraconazole and ritonavir
C

C Most common dose limiting toxicity of anticancer It follows zero order kinetics
drugs is C Nitrates are used in
Myelosuppression CCF, esophageal spasm and cyanide poisoning.
C Oligospermia reported in C In India, drugs given under supervised regimen are
Methotrexate Rifampicin, pyrazinamide and clofazimine
C Drug of choice in Zollinger-Ellison syndrome C Prodrugs are
Proton pump inhibitors Quinapril, fosinopril and benzopril
Pharmacology 33

C Least narcotic C True regarding acarbose


Papaverine It acts by inhibiting enzyme alpha glucosidase. It
C Anti-TNF used in decreases the progression of impaired glucose
RA with HIV, RA with HCV and RA with tolerance to overt diabetes mellitus and it reduces
pulmonary fibrosis. fibrinogen level.
C Function cox-2 is C Drugs used as immunosuppressants are
Cell proliferation Glucocorticoids, cyclosporine and azathioprine
C Drugs useful in detrusor instability
Solefenacin, tolterodine and flavoxate
AIIMS MAY 2009
C Drugs requiring dose reduction in cirrhosis are
C Drugs aggravating myasthenia gravis Diazepam, metronidazole and rifampicin
Phenytoin, tetracycline and aminoglycoside C Drugs causing neuroleptic malignant syndrome are
C True about cephalosporins Amantadine, haloperidol and metoclopramide
Ceftazidime is 3rd generation cephalosporin, C A 55 years old with breast cancer was prescribed
cefoperazone has antipseudomonal effect and herceptin (trastuzumab). True is
cephalosporin act by inhibiting cell wall synthesis. It is a monoclonal antibody produce by injecting
C True about warfarin is her-2 antigen.
It causes inhibition of Vit K dependent clotting C Anticancer drug excreted by lung is
factors, T½- 36 hours and crosses placenta. 5 Fluorouracil
C Drug used in heparin overdose (Note: as CO2)
Protamine sulfate C A girl on sulphonamides developed abdominal
C Anticancer drugs given orally pain and presented to emergency with seizure.
Mesna Probable cause is
C Tolazoline is used as Acute intermittent porphyria
Vasodilator in treating coronary artery stenosis
during angio procedures. AIIMS MAY 2008
C Cetuximab (EGFR antagonist) can be used in
Palliation in head and neck cancer C Cross resistance of isoniazid is seen with
C Most emetogenic drug is Ethionamide
C Drugs acetylated are
Cisplatin and high dose cyclophosphamide
Dapsone, hydralazine and INH (Mn: SHIP).
C MDR gene acts by causing
C Prodrug is
Efflux of drug
Enalapril
C Toxicity of amiodarone therapy are
C Free water clearance decreased by
Pulmonary fibrosis, corneal microdeposits and
Furosemide
cirrhosis of liver.
C An antibiotic acting by inhibiting cell wall synthesis
C Digoxin toxicity is increased by
Cefepime
Renal impairment, hypocalcemia and hypomagne-
C Drug treating both dermatophysis and candidal
semia.
infection are
C Cerebellar toxicity is seen with
Ketoconazole
Cytarabine
C SIADH is caused by
Vincristine, vinblastine and cyclophosphamide
AIIMS NOVEMBER 2008 C Teratogen causing deafness is
C Reaction occurring in phase one in metabolism of Isotretinoin
xenobiotics is C Imatinib is used in treatment of
Oxidation, reduction and hydrolysis Chronic myelomonocytic leukemia
(Note: Cyclization, Hydrolysis, Oxidation, Reduction C Sustained neutropenia is seen with
and Decyclization. Mn: CHORD) Carmustine
34 AIIMS and All India PGMEE—Review Questions

C Used in scabies are C Fibrates-True are


BHC, permethrin and Crotamiton Increase lipoprotein lipase activity through PPAR
C Used in Erectile dysfunction are and cause increased lipolysis of TG, cause urticaria,
Alprostadil, vardalafil and PGE2 rashes, alopecia, myopathy and GI distress and are
C 2nd generation antihistamines are first line of drugs in severe dysbetalipoproteinemia
oratidine and acrivastine and hypertigyceridemias.
Drugs to be discontinued on day of surgery are
True about aminoglycosides are
C
C
Metformin
Teratogenic, excreted unchanged in urine and
distributed only extracellularly. C Therapeutic drug monitoring is advised in
C Loading dose depends on Phenytoin, tacrolimus and cyclosporine
Triptans in migraine acts on
Volume of distribution
C
5 HT 1B/1D
C Drugs used in H. pylori are
C Thiazides true is
Omeprazole, metronidazole and amoxicillin
Used in CCF, causes hyperglycemia and increases
C Long acting beta-2 agonist is
uric acid concentration in serum.
Salmeterol
C True about protease inhibitors in HIV
Benzodiazepine antagonist is
Powerful enzyme inhibitor, cause hepatic toxicity
C
Flumazenil and all protease inhibitors are substrates for
C Cytochrome p450 inhibitor is glycoprotein coded by MDR gene.
Ketoconazole C Good clinical practice seen in
C True regarding carbamazepine is Phase I, II, IV trials
Used in trigeminal neuralgia. (Note: III also)
C Drug used in uncomplicated alcohol withdrawal C OCP should not be used with
Diazepam Rifampicin
C Drug causing hyperglycemia is
AIIMS NOVEMBER 2007 Glucocorticoid
C Bisphosphonates are used in
Post-menopausal osteoporosis, steroid induced AIIMS MAY 2007
osteoporosis and malignancy associated hypocal- C Gynaecomastia is caused by
caemia. Spironolactone
C Drugs used for thromboprophylaxis are C Impotence most commonly caused by which
Heparin, aspirin and warfarin antihypertensive drug
C True about frusemide is Beta-blockers
It is used in pulmonary edema C Endogenous catecholamines are
C Drugs acting through GABA receptors are Epinephrine, norepinephrine and dopamine
Benzodiazepines, barbiturates and zopiclone C Bone marrow apalasia is seen with
C Drug with sedation as side effects are Chloramphenicol, alpha-methyl hydantoin and
Antihistamines and antidepressants phenylbutazone.
C True about phenytoin are C Antiemetic are
Potent microsomal enzyme inducer, highly protein Domperidone, ondansetron and cyclizine
bound and with increasing dose t1/2 increases. C Antimetabolite drug is
C 5HT3 receptor blocker with highest receptor affinity is Methotrexate
Palonsetron C Hypertension is seen with
C True regarding benzodiazepines Erythropoietin, cyclosporine and NSAID
If taken in higher doses it is less toxic than other C Antihypertensive drugs contraindicated in pregnancy
sedatives. Spironolactone, sodium nitroprusside and ACE
(Note: Flat curve) inhibitor
Pharmacology 35

C Drug which increase NO are C Anticoagulant effect of warfarin is increased by drugs


GTN, hydralazine and sodium nitroprusside Amiodarone, cimetidine and phenylbutazone
C Carbimazole- teratognic effects are C Aromatase inhibitor is
Choanal atresia, scalp defect and neck swelling Letrozole
C Imatinib used in CML acts by C Drug associated with untoward side effect of renal
Competitive inhibition of ATP binding site of Abl tubular damage
Kinase Cisplatin
C FK 506 is C Anticancer drug causing secondary leukemia
A macrolide antibiotic Cisplatin
C Naloxone is not used during resuscitation of a child C Inhaled nitric oxide is used for
whose mother is on Reducing pulmonany hypertension
Methadone C Antiepileptic drug causing weight loss
C Ditoxin toxicity is precipitated by Topiramate
Electrolyte disturbance, acute myocardial infarction C A patient of hepatic vein thrombosis receiving
and renal disease. coumarin therapy for last 3 years has developed
C Drugs useful for MRSA are bleeding tendency. Effect of coumanin can be
Cotrimoxazole, vancomycin and ciprofloxacin reversed by
Vitamn K injection
C Imatinib mesylate is
AIIMS NOVEMBER 2006 Tyrosine kinase inhibitor
C Loading dose of aminophylline is
C Drugs metabolized by acetylation are
5–6 mg/kg
INH, hydralazine and procainamide (Mn: SHIP)
C Second generation antihistamines are AIIMS NOVEMBER 2005
Fexofenadine, loratadine and Atorvastatin
C Denosumab (a monoclonal antibody against RANKL C Antimicrobial therapy for ESBL producing Pseudo-
receptor) is used in treatment of monas aeuroginosa causing febrile neutropenia is
Imipenem and amikacin
Osteoporosis
C Side effect of a drug can be minimized by its
C Drugs with maximum interval in dosage in a
High specificity
patient with creatine clearance less than 10 is
C A drug can be used in low concentrations if it has
Vancomycin
High affinity
NSAID undergoing enterohepatic circulation is
Mechanism of action of tetracycline
C
C
Piroxicam Binds to 30s subunit and inhibits binding of
C Anti-arrhythmic agents which are class IC are aminoacyl tRNA
Propafenone, flecainide and encainide. C Drug causing granuloma in the liver
C ATT causing transient memory loss is Allopurinol
INH C Drug causing hyperkalemia
Cyclosporine, heparin and NSAIDS
Crohn’s disease can be treated by
AIIMS MAY 2006
C
Anti-TNF alpha antibody
C The most significant side effect of ACE inhibitors is C Drugs causing central anticholinergic syndrome
First dose hypotension Atropine sulphate, antihistaminic and tricyclic
C Neostigmine antagonizes non-depolarizing antidepressants
blockade by C True about heparin
Decreasing acetylcholine breakdown at motor end Most commercial preparations of heparin now
plate, preventing K + efflux from the cell and utilize pig intestinal slimes, act by antithrombin
depolarization of motor end plate. activation and produce thrombocytopenia.
36 AIIMS and All India PGMEE—Review Questions

C Antiepileptic drug which decreases the uptake of C Unconscious man brought to hospital suffering
GABA into neuron and glial cells from methyl alcohol poisoning. True are
Tiagabine Kussmaul’s breathing present, papilloedema
C The mu receptor of opioids is responsible for present and plasma bicarbonate might be very low.
Analgesia, sedation and respiratory depression C Drug of choice of Listeria meningitis in patient
C Anticancer drug with selective action on hypoxic allergic to penicillin
tumor cells Trimethoprim-sulphamethoxazole
Mitomycin C. C True about Vincristine
C Anticancer drug most commonly given by Alkaloid, neurotoxicity is side effect and used for
continuous infusion induction of remission in acute lymphoblastic
leukemia.
Cytosine arabinoside
C True about hydralazine
C Nephrotoxicity is side effect of
Causes direct relaxation of blood vessels, increases
Tacrolimus
plasma renin activity and postural hypotension is
C Drug having covalent interaction with target not a common problem.
Aspirin C True about antitubercular drugs
C Radioiodine destroys the neoplastic cell of papillary Ethambutol accumulates in renal failure,
carcinoma thyroid predominantly by hyperuricemia is recognized side effect of
Beta-rays pyrazinamide and red green colour impairment is
C Pemetraxed, useful in breast cancer is classified as an early sign of ethambutol induced optic neuritis.
Antimetabolite C Immediate reversal of warfarin can be done by
Fresh frozen plasma
AIIMS MAY 2005 C Opioids with 90% plasma protein binding capacity
Sulfentanyl
C Indications of ACE inhibitors are
C Indication of ropinirole (non-ergot dopamine agonist)
Hypertension, myocardial infarction and left
Parkinson’s disease
ventricular dysfunction.
C Antihypertensive drug contraindicated in pregnancy
C Zidovudine resistance develops due to
Enalapril
Mutation at reverse transcriptase
C Diuretic not requiring their presence in tubular
Drug of choice for SIADH
lumen for pharmacological effects
C
Demeclocycline Spironolactone
C Alpha adrenoceptor agonist
Guanabenz, guanafacine and clonidine AIIMS MAY 2004
C True about NSAIDS
C Drugs contraindicated in G6PD deficiency
Acetylsalicylic acid (aspirin) is irreversible cox
inhibitor, salicylic acid reduces in vivo synthesis of Cotrimoxazole, furazolidone and nalidixic acid
PGs, and anti-platelet effect of low dose aspirin is C Pro drugs are
related to presystemic cox inhibition. Chloral hydrate, enalapril and oxacarbazepine
C Slow acting schizonticide antimalarial drug is
Pyrimethamine
AIIMS NOVEMBER 2004 C Thalidomide can be used in
C Estrogen increases the risk of ENL
Breast carcinoma, endometrial carcinoma and C Aldosterone antagonist (Spironolactone) is used in
hepatocellular carcinoma. CHF, hirsutism and hypertension
C A 20 years old, 50 kg weight girl consumed 10 gm of C True about lithium
paracetamol with alcohol, 6 hours earlier. Serum Used in bipolar disorder, monitoring of lithium
paracetamol level now is 400 μg/dl. True is concentration in blood is necessary and thiazides
Hepatotoxicity is likely to occur are useful in treatment of lithium induced DI.
Pharmacology 37

C Fourth generation cephalosporins are C Drug of choice for thyrotoxicosis in pregnancy


Cefepime and cefpirome (Hint: Pi) Propylthiouracil
C Drug causing pseudotumor cerebri C Sterile haemorrhagic cystitis is caused by
Tetracycline Cyclophosphamide
C Drug requiring dose adjustment in a patient during C A 65 years old women with history of MI and CHF
radiotherapy to prevent radiation toxicity for 5 years has locally advanced breast cancer.
Antineoplastic drug not given
Dactinomycin
Anthracycline (Doxorubicin and daunorubicin)
C Immunosuppressant drugs are
(Note: Cardiotoxic)
Cyclosporine, azathioprine and steroids
C Hypervitaminosis causing congenital malformations
C Drug causing methemoglobinemia Vitamin A
Nitroglycerine, prilocaine and procaine C Peripheral neuropathy occurs with
(Note: and benzocaine) Vincristine, cisplatin and procarbazine
C Aminophylline inhibits C A manic patient on haloperidol has become restless
Phosphodiesterase and keep pacing in the room for a day, with tremors
C Most potent androgen is of hand
Dihydrotesterone Restless leg syndrome
C Women receiving tamoxifen should be periodically C Antimicrobial drug causing ergosterol biosynthesis
screened with inhibition is
Endometrial sampling Amphotericin B
C Pyronaridine is
AIIMS NOVEMBER 2003 Antimicrobial drug

C Nucleotide derivative prescribed in kidney trans-


AIIMS MAY 2003
plant for renal failure in a 60 years old male
Azathioprine C Diuretics requiring their presence in the tubular
C H3 receptor agonist causes lumen for therapeutic action
Inhibition of H1 receptor mediated wakefulness, Carbonic anhydrase inhibitor, Na +/K + symport
inhibition of H1 mediated bronchoconstriction and inhibitor and Na-Cl symport inhibitor.
negative chronotropic effect on atria C True regarding beta-blocker in heart failure
C Sparfloxacin and terfanadine can cause Initiated at very low dose, slow upward titration of
Ventricular arrhythima dose is required and carvedilol is most widely used
C Prolactin secretion is inhibited by in this condition.
Km of an enzyme is defined as
Bromocriptine
C
The substrate concentration at half maximal velocity
C Side effect of salbutamol
C An HIV positive patient on therapy with zidovu-
Hypokalemia, tachycardia and tolerance
dine, lamivudine, and indinavir has developed
C Side effects of ACE inhibitors are tuberculosis. Antitubercular drug not given in this
Angioneurotic edema, cough and skin rash setting
C An epileptic patient on phenytoin develops Rifampicin
weakness and fatigue, Hb-4.6 gm/dl, MCV 104 fl, C MESNA protects in
MCH 40 pg/dl. Diagnosis is Cancer chemotherapy
Megaloblastic anaemia C Antitubercular drugs needing dose modification in
(Note: Folic acid deficiency) chronic renal failure are
C Oral hypoglycemic drug causing dilutional Ethambutol, isoniazid and streptomycin
hyponatremia C True about sotalol
Chlorpropamide Non-selective beta-blocker, prolongs action
C Therapeutic uses of PGE1 potential and polymorphic ventricular tachycardia
Impotence, MTP, patency of PDA. is a common side effect.
38 AIIMS and All India PGMEE—Review Questions

C Antipseudomonal cephalosporins are C IV penicillin injection leads to generalized urticaria,


Ceftazidime, cefoperazone and cefepime swelling of lips, hypotension and bronchospasm.
C True about clonidine use in hypertension Treatment
Reduction in central sympathetic outflow, can be Epinephrine injection.
combined with vasodilators and sedation and C Drug of choice for all types of actinomycosis
xerostomia are common adverse effects. Penicilline G
C The time required by first order drug to achieve C Prolactin secretion inhibited by
steady state levels can be predicted from Dopamine
Half life C tRNA analogue (resembling aminoacyl tRNA)
C Recent onset generalized epilepsy patient on causing premature chain termination that inhibits
phenytoin develops drug reaction and skin rash. protein synthesis
Next done is Puromycin
Shift to valproic acid C The substrate concentration at which enzyme activity
C A 60 years old hypertensive patient on losartan is is maximum for an enzyme with Km = X will be
posted for hernia repair. Losartan should be 2X
Continued till the day of surgery C True about statins
Bioavailability is minimally modified when
AIIMS NOVEMBER 2002 provastatin is taken with food, simvastatin is
C Side effects of phenytoin rapidly metabolized but provastatin is least
Ataxia, hirsutism and lymphadenopathy metabolized and CNS accumulation of simvastatin
and lovastatin is high and that of provastain and
C Drug of choice for cryptococcal meningitis
fluvastatin is low.
Amphotericin B
C The drug used in prostatic carcinoma
C Pulmonany fibrosis occurs most commonly after
Finasteride, flutamide and diethylstilbestrol
Bleomycin
C Drugs causing myopathy
C Lymphocytopenia occurs after large dose of predni-
Betamethasone, chloroquine and zidovudine
sone to a patient of lymphocytic leukemia is due to
C Drugs causing pancreatitis
Massive lymphocytic apoptosis
Didanosine, furosemide and valproic acid
C Drug induced myopathy caused by
C Drugs used in acute bronchial asthma
Atorvastatin, chloroquine and d-penicillnamine
Hydrocortisone, ipratropium and salbutamol
C 5-lipoxygenase inhibitor is used in
C True about metformin
Bronchial asthma.
Causes little or no hypoglycemia in nondiabetic
C The mu opoid receptor is responsible for subjects, increases peripheral utilization of glucose
Bradycardia, hypothermia and miosis and decreases absorption of glucose from intestine
C Diuretic decreasing renal lithium clearance and when given with alcohol, increases risk of lactic
hydrochlorothiazide acidosis.
C True about antiparkinsonian drugs C Protein binding of drug least affected in patients
Amantadine causes ankle edema with nephrotic syndrome and hypoalbuminemia
C True about effects of prostaglandins Morphine (30% protein binding)
PGE2 and PGI2 inhibit platelet aggregation whereas C Drug causing hypertensive crisis in pheochromocy-
TXA2 facilitate aggregation. PGE2 and PGEI2 cause toma
vasodilation but TXA2 cause vasoconstriction. Propranolol, salfasalazin and captopril
C A patient of unknown poisoning presents with C Terminal cancer patient-effective and safe drug
tachycardia, hypotension and myoclonic jerk Oral morphine
seizures, ECG finding-HR 120/min, QRS interval C True about beta-blockers
0.16 and, arterial blood finding-pH 7.25, pCO2 30 Nadolol has longest half life, atenolol is longer
mm Hg and HCO3 of 15 mmol/L acting than metoprolol and labetalol has both alpha
Imipramine poisoning and beta-blocking action.
Pharmacology 39

C Analgesic drug contraindicated in head injury Clinical significance of drug interaction of


patients is clopidogrel and PPI has been given much attention.
Morphine The metabolizing enzyme common to these two
C Racemic mixture of two enantiomers with different drugs is
pharmacokinetic and pharmacodynamic properties Cyp2C20
is seen in C Cyanide toxicity can be caused due to overdose of
Verapamil Sodium nitroprusside
C A drug is more likely to cause toxicity in elderly C Mobius syndrome is caused by antenatal intake of
patients due to Misoprostol
Decreased renal excretion, decreased hepatic C The maternal intake of drugs associated with
metabolism, and increased sensitivity of receptors. hypospadias in the male infant
Diethylstilbestrol
ALL INDIA 2012 C A 33 years old female patient pregnant P3 + 0, 8
weeks gestation, asymptomatic on investigation,
Drug used for treatment of constipation dominant IBS
RPR and MHA- TP are positive. Most appropriate
C
Lubiprostone treatment
C The drug used to treat cerebral vasospasm Penicillin
Nimodipine C Bactericidal drugs are
C Apoxiban Isoniazid, moxifloxacin and daptomycin
Factor Xa inhibitor
C An old patient on medications is planned for
C The antineoplastic agent temozolomide belongs to surgery. Drug stopped
Alkylating agent Angiotensin 2 receptor blockers
Monitoring of plasma levels is done for drugs
Indications of thiazides
C
C
Lithium, cyclosporine and gentamycin
Hypertension, heart failure and nephrolithiasis due
C Drug induced osteomalacia to idiopathic hypercalcemia
Phenytoin
C Drugs causing sinus tachycardia
C A patient with multiple myeloma is suffering from
Amphetamine, nifedipine and theophylline
severe hypercalcemia. Drugs decreasing the serum
calcium levels are C Nowadays, method used for drug designing is
Prednisolone, disodium etidronate and furosemide Target structure based
C Fluoroquinolones with longest half life C Cardiotoxicity is caused by
Moxifloxacin Rafecoxib
C A HIV patient on HAART therapy presents with C Antidiabetic drug associated with vitamin B 12
lipodystrophy, hypertriglyceridemia and hyper- deficiency on long term use
cholesterolemia. Drug responsible Metformin
Ritonavir C Side effect of thalidomide
C A 20 years female patient suffering from diarrhea is Sedation, constipation and neuropathy
treated by a drug that inhibits pyruvate-Ferrodoxin C A patient of leukemia who was given chemo-
oxidoreductase. Diagnosis is therapy with dactinomycin and ara-C 6 weeks
Cryptosporodiosis before now presents with persistent pancytopenia.
(Note: nitazoxanide) Causes can be
C Side effect of isoniazid in a patient of pulmonary TB Vitamin B12 and folic acid deficiency, persistent leuke-
who is slow acetylators mia and myelosuppression due to chemotherapy.
Peripheral neuropathy C Alkaline diuresis enhances the elimination of drug
C Clopidogrel is given for patients with stroke, Phenobarbitone
myocardial infarction and peripheral vascular C A patient on ketoconazole for fungal infection has
disease. PPIs are also given along with it to prevent developed GERD. Drug avoided
the risk of increased GI erosion and bleeding. Cisapride.
40 AIIMS and All India PGMEE—Review Questions

C After taking some drug for acute attack of migraine, C True about Ranolazine are
a patient developed nausea and vomiting, deve- Piperazine derived antianginal agent, used as first
loped tingling and numbness in the tip of finger that line agent in chronic angina and may improve
also turned blue. Most likely drug implicated glycemic control.
Dihydroergotamine C True regarding selective estrogen receptor down
C An old aged male patient used a drug for allergic regulator (SERD), fulvestrant are
conjunctivitis that prevents the release of chemical Selective estrogen antagonist, used in treatment of
mediators from mast cells. Mechanism of action of breast cancer and administered as once a month
drug dose.
Inhibition of 5 lipoxygenase C True about erlotinib are
(Note: Zileuton) Tyrosine kinase inhibitor, rash may occur and used
in non-small cell lung carcinoma.
ALL INDIA 2011 C True about aprepitant are
Crosses the blood brain barrier, metabolized by cyp
C Actions of muscarinic antagonists are
3A4 pathway, and ameliorates nausea and
Decrease gastric secretions, decrease respiratory vomiting of chemotherapy.
secretions and facilitates AV conduction.
C Agents used for treatment of thrombocytopenia
Buprenorphine is classified as
secondary to myelosuppressive cancer chemotherapy
C
Partial agonist at mu receptor
Oprelvekin
C Opioid tolerance develops to
C Drugs that should not be used in a patient on
Analgesia, euphoria and nausea and vomiting.
Lithium in order to prevent lithium toxicity
(Exception: 3C convulsion, constipation and
Diuretics
constriction of pupil).
C True about epidural opioids are C Known adverse effects of thalidomide are
Acts on dorsal horn substantia gelatinosa, may DVT, teratogenicity and peripheral neuropathy
cause pruritis and may cause respiratory depre- C Alkalinization of urine is done is case of
ssion. Methotrexate
C Drug most commonly used worldwide in mainte- (Hint: Alkalinisation in acidic drug poisoning)
nance doses for opioid dependence is C Mineralocorticoid receptors are found in
Methadone (Mn: M for M) Colon, hippocampus and kidney
C True about serotonin syndrome are
Not an idiosyncratic reaction, can be caused by SSRI
ALL INDIA 2010
and associated with hyperthermia and hypertension.
C A patient prescribed primaquine develops dark C Narrow therapeutic index is seen in
coloured urine after administration of drug. Lithium
Diagnosis is C Proton pump inhibitor is
G6PD deficiency Omeprazole
C Prolonged treatment with INH leads to deficiency of C Mechanism of action of theophylline in bronchial
Pyridoxine asthma includes
C Amphotericin treatment mandates the monitoring Phosphodiesterase inhibition, Adenosine receptor
of antagonism and increased histone deacetylation.
K+ C Methyldopa is primarily used in
C Integrase inhibitor approved for HIV is Pregnancy induced hypertension
Raltegravir (Mn: tegravir are integrase inhibitor) C True about opioid receptor antagonists are
C True about meglitinides are Naloxone can be used for treatment of opiod
Act by stimulating insulin release, decrease post- induced constipation, naltrexone may be used for
prandial hyperglycemia, and hypoglycemia is less treatment of alcohol dependence and nalmefine has
common than with sulfonylurea. a longer half life than naloxone.
Pharmacology 41

C L-Dopa is combined with carbidopa in the C True about mycophenolate mofetil


treatment of parkinsonism to Used in transplant rejection, it is a prodrug and
Inhibit peripheral decarboxyation of levodopa converted to mycophenolic acid and it is not used
C Drug that should not be used with rivastigimine in with azathioprine.
patients with Alzheimer’s disease C Drug used for treatment of heparin induced
TCA thrombocytopenia
C Ethosuximide is the drug of choice for Lepirudin
Absence seizures C True about trientine are
C True about phenytoin Alternative to penicillamine in non-tolerant, not
Follows saturation kinetics, is teratogenic and is given with iron within 2 hours of ingestion and may
highly protein bound (Mn: Zero watt power) cause iron deficiency anemia.
C Fetal hydantoin syndrome is caused by C Allopurinol is used in treatment of
Phenytoin Gout
C Correctly matched teratogenic effects C In equivalent concentrations, topical steroids are
Phenytoin-cleft lip/palate, valproate-neural tube more potent in form of
defects and warfarin-nasal bone dysplasia.
Ointment
C Drug causing hirsutism
Danazol, phenytoin and norethisterone
ALL INDIA 2009
C Most common congenital anomaly associated with
lithium C Effects of estrogen are
Cardiac malformation Reduces LDL and increases triglycerides
C Agents used for prophylaxis of migraine C Adverse effects of valproate are
Propranolol, valproate and topiramate Alopecia, liver failure and weight gain
C Serotonin syndrome may be precipitated by C Nitroglycerine causes
Pentazonice, buspirone and meperidine Methemoglobinemia, hypotension with tachycardia
C MAO inhibitors should not be used with and vasodilatation.
Pethidine C Cephalosporin not needing dose modification in
C A young male allergic to penicillin presents with presence of reduced GFR
meningococcal meningitis. Most suitable treatment is Cefoperazone
Chloramphenicol C Antimicrobial drug needing dose reduction in
C Monitoring in patient receiving linezolid therapy of patient with renal failure
Platelet count Fluconazole, vancomycin and imipenem
C True about Thalidomide C Adverse effects of tacrolimus are
Reintroduced for its activity in ENL, developed as Nephrotoxicity, neurotoxicity and hepatotoxicity
antiemetic in pregnancy but withdrawn because of
C Ifosfamide is an
phocomelia and used for new and relapsed cases of
multiple myeloma. Alkylating agent
C Pancreatitis occurs with C Anticancer drug that is cell cycle phase specific
Didanosine Bleomycin
C Drug with activity on both HER 1 and HER 2 new C Bleomycin toxicity is characterized by
receptors Hyperplasia of type II pneumocytes
Lapatinib C Hand foot syndrome can be caused by
C Tyrosine kinase inhibitors are used in the treatment of Capecitabine
GIST C Drug not used in acute angle closure glaucoma
C Drugs recommended for treatment of GIST is Clozapine
Imatinib C Treatment of choice for severe falciparum malaria
C Amphotericin B toxicity can be lowered by using is
Liposomal delivery system Intravenous artesunate
42 AIIMS and All India PGMEE—Review Questions

ALL INDIA 2008 C Drugs given in enteric fever are


Cotrimoxazole, ciprofloxacin and ceftriaxone
C Phase II in a clinical drug trial is done
C The drug of choice for treatment of neurosyphilis
To assess therapeutic efficacy is
C Therapeutic index is a measure of a drug’s Procaine penicillin G
Safety C Antifungal drugs are
C ED 50 is a measure of Ciclopirox olamine, ketoconazole and undecylenic
Potency acid
C Mechanism of action of pralidoxime is C Anti-tumor peptide is
Reactivation of cholinesterase Bleomycin
C Ipratropium bromide is contraindicated in C Mechanism of action of Paclitaxel is
Urinary retention Stabilization of tubules
C Tiotropium is used for C Leucovorin is given with
Treating asthma Methotraxate
C Drugs used to relieve urinary spasm after C Adverse effects of tacrolimus include
urological procedure Neurotoxicity, nephrotoxicity and diabetes
Darifenacin, oxybutynin and tolteradine
C Immunomodulator used for genital warts is
Use of ergotamine is contraindicated in
Imiquimod
C
Ischemic heart disease
C Enzymes and reactions involved in metabolism of
C Mechanism of action of clomiphene citrate is xenobiotics are
Binds estrogen receptors and prevents negative
Cytochrome P450, methylation and hydroxyla-
feedback at hypothalamus.
tion
C Drugs used in anxiety are
SSRI, clonazepam and buspirone
C Cell wall synthesis is inhibited by ALL INDIA 2007
Amoxicillin, penicillin G and cefotetan C Conditions associated with beta-2 agonist treatment
C Antipseudomonal penicillin is are
Piperacillin Hyperglycemia, detrusor relaxation and relaxation
C True regarding penicillin G are of gut and bronchial muscles.
Active against Gram-positive organisms, proben- C Conditions associated with use of beta agonist in
cid given along with penicillin G increases its preterm labour are
duration of action and acts by inhibition of cell wall Hyperglycemia, tachycardia and relaxation of
synthesis. uterine muscles.
C True regarding methicillin in MRSA C Nonselective beta-blockers with additional actions
Resistance is produced as a result of alteration in are
PBP, resistance may be produced by hyperproduc-
Carvedilol, cartelol and labetalol
tion of beta lactamase, resistance may be missed at
incubation temp. of 37°C during susceptibility C True regarding clonidine are
testing. Increases parasympathetic outflow, decreases
C True regarding adverse effects of tetracycline are sympathetic outflow by stimulating central alpha
May lead to discoloration of teeth, are a common receptor and used in hypertension.
cause of superinfections and may precipitate liver C Finasteride is a
damage. 5 alpha reductase inhibitor
C Nephrotoxic antibiotics are C Drugs used in erectile dysfunction are
Streptomycin, gentamycin and polymyxin B Apomorphine, Alprostadil and PGE1 analogues.
C A drug of choice for prophylaxis in diphtheria is C Drugs used in obesity are
Erythromycin Orlistat, olestra and sibutramine
Pharmacology 43

C Leukotrine antagonist is C Drug causing hypercoagulability is


Montelukast L-asparaginase
C Drug used for NSAID induced gastric ulcer is C Methotrexate is used in high doses in
PGE 1 agonist (But DOC-PPI) Osteosarcoma
C Hormones with transcription factors as receptors C Filgrastim in used in treatment of
are Neutropenia
Estrogen, glucocorticoids and vitamin D
C Short and rapid acting insulins are ALL INDIA 2006
Lispro, aspart and glulisine (Mn: Asli Glu)
C True about octreotides are C When a drug is evaluated for its usefulness in
controlled condition, it is called
Suppresses growth hormone secretion, useful for
variceal bleeding and useful in secretory diarrhoea Efficacy
(Mn: SOMA–Secretory diarrhea, oesophageal C Misoprostol is a
variceal bleeding, malignancy and acromegaly) PGE1 analogue
C Antiepileptic agents are C Enzyme inhibited by aminophylline
Phenytoin, topiramate and carbamazepine. Phosphodiesterase
C Antiepileptic drugs acting via Na+ channels are C Bisphosphonates acts by
Phenytoin, valproate and lamotrigine. Decreasing the osteoclast mediated resorption of
C Nesiritide is bone
Brain natriuretic peptide analogue. C Drug causing constipation
C INR of a patient is Verapamil
Sensitivity index C Benzodiazepines of choice in elderly and those with
PT of patient liver disease
PT of control Lorazepam, oxazepam and temazepam
C Pro drug is C True regarding benzodiazepines
Ticlopidine and clopidogrel Have active metabolites, decreases natural gastric
C True about heparin are secretion in human being and extensively meta-
Causes alopecia, nonteratogenic and releases bolized by cytochrome P450 enzyme 3.
lipoprotein lipase. C Ethosuximide is used in the treatment of
C GP IIb/IIIa antagonists are Absence seizure
Eptifibatide, tirofiban , and abciximab C Combination of clonazepam with valproic acid
C Orange coloured urine is due to precipitates
Absence status
Rifampicin
C Nevirapine belongs to C Drugs interfering folic acid metabolism
NNRTI Phenytoin, phenobarbitone and primidone
C Drug with maximum propensity to cause peripheral C Common side effect of fluoxetine
neuropathy is Loose stools
Stavudine C Side effects of clozapine
C True about immunosuppressants are Granulocytopenia, seizures and sedation
Tacrolimus inhibits calcineurin pathway, steroids C The most common side effect of haloperidol is
bind to cytosolic receptors and heat shock proteins Akathisia
and sirolimus will block kinase in the IL1 receptor C Antibacterial agents acting by inhibiting cell wall
pathway. synthesis
C Chlorambucil is an Carbapenems, monobactums and cephamycin
Alkylating agent C Drugs contraindicated in patients with allergy to
C Drug acting by hypomethylation is sulphonamides
Decitabine Brinzolamide
44 AIIMS and All India PGMEE—Review Questions

C Therapeutic uses of penicillin G C The major difference between typical and atypical
Bacterial meningitis, syphilis and anthrax antipsychotics is that
C Antipseudomonal penicillin Typical antipsychotic drugs cause tardive dyski-
Piperacillin nesia
C A diabetic patient developed cellulitis due to C Antipsychotic drug induced parkinsonism is
MRSA. Drugs appropriate are treated by
Vancomycin, teicoplanin and linezolid Anticholinergics
C True about mycophenolate mofetil C Oculogyric crisis is known to be produced by
It is a prodrug, it is a selective, uncompetitive and Trifluoperazine, prochlorperazine and perphenazine
Reversible inhibitor and selectively inhibits C Inverse agonist of benzodiazepine
lymphocyte proliferation (Mn: SURI). Beta-carboline
C Protease inhibitors are C Hallucinogens are
Nelfinavir, saquinavir and ritonavir LSD, phenylcyclidine and mescaline
C Nevirapine is a C Topically used sulphonamides are
Non-nucleoside reverse transcriptase inhibitor Sulphacetamide, silver sulphadizaine and mafe-
C Pre-treatment evaluation for lithium therapy includes nide.
Serum creatinine C Penicillinase susceptible are
C Treatment of lithium toxicity includes Amoxicillin, piperacillin and penicillin G
Treating dehydration, ingestion of polystyrene C Antibiotic with anti-inflammatory and immuno-
sulfonate and hemodialysis. modulatory activities is
Macrolides
ALL INDIA 2005 C Best associated with lumefantrine
Antimalarial
C A highly ionized drug
C Drug therapy of toxoplasmosis
Is excreted mainly by kidney
Pyrimethamine
C Hormones with cell surface receptors
C Topoisomerase inhibitor is
Adrenaline, growth hormone and insulin
Irinotecan
True of adrenal suppression due to steroid therapy.
Etanercept acts
C
C
It should be expected in anyone receiving
By blocking TNF
> 5 mg, prednisone daily
C Theophylline has an antagonistic interaction with
Adenosine receptors ALL INDIA 2004
C Drug increasing gastrointestinal motility C Ethical clearance is required in
Neostigmine Phase I, II and III of clinical trial of drugs
C Drug protecting from gastric aspiration syndrome C Sympathomimetic drugs are useful in
in a patient with symptom of flux Acutely decompensated heart failure, hypotension
Metoclopramide and hypertension.
C Drugs with significant drug interaction with C True about biguanides
digoxin are Don’t stimulate insulin release, decrease hepatic
Cholestyramine, thiazide diuretics and guanidine glucose production and can be combined with
C True about nesiritide sulfonylureas.
Brain natriuretic peptide analogue used in C True about an alpha glucosidase inhibitor
acutely decompensated heart failure and has Reduces intestinal absorption of carbohydrates,
short half life effective in both type of diabetes and can be used
C Dry mouth during antidepressant therapy is caused with other oral hypoglycemic drugs.
by blockade of C Anticancer drug causing bone marrow depression
Muscarinic acetylcholine receptor Chlorambucil, daunorubicin and doxorubicin
Pharmacology 45

C Advantages of using Raloxifen over estrogen in C Single most active anticancer drug for leiomyo-
post-menopausal women sarcoma is
Reduces fracture rates, avoids endometrial Adriamycin (Doxorubicin)
hyperplasia and no increase in incidence of breast C Clinically significant drug interaction occurs
carcinoma. between pyridoxine and
C Drugs binding to GABA receptor chloride channel Isoniazide, levodopa, hydralazine
Ethanol, Zolpidem and alphaxolone C True about tacrolimus
C A 60 years old male patient of rheumatic mitral It is a macrolide antibiotic indicated for prophylaxis
stenosis with artial fibrillation is on treatment for fast of organ transplant rejection and glucose intolerance
ventricular rate. He develop a pulse rate of 64/ min, is a well recognized side effect.
regular. Drug given was C BAL used for treating
Digoxin Lead, arsenic and organic mercury poisoning
C Side effects of chronic amiadarone therapy (Note: Not used in Iron and Cadmium poisoning)
Pulmonary fibrosis, hyperthyroidism and hypo-
thyroidism.
C Ranitidine and sucralfate are not prescribed together ALL INDIA 2003
because C True regarding bioavailability of a drug
Sucralfate inhibits absorption of ranitidine Fraction of unchanged drug reaching the systemic
C True about interactions of levodopa circulation, bioavailability can be determined from
It is a prodrug, pyridoxine reduces effects of plasma concentration or urinary excretion data and
levodopa in parkinsonism and phenothiazines bioavailability of oral drug is calculated by
reduce efficacy of levodopa in parkinsonism. comparing the area under curve (0-alpha) after oral
C QT prolongation in a premature baby by and intravenous administration.
Cisapride C Food interferes drug absorption by slowing gastric
C Action ascribed to delta type of opioid receptors emptying and/or by altering degree of ionization of
Supraspinal analgesia drug in stomach. True are
C Morphine can be used in Digoxin absorption delayed by food, food severely
Asthma, hypothyroid and diabetes reduces phenytoin absorption and halofantrine is
more extensively absorbed with food.
C True regarding sulfonamides
Intravenous therapy for hypertensive emergencies
Crystalluria can occur with sulphonamide therapy,
C
are
sulfonamide administration to newborn may cause
kernicterus and sulfonamide can be used in Enalapril, urapidil and fenoldopam
nocardia infection. C CNS/CVS toxicity may result when standard lido-
C Mutually antagonistic antibiotics when used in caine doses are given in patients with circulatory
combination are failure due to
Ampicillin and chloramphenicol Lignocaine concentrations are initially higher in
relatively well perfused tissues like cardic and brain
C Fluoroquninolone not requiring dose adjustment in
tissue.
a patient with creatine clearance of < 50 mg/min
Hemorrhage secondary to heparin use corrected
Trovafloxacin
C
by
C Antipseudomonal cephalosporin
Cefoperazone Whole bloods
C Antitubercular drug not used in pregnancy C Hormone present intracellularly in muscle cells
Streptomycin Corticosteroids
C Patient suffering from multidrug resistant C Hirsutism, gum hyperplasia and granulocytopenia
tuberculosis can be treated with are side effects of
Amikacin, clarithromycin and ciprofloxacin phenytoin
C Hormonal drugs acting against breast cancer C Drugs acting on cell membrane
Letrazole, exemestane and tamoxifen Nystatin, amphotericin B and polymyxin B
46 AIIMS and All India PGMEE—Review Questions

C Bacitracin acts on C Barbiturate is contraindicated in


Cell wall Acute intemittent prophyria
C Most effective drug against Mycobacterium leprae C Mechanism of action of tianeptin
is Selective serotonin reuptake enhancer
Rifampicin C PPI are effective when they are given
C Chelation therapy in beta-thalassemia major shortly before meals
Better drug is oral deferiprone C Correctly matched pair
C Endothelin receptor antagonist is Penicillamine-copper
Bostentan C Digoxin is contraindicated in
C Extent of ionization of a drug depends upon pKa of HOCM
the drug and pKa of the solution drug is dissolved C Drug causing renal toxicity
in. True are Cephalosporins, gentamycin and amphotericin B
pKa of a drug is the pH at which the drug is 50% C True about losartan
iodized, knowledge of pKa of a drug is useful in Competitive angiotensin receptor antagonist,
predicting its behaviour in various body fluids and negligible cough and has long acting metabolite.
phenobarbitone (pKa 7.2) is largely ionized at acid C Gemcitabine is used in
pH and will be about 40% nonionized in plasma.
Pancreatic cancer
C Drugs crossing placenta
ALL INDIA 2002 Phenytoin, diazepam and morphine
C Entropy in a biological system does not increase C Truck driver with profuse rhinorrhoea and sneezing.
because Drug given
It is an open system Cetirzine
C True about a system favouring oscillatory respon- C Mechanism of action of sodium nitroprusside is
ses Increased guanylate cyclase
Positive feedback system C Steroid receptor super family includes
C True about potency and efficacy of a drug Thyroid receptor, retinoid receptor and vitamin D3
In a clinical setup, efficacy is more important than receptor.
potency, in log dose response curve, height of the C Drugs undergoing hepatic metabolism
curve indicates efficacy and drugs that produce a Phenytoin, diazepam and cimetidine.
similar pharmacological effect can have different C In a woman on OCP, chance of pregnancy increases
levels of efficacy. after simultaneous use of drugs
C Selective beta-blockers are Ampicillin, griseofulvin and phenytoin
Atenolol, bisprolol and esmolol C The primary mechanism of action of fluoride on
C Aminoglycoside renal toxicity is increased in topical application is
Elderly person, hypokalemia and aminoglycoside Conversion of hydroxyapatite to fluoroapatite by
administration in the recent past replacing the OH-ions.
Microbiology 47

Microbiology

AIIMS NOVEMBER 2015 C Adult worm of which of the following parasites


may cause lymphatic filariasis
C Most cost effective way to prevent infection Oncocerca volvulus, Brugia malayi and Wuchereria
Proper hand washing bancrofti.
C 30 years old male farmer presents with multiple C A child is suffering from Haemophilus influenzae
discharging cervical sinuses. Stain to diagnose encephalitis. He was given cefotaxime in place of
Acid fastness penicillin as resistance to penicillin may have
C Antibody dependent enhancement is observed in been developed. Mechanism of resistance most
Dengue hemorrhagic fever likely is
C Chromosomal transfer through bacteriophage in β lactamase property acquired through plasmid.
corynebacterium diphtheria to make it virulent by
the process AIIMS NOVEMBER 2014
Transduction C A 26 years old sexually active woman presents with
C Patient developing gastroenteritis after eating purulent vaginal discharge and dysuria for 7 days
seafish/shellfish by virus after an unprotected sexual intercourse with a new
Noro virus/Norwalk virus partner. Investigation most sensitive to diagnose
C Stain used for actinomycetes the condition
Acid fast stain Polymerase chain reaction for nucleic acids.
(Note: Chlamydial infection)
AIIMS MAY 2015 C Microbe with pathogenic strains termed entero-
C Reverse transcriptase is present in aggregative, entero-toxigenic, entero-hemorrhagic
Hepatitis B virus and entero-invasive
(Note: Pararetrovirus) Escherichia coli
C Tropical spastic paraparesis is caused by C Sputum of a patient with cystic fibrosis grew
HTLV Pseudomonas aeruginosa that produced mucoid
C A patient presents with vesicles on the shin. Tzank colonies. It implies
smear microscopy shows giant cells. Causative The Pseudomonas aeruginosa has produced a biofilm
agent is in the patient’s airway.
Varicella zoster C Non-typable Haemophilus influenzae can cause
C HPV cause Exacerbation of COPD, otitis media and puerperal
Base of tongue carcinoma, tonsillar carcinoma and sepsis.
carcinoma cervix C Virus neutralizing antibodies
(Note: Nasopharyngeal carcinoma is caused by Are directed against viral protein determinants on
Epstein Barr virus) the virus particle.

47
48 AIIMS and All India PGMEE—Review Questions

C A 70 years old woman refused to take influenza surrounded by a clear halo. The CD4 count was less
vaccine and later developed influenza and died than 100/mm 3 before demise. The most likely
after 1 week due to pneumonia. The most diagnosis
common cause of acute post-influenza pneumo- CMV pneumonia
nia C Giemsa stained smear of epithelial cells obtained
Staphylococcus aureus from a newborn with hepatosplenomegaly shows
C The avian influenza has the potential to cause intranuclear inclusion surrounded by a clear halo.
human infections with high mortality. Probable Most likely cause of congenital infection
reason for not causing any pandemics CMV
No human to human transmission C An elderly male patient presents with nephropathy,
C True about DHF/DSS 2 months after renal transplantation. Viral etiologi-
It is caused by flavivirus of the arthropod borne cal agent
virus group, Aedes aegypti is the vector and Polyoma virus BK
malnutrition may exert protective effect. C The causative organism for AIDS was identified in
C True about Entamoeba histolytica infection the year
Most infections are asymptomatic, amoebic liver 1983
abscess is more common in man than woman and
C The term recrudescence in malaria refers to
extra-intestinal involvement may include brain,
skin and lungs. Recurrence of sexual parasitemia after completion
of treatment.
C A 15 years old girl returned to her village from
another village after a short stay and complained of
severe headache and fever and was diagnosed as a AIIMS NOVEMBER 2013
case of pyogenic meningitis and admitted but died
C The role played by MHC 1 and MHC 2 is
5 days later. Organism that can be considered in the
diagnosis Present antigens for recognition by T cell antigen
receptors.
Naegleria fowleri.
C Neonatal sepsis is caused by
Group B Streptococci, E. coli and Staphylococcus.
AIIMS MAY 2014
C True regarding staining of microbes AIIMS MAY 2013
Gram-negative bacteria stain red C Rapidly growing atypical Mycobacteria causing
C Both DNA and RNA are present in lung infections
Bacteria M. chelonie, M. fortuitum and M. abscessus
C True about IgE antibodies C A young lady presents with sore throat, fever and
It mediates release of histamine and other chemical headache for 3 days. On examination—severe dehy-
mediators, it is the primary antibody involved in dration, BP—90/50 mm Hg and small red spots on
allergic reaction and it is involved in anti-parasitic the distal aspect of the cuff. Most probable etiology
immune responses. Neisseria meningitides
C A 10 years old child presented with severe sore C A patient with conjunctival infection leading to
throat and a gray pseudomembrane over pharynx corneal perforation, on Gram stain was positive for
and tonsils. The organism causing this infection is Gram-negative coccoid appearance with further
most likely investigation revealing small translucent colorless
A Gram-positive bacillus organism that is oxidase positive. Most probable
(Note: Corynebacterium diphtheriae) causative organism
C The H and E staining of a lung section during Neisseria gonorrhea
autopsy of a patient suffering from AIDS shows C The most sensitive and rapid test for diagnosis of
desquamation of type 1 pneumocytes with CMV retinitis is
prominent intranuclear basophilic inclusion bodies Nucleic acid detection from the intraocular fluid
Microbiology 49

C A method for acquiring infection by Legionella C An abattoir worker presented with a pustule on his
pneumophila hand progressed to ulcer. Smear stain to determine
Consumption of contaminated water the causative agent of the ulcer
C A young male patient presents with loose motion Polychrome methylene blue
and intermittent abdominal pain over the past (Note: Cutaneous anthrax)
1 year. Wet mount stool specimen showing multiple C True about antibiotic resistance
ova > 100 μ in diameter. Causative organisms may be The most common mechanism is production of
Fasciola gigantica, Gastrodiscoides hominis and neutralizing enzymes by bacteria, complete
Echinostima ileocanum. elimination of target is the mechanism by which
enterococci develop resistance to vancomycin and
AIIMS NOVEMBER 2012 alteration of target lesions leads to development of
resistance to antibiotics in Streptococcus pneumonia.
C A farmer from the sub-Himalayan region presents
C Sputum of a pneumonia patient sent for culture
with multiple leg ulcers. The most likely causative
showed Gram-positive cocci in chains and alpha
agent is
hemolytic colonies on sheep agar. Test for confirming
Sporothrix schenkii
the diagnosis
C The endotoxin of following Gram-negative bacteria
Optochin sensitivity
plays a part in the pathogenesis of the natural
A young male patient presented with urethral
disease
C
discharge, pus cells but no organisms on urine
E. coli, Klebsiella and Pseudomonas.
examination. Method best for the culture
C Which of the following features are shared between
McCoy cell line
T cells and B cells
(Note: Chlamydia)
Class 1 MHC expression and antigen specific
receptors. C Synthesis of an immunoglobulin in membrane
bound or secretory form is determined by
C True about Bacteroides fragilis
Differential RNA processing
It is the most frequent anaerobe isolated from
clinical samples, it is not uniformly sensitive to
AIIMS NOVEMBER 2011
metronidazole and lipopolysaccharide formed by it
is structurally and functionally different from the C True about antibiotic resistance
conventional endotoxin. Most common mechanism of resistance is produc-
C True about prions tion of neutralizing substances (enzymes), alteration
Long incubation period of target lesion leads to pneumococcal resistance
C An abattoir worker presented with a malignant and complete elimination of target is cause of
pustule on his hand that progressed to form an resistance to vancomycin.
ulcer. Smear was taken and sample sent for the lab.
Diagnosis AIIMS MAY 2011
Cutaneous anthrax C True about Brucella are
C Aerosol spread leading to epidemics is seen in B. abortus is capnophilic, transmission by aerosol can
infection with occur occasionally and pasteurization destroys it.
Infuenza C True about non-typhoid Salmonella
Poultry is source, can cause invasive diseases in neo-
nates and resistance to fluoroquinolones has emerged.
AIIMS MAY 2012
C Lambda phage true is
C Sputum can be disinfected by Lysogenic phage incorporates in host DNA and
Autoclaving, boiling and cresol remains dormant.
C In a 5 years old male child with history of pyogenic C Streptococcus true are
infections by bacteria with polysaccharide rich Streptodornase cleaves DNA, streptolysin O active
capsules, investigation done is in reduced state and streptokinase is produced
IgG2 deficiency from serotype A, C and K.
50 AIIMS and All India PGMEE—Review Questions

C Lymes disease true are AIIMS MAY 2010


Borrelia burgdorferi replicates locally and invades
locally, infection progresses in spite of good C Wright Giemsa’s stain of a male patient presented
humoral immunity and IgA intrathecally confirms with granulomatosus penile ulcer shows tiny
meningitis. organisms of 2 μ within macrophages
C True about bacteriophage Causative agent is Calymmatobacterium granulomatis.
It imparts toxigenicity to bacteria C Diagnostic of rabies
C True regarding disinfectants Negri bodies
Glutaraldehyde is sporicidal, hypochlorites are C Superantigens, true is
virucidal and ethylene oxide is intermediate Directly attached to lateral aspect of TCR beta region
disinfectant. C Used in diagnosis of leptospirosis
C A cellular pertussis vaccine contains Microscopic agglutination test, macroscopic
Flagillary hemagglutinin, fimbriae and pertussis toxin. agglutination test and dark filled illumination.
C HIV patient presenting with malabsorption,
chronic diarrhea, with acid fast organism. Causative
AIIMS NOVEMBER 2010
organism is
C A boy with skin ulcer on leg, culture reveals beta- Isospora
hemolysis, cultures from school children with sore C Most sensitive test for Treponema pallidum is
throat some days back also revealed beta- FTA-ABS
hemolysis. Similarity between both is
C Maltes cross seen on polarizing microscopy in
C Carbohydrate antigen is same
Cryptococcus neoformans (more important in Babesia
C Amoebiasis is transmitted by microti if given in option).
Cockroach, feco oral and oro rectal C Irradiation used to sterilize are
C True about ETEC Bone graft, suture and artificial tissue graft
Common cause of acute watery diarrhea in children C Malabsorption caused by
in developing countries.
Giardia, Strongyloides and Capillaria phillipensis
C Characteristic of Bacillus cereus food poisoning is
C Antigen–antibody precipitation reaction maximum
Presence of pain in abdomen. in Zone
C In plasma sterilization control used is Equivalence of antibody and antigen. (Mn: PROBE—
Bacillus stearothermophilus Prozone is Antibody excess)
C What happens if a bacteria acquires genes coding to
restrict endonuclease
Bacteria will die as it lacks methylase AIIMS NOVEMBER 2009
C Reassortment is seen in C Most sensitive investigation for diagnosis of
Rotavirus asymptomatic Chlamydia infection is
C Malignant pustule is seen in Nucleic acid amplification.
Cutaneous anthrax C Infective form for mosquito in Plasmodium falciparum is
C True about Haemophilus influenza is Gametocytes
It needs factor X and V for growth in culture media. C True about Neisseria gonorrhea is
Its rare cause of meningitis in first-two months of Exclusive human pathogen, some strains may cause
life and most common invasive manifestation is disseminated disease and acute urethritis is the
meningitis. most common manifestation in males.
C True about Clostridium perfringens C Correct regarding widal test
Food poisoning causing strains of C. perfringens Baseline titres differ depending on the endemicity
produce heat resistant spores, show positive of the disease, high titers value in a single widal test
Naegler’s reaction and C. perfringens is the most is not confirmatory and H antibody cannot
common cause of gas gangrene. differentiate between types.
Microbiology 51

C A 25 years old man with 3 weeks fever presented with C Congenital toxoplasmosis, true are
tricuspid valve vegetation. Most common cause is IgA is better than IgM in detection, diagnosed by
Staphylococcus aureus. detection of IgM in cord blood.
C A young child presenting with intermittent
abdominal cramps, loose stools and ova of size AIIMS NOVEMBER 2008
100 μm on stool examination causes are C Bio safety precaution grade III is practiced in
Fasciola gigantica, Echinostorum ileocaxnum and St. Louis encephalitis virus, Coxiella burnetii and
Gastrodiscoides hominis. (Mn: FEG) Mycobacterium tuberculosis. (Mn: MSC)
C An adult labourer presented with penile ulcer C Most common species of Pseudomonas causing
3 years back not treated. Then he presented with intravascular catheter related infections is
neurological symptom for which he got treated. P. aeruginosa
Test to monitor treatment response is C Most sensitive diagnostic test for dengue is
VDRL Neutralization test
(Note: Neurosyphilis) C A gardener has multiple vesicles on hand and
C Stain used for fungal elements multiple eruptions along the lymphatics. Most
Methanamine silver common fungus responsible is
C Recent Nobel prize for Sporothrix schenckii
RNA viruses are
iRNA
C
Ebola, rabies and vesicular stomatitis virus
C Regarding prior protein true is
AIIMS MAY 2009 It catalyzes abnormal folding of other proteins
C Dimorphic fungi are C True about bacteriophage
Penicillium marneffi, Histoplasma and Blastomyces It is a virus that infects bacteria, it helps in
dermatidis. transduction of bacteria and it imparts toxigenicity
to bacteria.
C Malabsorption caused by
C A girl from Shimla with fever, hypotension, malaise
Giardiasis, Strongyloides and Capillaria phillipen- and axillary and inguinal lymphadenopathy.
sis. Culture in glucose broth shows stalactite growth.
C Gas gangrene is caused by Most likely organism is
Cl. histolyticum, Cl. novyi and Cl. septicum. Yersinia pestis
C Culture media of leptospirosis C True about carbohydrate antigens are
Korthof It has lower immunogenicity, cause polyclonal
C Intermediate host for hydatid disease B cell stimulation, does not require stimulation by
T cells.
Man
C Pertussis toxin acts by mechanisms
C A 25 years old male presented with diarrhea for
ADP ribosylation of proteins associated with
6 months. On examination the causative agent was
receptors, increase cAMP, and acts thorough
found to be acid fast with 12 micrometre diameter.
G alpha submit.
Agent is
C True about isotypic variation
Cyclospora
Changes in heavy and light chain in constant region
C A man on return from a country complains of pain is responsible for class and subclass of immuno-
in abdomen, jaundice, with increased alkaline globulins.
phosphatase and conjugated hyperbilirubinemia. C True about hybridoma technology are
USG shows blockage in billiary tree. Cause is
Specific antibody producing cells are integrated
Clonorchis sinensis with myeloma cells, aminopterin, a folate antago-
C Highest risk of nosocomial infection to a patient is nist inhibits de-novo pathway and HGPRT ase and
in thymidylate synthetase are required for salvage
Patient admitted for elective surgery. pathway.
52 AIIMS and All India PGMEE—Review Questions

AIIMS MAY 2008 C Lancefield classification depends on


Carbohydrate antigen
C Nocardia is stained by
C True about Corynebacterium diphtheriae
Acid fast stain, alcian blue and mucin stain
Has metachromatic granules, does not invade
Selective media for vibrio is
deeper tissue and toxigenecity demonstrated by
C
TCBS Elek’s test.
C Mark true in the following C Positive Schick test indicates
Hanta virus pulmonary syndrome is caused by
Susceptible to diphtheria
inhalation of rodent urine and faeces.
C True about respiratory viruses
C True about mycoplasma are
RSV-common cause of bronchiolitis in infants,
Sterol enhances growth, can grow in cell free media
SSPE is a late complication of measles and para
and when grown in liquid medium do not produce
influenza attaches to cell by haemagglutinin.
turbidity.
C True about HUS are
Not commonly caused by VTEC in Asia, recurrence AIIMS MAY 2007
rare and transient thrombocytopenia. C Only hepatitis virus which can be cultured is
C True about MOTT HAV
Causes decreased efficacy of BCG due to cross C Most common cause of Traveller’s diarrhoea
immunity. ETEC
C Malta fever is caused by C A child with pustural lesion on leg. On Gram
Brucella melitensis staining, Gram-positive cocci are seen. Diagnosis of
C Acute primary amoebic meningoencephalitis, true group A streptococcal erythroderma can be
is established by
Diagnosis is by demonstration of trophozoites in Bacitracin sensitivity
CSF C Pseudomembranous colitis, true are
C Parvovirus B19 causes Toxin A is responsible for clinical manifestation,
Aplastic anemia in sickle cell disease and fetal Toxin B is responsible for clinical manifestation and
hydrops. summit lesion in early histopathological finding.
C True about histoplasmosis
In early stages it is indistinguishable from TB AIIMS NOVEMBER 2006
C True about polio
Increased muscular activity leads to increased C Most suitable dye for showing fungus in biopsy is
paralysis PAS
C Part of HACEK
Haemophilus aphrophilus, Cardiobacterium hominis, AIIMS MAY 2006
Kingella kingae and Eikenella corrodens C Cholera toxin
(Note: also Actinobacillus Actinomyces comitans) Causes continued activation of adenylate cyclase.
C Anaerobic: aerobic bacteria ratio in GIT
AIIMS NOVEMBER 2007 1000:1
C Most suitable dye for showing fungus in biopsy is
A 25 years old sexually active male with burning
PAS
C
micturition. On clinical examination no ulcer in the
genital. Urine examination shows 50 WBC/HPF, no
RBC, leukocyte esterase positive, gonococci culture AIIMS NOVEMBER 2005
negative. Most likely causative agent is C A patient of acute lymphocytic leukemia with fever
Chlamydia trachomatis and neutropenia develops diarrhea after given
C Scrub typhus vector amoxicillin. Most likely organism responsible
Trombiculid mite Clostridium difficile
Microbiology 53

C True about mycoplasma C Cell type lacking HLA antigen is


Smallest prokaryotic organisms that can grow in Red blood cell
cell free culture media, lack a cell wall and resistant C True about mycoplasma
of beta lactam drugs. Can reproduce in cell free media, can pass through
C Yeast like morphology is found in Gram stain of 450 nm pore filters and have an affinity for
lung aspirate in HIV infected individual. Organism mammalian cell membranes.
possible are C Intra species competition is competition among
Candida tropicalis, Cryptococcus neoformans and individuals of a population.
Penicillium marneffei. C Recurrent facial/oropharyngeal/laryngeal edema
C Cervicitis and mucopurulent cervical discharge in a patient with low C4, normal C3 and normal
was found on examination in a 40 years old woman factor B. Diagnosis
presenting with lower abdominal pain and vaginal C1 esterase inhibitors deficiency
discharge. Organism best isolated by C Reduvid bug transmits
Culture on McCoy cells Chagas’ disease
C Pathogenesis of Listeriosis C Rocky Mountain spotted fever is caused by
Survival and multiplication of Listeria within Rickettsia rickettsii
mononuclear phagocytes and host epithelial cells. C Labelling of antibodies is done in
C Method of isolation of Chlamydia from clinical ELISA, radioimmunoassay and immunofluorescence
specimens C True regarding Chlamydia
Tissue culture using irradiated BHK cells, tissue Erythromycin is effective for therapy of chlamydial
culture using irradiated McCoy cells and yolk sac infections, cell wall lacks peptidoglycan, obligate
inoculation. intracellular bacteria.
C Sputum specimen of a patient cultured on 5% sheep C Hyperacute rejection of renal transplant
blood agar shown alpha hemolytic colonies. Other Type III hypersensitivity reaction
findings are
Gram-positive cocci in pairs, catalase negative and
AIIMS NOVEMBER 2004
bile soluble.
C Adult farmer presents with high grade fever and C True about chronic granulomatous disease
altered sensorium comatosed, conjunctival AD, abnormal bacterial phagocytosis and recurrent
hemorrhage and Hb 11g/dl, bilirubin 7 mg/dl and streptococcal infections.
urea 80 mg/dl. Malarial parasite negative. Most C Antigenic variation as a means of evading host
likely diagnosis defences used by
Weil’s disease Borrelia recurrentis
AIIMS MAY 2005 C Warm cycle of ethylene oxide sterilization
49°–59°C
Motile unicellular microorganisms were found on
Young boy with fever, icterus, conjunctival
C
C
wet mount microcopy of CSF of a 30 years old
suffusion and hematuria for 3 weeks. Diagnostic
patients of acute meningoencephalitis. It is
serological test used in this case
Naegleria fowleri
Microscopic agglutination test
C Late onset perinatal infection causing meningitis in
3 weeks old infant with CSF positive for Gram- C Bacteria requiring synergistic activity of penicillin
positive bacilli. Further differentiated by and amino glycoside
Motility at 25°C Enterococcus faecalis
C Part of innate immunity are C Campylobacter jejuni can be isolated by culture of
Complement, NK cells and macrophages stool of diarrhea patient by the method
C Autoimmunity can be caused due to Culture on skirrow’s medium, incubated at 42°C
Expression of cryptic antigens, inappropriate under microaerophilic conditions.
expression of MHC proteins and the pressure of C Throat swab was collected in a child immunized
forbidden clones. against diphtheria and showed Corynebacterium
54 AIIMS and All India PGMEE—Review Questions

diphtheria like organisms on Albert staining. Further C Sulfur granule of actinomycosis contains
processing reveals Organisms
It can grow on potassium tellurite medium. C Conjugate vaccine are available against
C The most suitable clinical sample for ante mortem H. influenzae, S. pneumoniae and N. meningitidis
diagnosis of rabies (group C).
Corneal smear for immunofluorescence stain. C Best method for selective isolation of Nocardia
C Diagnosis of Plasmodium falciparum malaria can be paraffin bait technique
confirmed by detection of antigen C Methods of antitubercular drug susceptibility are
Histidine rich protein II Molecular method, resistance ratio method and
C Urethritis in a 25 years old male patient can be radiometric broth method.
caused by
Neisseria gonorrhoea, Chlamydia trachomatis and
AIIMS NOVEMBER 2003
Trichomonas vaginalis.
C True about Legionella C True about H. influenzae
Can be grown on complex media, not effectively Part of normal flora in some patients, requires
killed by polymorphonuclear leukocytes and haemin and NAD for growth in culture media and
Legionella pneumophila sero group 1 is the most Type b is responsible for invasive disease.
common sero group isolated from human. C Fungal culture of skin scraping of lesion of a patient
C True about Staphylococci showed slow growing colony, which produced a
A majority of infection caused by coagulase few small microconidia. It is a dermatophyte of genera
negative staphylococci and Staph. epidermidis, beta- Trichophyton
lactamase production is under plasmid control and C Most common nosocomial infections are
methicillin resistance is independent of beta- Staphylococcus aureus, P. aerogenes and enterobacteriaceae.
lactamase production. C Bilateral diffuse interstitial pneumonia, 2 months
C The transport media of choice for streptococcal after kidney transplant in a 40 years old man is most
pharyngitis likely caused by
Pike’s media Cytomegalovirus
C Epstein-Barr virus C Irregular branching septate hyphae in Gram smear
Nasopharyngeal carcinoma of the orbital exudates of elderly diabetic patient
C Bilateral fluffy infiltrates on chest X-ray in a patient with left-sided orbital cellulitis and left maxillary
of acute leukemia presenting with febrile neutro- sinusitis on paranasal sinuses CT scan is most
penia. Drug of choice probably
Cotrimoxazole should be added Aspergillus
C Patient from Shimla presents with a series of ulcers
in a row on his leg and biopsy was cultured on
AIIMS MAY 2004 Sabouraud’s dextrose agar. Causative organism
C The protection from smallpox due to previous Sporothrix schenckii
infection with cowpox can be explained by C Gram-negative bacilli with bipolar staining on
Antigenic cross reactivity direct microscopy and rough corrugated grey-
C Class of immunoglobulins is determined by white colonies on blood agar in culture of pus
H-chain aspired from subcutaneous nodules on legs and
C Hepatitis C virus is forearms of a 50 years old alcoholic farmer
Flavivirus presenting with fever, chills and rigor and bilateral
C Painless indurated ulcer on the glans exuding clear crepts and scattered rhonchi. Diagnosis is
serum on pressure and painless inguinal lympha- Meliodosis
denopathy in both groins two weeks after unpro- C True statements are
tected sex in 20 years old male can be diagnosed by Ingested erythrocytes are seen only in Entamoeba
Dark field microscopy of ulcer discharge. histolytica, invasive amoebiasis most commonly
Microbiology 55

affects young adult male of low socio-economic CSF finding of polymorphs 2000/μl, protein
status and electrophoretic study of zymogenes 100 mg/dl, and glucose 10 mg/dl. The possible agent is
differentiates pathogenic and non-pathogenic Haemophilus influenzae
strains of Entamoeba histolytica. C True for Mycoplasma
C Phenomenon responsible for antibiotic resistance in Can grow in cell free media, require sterol for their
bacteria producing slime growth and multiply by binary fission.
Biofilm formation C A man presenting with urethritis and urethral
C Thermally dimorphic fungi are discharge with Gram stain showing numerous pus
Blastomyces dermatitidis, Histoplasma capsulatum and cells but no microbes and also culture negative. The
Sporothrix schenckii most likely responsible organism is
Chlamydia trachomatis
C Most common organism causing cellulitis is
AIIMS MAY 2003
Streptococcus pyogenes
C Virulence factor causing streptococcal toxic shock C Chlamydia trachomatis serovars D-K cause
syndrome Urethritis
Pyrogenic exotoxin C 8 out of 10 develops gastroenteritis within 4–6 hrs of
C Virulence factors of Gonococcus are eating sandwiches. Cook may be the carrier of
IgA protease, outer membrane protein and pilli Staphylococcus aureus
C Malignant pustule is a term used for C Diagnostic test for chronic carrier state in a cook
Anthrax of the skin with history of enteric fever 2 years ago
C True about botulism Vi agglutination test
Botulism is caused by ingestion of preformed toxin,
gene for botulinum toxin encoded by bacteriophage AIIMS MAY 2002
and Clostridium barati may cause botulism. C Antibiotics resistant multiple discharging sinuses
C Pus cells and red cells on stool examination of a patient in the legs of a farmer is
presenting with pain abdomen and mild fever Madurella
followed by gastroenteritis. Organism responsible is C True about Vibrio cholerae is
EIEC Pathogenicity of 0–139 vibrio is due to 0–139 antigens.
C Enrichment medium of choice for stool samples in C Staphylococcus auerus causes vomiting in 16–18 hours.
an outbreak of food borne Salmonella gastroenteritis is Cause is
Selenite F medium/tetrathionate broth. Vagal stimulation.
C Catalase positive and coagulase negative gram- C True about poliovirus
positive cocci on blood culture of a patient in ICU on Very difficult to eliminate type I, type I is responsible
CVP line-organism responsible is for most epidemics and type I is most commonly
Staphylococcus epidermidis associated with paralysis.
C DNA hepatitis virus C Foul smelling pus showing red fluorescence on UV
examination is aspirated from frontal abscess of a
Hepatitis B virus
patient. Most likely organism is
Bacteroides.
AIIMS NOVEMBER 2002 C Detection of Rota virus by
C Young child presenting with fever and cervical Antigen in stool
lymphadenopathy and showing grey membrane on C The most avidly complement fixing antibody is
the right tonsil extending to the anterior tonsil. Ideal IgM
medium for culture of throat swab C The granules discharged in mycetoma contains
Loeffler’s serum slope Fungal colonies
C Gram-negative coccobacilli showing growth of C Hepatitis B vaccination is given to a patient. His
bacteria only on chocolate agar causes fever, serum will reveal
vomiting and neck rigidity in a 2 years old child with Anti-HBsAg antibody
56 AIIMS and All India PGMEE—Review Questions

ALL INDIA 2012 C A 8 years old child with history of pain and
discharge from right ear presents with fever, neck
C A series of ulcers in lower extremities in sub- rigidity and positive Kernig’s sign. Staining reveals
Himalayan area is often caused by Gram-positive cocci. Most likely it is
Sporothrix schenckii Pneumococcus
C Vaccination is based on the principle of C Bacteria having polysaccharide capsule related
Immunological memory antigen–antibody responses are
C EB virus causes autoimmunity by Pneumococus, Meningococcus and H. influenzae.
Polyclonal B cell activation C ‘C’ in CRP stands for
C Cholera toxin stimulates secondary messenger Capsular polysaccharide in Pneumococcus.
cAMP C True about C. diphtheriae
C A characteristic infection of Nocardia asteroides Organism may be identified by tests for toxigenicity,
Brain abscess toxin acts by inhibiting protein synthesis and toxin
C RF in the synovial fluid of patients with RA is most may affect heart and nerves.
frequently found to be C True about Campylobacter jejuni
IgM reacting with IgG Commonest agent responsible for campylobacte-
C True about HUS riosis, paultry is a common source of infection, and
Infection may be transmitted by food, HUS is associated with GB syndrome.
caused by serotoxin producing E. coli and HUS is C Farmer with history of fever and flu-like symptoms,
more common in children. painful inguinal lymphadenopathy and ulcer in the
C Gram-negative bacteria with no role of endotoxin in leg. Stain used to detect suspended bipolar organism
pathogenesis of disease Wayson staining
Vibrio cholerae. C True regarding Clostridium tetani are
C True about interferon β release assays used for the Spores are resistant to heat, incubation period is
diagnosis of TB 6–10 days and person to person transmission does
Second generation quantiferon-TB (gold) used not occur.
ESAT 6 and CPF 10. C MRI scan of a girl with meningitis reveals ring
C Most resistant to actions of antiseptics and disinfec- enhancing lesion in frontotemporal region while
tants CSF shows gram-positive bacilli. Organism is
Prions Listeria monocytogenes
(Note: if cell is mentioned, answer is spore). C True regarding Chlamydia trachomatis
C Gene for bacterial resistance to antibiotics is located It is biphasic, reticulate body divides by binary
in part of the bacterial cell fission and inside the cell it evades phagolysosome.
Chromosome C True about parvovirus B19 is
C Person with AIDS related complex (ARC) is most Respiratory route is primary mode of transmission,
likely suffering from It is a DNA virus and affects erythroid progenitors.
Opportunistic infections C Most common fungal infection in immune competent
patient
ALL INDIA 2011 Candida
C A patient presents with signs of pneumonia. The C Aflatoxins are produced by
bacterium obtained from sputum was a Gram- Aspergillus flavus
positive cocci which showed alpha hemolysis on C A patient with history of persistent fever and
sheep agar. Test to confirm diagnosis is cough, X-ray suggestive of pneumonia, sputum
Bile solubility > Optochin sensitivity shows aerobic branching Gram-positive filaments
C True about Pneumococcus that are partially acid fast. Likely agent is
Capsule aids in virulence, commonest case of otitis Nocardia asteroides
media and respiratory tract of carriers is most C Visceral larva migrans is associated with
important source of infection. Toxocara canis
Microbiology 57

C Complement component that is common link in C Biopsy from multiple polyps with muscle thickening
classical and alternate pathway of complement and impacted secretions in nasal cavities of a young
system is woman complaining of recurrent rhinitis, nasal
C3 discharge and bilateral nasal blockage since one
C Most potent activators of naïve T cells are year shows hyphae with dichotomous branching
Mature dendritic cells typically at 45°.
C True about carbohydrate antigens are Organism is Aspergillus.
Poor immunogenicity, T cell independent immunity
and poyclonal response.
ALL INDIA 2009
C Ovalbumin was injected into a Rabbit. Class of
immunoglobulin likely to be produced initially C True regarding disinfectants
IgE Hypochlorites are bactericidal and inactivated by
organic matter, glutaraldehyde is sporicidal and
ALL INDIA 2010 inactivated by organic matter and formaldehyde is
C Peptide binding site on class I MHC molecules for bactericidal, spermicidal and virucidal.
presenting processed antigens to CD8 T cells is C Culture media used for diagnosis of EHEC 0157: H7 is
formed by Sorbitol MacConkey media
Distal domain of alpha subunit. C True about non-typhoid Salmonella
C True about penicillin binding proteins (PBP) are Transmission is most commonly associated with
PBPs are essential for cell wall synthesis, PBPs act as eggs, poultry and undercooked meat, common in
carboxypeptidase and transpeptidase, alteration in immunocompromised individuals and resistance
PBPs is the primary basis of resistance in MRSA. to fluoroquinolone.
C True regarding penicillin resistances are C True about leptospirosis
B-lactamase production is the most common mecha- Infection acquired by direct contact with infected
nism of resistance, alteration in permeability/ urine, mortality is 5–15% in severe cases, and IV
penetration of antibiotic causes resistance only in penicillin is recommended for treatment of severe
Gram negative bacteria and B lactmase production cases.
causes resistance in both Gram-positive and Gram- C True about transmission
negative bacteria. Legionella may be transmitted through water
C True about Staphylococcus is aerosols, Listeria may be transmitted through
About 30% of general population is healthy nasal refrigerated food and leptospirosis may by trans-
carriers, Epidermolysin and TSS toxin are super mitted through water contaminated with rat urine.
antigens and methicillin resistance is chromoso- C Species of Borrelia not associated with tick-borne
mally mediated. relapsing fever is
C A child presents with sepsis. Bacteria isolated showed Borrelia recurrentis
beta-hemolysis on blood agar, resistance to bacitracin
C Most common mode of transmission of Pasteurella
and a positive cAMP test. Most likely agent is
multocida is
S. agalactiae.
Animal bites or scratches
C True about El Tor Vibrios are
C Maltese cross is a characteristic feature of
Human are the only reservoir, can survive in ice
cold water for 2–4 weeks and killed by boiling for Babesia microti > Cryptococcus neoformans
30 seconds. C True about Chlamydia trachomatis are
C Isolation of Chlamydia from tissue specimen can be Genital chlamydial infections are often asympto-
done by matic, can be cultured and inclusion conjunctivitis
Yolk sac inoculation is caused by C. trachomatis serotypes D-K.
C Varicella zoster remains latent in C True about hepatitis C
Trigeminal ganglion Most common indication for liver transplantation
C Most common genital lesion in HIV patient is C Enteroviruses are associated with
Herpes Aseptic meningitis, pleruodynia and herpangina
58 AIIMS and All India PGMEE—Review Questions

C True about parvovirus B19 C Components of innate immunity that are active
DNA virus, can cause severe anemia and can cause against viral cells include
aplastic crisis. NK cells
C Dengue hemorrhagic fever is caused by C Culture of infective skin lesions of the leg of a child
Reinfection with a different serotype of dengue virus. shows Gram-positive cocci in chains and hemolytic
C Primary receptor for HIV is colonies. Test to identify the organism is
CD4 Bacitracin sensitivity
C Function of CD 4 helper cells are C Vancomycin resistant Gram-positive cocci in
Immunogenic memory, activate macrophages and chains with catalase negative organism causing a
activate cytotoxic cells. spike of fever in a patient in an ICU on central
venous line and on ceftazidime and amikacin for
ALL INDIA 2008
last 7 days is
Enterococcus faecalis
C Most resistant to antiseptics C True about Vibrio cholerae
Prion Nonhalophilic, grows on simple media, and man is
C True about Gram-positive cocci the only natural host.
Staphylococcus saprophyticus causes UTI in females, C Bacteria acting by increasing cAMP are
micrococci are oxidase positive and pneumococci
Vibrio cholerae
are capsulated.
C Component of Streptococcus pyogenes showing cross C Sputum culture showing growth on BCYE in an
reactivity to synovium of human tissues elderly male patient presenting with fever, chest
pain and dry cough. The causative agent is
Capsular hyaluronic acid
C True regarding Enterococcus Legionella
Common species are E. faecalis and E. faecium. It is a C A person working in an abattoir presented with a
cause for peritonitis and it is a cause for intra- papule on hand which turned into an ulcer. Stain
abdominal abscess. used for diagnosis is
C True about V. cholerae 0139 Trichrome methylene blue
Clinical manifestations are similar to Eltor strain, C Brucella is transmitted by
first discovered in Chennai and epidemiologically Through the placenta of animals, aerosol
indistinguishable from 01 Eltor strain. transmission and eating uncooked food.
C True about Pneumocystis jirovecii is C Botulinum affects
May be associated with pneumatocele, usually Neuromuscular junction, preganglionic junction
diagnosed by sputum examination and cause and postganglionic nerves.
disease only in the immunocompromised host. C True about Bacteroides fragilis
True about prions is
It is a frequent anaerobe isolated from clinical
C
They are infectious proteins samples, it is not uniformly sensitive to
C H1N1 may be best described as a metronidazole and LPS formed by Bacteroides
Bird flu virus fragilis is structurally and functionally different
C Parasites causing biliary obstructions are from conventional endotoxins.
Clonorchis, Ascaris and Fasciola C True about Chlamydia psittacosis are
ALL INDIA 2007
Acquired by bird’s droppings, causes pneumonia
and treatment is tetracycline.
C True statement is
Nutrient broth is a basal medium
ALL INDIA 2006
C Urine examination of a young male patient
presented with UTI shows pus cells but no C A veterinary doctor had PUO due to Gram-negative
organism. Method best used for culture is short bacilli oxidase positive. Likely causative agent
McCoy culture Brucella spp
Microbiology 59

C Smear staining done for a young boy with flea bite C Bacterial disease associated with rats, rice fields and
in a wheat grain godown and fever and axillary rainfall is
lymphadenopathy after 5 days Leptospirosis
Wayson staining C Correct incubatory conditions of the culture plates
(Note: Plague) of the stool sample for Campylobacter jejuni is
C True about Mycobacteria Temperature of 42°C and microaerophilic
M. Kansasii can cause a disease indistinguishable C True about meliodosis
from tuberculosis. Caused by Gram-negative aerobic bacteria, bipolar
C VDRL positive woman gave birth to an infant. Risk staining is seen with methylene blue stain, most
of transmission is determined by common form of meliodosis is pulmonary infection.
TPHA test on serum sample of mother, VDRL on C Organisms known to survive intracellularly
paired serum sample of infant and mother and time Neisseria meningitidis, Legionella pneumophila and
interval between treatment of the mother and Salmonella typhi.
delivery. C The most common organism causing acute
C A patient complained of chills and fever for 2 week meningitis in an AIDS patient is
after louse bite with rashes all over the body, Cryptococcus neoformans
delirious and went into coma. Vasculitis due to C The most common pathogen responsible for
Rickettsial infection by nosocomial pneumonias in the ICU are
Rickettsia prowazekii Gram-negative organisms.
C Associated with Epstein-Barr virus infection C Normal microbial flora present on the skin and
Infectious mononucleosis, nasopharyngeal mucous membranes
carcinoma and oral hairy leukoplakia. Endemic trachoma, inclusion conjunctivitis and
C Parasitic infestation leading to malabsorption lymphogranuloma venerum.
syndrome C True about Chlamydia pneumoniae
Giardiasis The group specific antigen is responsible for the
C A vitreous aspirate from a case of metastatic production of complement fixing antibodies.
endophthalmitis on culture yields Gram-positive C High fever, headache, generalized lymphadeno-
round to oval cells, 12–14 micron in size. Gram pathy and erythematous lesion of about 1 cm on leg
staining shows pseudohyphae. Fungus responsible surrounded by small residues in an army jawan
is posted in remote forest area was provisionally
Candida diagnosed as Rickettsial disease. Weil-Felix
reaction diagnostic in setting
High OX-K
ALL INDIA 2005
C Viruses can be cultivated in
C Cholera toxin acts in the small intestine by Animals, embryonated eggs and tissue culture
ADP ribosylation of G-regulatory protein C Virus mediated transfer of genetic material is called
C Halophilic species of Vibrio are Transduction
Vibrio parahemolyticus, V. fluvialis and V. alginolyti- C HIV can be detected and confirmed by
cus RT-PCR
C True regarding Clostridium perfringens C Neuroparasites are
Commonest cause of gas gangrene, normally Acanthamoeba, Naegleria and Taenia solium
present in human faeces, the principal toxin is alpha C The capsule of Cryptococcus neoformans can be seen
toxin. in CSF by
C True about E. coli India ink preparation
EAEC-persistent diarrhea. C The maximum serum concentration of subclass of
EHEC-HUS human IgG is
ETEC-Traveller’s diarrhea. IgG1
60 AIIMS and All India PGMEE—Review Questions

C Type of hypersensitivity reaction mediated by C Laboratory diagnosis of viral respiratory tract


IgE infection can be established by
C Ovary transplant from an identical twin to her sister is Detection of virus specific IgM antibodies in single
Isograft serum, detection of viral antigens by indirect
C Adenosine deaminase deficiency is seen in immunofluorescence assay in nasopharyngeal
Severe combined immunodeficiency washings and isolation of viruses using centrifu-
gation enhanced culture.
True about poliovirus
ALL INDIA 2004
C
Transmitted by faeco-oral route, asymptomatic
C True about hapten infections are common in children and live
It needs carrier protein attenuated vaccine produces herd immunity.
C The serotype of E. coil likely to cause haemorrhagic C Young pregnant woman presenting with fulminant
colitis in 20 years old man—0157:H7. hepatic failure
C Toxin inhibiting protein synthesis Hepatitis E virus
Shiga toxin C Clinical features associated with enteroviruses
C Fimbrial adhesives of diarrheagenic E. coli which Myocarditis, pleurodynia and herpangina
attaches to specific receptors in the GI track are
C True about Arbo viruses
CFA-1, CS-2 and K88
KFD is transmitted by ticks.
C Toxins are implicated as the major pathogenic in
diarrhea caused by
Vibrio cholerae, Shigella spp and Staphylococcus ALL INDIA 2003
aureus. C The most appropriate method to disinfet the
C An abattoir worker develops pustule on his hand. On endoscope in between two patients is
examination gram positive bacilli in long chains 2% gluteraldehyde for 20 minutes
positive for McFadyean’s reaction are found. C Heat labile surgical instruments is best sterilized by
Organism is Bacillus anthracis. Ethylene oxide gas
A farmer presenting with fever off and on for past
True about Bordetella pertussis
C
C
4 years was diagnosed chronic brucellosis.
Strict human pathogen, cultured from the patient
Serological tests applied are
during catarrhal stage and invades respiratory
Complement fixation test, Coombs’ test and 2ME test.
mucosa.
C True about gas gangrene
C A sewage drain worker presents with fever,
Gas is invariably present in the muscle compartment.
jaundice and renal failure. Diagnosis by
C A 30 years old woman with a bad obstetric history
Microscopic agglutination test
presents with fever. Blood culture-Gram-positive
small to medium coccobacilli, pleomorphic, in short C After attending a convention, an old man develops
chains and direct wet mount shows tumbling Legionnaires disease. It is acquired from
motility Inhalation of aerosol in the air conditioned room at
Listeria monocytogenes convention centre.
C Culture media for genital ulcer in an adult male C True about Lyme’s disease
patient with smear showing gram-negative Transmitted by ixodes tick, rodents are natural
coccobacilli is hosts and erythema chronicum migraines may be
Chocolate agar with isovitale X found.
C The most sensitive method for detection of cervical C Twenty out of 40 children developed abdominal
Chlamydia trachomatis infection is cramps, vomiting and watery diarrheas after
PCR 6–10 hours after a picnic party. Causative organism
C EB virus may cause Clostridium perfringens infection
Hodgkin’s disease, non-Hodgkin’s lymphoma and C True about rabies virus
nasopharyngeal carcinoma. RNA has negative polarity
Microbiology 61

C Infection associated with acute intravascular C Heat stable enterotoxin causing food poisoning is
hemolysis caused by
Babesia microti, Bartonella bacilliformis and Plasmo- Bacillus cereus, Yersinia enterocolitica and Staphylo-
dium falciparum. coccus.
C The earliest Ig synthesized by fetus is C HIV virus contains
IgM ssRNA
C True about HIV
ALL INDIA 2002
Reverse transcriptase present, infect CD4 cells and
C Bacteria may acquire characteristics by T-lymphocytes, cause CD4 count reduction at late
Conjugation, bacteriohage mediated and by taking stage.
up soluble DNA fragments across the cell wall. C CD4 count when CMV retinitis occurs in HIV
C Neonatal thymectomy leads to positive patient is less than
Decreased size of paracortical areas 50
C Staphylococcus aureus differs from Staphylococcus C Narrow angled septate hyphae in corneal scraping.
epidermidis by Fungus responsible is
Coagulase test Aspergillus
C In a patient with typhoid, diagnosis after 15 days of C True about globi in lepromatous leprosy
onset of fever is best done by Consists of macrophages filled with acid fast
Widal test bacilli
C Transmitted by rat urine C Correct matches are
Leptospira Ziehl-Neelsen staining-detection of Mycobacteria
C True about Listeria specific IgM antibodies—detection of acute
Transmitted by contaminated milk, causes abortion infection, immunofluorescence—detection of
in pregnancy and causes meningitis in neonates. Influenza virus.
C True about Bacteroides C IL-1 produces
May cause peritonitis T lymphocyte activation
62 AIIMS and All India PGMEE—Review Questions

Pathology

AIIMS NOVEMBER 2015 C Figure showing karyotype with 47 chromosome


with XXY. What will be found in the patient
C Main mechanism of genomic imprinting Gynaecomastia and long thin limbs.
Methylation
C Most important prognostic factor in ALL among
options AIIMS NOVEMBER 2014
Response to treatment C Factors responsible for apoptosis
C Cell cycle has specific checkpoints in the cell cycle Glucocorticoids
which can stop cell growth. Primary point in
regulation of cell growth C A 36 years old woman underwent cholecystectomy
for acute abdomen. The wound was closed with
End of G1
sutures. Step responsible for increasing the tensile
True about intraoperative histopathological
strength of the wound
C
analysis are
Cross-linking of tropocollagen
Used for detecting positive margins after resection
Used to confirm suspected metastasis C Epigenetics is the study of genetic changes without
Sentinel lymph node biopsy in breast carcinoma involving DNA sequences. Molecular techniques
is an example. used in epigenetic research
Bisulfite sequencing, methylation sensitive—RE
analysis and ChIP on chip.
AIIMS MAY 2015
C A patient presented with wheeze, pallor and
C ABO blood grouping shows the type of inheritance fatigue. He was given penicillin for an infection
Co-dominance 48 hours earlier but no history of penicillin allergy.
C More severe and early presentation in next generation Lab examination showed antibodies to penicillin
Anticipation and hemolytic anemia. Type of hypersensitivity
C Autoimmune conditions are reaction most likely developed
Grave’s disease, rheumatoid arthritis and systemic Type 2
lupus erythematosis. C The role played by MHC1 and MHC2 is
C IgE attaches to Present antigens for recognition by T cell antigen
Mast cells receptors.
C A patient presents with night sweat, fever and C Cancers are usually associated with
weight loss. On investigation, casseous necrosis Methylation of tumor suppression genes
mass was found in the apex of left lung. Cause is C Pairs of antigen and tumors correctly matched
Hypersensitivity reaction with modified macro- Vimentin in sarcoma, S100 protein in melanoma
phages, lymphocytes and giant cells. and leukocyte common antigen in lymphoma.

62
Pathology 63

C An old age man presents with chronic cough and C Example of a syndrome caused by uniparental
hemoptysis. On investigations, a grayish white disomy
tumor located at the central bronchus was found Angelman syndrome, Prader-Willi syndrome and
causing distal bronchiectasis and recurrent Russel-Silver syndrome.
pneumonia. Biopsy may show C ANCA is seen in
Small round cells and hyperchromatic nuclei with Wegener’s granulomatosis.
nuclear moulding. C ARDS is due to a defect in
(Note: Small cell lung cancer) Type 2 pneumocytes.
C A 70 years old man with history of working in the C A 50 years old male patient presents with large
asbestos factory for a long time, on routine bowel type diarrhea and rectal bleeding. On
examination shows a mass in the apex of right sigmoidoscopy, a cauliflower-like growth is seen in
upper lobe. Characteristic electron microscopic the rectum. Colectomy specimen on histopathology
appearance of the mass shows adenocarcinoma. Prognostic investigations
Large tubular cells with long villi on their surface required are
(Note: Mesothelioma) Microsatellite instability, c-myc mutations and
k-RAS mutations.
C Premalignant lesions are
C Verocay bodies are seen in
Barret esophagus, ulcerative colitis and Crohn’s Schwannoma.
disease.
C Histopathologically, rosettes are seen in
C A 16 years old girl underwent FNAC for breast Retinoblastoma, medulloblastoma and PNET.
lump that was well defined, non-tender and mobile.
Pathological feature suggesting benign lesion
AIIMS NOVEMBER 2013
Tightly arranged ductal epithelial cells with
dyscohesive bare nuclei. C Which indicate megaloblastic anemia
C The molecular classification of breast cancer includes Raised bilirubin, mild splenomegaly and nucleated
Hormonal receptor and Her-2 status RBC.
C The characteristic histopathological feature of C Platelet adhesion to collagen occurs via
schwannoma is von Willebrand factor
Antoni A and B areas C Markers of acute kidney injury includes
Cystatin- C, N-gal and kim-1
C Indoor air pollution leads to
AIIMS MAY 2014
Chronic lung disease, adverse pregnancy outcome
C A 50 years old patient presents with weakness and and pneumonia.
bleeding episodes, Hb–10.5 gm/dl, TLC–48 × 109 C Chromosomes are visualized through light micro-
cells/L, platelet 50 × 10 9 cells/L, dysplasia of scope with resolution of
neutrophils with a differential count showing 76% 5 mb
neutrophils, 8% blast cells, 12% myelocytes and C Tensile strength of wound after laparoscopic chole-
metamyelocytes and 4% other cells. Bone marrow cystectomy in a 30 years old women depends upon
showed 14% blast cells. Cytogenetics showed-t(8, 21).
Extensive crosslinking of tropocollagen
The most likely diagnosis
C In a 70 years old person who has worked in asbestos
Chronic myeloid leukemia. factory for 10–15 years, has a mass in right apical
C True about cellular events in acute inflammation region of the lung on routine X-ray. Electron
Components of complement can assist in microscopic finding after biopsy
chemotaxis, neutrophil margination is assisted by Numerous long slender microvilli
selectins and ICAM-1/VCAM-1 is responsible for C Which induces apoptosis
neutrophil adhesion. Glucocorticoid
C Which is morphogenic as well as mitogenic C A 45 years old lady presents with painless
Insulin like growth factor. supraventricular lymphadenopathy, on biopsy
64 AIIMS and All India PGMEE—Review Questions

showing binucleated acidophilic owl eye C 70-years-old patient presents with generalized
appearance with floating lymphocytes in empty lymphadenopathy, WBC count—20,000/mm3 and
space, that was CD 15 and CD 30 positive blood film showing > 70% mature looking
Nodular sclerosis Hodgkin lymphoma lymphocytes. Next done
C Real time polymerase chain reaction is done for Peripheral immunophenotyping
Monitoring amplification of target nucleic acid C Most common fixative used in electron microscopy
Formalin
C Mutation in COL4A5 chain is seen in
AIIMS MAY 2013
Alport’s syndrome
C Apoptosis is characterized by C Person with heterozygous sickle cell trait is
DNA fragmentation, Chromatin condensation and protected from the infection of
cell shrinkage. Plasmodium falciparum
C The role of MHC proteins is to C Hyperacute rejection is due to
Present antigens for recognition by T cell antigen Preformed antibodies
receptors. C In kidney specimen, fibrinoid necrosis is seen and
C Special stain used to diagnose fungal hyphae in onion peel appearance is also present. Most
tissues probable pathology is
Silver methenamine Hyaline degeneration
C In Alzheimer’s disease, the pathology seen in the C CD 95 is a marker of
brain is Extrinsic pathway of apoptosis
Atrophy of parietal and temporal lobes C Staining done for sebaceous cell carcinoma
C A 28 years old lady presents with recurrent Oil red O
abortions and pain in the calves. She is most likely C Flow cytometry is done on
suffering from the deficiency of Lymphocytes
Protein C C Receptor on neuronal membrane that induces
C Special stain used for the diagnosis of rhabdomyo- development of glioma
sarcoma CD 133
Desmin
C A 17 years old girl underwent FNAC for a breast AIIMS MAY 2012
lump, well-defined, non-tender and mobile. The C A young female patient, with no history of RHD, on
diagnosis of benign lesion can be considered by routine examination mid-systolic click is found.
feature Histopathology will show
Tightly arranged ductal epithelial cells with Myxomatous degeneration and prolapse of the
dyscohesive bare nuclei. mitral valve.
C Infection with HIV is associated with atrophy in C Characteristic of irreversible injury on electron
parts of the brain microscopy
Caudate nucleus, lower white matter nucleus and Amorphous densities in mitochondria
globus pallidus. C True statement is
Acute lymphoid leukemia in less than one year old
AIIMS NOVEMBER 2012 has poor prognosis.
C True about Bombay blood group
C Immunological stain used for the diagnosis of Lack of H, A, and B antigens on RBCs, lack of H, A
lymphoma and B substance in saliva and H, A and B antibody
Leukocyte common antigen will always be present in serum.
C Myelofibrosis leading to a dry tap on bone marrow C Hematopoetic stem cells differ from progenitor
aspiration is seen with stem cells in that they can
Acute megakaryocytic leukemia Long term reconstitution of bone marrow
Pathology 65

C In an ablated animal, myeloid stem cell series are reports show transmural coagulative necrosis.
injected. Cells seen after incubation period Further microscopic examination reveals
RBC Neutrophilic infiltration surrounding coagulative
C The role of bradykinin in process of inflammation is necrosis.
Increased vascular permeability > pain. C A 7 years old girl with complaints of generalized
C A 23 years old female patient presents with jaundice swelling of the body. Urinary examination reveals
and pallor for 2 months with peripheral blood grade 3 proteinuria and presence of hyaline and
smear showing spherocytes. Investigation done for fatty casts. She has no history of hematuria. True is
diagnosis No IgG or C3 deposition on renal biopsy.
Coombs’ test C Infraclavicular lesion of TB is called
C The fixative used in histopathology Assmann’s focus
10% buffered neutral formalin C CD4 is important for
C A newborn baby presents with profuse bleeding Antibody production, cytotoxicity of T cells and
from the umbilical stump after birth. Rest of the memory B cells. (Mn: MAC)
examination and PT, APTT are within normal C Cardiac polyp means
limits. Most probable diagnosis Fibrinous clot
Glanzmann thrombasthenia
AIIMS NOVEMBER 2010

AIIMS NOVEMBER 2011


C True about bacterial killing
MPO helps in formation of OCl, Chiediak Higashi
C Anticoagulant of choice for anticoagulation testing syndrome is due to failure of phagolysosome
Trisodium citrate 3.2 formation and NADPH oxidase helps in formation
C Pre-leukemic conditions of superoxide anion.
PNH, aplastic anemia and myelodysplastic C Disease with maternal disomy of chromosome 15
syndrome Prader-Willi syndrome.
C After binding of complement and antibody on C True about fibronectin nephropathy
surface of encapsulated bacteria, phagocytosis by Glomerular enlargement and PAS + trichrome
polymorphonuclear leukocytes involves mesangial deposit, Glomerulus do not consistently
Fc and C3b stain for Ig and complement and ultra structure
C Histological finding in a patient with radiologically feature is presence of large electron dense
confirmed reflux nephropathy developing nephrotic mesangial or subendothelial deposit.
range proteinuria C Edema in nephrotic syndrome is due to
FSGS Hypoalbuminemia
C True about subendocardial hemorrhage are
C Urine analysis of a patient of hematuria with
May be seen after head injury, continuous pattern
hypercalciuria
of sheet like and flame-shaped hemorrhage.
Isomorphic RBC
C Pale infant seen in
C Associated with Down syndrome Kidney, spleen and hearts
Mosaic 21, trisomy 21 and translocation t(13, 21), C Channelopathy are
t(21, 21). Cystic fibrosis, Liddle’s syndrome and hypokalemic
periodic paralysis.
Onion bulb appearance on nerve biopsy seen in
AIIMS MAY 2011
C
CIDP
C Commonest histological finding in benign hyper- C In Wegener’s glomerulonephritis, characteristic
tension is feature is
Hyaline arteriosclerosis Focal Necrotizing
C A 45 years old male patient died 4 days after severe C Mediators of inflammation are
chest pain and diagnosis of AMI was done. Autopsy TNF, IFN and PG/IL
66 AIIMS and All India PGMEE—Review Questions

C Inclusion in oligodendrocytes is a feature of C B cell markers are


Progressive multi focal leucoencophalopathy. CD10, CD 19 and CD 20
C A 9 years old boy with elevation in both PT and C Burkitt’s lymphoma, true is
APTT. Diagnosis is CD 34 -ve and sIg +ve
Defect in common pathway C True about P53 are
C Endotoxic shock is initiated by Encodes 53k Da protein, located on chromosome 17
Endothelial injury and arrests cell cycle at G1 phase.
C Conventional cytogenetics are difficult in solid C ANCA positive is
tumors especially in case of carcinoma cervix due to Wegener’s granulomatosis.
Bacterial contamination of specimen. C A 60 years old smoker has mass in bronchus
C A 60 years old lady, history of 8 blood transfusions resected. Most useful immunohistochemical
in last 2 years, Hb—6 g/dl, TLC—5800/cmm, marker to make a proper diagnosis would be
platelet—3.4 lakhs/cmm, MCV—60 , RBC—2.1 Cytokeratin
lakh/mm3 with hypochromic microcytic anemia. C Lipid in tissue detected by
Investigations needed are Oil Red O
Urinary hemosiderin, bone marrow examination C Benzopyrene change to carcinogen in animal occurs
and gastrointestinal endoscopy. due to
C A patient with microcytic hypochromic anemia, Epoxide formation, P53 formation and by inducing
Hb—9.1, serum iron is 20 mc/dl, ferritin level is metabolism of cyt P 450.
800 mg/ml and transferin saturation is 64. Possible C DNA repair defect associated with
diagnosis is Xeroderma pigmentosum
Atransferrinemia C Gene instability associated with malignancy is
C Best predictor of coronary heart disease Ataxia telangiectasia
LDL C HLA is located on
Short arm of chromosome 6.
AIIMS MAY 2010 C True is
C Main feature of chemotaxis is BCL 6 Burkitt’s lymphoma.
Unidirectional locomotion of neutrophil. C Low serum haptoglobin in hemolysis is masked by
C Coagulative necrosis is seen in Bile duct obstruction.
TB
AIIMS MAY 2009
C Best investigation for diagnosing amyloidosis is
Rectal biopsy C True about metastatic calcification
C Caspases are required for Calcification starts in mitochondria.
Organogenesis C True about neurofibromatosis are
C Two siblings have osteogenesis imperfecta but their Scoliosis, multiple fibroma and associated with
parents are normal. Mechanism of inheritance in cataract.
this case is C True about pilocytic astrocytoma
Germ line mosaicism Long survival, total surgical resection possible and
C CD 4 marker specific for myeloid series can involve posterior fossa.
CD 117 C Both hyperplasia and hypertrophy is found in
C In Berry’s aneurysm, there is congenital defect in Pregnancy uterus
Degeneration of media/muscle cell layer C Annexin V is a marker of
Apoptosis
AIIMS NOVEMBER 2009 C Seen in HIV involvement of CNS
Perivascular giant cell, vacuolar degeneration of
C NARP is a posterior column and microglia nodule formation.
Mitochondrial disorder C Langerhans cell histiocytosis, true is
C MIC-2 is marker of Peak incidence less than 3 year of age, radiosensitive
Ewing Sarcoma and diffuse form is known as Letterer Siwe disease.
Pathology 67

C Tumerogenesis in aging is due to C A 30 years old female, RBC count 4.5 million, MCV
Telomerase reactivation 5.5 micron, TLC 8000, no history of blood transfusion
C Finnish of nephrotic syndrome is caused by defect Thalassemia minor
in C Seen in ulcerative colitis is
Nephrin Cryptitis
C MHC restriction is part of C Hematoxylin bodies seen in
Antiviral cytotoxic T cell, antibacterial helper T cell SLE
and allograft rejection. C CD 99 is for
C Paucimmune glomerulonephritis is seen in Ewing’s sarcoma
Microscopic polyangitis C Loss of heterozygosity is associated with
Retinoblastoma
AIIMS NOVEMBER 2008 C AML with gum infiltration, hepatosplenomegaly
C Gene for MHC complex is located on M4
Chromosome 6 C Bone resorption markers are
C BRCA 1 gene is located on TRAP, cross linked N-telopeptide and urine total
free deoxypyridinoline.
Chromosome 17
C FSGS with worst prognosis is
C Gene for folate carrier protein is located on
Diffuse alveolar damage
Chromosome 21
C Tumor suppressor genes are
C Essential for tumor metastasis is
WT1, Rb and p53
Angiogenesis
C Steroid resistant nephrotic syndrome is
C Virchow’s triad includes
Podocin
Injury to vein, venous stasis and hypercoagulability
A patient with Hb-6 gm/dl, TLC-12,500, platelet
of blood. (Mn: HIS)
C
60,000, MCV-112 fl. Diagnosis is
C Minor diagnostic criteria for multiple myeloma are
Megaloblastic anemia
Lytic bone lesions, plasmacytosis greater than 20%
and monoclonal globulin spike on serum
electrophoresis of 2.5 g/dl for IgG and > 1.5 g/dl for AIIMS NOVEMBER 2007
IgA. C Collapsing glomerulopathy, features are
C Antibody independent and dependent complement Hypertrophy and necrosis of visceral epithelium.
pathway converge on C In MVPS, mitral valve histopathology shows
C3 Myxomatous degeneration
C Good prognostic factors for acute lymphoblastic C Prion includes
leukemia are
Proteins
Age of onset between 2 and 8 years, initial WBC
C Councilman bodies are seen in
court > 50000 and hyperploids.
Acute viral hepatitis
Nitroblue tetrazolium test is used for
Stain used for lipid
C
C
Phagocytosis (Mn: PhaNi) Oil red O, Sudan III and Sudan black.
C Hypersensitivity vasculitis are seen most commonly C Acrodyne orange is a fluorescent dye used to bind
in DNA and RNA
Postcapillary venules C PAS stains
Glycogen, lipid and fungal cell wall.
AIIMS MAY 2008
C Ability of stem cells to cross barrier of differentia-
C A patient presenting with mediastinal mass with tion to transform into a cell of another lineage
sheets of epithelial cells giving arborizing pattern of expressing the molecular characteristic of different
reactivity along with interspersed lymphoid cells. cell type with the ability to perform function of new
The diagnosis is cell type is referred as
Thymoma Trans differentiation
68 AIIMS and All India PGMEE—Review Questions

C Components of basement membrane are C Why fetal cells continue to divide but terminally
Nidogen, laminin and entactin. differentiated adult cells don’t divide
C Ultra structural finding of irreversible cell, injury There are many cyclin inhibitors which prevent cell
Amorphous densities in mitochondria to enter into S phase in adult.
C Toll like receptors recognize bacterial products and
C Reye’s syndrome—histological finding is
stimulate immune response by
Glycogen depletion
Transcription of nuclear factor mediated by NFK
C Pancytopenia with cellular marrow is seen in beta which recruit cytokinines.
PNH, megaloblastic anemia and myelodisplastic C C-C beta chemokine includes
syndrome. Eotaxin
C Contraction of endothelial cell cytoskeleton in acute
AIIMS MAY 2007 inflammation results in
C Michaelis Guttman bodies are seen in Early transient increase
Malakoplakia
C Histopathological feature of liver in malaria is AIIMS MAY 2006
Kupffer cell hyperplasia with macrophage
Right ventricular endo-myocardial biopsy of a
infiltration around periportal area laden with
C
50 years old male patient of restrictive heart disease
pigments.
revealed deposition of extra cellular eosinophilic
C Affected in graft versus host disease
hyaline material. On electron microscopy, finding is
Skin, GIT and liver
Non-branching filaments of indefinite length
Brain natriuretic peptide is degraded by
The ideal immunohistochemistry panel for a 30
C
C
Neutral endopeptidase years old female with histopathologic finding from
C B cell markers are solid-cystic unilateral ovarian tumor showing
CD19, CD21 and CD24. diffuse sheets of small cells with doubtful nuclear
C DIC is seen in grooving and scanty cytoplasm (and no Call-Exner
Acute promyelocytic leukemia bodies are seen)
C Gene defect in idiopathic steroid resistant nephrotic CD99, epithelial membrane antigen, inhibin and
syndrome vimentin.
NPHS2 C True about carcinogenesis
Asbestos exposure increases incidence of lung
AIIMS NOVEMBER 2006 cancer, exposure to aniline dyes predisposes to
cancer of the urinary bladder and hepatitis B virus
C Antiapoptotic gene is has been implicated in HCC.
Bcl-X C Histological feature most helpful to differentiate
C Cytosolic cytochrome C plays an important role in between possible enchondroma or chondrosarcoma
Apoptosis in a 50 years old lady presenting with 3 months
C Acid phosphatase is specific for history of pain in lower third of right thigh is
Monocyte Tumor permeation between bone trabeculae at
C Proliferation of which cell occurs in post-transplant periphery.
lymphoma C True about lymphoma
B cell A single classification system for Hodgkin’s disease
is almost universally accepted, HD more often
C Beta pleated sheet is seen in amyloidosis in
tends to remain localized to a single group of lymph
X-ray crystallography nodes and spreads by contiguity and several types
C Natural killer cells attack of NHL may have a leukemic phase.
Cells which are not able to express MHC 1. C Characteristically associated with development of ILD
C Pigments involved in free radical injury Organic dusts, inorganic dusts and toxic gases (e.g.
Lipofuscin chlorine, sulphur dioxide).
Pathology 69

C Peripheral smear with increased neutrophils, C Internucleosomal cleavage of DNA is characteristic


basophils, eosinophils and platelets is highly of
suggestive of Apoptosis
Chronic myelogenous leukemia C On electron microscopy, amyloid characteristically
C Immunophenotype of blasts of a 18 years old boy exhibits
presenting with TLC of 138 × 109/L with 80% blasts 7.5–10 nm fibrils
of peripheral smear and large medistrial mass on C Feature of acute diffuse proliferative GN are
X-ray would most likely show Microscopic hematuria, raised blood urea level, and
An immature T cell phenotype (Tdt/CD34/CD7 raised serum creatinine level.
positive). C Mediators of acute inflammation
C Amyloid deposits stain positively with PG E2, kallikrein and C3a
Congo red, crystal violet and thioflavin T. C Surface glycoprotein most often expressed in
C The curetting from fracture site of a 70 years old human hematopoietic stem cell
male presenting with pathological fracture of femur CD34
with X-ray showing lytic lesion and circumscribed C Cellular content of DNA is doubled during
punched out appearance would most likely show S phase
Sheets of atypical plasma cells. C The tumor suppressor gene p53 induces cell cycle
C A 15 years old boy presented with 1 day history of arrest at
bleeding gums, subconjunctival bleed, and G1S phase
purpuric rash. Lab finding –Hb↓, TLC↑, platelet ↓,
C Features compatible with diagnosis of juvenile
PT ↑, aPTT ↑, fibrinogen ↓ and peripheral smear
myelomonocytic leukemia
suggestive of acute myeloblastic leukemia. Diagnosis
GM-CSF hypersenstivity of myeloid progenitors in
Promyelocytic leukemia
vitro, increased HbF levels for age and peripheral
A normal couple has two children affected with
blood monocytes, more than 1 × 109/L.
C
tuberous sclerosis. This is explained by
C Programmed cell death is known as
Germ line mosaicism
Apoptosis
AIIMS NOVEMBER 2005 C The most potent stimulator of naïve T cells is
C The most common histological variant of renal cell Mature dendritic cells
carcinoma
Clear cell carcinoma AIIMS MAY 2005
C Diseases with DNA repair mechanism defect
Chemotherapeutic drugs can cause
Fanconi syndrome, HNPCC and xerodema
C

pigmentosa. Both necrosis and apoptosis


C Carcinoma most frequently metastasizing to brain C Gastrin is a biochemical marker of
Small cell carcinoma lung Pancreatic neuroendocrine tumor
C A metastatic carcinoma in the brain of an adult most C The tissue of origin of Kaposi’s sarcoma is
often comes from a primary in the Vascular
Lung C Guardian of the genome
C The lymphocytic and histiocytic variant of Reed- p53
Sternberg cell is seen in C True about tumor suppressor gene p53
Lymphocyte predominant Hodgkin’s disease It regulates certain genes involved in cell cycle
C Example of round cell tumor regulation, its increased levels can induce apoptosis
Ewing’s sarcoma, neuroblastoma and NHL and mutations of p53 gene are the most common
C The inheritance pattern of familial retinoblastoma genetic alteration seen in human cancer.
AD C Increased susceptibility to breast cancer is likely to
C Common site for metastatic calcification be associated with a mutation in
Gastric mucosa, kidney and lung. p53
70 AIIMS and All India PGMEE—Review Questions

C Conditions causing megekaryocytic thrombocyto- C Coagulation parameters of a patient with cirrhosis


penia are of liver are platelet count 2,00,000, PT ↑, aPTT ↑,
ITP, SLE and DIC thrombin time 15s/15s. In this patient
C Ladder pattern of DNA electrophoresis in apopto- D-dimer will be normal
sis is caused by the action of C The liver biopsy in acute hepatitis due to hepatitis B
Endonuclease virus is likely to show
C HIV associated nephropathy is a type of Acidophil bodies, ballooning change of hepatotytes
Collapsing glomerulopathy and focal or spotty necrosis.
C In renal disease, albumin is first to appear in urine
C Tumor markers
because
AFP, β2 microglobulin and thyroglobulin
Has moleculer weight slightly greater than the
C True about HbS molecules normally getting filtered.
HbS differs from HbA by substitution of Val for Glu C Tumor with elevated level of PLAP in serum along-
in position 6 of beta chain, one altered peptide of wtih positive immunolistochemical staining for
HbS migrates faster torwards the cathode (–) than PLAP
the corresponding peptide of HbA and lowering Seminoma
the concentration of deoxygenated HbS can prevent C A baby girl with ambiguous genitalia have
sickling. 21-hydroxylase deficiency of salt wasting type.
C IL characteristically produced in a TH1 response Karyotype
IL-2 46, XX
C Calcification of soft tissues without any disturbance
of calcium metabolism is called
AIIMS NOVEMBER 2003
Dystrophic calcification
C Disease transmitted from father to daughter and C Transition from G2 to M phase of the cell cycle is
from daughter to both her sons and daughters controlled by
shows inheritance pattern of Cyclin B
X-linked dominant C Cyclin dependent kinase inhibitors are
C Aniridia and hemihypertrophy are most likely P21, P27, P57
present in C Hypercoagulability due to defective factor V gene is
Wilms’ tumour called
C In the mitogen activated protein kinase pathway, Leiden mutation
the activation of RAS is counteracted by C The presence of small-sized platelets on the
GTPase activating protein peripheral smear is characteristic of
C Wire loop lesions are often characteristic for class of Wiskott-Aldrich syndrome
lupus nephritis C In malignant hyperthermia the increased heat
Diffuse proliferative GN (class IV WHO) production is due to
C Best morphological feature to distinguish UC from Increased muscle metabolism by excess of calcium
CD ions.
Diffuse distribution of pseudopolyps C 7 year old boy presented with generalized edema.
C In chronic viral hepatitis Urine examination showed marked albuminuria,
Grading refers to the extent of necrosis and serum biochemical examination—hypoalbuminemia
inflammation. with hyperlipidaemia. Kidney biopsy, light micro-
C Mesangial deposits of monoclonal kappa/lambda scopy examination normal, electron microsocopy
light chain is indicative of finding reveals
Amyloidosis Fusion of foot processes of the glomerular epithelial
C Immunophenotype pattern in a patient with acute cells.
leukemia is CD10+ve, CD19+ve, CD33+ve, CD C Cereals avoided in coeliac disease
13+ve. Diagnosis Barley, rye, wheat (Mn: Brow–Barley, ryes, oat and
Biphenotypic leukemia wheat)
Pathology 71

C Obstruction to the flow of CSF at the aqueduct of DLC—Neutrophils 55%, lymphocytes 4%, mono-
Sylvius most likely lead to enlargement of cytes 2%, basophils 6%, Metamyelocytes 10%,
Both lateral and third ventricles. myelocytes 18%, promyelocytes 2% and blasts 3%.
C CD 95 has a major role in The most likely cytogenetic abnormality t(9, 22)
Apoptosis CML
C Example of C-X-C or α chemokine C A 50 years old woman presents with progressive
IL-8 renal failure for past 3 years, renal biopsy showing
glomerular and vascular deposition of pink
C Tectel breaking is found in
amorphous material. It shows apple green
Arnold-Chiari malformation birefringence and polarized light after congo red
C Compatible with diagnosis of chronic myelomono- staining. These deposits are positive for Lambda
cytic leukemia light chain. Diagnosis
Peripheral blood monocytosis more than 1 × 109/L, Multiple myeloma
absence of Philadelphia chromosome and absent or C A 40 years old man has chronic cough with fever for
minimal dysplasia in myeloid lineages. several months. X-ray showing diffuse reticulono-
C Complication of infective endocarditis dular pattern. Focal areas of inflammation
Myocordial ring abscess, myocardial infarction and containing epitheloid cell granuloma, Langerhans
focal and diffuse GN. giant cells and lymphocytes are seen micro-
C A 1 year old boy presented with heptospleno- scopically on transbronchial biopsy. Type of hyper-
megaly and delayed milestones, liver biopsy and sensitivity reaction is
bone marrow biopsy revealing presence of Type IV
histiocytes with PAS positive diastase resistant C Most important factor responsible for rapid healing
material in the cytoplasm. Electron microscopic of a burn injury of hand of an adult man over a few
examination of these histiocytes most likely reveals weeks without the need for grafting
the presence of Remnant skin appendages.
Parallel rays of tubular structures in the lysosomes C An organ section at autopsy shows focal, wedge
Niemann-Pick’s disease shaped, firm area accompanied by extensive hrge
C Classical presentation of craniovertebral junction with red appearance with base on surface of organ.
anomalies are This is typical of
Pyramidal signs, low hairline and short neck Lung with pulmonary thromboemobolism
C In a case of Rh incompatibility, a few days after birth (Hint: Red infarct).
neonate developed jaundice, ascites, hepatomegaly C Tophus is the pathognomonic lesion of
and edema. The likely substance(s) deposited in Gout
skin and sclera in jaundice is C Infertility is a common feature in Sertoli cell syn-
Conjugated and unconjugated bilirubin drome because
There are no germ cells in this condition.
AIIMS MAY 2003 C The most common infectious agent associated with
chronic pyelonephritis is E. coli.
C The light brown perinuclear pigment seen on H/E
staining of the cardiac muscle fibres in grossly
AIIMS NOVEMBER 2002
normal appearing heart of 80 years old man at
autopsy is due to deposition of C Mixed lymphocyte culture is used to identity
Lipochrome MHC Class II antigen.
C Dystrophic gene mutation leads C Feature seen in asbestosis
Duchenne muscular dystrophy Calcific pleural plaques, diffuse pulmonary inter-
C A 60 years old man presented with fatigue, weight stitial fibrosis and fibrous pleural thickening (Mn:
loss, and heaviness in left hypochondrium for 6 CDF).
months with hemogram showing Hb 10gm/dl, C Aschoff bodies in rheumatic heart disease shows
TLC 5 lakhs/mm 3, platelet count 4 lakhs/mm3, Anitschkow cells, fibrinoid necrosis and giant cells.
72 AIIMS and All India PGMEE—Review Questions

C Distinguishing factor between hydrocephalus due C Baby’s blood group is O Rh negative. Blood group
to aqueductal stenosis and that due to Dandy- baby’s parents will not have
Walker malformation AB Rh negative.
Posterior fossa volume C An accident victim urgently needs blood. The blood
(Hint: 4th ventricle is in posterior fossa). bank is unable to determine his ABO group.
C Dystrophic calcification is seen in Emergency transfusion of the patient should be
Atheromatous plaques. O positive RBC and colloids/crystalloid.
C Familial amyloidotic polyneuropathy due to C ABO blood group system is most important in
amyloidosis of nerves is caused by deposition of clinical medicine out of 400 blood groups because
Mutant transthyretin. ABO (H) antibodies are invariably present in plasma
C Lardaceous spleen is due to deposition of amyloid in when person’s RBC lacks the corresponding antigen.
Sinusoids of red pulp. C Complement fixed best by
C A 40 years old man presented with 2 week history IgM
of fever, weakness, bleeding gums, peripheral C The most common cause of Addison’s disease
blood smear showing pancytopenia, bone marrow
Autoimmune adrenalitis.
showing 26% blasts frequently exhibiting. Auer
Gastrointestinal stromal malignancy arises from
rods of mature myeloid cells with occasional
C

neutrophil and pseudo Pelger-Huet anomaly. Interstitial cells of Cajal.


Cytochemical stain found positive is C Antigen presenting cells
Myeloperoxidase Langerhan’s cells.
(Hint: AML) C Asbestosis of lung is associated with
C A patient presented with cough, homoptysis, Asbestos bodies in sputum, mesothelioma and
glomerulonephritis and raised C-ANCA levels in progression of lesion even after stopping exposure
serum. Diagnosis to asbestos.
Wegener’s granulomatosis C A female patient presented with a firm mass of 2 × 2
C The gene regulating normal morphogenesis during cm in upper outer quadrant of breast. She gives
development family history of ovarian carcinoma. Investigation
needed to assess for mutation
Homeobox gene
BRCA-2
C The Fenton reaction leads to free radical generation C Associated with hypersensitive pneumonitis
when Byssinosis
Ferrous ions are converted to ferric ions. C A young girl presented with bilateral nodular
C Electron microscopy finding indicating irreversible lesions on skin. She was also found to have bilateral
cell injury hilar lymphadenopathy on chest X-ray. Mantoux
Flocculent densities in mitochondria. test negative. Skin biopsy may reveal
C Familial syndromes associated with pheochro- Non-caseating granuloma
mocytoma (Hint: Erythema nodosum—Sarcoidosis).
MEN-II, Sturge-Weber syndrome and von Recklin-
ghausen disease. ALL INDIA 2012
C The approximate number of genes in human C A young boy presents with dyspnoea, mediastinal
genome is 30,000. mass on CT scan and lymphoma on histopathology.
C In posthepatic jaundice, the concentration of Diagnosis
conjugated bilirubin in the blood is higher than that T cell lymphoblastic lymphoma
of unconjugated bilirubin because C A person suffering from normal MCHC and high
The conjugation process of bilirubin in liver remains MCV may be suffering from
operative without any interference. Cobalamine deficiency
C The pathogenesis of hypochromic anemia in lead C A 12 years old female patient presents with fever,
poisoning leucopenia and splenomegaly suddenly died due to
Inhibition of enzymes involved in heme biosynthesis. splenic rupture. Gross postmortem finding
Pathology 73

Longitudinal ulcers of small intestine alongside of C An infant presents with generalized skin rash with
lymphoid tissues. biopsy suggestive of Langerhans cell histiocytosis.
C Splenomegaly is least likely associated with Immunohistochemical stain to confirm the diagnosis
Essential thrombocytosis CD 1a
C Inherited aplastic anemia is C In normal healthy person blood is prevented from
Fanconi’s anemia clotting due to
C A 2 years old boy presents with skin rashes, Binding of thrombin to antithrombin 3
bleeding, very low platelet counts and decreased C Bone marrow derived stem cells are
IgM. Diagnosis Endothelial progenitor cells, hematopoetic stem
Wiskott-Aldrich syndrome cells and mesenchymal stem cells.
C The amyloid protein in secondary amyloidosis is C A 50 years old male patient with history of
most likely derived from bronchiectasis 5–10 years back, now presents with
Serum amyloid associated protein pedal edema, albuminuria and hypoalbuminemia.
Renal biopsy will show
C Immunohistochemical marker most useful for
establishing diagnosis of a poorly differentiated Amyloid nephropathy (Secondary amyloidosis in
carcinoma brochiectasis—Nephrotic syndrome).
Features shared by B and T lymphocytes
Cytokeratin
C
Positive selection during development, antigen
C A 18 years old male hypertensive patient suddenly
specific receptors and dependence on cytokines
developed severe headache, vomiting, became
secreted by other cells.
unconscious and died within 24 hours. Gross
pathological finding of kidney C A patient with renal tumor is diagnosed by
exfoliative cytology. The most likely histological
Petechial hemorrhages on the surface
subtype
Histopathology of adults with respiratory distress
Transitional cell carcinoma, adenocarcinoma and
C
syndrome
well-differentiated carcinoma—all three subtypes
Diffuse alveolar damage can be easily detected by exfoliative cytology.
C True about DIC C Pyrogenic cytokines are
Increased FDP, prolonged PT, increased thrombin- TNF, IF-α and IL-6
antithrombin complex, decreased antithrombin 3,
reduced platelet count.
C Immune hypersensitivity reaction in myasthenia ALL INDIA 2011
gravis C Organelle playing a pivotal role in apoptosis
Type 2 hypersensitivity reaction. Mitochondria
C Autosomal dominant conditions are C Enzyme contributing in generating free oxygen
Gardener’s syndrome, neurofibromatosis and radicals within neutrophils for killing intracellular
Peutz-Jegher’s syndrome. bacteria is
C A biopsy report shows inflammation of the airways Superoxide dismutase, Fenton’s reaction and
with increased Reid index. Diagnosis NADPH oxidase.
Chronic bronchitis C Enzymes responsible for generating oxygen burst
C An acidified urine sample contains proteins that within neutrophils for killing intracellular bacteria
precipitate on heating at 40–50°C but redissolves on are
further heating. The protein is Oxidase/NADPH oxidase.
Bence-Jones protein C Most important characteristic feature of acute
C A 9 years old female patient presents with primary inflammation is
small round cell tumor in tibia. Most likely genetic Vasodilatation and increased vascular permeability.
abnormality C True about xanthogranulomatous inflammation is
22q12 translocation Foam cells, yellow nodules and multinucleated
(Hint: Ewing’s sarcoma) giant cells are seen.
74 AIIMS and All India PGMEE—Review Questions

C True about blood coagulation C Translocation t (2,8) (p 12: q24) is associated with
Factor X is part of both intrinsic and extrinsic path- Burkitt’s lymphoma
ways, intrinsic pathway can be activated in vitro, and C Plasmacytoid lymphoma is associated with
calcium is required in several steps of coagulation. IgM
C HbH is associated with deletion of C Most friable vegetations in
3 alpha genes Infective endocarditis
C True about platelet function defects C Characteristic pathological finding in carcinoid heart
Normal platelet count with prolonged bleeding disease is
time. Fibrous endocardial thickening of RV, tricuspid
C True regarding Bernard-Soulier syndrome valve and pulmonary valve.
Aggregation with collagen and ADP is normal, C A female patient presents with history of
large platelets and thrombocytopenia. progressive breathlessness. Histology shows
C An antibody most frequently seen in antiphaspho- heterogenous patchy fibrosis with several
lipid syndrome is fibroblastic foci. Diagnosis
Anti-beta 2 glycoprotein antibody Usual interstitial pneumonia.
C Autoimmune diseases are C Chromophobe variant of RCC is associated with
Monosomy 1 and Y
SLE, Graves’ disease and myasthenia gravis.
C Hypercoagulation in nephrotic syndrome is caused
C Necrotizing lymphadenitis is characteristically
by
seen in
Loss of antithrombin III
Kikuchi disease
GIST specific marker is
Neuronal tumors are
C
C
CD 117
Gangliocytoma, ganglioglioma and neurocytoma.
Granulomatous vasculitis are associated with
Psammoma bodies may be seen in
C
C
Wegener’s granulomatosis, Takayasu arteritis and
Papillary carcinoma thyroid, meningioma and
giant cell arteritis.
serous cystadenocarcinoma of ovary.
Electron microscopy is diagnostic in
Most commonly used fixative in diagnostic
C
C
pathology is Alport’s syndrome
Formaldehyde C Most characteristic ultra structural feature of
paraganglioma on electron microscopy is
ALL INDIA 2010 Dense core neruoendocrine granules.
C Marker for Langerhan’s cell histiocytosis is
C Characteristic feature of apoptosis is
CD 19
Nuclear compaction
C Down’s syndrome is most commonly caused by
C Features of apoptosis are
Maternal nondisjunction
Nuclear compaction, intact cell membrane and
A normal couple has one daughter affected with AR
cytoplasmic eosinophilia.
C
disease cystic fibrosis. Chance of another child
Caspases are involved in
being affected by the disease is
C
Apoptosis
¼
Actions of bradykinin include
Males are more commonly affected in
C
C
Vasodilatation, increased vascular permeability
X-linked recessive
and pain.
C Heterozygous sickle cell anemia is protective
ALL INDIA 2009
against
Malaria C Most effective bactericidal system within
C PNH is associated with a deficiency of phagocytes is
DAF, MIRL and GPI anchored protein Reactive oxygen metabolite mediated.
C Burkitt’s lymphoma is associated with C Primary structural defect of an organ is termed
t (8,14) Malformation
Pathology 75

C Associated with increased aging is C Crescentric glomerulonephritis may be seen in


Increased superoxide dismutase PSGN, HSP and anti-basement membrane disease.
C Paraneoplastic syndromes associated with RCC are C Non-proliferative glomerulonephritis includes
Polycythemia, hypercalcemia and hypertension. FSGS, membranous glomerulonephritis and
C Coombs’ positive hemolytic anemia is associated amyloidosis.
with C True malignant tumors are
SLE Plasmacytoma, Askins tumor and chloroma.
C Conditions in association with Coomb’s positive
Rh factor is a
hemolytic anemia are
C

SLE, PAN and scleroderma Antibody


C Hypersensitivity pneumonitis is associated with C True about graft versus host disease
Type III (immune complex) hypersensitivity. Occurs when host is immunocompromised, also
C True about Hashimoto’s thyroiditis are called Runt disease and common cause is stem cell
Follicular destruction increase in lymphocytes and transplantation.
oncocytic metaplasia. C True regarding super antigens
C Rate of newly synthesized osteoid mineralization Binds T cell irrespective of antigenic specificity of
can be best estimated by TCR, bind directly to both MHC II and T cell
Tetracycline labeling receptor causing T cell activation and binds directly
C Karyotyping is useful in diagnosis of to lateral aspect of T cell receptor.
Chromosomal abnormalities. C Ferruginous bodies are commonly seen in
C If a functional gene is inherited from one parent Asbestosis
only, the condition is known as C Degeneration of basement membrane is mediated by
Genomic imprinting Metalloproteinase

ALL INDIA 2008 ALL INDIA 2007


C Reactive oxygen intermediates are released by C Earliest transient change following tissue injury
NADPH oxidase will be
C Delayed prolonged leakage following tissue injury Neutrophilia
is due to
C Antibody independent and dependent complement
Direct effect of the injuring agent. pathway converge on
C Antigen presenting cells are C3
Langerhans’ cell, dendritic cells and B cells.
C Component present in final common terminal
C Hemodialysis associated amyloid is complement pathway is
Beta-2 microglobulin. C5
Oncogenes that is a growth factor is
Ig involved in Type I hypersensitivity reaction is
C
C
Sis
IgE
C HMB 45 is an immunohistological marker for
C Type of hypersensitivity of Arthus reaction is
Melanoma.
Localized immune complex
C Specific marker for synovial sarcoma is
t (×, 18) C Protein defect causing hereditary spherocytosis is
C Characteristic markers for Hodgkin’s lymphoma are Ankyrin, palladin and anion transport protein.
CD 15 and CD 30. C ALL L3 morphology is a malignancy arising from
C Rosettes are characteristic in Mature B cell
Retinoblastoma C Non-specific esterase is positive in
C Change occurring in malignant HTN M3, M4, M5
Petechial hemorrhage on cortical surface, fibrinoid C Bone marrow in AL amyloidosis shows
necrosis of arterioles and intimal concentric thickening. Bone marrow plasmacytosis.
76 AIIMS and All India PGMEE—Review Questions

ALL INDIA 2006 C Substance released from mitochondria which


activates Apaf-1 in apoptosis
C The normal cellular counter parts of oncogenes are Cytochrome C
important for C In situ DNA nick end labeling can quantitate
Promotion of cell cycle progression, promotion of Fraction of cells in apoptotic pathways
nuclear transcription and inhibition of apoptosis.
C All endothelial cells produce thrombomodulin
C Most significant risk factor for development of when present in
gastric carcinoma Renal circulation, cutaneous circulation and
Intestinal metaplasia hepatic circulation.
C Stomach cancer developing secondarily to pernicious C Example of tumor suppressor gene is
anemia is usually situated in the Rb
Fundus C Feature of malignant transformation by cultured cells
C Acinic cell carcinoma of salivary glands arise most Alteration of cytoskeletal structures, increased cell
often in the density and loss of anchorage.
Parotid gland C A simple bacterial test for mutagenic carcinogens is
C The most important criterion for malignant Ames test
behavior of leiomyosarcoma C Tumor marker used in testicular tumors
The number of mitoses per high power field AFP, hCG and LDH
C The type of mammary DCIS most likely to result in C Stain specific for amyloid
a palpable abnormality in the breast is Congo red
Comedo DCIS C Type of amyloidosis caused by mutation of
C Immunohistochemical marker positive in neoplastic transthyretin protein
cells of granulocytic sarcoma are Familial amylodotic polyneuropathy
CD43, myeloperoxidase and lysozyme C The classification for NHL proposed by international
C Mantle cell lymphomas are positive for lymphoma study group
CD5, CD20 and CD43 REAL classification
C Neointimal hyperplasia causes vascular graft failure C True about leukemia
as a result of hypertrophy of Tartarate resistant acid phosphatase activity is
Smooth muscle cells typically seen in hairy cell leukemia.
C True about subendocardial infarction C True statements are
Multifocal in nature, often result from hypotension Patients with IgD myeloma may present with no
or shock and epicarditis is not seen. evident M-spike on serum electrophoresis, in
C The percentage of pulmonary emboli proceed to smoldering myeloma plasma cells constitute
infarction is 10–30% of total bone marrow cellularity and a
5–15% patient with multiple myeloma, a monoclonal light
chain may be detected in both serum and urine.
ALL INDIA 2005 C Aschoff’s nodules are seen in
Rheumatic carditis
C True about reversible cell injury C Features seen in viral pneumonia
Diminished generation of ATP, formation of blebs Bronchiolitis, multinucleate giant cells in the
in the plasma membrane and detachment of bronchiolar wall and presence of interstitial
ribosomes from granular endoplasmic reticulum. inflammation.
C Vascular changes observed in acute inflammation C Atypical pneumonia can be caused by
are Human corona virus, Legionella pneumophila and
Increased vascular permeability, vasodilatation Mycoplasma pneumoniae.
and stasis of blood. C Bilateral contracted kidneys are characteristically
C Fibrinoid necrosis may be seen in seen in
Aschoff’s nodule, malignant HTN and PAN. Benign nephrosclerosis
Pathology 77

C Kidney biopsy from a child with HUS characteris- C Most common cytogenetic abnormality in adult
tically most likely presents with features of myelodysplastic syndrome
Thrombotic microangiopathy Monosomy 7
C Disease characteristically causing fatty change in C True about hairy cell leukemia
liver Cells are positive for TRAP, splenomegaly is
Chronic alcoholism conspicuous, and cells express CD 25 consistently.
C Feature of liver histology in NCPF C Good prognostic markers for Hodgkin’s disease are
Fibrosis in and around portal tracts, non-specific Hb > 10 gm/dl, WBC count < 15,000/mm3 and age
inflammatory cell infiltrates in the portal tracts and < 45 years.
thrombosis of the medium and small portal vein C A 65 years old male patient presented with acute
branches. chest pain of 4 hrs duration ECG-new Q wave with
C Differential expression of same gene depending on ST wave depression and died within 24 hrs. The
parent of origin is referred to as heart revealed presence of a transmural hemorrhagic
Genomic imprinting area over the septum and anterior wall of left
C The membrane protein, clathrin is involved in ventricle. Light microscopic examination reveals
Receptor mediated endocytosis Necrotic myofibers with presence of neutrophils
C Cellular and flagellar movements are carried out by C Serum C3 is persistently low in
Actin, myosin and tubulin MPGN, lupus nephritis and GN related to bacterial
endocarditis.
C Associated with low complement levels
ALL INDIA 2004 MPGN, lupus nephritis and post-infectious GN.
C Lipoxin belongs to family of chemical mediators of C A 70 years old male addicted to tobacco chewing for
inflammation last 52 years presented with six months history of
Arachidonic acid metabolites. large fungating, soft papillary lesions in the oral
C Procoagulant protein is cavity with lesion penetrating into mandible,
Thrombin lymph nodes not palpable and two biopsies taken
from lesion proper show benign appearing
C Antiapoptotic gene is
papillomatosis with hyperkeratosis and acanthosis
bcl-x
infiltrating subjacent tissues. Diagnosis
The most abundant glycoprotein present in
Verrucous carcinomas
C
basement membrane is
C The blood in the vessels normally does not clot
Laminin because
C Good outcome in neuroblastoma is associated with Vascular endothelium is smooth and coated with
Trk A expression glycocalyx.
C Gene defect associated with medulary carcinoma of
thyroid ALL INDIA 2003
Ret proto-oncogene C Correct sequence of cell cycle
C Males who are sexually underdeveloped with Go-G1-S-G2-M
rudimentary testes and prostate glands, sparse C Procedure used as routine technique for karyotyping
pubic and facial hair, long arms and legs and large using light microscopy
hands and feet have chromosomes G-banding
46, XXY C Trisomy 21 in karyotype suggests
C Possible mode of inheritance in a family where Mrs Down’s syndrome
A is hesitant about having children as her two C An 18 years old female with short stature, wide spread
sisters had sons who died from kinky hair disease as nipples and primary amenorrhea has karyotype of
also her mother’s brother 45X
X-linked recessive C Chances of affected and carrier child when an
C Autosomal dominant metabolic disorder is albino girl marries to a normal boy
Familial hypercholesterolemia None affected, all carriers
78 AIIMS and All India PGMEE—Review Questions

C Risk of child with beta thalassemia for a couple with C Peripheral blood smear examination of a 40 years
family history of beta-thalassemia in a distant old male who had undergone splenectomy 20 years
relative, with husband having HbA2 of 4.8% and ago shows
wife having HbA2 of 2.3% Howell-Jolly bodies
0%
C Chromosomal deletion in hereditary retinoblastoma ALL INDIA 2002
13q14
C The epitheloid cells and multinucleated giant cells
C Gluten sensitive enteropathy is most strongly of granulomatous inflammation are derived from
associated with
Monocytes-macrophages
HLA-DQ2
C Host tissue responses seen in acute infections
C Pan T lymphocyte marker are
CD 3
Exudation, margination and vasodilatation
Memory T cells can be identified by
Feature common to both cytotoxic T cells and NK cells
C
C
CD 45RO
Effective against virus infected cells
C True about NK cells
C A myocardial infract showing early granulation
Derived from large granular cells, comprise about tissue has most likely occurred
5% of human peripheral lymphoid cells and they
within 1 week
express IgG Fc receptors.
Findings expected at autopsy in 10 years old boy
MHC class III gene encodes
C
C
dying of acute rheumatic fever
TNF
Aschoff’s nodules, fibrinous pericarditis and
The HLA class III region genes are important
McCallum patch.
C
elements in
C Seen in asbestosis
Governing susceptibility to autoimmune diseases
Calcify pleural plaques, diffuse pulmonary
C True about lactoferrin
interstitial fibrosis and mesothelioma.
Great affinity for iron, present in secondary
C Macrophages containing large quantities of undi-
granules of neutrophil and present in exocrine
gested and partial digested bacteria in intestine are
secretion of body.
seen in
C The primary defect leading to sickle cell anemia is
Whipple’s disease
Replacement of glutamate by valine in β-chain of
The histological features of coeliac disease include
HbA
C

C Raised serum level of lipoprotein A as a predictor of Crypt hyperplasia, increase in intraepithelial


lymphocytes and increase in inflammatory cells in
Atherosclerosis
lamina propria.
C Recurrent ischemic events following thrombolysis
C Liver granulation may be associated with
has been pathophysiologically linked to
Candida, halothane and sarcoidosis
Fibrinopeptide A
C Disease increasing the susceptibility to coronary C Seen in liver in chronic alcoholic
artery disease Chronic hepatitis, cholestatic hepatitis and fatty
Nephrotic syndrome degeneration (Mn: CCF).
C Medullary carcinoma of thyroid is associated with C Crescent formation is characteristic of
MEN II RPGN
C True about Berger’s disease C Necrotizing papillitis may be seen in
Hematuria may be gross or microscopic, on Sickle cell disease, diabetes mellitus and analgesic
immunofluorescence deposits contain both IgA nephropathy.
and IgG and pathological changes are proliferative C Disease or infarction of neurological tissue causes it
and usually confined to mesangial cells, usually to be replaced by
focal and segmental. Neuroglia
Forensic Medicine and Toxicology 79

7
Forensic Medicine and
Toxicology
AIIMS NOVEMBER 2015 C A man throws sulphuric acid on his wife’s face.
True about chemical burns
C A married woman attempts to suicide and is Ulcerated patches are present
admitted in a hospital. The treating doctor gives
Absence of singeing of hairs
false evidence to the police. He can be punished
under Coagulative necrosis.
IPC 193 (Note: Vesicals and blisters are usually absent in
C Boiled lobster syndrome is seen in poisoning with chemical burns)
Boric acid poisoning
C Shoe polish smell AIIMS MAY 2015
Nitrobenzene
C According to criminal amendment act 2013, the age
Autopsy in death due to accident shows two linear
of consent for sexual contact is
C
parietal fractures joining in the midline. Rule to
determine which line appeared first 18 years
Puppe’s rule C Lower two parts of sternum fuse by
(Note: Fracture line appearing later will not cross 14 years
the earlier fracture line) C Postponement of capital punishment in a case of
C Ashley rule is used for pregnant woman comes under Cr. P. C.
Sex determination 416
C Marshal’s triad for blast injury includes C Judicial hanging
Abrasions, bruise and laceration Side of angle of jaw.
C Acids causing coagulative necrosis C A child presents with fatigue and pallor. On
HCl, HNO3 and H2SO4 investigation, microcytic hypochromic anemia was
(Note: HF does not cause coagulative necrosis) found. Serum lead level was also found to be
C Ewing’s postulate is related to elevated. Enzyme level likely to be elevated
Growth at the site following trauma. δ ALA synthatase
C Most common mode of suicide in India C Rat hole tear is found in
Hanging Gunshot injury.
C True about snake bite C Mr X fired gunshot at Mr Y. Mr Y escaped with
Cobra snake is neurotoxic grazing of bullet with thigh which healed but he
Neostigmine can be used in krait bite filed FIR. Mr X is punishable under
Atropine is given before neostigmine. Section 324 IPC
(Note: Polyvalent antisnake venom is NOT effective (Note: Non-grievous hurt by a dangerous wea-
against pit viper) pon)

79
80 AIIMS and All India PGMEE—Review Questions

C A car met with an accident. Driver and front seat C A person is liable for punishment for perjury under
passenger both were lying beside the upturned car. Section 193 IPC
Injuries that will help differentiate between driver C The most important sign of age determination as
and the front seat passenger per Gustafson’s method is
Steering wheel injuries, left shoulder seat belt injury Root transparency
and whiplash injury. C A person of eonism derivs pleasure from
C An orthopedician gave proper instructions to the Wearing clothes of opposite sex.
patient about dressing of his wound. During C Cannabis is the most commonly used illicit drug in
round in the ward, he saw the patient dressing the India. Derived from cannabis
wound with wrong technique but did not say Bhang, ganja and charas
anything to him due to hurry and patient’s wound C The characteristic burnt rope odour is caused by
got infected toxin of
Doctor is guilty under last clear chance doctrine. Cannabis
C After an incised wound, new collagen fibrils are C The active principle of white oleander is
seen along with growing epithelium. The age of the Nerin
wound is C A doctor is treating a patient with viper snake bite.
24–72 hours. Viper venom is
Vasculotoxic (Mn: V for V)
C Fragmentation or segmentation of the blood
columns in the retinal vessels appearing within AIIMS NOVEMBER 2013
minutes after death and persisting for about an
hour C True about methanol poisoning
Kevorkian sign Minimum lethal dose of methanol is 1.25 ml/kg
body weight, formic acid is mainly responsible for
(Note: Rail road sign)
toxicity and methanol causes snow field vision.
C A bomb blast took place in Delhi. 2 persons died
C Disclosing the identity of rape victim is punishable
after sustaining injuries. Following statements can
under IPC
be said to be true about their injuries
Section 228A
Injuries due to burns or air blast, force of explosion
C Lateral traction test is done in unnatural sexual
decrease rapidly and bruise, laceration and
offences in
fractures are triad of main injuries seen.
Habitual passive agent
C According to Delhi Anatomy Act, 1957, a person
AIIMS NOVEMBER 2014 died in road traffic accident, the dead body is said
C If a homicide case comes to the doctor, he/she to be unclaimed after
should inform to the police under Cr. P.C. 48 hours
39 C Imbalming without issuing death certificate is
C Hara-kiri is a form of suicide that consists of cutting punishable under
Abdomen with a sharp weapon. IPC 201
C IPC section dealing with illegal abortion with
C The most common method of parasuicide is
women consent
Drug ingestion
312
C Caliber of a rifle is C Rigor mortis is seen in
Distance between opposite lands Well built male, well built female and old patient >
C On postmortem examination, a person was found 80 years.
to be due to arsenic poisoning. The fatal dose is
120 to 200 mg AIIMS MAY 2013

AIIMS MAY 2014


C The assessment and differentiation of entry and exit
wound is difficult during postmortem examination
C The legal responsibilities of an intoxicated person is of a case of death due to bullet injury due to surgical
given in IPC section alterations. It is known as
85 Kennedy phenomenon
Forensic Medicine and Toxicology 81

C A driver wearing seat belt, suddenly applies brakes C A person found dead with suicidal shot on the right
to avoid a collision. Body part most likely to be injured temple with gun in the right hand and skull burst
Mesentery open. There was charring and cherry red coloration
C A young male patient brought to the emergency in the tract inside
with nausea, vomiting and breathlessness 2 hours It is a contact shot.
after insecticide ingestion. On examination there is C Diagnosis of a man found dead with bluish green
pin point pupil and kerosene like smell emiting frothy discharge at the angle of mouth and nostrils
from the patient. True about the patient Copper poisoning.
Activated charcoal has no proven therapeutic role,
Organ tested in Breslau’s second life test
cholinesterase levels do not have prognostic
C

significance and atropine is the antidote of choice. Stomach and intestine.


C True about methanol poisoning C Last organ to be dissected during autopsy in death
Critical level is 1.25 ml/kg body weight, formic acid due to asphyxia
is mainly responsible for toxicity and methanol Neck
causes snow field vision. C A person breaks someone’s mandible in alleged
C Priapism occurs with fight. Police can
Spanish fly Arrest without warrant.
C Heat stiffening occurs in muscles above temperature
AIIMS NOVEMBER 2012 (°C)
C Assault on a woman by her neighbor led to 60°C
fractures of the middle tooth, injury extending C Active partner in lesbianism is called as
medially to the mouth and contusion to the bilateral Dyke
leg. Nature of injury C According to a recent Supreme Court judgement,
Grievous injury. doctor can be charged for medical negligence under
C The age of the fetus of the aborted product of section 304A, only if
conception of size 2 cm and weight 10 gm Gross negligence
6 weeks C A patient comes with pin point pupil, salivation,
C If a person keeps a man under his power and tremors and red tears. Cholinesterase activity was
threatens him to cause harm without any intention 30% of normal. Probable diagnosis is
to kill comes under IPC section
Organophosphate poisoning
351
C Double based smokeless gun powder consists of
AIIMS NOVEMBER 2011
Nitrocellulose and nitroglycerine.
C A forensic laboratory sample examined under C Palatoprints for identification of human beings are
ultraviolet light is found bluish white color. Most taken from
probable content Anterior part of palate
Semen C Stack’s formula for age estimation from teeth is
C A young known heroine addict was brought uncon- used in
scious with pin point pupils. Treatment of choice Infants
IV naloxone C Issuing or attesting a false medical certificate by a
C Choking is seen in doctor is punishable under
Shotgun Section 197 IPC
C Maximum punishment for medical negligence
AIIMS MAY 2012 under section 304A of IPC
C Choking is seen in 2 years imprisonment.
Shotgun C Punishment of imprisonment to erring doctor
C Primary and secondary markings on a metal bullet according to Human Organ Transplant Act 1994
recovered from the body in a case of murder with 2 to 5 years
gunshot can be used for C Flaying is seen in type of laceration
Identification of weapon Avulsion
82 AIIMS and All India PGMEE—Review Questions

C Hostile witness is punishable under section of IPC AIIMS MAY 2010


Section 191
C A 45 years old chronic alcoholic male patient C Characteristics of heat rupture is
brought to the emergency with an alleged history of Clotted blood vessels
assaulting his neighbor. On examination- irrelevant C Motorcyclist fracture is
talk, not taking alcohol for last 4 days. He is Fracture base of skull into anterior and posterior
diagnosed as delirium tremens. In this condition halves.
He is not responsible for his Act under Section 84 C A child with suspected ingestion and dry mouth ,
IPC. dilated pupil, difficulty in swallowing, delirium,
C During autopsy, most preferred organ for DNA dry and warm skin, the substance is
extraction is Anticholinergic
Spleen C Incisional wound on genital is seen most commonly
C Correctly matched IPC-Murder-300, attempt to in
Murder-307 and culpable homicide not amounting Homicides
to Murder-304. C Destructive power of bullet is determined by
Velocity of bullet
AIIMS MAY 2011
C During autopsy, most commonly used approach AIIMS NOVEMBER 2009
for examination of spinal cord C Section IPC 377 deals with
Posterior Unnatural sex offences.
C Middle aged man with paraesthesia of hands C Antemortem burn differs from postmortem burn
and feet, hyperkeratosis of palms, rain drop by
pigmentation and transverse lines on nails.
Pus in vesicle, vesicle with hyperemic base and
Diagnosis is
inflammatory red line.
Arsenic poisoning
A student suffered from 25% hearing loss in left ear
In civil negligence, onus of proof lies on
C
C
after slapping by teacher but corrected after a
Patient
surgery. Type of injury is
C Condition promoting adipocere formation
Hot and humid Grievous
C According to MCI Act, medical education granted C Dried semen stain in clothes is identified by
by Indian universities comes under UV light
Schedule 1 of MCI Act. C Caustic poison corrodes mucosa because of
It is hygroscopic nature.
AIIMS NOVEMBER 2010 C Hatter’s shake is seen in poisoning of
C Widmark’s formula is used in calculating Mercury
Quantity of alcohol in body.
C Spanish windlass practiced in Spain as a method of AIIMS MAY 2009
execution is a type of C A man working as a pest killer comes to OPD with
Garroting pain abdomen, garlic order in breath and transverse
C Patient with bronchodialation, increased tempera- lines on nails. Most likely the person is having
ture, constipation and tachycardia. Diagnosis is Arsenic poisoning.
Atropine poisoning. C In which part of the body a lesser impact causes
C Rickshaw tyre passed over an 8 years old child. Tyre maximum bruise
marks produced is a type of Face
Imprint abrasion. C Antemortem and postmortem wounds could be
C Aluminium phosphide poisoning, true are differentiated by
Subendocardial infarction, produce phosphine gas Everted margins, blood clots in surrounding tissue
and esophageal stricture. and swollen edges.
Forensic Medicine and Toxicology 83

C Fornication and delusion of persecution occurs C Shape of nulliparous cervical canal is


together in Transverse
Cocaine C Aryl phosphates are
Parathion, paraxon and tik 20.
AIIMS NOVEMBER 2008 C Dialysis is useful in poisoning with
Methyl alcohol, ethylene glycol and barbiturates.
C Study of death in all its aspects is known as
C Hemoperfusion with charcoal is useful in
Thanatology
poisoning with
Hydrocution is
Barbiturate poisoning.
C
Drowning in cold water.
C Falagna is AIIMS NOVEMBER 2006
Beating on soles with blunt object.
C Constituents of a typical embalming solution are C Sexual asphyxia is associated with
Ethanol, formalin and glycerine (Mn: EFG) Masochism
C Preservation of brain is required in
Alkaloid poisoning, phosphorus poisoning and
AIIMS MAY 2008 volatile organic poisoning.
C Sweating is not present in C The knot in judicial hanging is placed at the
Heat stroke Side of the neck.
C Fingerprint bureau was first established in C A person presents with rigors and chill and a
India clinical picture resembling malaria, likely
C Corpus delicti means poisoning is
Essence of crime. Zinc
C Last organ to be dissected during autopsy in
asphyxia death is
AIIMS MAY 2006
Neck
C Irrestible sexual desire in male is known as C A dead body has cadaveric lividity of bluish green
Satyriasis color
Hydrogen sulphide poisoning.
C Autopsy specimen sent for virological examination
AIIMS NOVEMBER 2007
is preserved in
C Not true about consent is 10% formalin
Blanket consent authorizes the attending physician C Fracture ala signature is
or surgeon to do whatever he thinks best for him Depressed fracture
under the circumstances.
C The fingerprint pattern impaired permanently in
C Griveous injuries are
Loss of teeth, loss of hearing of one ear and Leprosy
permanent disfiguration of face. C Priapism occurs in
C Telefono is Cantharide poisoning
Beating of the ears C The ideal place to record body temperature in dead
C In tandem bullet, number of bullet coming out of body is
the gun are Rectum
2
C In autopsy, preservative used for vitreous is
AIIMS NOVEMBER 2005
Fluoride
C La facies sympathique is seen in
AIIMS MAY 2007 Hanging
C Spalding sign is seen in C Vagitus uterinus is seen in
Maceration Cry of unborn baby from uterus.
84 AIIMS and All India PGMEE—Review Questions

C Gunshot residue on hand can be detected by AIIMS MAY 2004


Dermal nitrate test.
C In freshwater drowning the death occurs within
Dead body has markings like branching of a tree
4–5 minutes of submission due to ventricular
C
found on chest. Cause of death
fibrillation. Cause is
Lightening injury Hemodilution, overloading of heart and hemolysis
C Dialysable poison is resulting in release of potassium.
Barbiturates, methanol and ethylene oxide. C A 25 years female was found in room with 100%
burns on her body. Findings that establish that the
AIIMS MAY 2005 burns were antemortem in nature
Carboxyhemoglobin (25%) and soot particles in
C Diffusion of oxygen at the tissue level is affected in trachea.
CO, phosgene and cyanide poisoning. C Suspended animation may be seen in
C Patterned abrasion is variety of Drowning
Pressure abrasion. C Unnatural death of a female within 7 years of
C Postmortem caloricity is seen in marriage. Inquest done by
Tetanus, strychnine poisoning and septicemia. Subdivisional magistrate.
C When a group of muscles of a dead body in state of C Cheiloscopy is the study of
strong contraction immediately prior to death and Prints of lips
remaining so even after death is termed as
Cadaveric spasm AIIMS NOVEMBER 2003
C Tissue putrefaction late in
AIIMS NOVEMBER 2004 Prostate
C Rule of Hasse
True about blood grouping
To determine age of fetus in uterus.
C
Used to resolve confusion of identity in alleged C Empty cartridge case is ejected after firing
exchange of babies in maternity unit, assist in
Pistol
making fragmented human remains in mass
disaster and help to show whether blood stain on
AIIMS MAY 2003
the weapon belongs to the suspected victim.
C True regarding dead body of a murdered person C Species identification by
brought for postmortem in mortuary Precipitin test.
Stored at 4°C on average, never undressed before C A dead born fetus have
the forensic doctor has seen it and can be stored at Rigor mortis at birth, maceration and mummification.
20°C to preserve it for long duration.
C When a person has suspended himself by applying AIIMS NOVEMBER 2002
ligature around neck so that the point of suspension C Hypostasis of red brown or deep blue in color is
(knot) is situated in the region of the occiput suggestive of poisoning due to
Typical Nitrates
C An 11 years old rape victim brought to casuality for C False about consent
medical examination. Medical officer required to do For artificial insemination consent of the patient
The patient should be examined in presence of a alone is required.
female attendant, she should be given necessary C Thanatology is the science that deals with
treatment and all the necessary forensic samples Death in all its respects.
should be collected. C A 5 years old child presenting with confusion,
C Autopsy report shows dark brown postmortem increased salivation, lacrimation, fasciculations,
staining in stomach and garlic odour. It is due to miosis, tachycardia and hypertension. It is a case of
Phosphorus poisoning Organophosphorus poisoning.
Forensic Medicine and Toxicology 85

C Certain delegation on the part of doctor under- C The anteroposterior diameter of the skull is minimum
taking postmortem examination in
Meticulous and complete examination, routinely Brachycephaly
recording all positive findings and important
negative findings, and preserving viscera and ALL INDIA 2011
sending for toxicological examination in poisoning.
C Sparrow footmark is seen in
Windshield injury
AIIMS MAY 2002
C Gunpowder and/or soot on blood stained garments
C Sexual asphyxia is associated with can be visualized by
Masochism Infrared rays
C A bullet fired from a gun not released and ejected C Rickshaw tyre passed over the body of an 8 years
out with the subsequent shot is known as old child leaving distinct markings of the tyre
Tandem bullet treads. It is example of
C A patient presenting with colic and constipation, Patterned abrasion
wrist drop and encephalopathy with basophilic C Fracture ala signature or signature fracture is
stippling of RBC in blood Depressed fracture of skull.
Lead poisoning (Mn: Lead poisoing-ABCDE) C A child is brought to the casuality with reports of
C Unknown patient presents with pyrexia, violent shaking by parents. Most likely injury is
constricted pupils, hypotension, cyanosis and Subdural hematoma.
stuper progressing to coma. C Dental numbering is done by
Poisoning is due to phenobarbitone. FDI has two digit system, anatomic and diagrammatic
charting and palmar notation.
C Illuminous, translucent and waxy poison is
ALL INDIA 2012
Yellow phosphorous.
C The Japanese detergent suicide technique involves C Patient with bluish discoloration of conjunctiva,
mixing of household chemicals to produce mucous membranes and nails one hour after
H2S and other poisonous gases ingestion of a poison. Examination reveals tachy-
C Degree of rupture of hymen and whether the cardia and hypotension. Poison likely is
rupture is recent or old in a case of rape is determi- Mercury
ned by C Commonly used as a rave drug
Glastair Keene rod Ecstasy
C For MTP, consent is taken from C Organs/tissues presently being used for transplant
Wife only Blood vessels, lung and liver.
C A man on provocation hits another person with a
ALL INDIA 2010
wooden stick on the arm causing a bruise of size
3 × 3 cm. he will be punished by C Primary impact injury is commonly seen on
Imprisonment of one month or with fine which may Legs
extend to 500 ` or both. C Bullet leaving a visible mark so that a person can see it
C Old blood stains can be best detected by Tracer bullet
Luminal spray C Lightening flash can cause injury by
C The identity of rape victim is not disclosed under IPC Direct effect of electric, superheated air and
228A expanded and repelled air.
C A sodomy accused person brought to you, C Death of woman within 5 years of marriage under
confesses that he is an active agent regularly suspicious circumstances. Her parents complained
practicing it. Findings of medical examination are that her in laws used to demand for dowry. Under
Faecal smell, tear of frenulum and relative constric- which section, magistrate authorize autopsy
tion of the shaft of penis. Section 176 CrPC.
86 AIIMS and All India PGMEE—Review Questions

C A factory worker with excessive salivation, blue C Mechanism of induction of labor with use of
line on gums, tremors, disturbed personality, abortion stick is
insomnia and loss of appetite. Likely poisoning is Stimulation of uterine contraction.
Mercury C False negative hydrostatic test in a liveborn fetus
C Acute poisoning causes may be seen in
Hypersalivation, tingling and numbness and chest Atelectasis
pain. C Corporo basal index is useful for determination of
C Drugs used for narcoanalysis Sex
Scopolamine C Example of polychlorinated hydrocarbons
Endrin
ALL INDIA 2009 C Yellow fatty liver is characteristic of poisoning with
Phosphorus
C Privileged communication is made between
C Heavy metal poisoning that may cause colitis and
Doctor and concerned authority.
resembles diphtheritic colitis is
Disease which permanently alters the fingerprints is
Mercury
C
Leprosy C Alkalinization of urine may be done in cases of
C Casper’s dictum is poisoning with
Estimation of time since death. Barbiturates
C Antemortem burns can confused with
Ant bite marks ALL INDIA 2007
Extensive abrasions are found all over the body of a
Burtonian line is seen in poisoning of
C
C
pedestrian lying by the roadside. Likely cause is
Lead
Secondary injury. C Acrodynia is associated with
C Stellate wounds may be seen in following bullet Mercury
entry wounds C Mercury affects part of renal tubule
Contact shot PCT
C Commonest organ to be injured in primary blast C The drug of choice for mushroom poisoning is
injuries is Atropine
Lung C Aryl phosphates are
C Characteristic of heat rupture is Follidol, parathion and tik 20.
Irregular margins
C Following may cause traumatic asphyxia ALL INDIA 2006
Railway accident, road traffic accident and stampede C Spalding sign occur after
in crowd. Death of fetus in uterus.
C Scab or crust of abrasion becomes brown
ALL INDIA 2008 Between 2 and 3 days.
C Cause of death due to suffocation are C Tests used to detect semen
Choking, gagging and smothering. Barberio’s test
C Fracture of the hyoid bone, thyroid cartilage and C Medical degrees awarded by institutes outside
cricoid cartilage along with extensive bruising of India and recognized by MCI are registered in
neck muscles is suggestive of Second schedule of Indian Medical Council Act.
Fingerprint bureau was first established in
Manual strangulation.
C

C Fracture of hyoid bone and thyroid cartilage along India


with neck muscle bruising is suggestive of
ALL INDIA 2005
Strangulation
C Test done for detecting vaginal cells from the C In India magistrate inquest is done in
accused in a case of rape is Dowry death, death in police custody and
Lugol’s iodine exhumation cases.
Forensic Medicine and Toxicology 87

C Minimum age at which an individual is responsible C Postmortem caloricity is seen in death due to
for his Criminal Act is 7 years. Septicemia
(Note: According to IPC) C Conter-coup injuries are seen in
C The most reliable method of identification of an Brain
individual is C Disputed maternity can be solved by
Dactylography Blood grouping, HLA typing and DNA finger
C The most comman pattern of fingerprint is printing.
Loop (Mn: LWAC-in decreasing order) C Hydrogen peroxide is used in chemical tests for
C Most reliable criteria in Gustafson method of blood
identification is Benzidine tests, phenolphthalein test
Transparency of root. C A person brought by police from the roadside lying
C True about diatoms is talking irrelevant with dry mouth, hot skin,
Diatoms are aquatic unicellular plants, acid dilated pupil, staggering gait and slurred speech.
digestion technique is used to extract diatoms Diagnosis
and presence of diatoms in the bone marrow is Dhatura poisoning.
an indication of-antemortem inhalation of C A middle aged man presents with paresthesia of
water. hands and feet. On examination, presence of Mee’s
C In a firearm injury, there is burning, blackening, lines on nails and rain drop pigmentation on hands,
tattooing around the wound, along with changed poison causing it is
color of the surrounding tissues, the injury is Arsenic
Close shot injury. C Most reliable method for detecting blood alcohol
C CNS depression, cardiac depression and optic level
nerve atrophy in methyl alcohol poisoning is Gas liquid chromatography.
produced due to C A 39 years old carpenter has taken two bottles of
Formaldehyde and formic acid. liquor from the local shop. After about an hour, he
C In chronic arsenic poisoning, sample sent for lab develops confusion, vomiting and blurring of
examination are vision. He should be given
Nail clippings, hair samples and bone biopsy. Ethyl alcohol
C At autopsy, feature of cyanide poisoning are C Heroin addict labourer was found unconscious. On
Congested organs, erosion and hemorrhage in examination, tachycardia, shallow breathing and
esophagus and stomach and skin may be pinkish or constricted pupils, BP-100/70 mm Hg, brisk
cherry red in color. bilateral deep tendon reflexes and plantar reflex
flexor on both sides. Best treatment given to him can
ALL INDIA 2004 be
C A convict with no known relation and no biological Naloxone
sample escaped from jail. Dead body resembling
ALL INDIA 2003
the convict but with mutilated face was found. The
identity can be established by C Mummification refers to
Anthropometry Desiccation of a dead body.
C Cephalic index of Indian people are between C Cobra snake venom is
70–75 Neurotoxic
C Following injury in right eye, a person developed C Rules related to legal responsibility of an insane
corneal opacity. Vision was restored following person
surgery. Medicolegal injury is McNaughton’s rule, Durham’s rule and Curren’s rule.
Grievous C In a suspected case of death due to poisoning where
C Poison causing deep blue color of hypostasis after cadaveric rigidity is lasting longer than usual, it
death may be a case of poisoning of
Aniline dyes Arsenic
88 AIIMS and All India PGMEE—Review Questions

C Blackening and tattoing of skin and clothing can be C A dead body with copious fine leathery forth in
best demonstrated by mouth and nostrils with increased pressure over
Infrared photography chest. Death was likely due to
C Postmortem lividity develops in Drowning
Drowning in well, postmortem submersion and C Entry wound blackening in firearm injury is due
drowning and drowning in chlorinated swimming to
pool. Smoke
C Postmortem caloricity is seen in C Tentative cut is a feature of
Heat stroke, pontine hemorrhage and septicemia. Suicidal attempt
C Prenatal diagnostic technique 1994, ground for
C Gastric lavage is indicated in all cases of acute
carrying out prenatal test are
poisoning, ideally because of
Pregnant women above age of 35, history of two or
Fear of aspiration
more spontaneous abortion or fetal loss and history
of exposure to potentially teratogenic drugs. C Tests for detection of heavy metal
C Punishment for perjury (Giving wilful false Harrison and Gilroy test, neutron activation
evidence by a witness while under oath) may analysis and atomic adsorption spectroscopy.
extend to C A 60 years old man with opium addiction has given
Imprisonment up to 7 years and comes under IPC up opium for last 2 days. Withdrawal symptom
193. Rhinorrhoea
C The sensations of creeping bug under the body is
ALL INDIA 2002 found in
C Race of individual with skull bone features- Cocaine poisoning
rounded nasal opening, horse shoe shaped palate, (Note: Tactile hallucination)
round orbit and cephalic index above 80 C Linseed plant ingestion causes cattle poisoning of
Mangol Hydrocyanic acid
C A sample to look for uric aid crystal (gouty typhus) C A 10 years old child presents with snake bite since
is submerged in six hours. No systemic signs are found. Lab
Alcohol investigation is normal. There is localized swelling
C Features of brain death over the leg of < 5 cm. Next done is
Complete apnea, absent pupillary reflex and heart Observe the patient for progression of symptoms
rate unresponsive to atropine. and wait for antivenom therapy.
Preventive and Social Medicine/Community Medicine 89

8
Preventive and Social
Medicine/Community Medicine
AIIMS NOVEMBER 2015 C An epidemiologist selected all possible samples
from a population and plotted the means of each
C True about Rashtriya Swasthya Bima Yojna sample on a linear graph. The graph is known as
Cashless benefits in hospitals. Sampling distribution
C Trivalent flue vaccine contains- C According to WHO 2013 malaria treatment
H1N1, H3N2 and Influenza B. guidelines, true statement is
(Note: H1N1 and H3N2 are Infuenza A viruses.) Primaquine is contraindicated in infants and
C Recently approved latex agglutination test shows pregnancy.
following results when compared to previous tests C A patient of diabetes and hypertension comes to the
Test result (+) Test result (–) clinic. Best tool to demonstrate the complications
Diseased 27(TP) 3(FN) and explain him the risks of various complications
Non diseased 5(FP) 95(TN) Venn diagram
Sensitivity and specificity respectively C Temperature for a medicine with a label of ‘store at
90% and 95% a cool place only’
(Note: Sensitivity = TP/TP + FN × 100 8–15°C
Specificity = TN/TN + FP × 100) C True about randomization in a clinical trial
C Free transportation of a 15 years old neonate Reduces confounding
suffering from cough, sneezing and fever comes Ensures comparability of 2 groups
under Decreases selection bias.
Janani Sishu Suraksha Karyakram (JSSK) (Note: External validity can not be ensured by
C Sensitivity means True positives. randomization.)
(Note: Specificity is True negative. Mn: P for N. N C DBP of 125 female patient was noted. Mean = 70
for P) mmHg and SD= 10 mmHg. 5th percentile value
C A method decreasing the mortality of disease but 50.4
not curing it (Note: Mean +/– 1 SD covers 68.4%
Prevalence will increase. Mean +/– 2 SD covers 95.3%
(Note: Prevalence = Incidence × Duraqtion of disease) Mean +/– 3 SD covers 99.7%)
C Best test for the association between exposure and
outcome
AIIMS MAY 2015
Cohort study
C Investigation for latent TB to be done according to C Sensitivity is
WHO in True positives
Patient on TNFα inhibitor, patient on silicosis and C Interpretation of tuberculin test becomes difficult in
patient on dialysis. High BCG coverage area.

89
90 AIIMS and All India PGMEE—Review Questions

C Data is collected and analysed. By mistake, highest more likely to do regular exercise. What type of bias
value is written as 85 which is actually 58. What is likely in this study
change will occur Selection bias
Mean will increase but the median will remain the C True about Japanese encephalitis vaccine are
same. Not given in less than 6 months of age, two primary
C HDI includes doses should be given and protection lasts for 3
Life expectancy at birth, knowledge and decent years.
standard of living.
C Test used for comparing birth weight measurements AIIMS NOVEMBER 2014
of babies of mothers belonging to two group in
which mother of one group received supplemental C Population attributable risk is
diet while mother in other group did not receive Incidence of disease/death in the total population
supplemental diet to those who were not exposed in relation to the
total population.
Student’s T test.
C If the association between smoking and CHD
C DBP of 200 people has been measured and arranged.
increases from low level of physical exercise to high
1st quartile 85, 3rd quartile 110. How many people
level of physical exercise, then physical exercise in
have DBP above 110–50.
relation to the association between smoking and
C Haddon matrix can be used for
CHD is known as
Prevention of injuries and accidents.
Effect modifier
C Nikshay is the software prepared by the government
The objective criteria for detecting community
for
C
spread of pandemic influenza A H1N1 are
Surveillance of TB cases.
One or more cases should be laboratory confirmed
C Food safety and standard authority of India comes
with pandemic influenza A H1N1, cases should be
under
reported over 2 weeks and presence of at least 25
Ministry of Health and Family Welfare.
linked epidemiologically suspect cases.
C Immunization status of children is assessed by
C School closure is a common strategy to keep
District level household survey
pandemics of influenza under control in the
C A child is bitten by the dog unprovoked. Dog is
absence of vaccines in developing countries. This
caught and appears healthy. What should be done
rationale is based on
Start post-exposure prophylaxis and observe the Children are the most susceptible age group,
dog. children are the primary transmitters of influenza
C In a community of 100 people, average of GFR is and contact rates are particularly high in school
measured to be 85 ml/min with standard deviation context.
25, what is 90% confidence range of GFR C Form of vaccine derived polio virus (VDPV)
81–89 iVDPVs, aVDPV and cVDPVs
C A cigarette smoker thinks that he will not have C True about Chandler’s index
cancer as he takes less number of cigarettes and eats Calculates the average number of hookworm eggs
healthy diet and does regular exercise. This theory per gram of faeces for the entire community,
is called as indicates magnitude of public health problem and
Self exclusion indicates effectiveness of mass treatment.
C Over few years, trend is changing from communi- C Under RNTCP, test used for diagnosis of a TB case
cable diseases to noncommunicable diseases in a Sputum staining
community. What type of trend is noted C Strategy employed by the NPCB to evaluate post-
Secular trend operative visual outcomes following cataract
C An epidemiologist is conducting a Cohort study. surgery extraction
He selects a group that includes non-smokers and Passive surveillance
another group of smokers. But he thinks that non C The outcome indicator for NPCB is
smoker persons may be more health conscious and Number of cataract surgeries with restored vision.
Preventive and Social Medicine/Community Medicine 91

C The Chairman of District Blindness Control Society C True about the calculation of HDI
is The minimum value for combined gross enrolment
District Collector ratio is fixed at 0%, the minimum value for adult
C True about cataract surgical rate (CSR) literacy rate is fixed at 0 and the maximum value for
It is the number of cataract surgeries performed per per capita income is fixed at 40,000.
million population per year, a CSR of 3000 is C True about education in HDI
recommended under vision 2020 and it indicates Country’s achievement is computed on the basis of
the effectiveness of cataract services delivery. adult literacy.
C True about Rashtriya Swasthya Bima Yojna(RSBY) C Immune thrombocytopenic purpura is observed as
Only below poverty line families to be covered a complication with vaccine
under RSBY. MMR
C A medical officer wants to estimate the number of C Chemoprophylaxis is indicated for
children in a village. He can get a rough estimate Cholera, meningococcal meningitis and conjuncti-
quickly from vitis.
Adult Folifer consumption in the last 1 year. C True about INH chemoprophylaxis
C The nutritional supplementation recommended It is not feasible to apply on a large scale, it has risk
under ICDS for pregnant woman is of drug induced hepatitis and it is not highly
600 calories and 18–20 grams of protein per day. clinically effective.
C The most effective antilarval measure for control of C Positive predictive value is a function of sensitivity,
urban malaria is specificity and
Filling up of all ditches and cess pools. Prevalence
C True about inertization C True about period of isolation
Waste is mixed with cement, it is relatively inexpensive Chicken pox-6 days after onset of rash, measles-3
and used for disposal of pharmaceutical waste. days after onset of rash and Herpes zoster- 6 days
C Objectives of the National Mental Health Programme after onset of rash.
Availability and accessibility for minimum health C True about OPV
services to all, promote community participation Useful in epidemics, rapid antibody response and
in mental health service development and protective even in presence of maternal antibo-
application of mental health knowledge to general dies.
health care. C HIV sentinel surveillance provides data
C True about Employees State Insurance Act 1948 To monitor disease trends.
The beneficiaries are entitled to funeral expenses up C A boy complained of unprovoked bite by a local
to ` 5000. dog in the community. The dog was later caught by
C In a sampling, from a given sampling frame, every the local animal authorities and appeared to be
10th person is selected starting from a random healthy. The most appropriate action
person. This method of sampling is called as Give post-exposure prophylaxis to the bitten
Systemic sampling person with cell culture derived vaccine.
C The measure of variation used to compare two data C Evidence of decreased risk of cardiovascular
with different scales of measurement is disease is associated with
Coefficient of variation
Low to moderate daily alcohol consumption,
regular physical activity and potassium.
AIIMS MAY 2014
C Under the school vision screening programme,
C The prospectively evaluated, double-blinded, vision screening in schools is done by
randomized clinical trial represents the gold Teacher
standard for providing evidence for therapeutic C Under NRHM, an ASHA will receive financial
decision making. This was first proposed by the renumeration for
father of evidence based medicine Institutional deliveries, 1st dose of DPT and OPV,
Sackett registration of birth.
92 AIIMS and All India PGMEE—Review Questions

C The provision under Janani Shishu Suraksha C True about roll back malaria
Karyakram include Insecticide treated mosquito nets, strengthening
Free food to the mother in the hospital, free health system and training of health workers.
transport to the mother and baby to/from hospital C Recommended daily intake of iodine in pregnancy
and free treatment of the neonate and infant in all 250 μg
public health facilities. C To evaluate post-operative vision effects after
C Calculate the neonatal mortality rate for a cataract extraction surgery under NPCB, what is
population with number of neonatal deaths-450, used
number of stillbirths-212, total number of live Routine check up of all operated cases.
births-12, 450 in the year 2012. C Common causes of epidemic after a disaster
36 Leptospirosis, Rickettsia and acute respiratory
C Infant mortality rate includes infections.
Early neonatal death, late neonatal death and post- C GDP proportion spent on public health in India
neonatal death. 0.012
C A girl is having white hair. 40% of her classmates C Under 5 mortality rate in the world by 2010
are also suffering from the same problem while 8 million
none of the other in the village is suffering from it C According to WHO classification, highly toxic
and her brother living in neighboring village with a insecticide are coded as
aunt is also normal. Which explains the probable Red
cause of her white hair C True about screw feed technology
Environmental Reduce weight by 30%, reduce volume by 80% and
C Severe acute malnutrition (SAM) in India is non-burn heat sterilization technique.
Very low weight for height (below-3Z scores of the C Vaccines given according to national immunization
mean WHO standards for growth), by visible schedule at 5 years of age
severe wasting, or presence of nutritional edema.
DT booster
C The most sensitive indicator of environmental
C Measles vaccination to all 9 months to 14 years
iodine deficiency is
children is part of WHO strategy
Urine iodine excretion
Catch up
C Which can cause epidemic in post-disaster period
Leptospirosis, rickettsiosis and acute respiratory C True about ESI act
infection. State govt’s share of expenditure on medical care is
C The hemoglobin levels of pregnant females in a 1/8, ESI’s share is 7/8 of total cost.
community was found to have a mean of 10.6 g/dl, C Area under normal curve with 1 standard deviation
and a standard deviation of 2 g/dl. What is 0.34
minimum Hb level below which 5% of the pregnant C In a study of 2000 population, mean value of
women would have their Hb levels hemoglobin is 13.5 gm, which follows normal
6.68 distribution curve. Percentage of people having
C Universal health coverage in India was recommen- higher hemoglobin than 13.5 gm is
ded by 50%
High level expert committee C Statistical Q test is used for
C Provisions included in the primary health care To determine outliers
according to the Alma Ata declaration
C In assessing the literacy, the parameter used is
Adequate supply of safe drinking water, provision
Age above 7 years
of food supply and basic sanitation.
C True about folic acid
Present in all grean leafy vegetables, proven to
AIIMS NOVEMBER 2013
decrease the occurrence of neural tube defects when
C True about Rashtriya Swasthya Bima Yojna taken preconceptionally and methyl folate trap is
Applies to BPL only because of methionine synthase defect.
Preventive and Social Medicine/Community Medicine 93

C True about polio C Human poverty index includes


Last case of wild polio virus was found on January Probability at birth of not surviving to age 40,
13, 2011 percentage of underweight children and percen-
tage of people without sustainable access to an
AIIMS MAY 2013 improved water source.
C The prevalence of Candida glabrata infection is C In the integrated disease surveillance project (IDSP)
found to be 80% in a diabetic sample of 100. If the diseases included
test is repeated for 95% confidence limits, then the Acute respiratory infection, snake bite and
range of prevalence for the Candida glabrata infection leptospirosis.
72–88 C Polyvalent meningococcal vaccine is routinely
C The prevalence of a disease in a population is recommended to
expected to be 50%. An investigator is interested in Laboratory workers
assessing the prevalence in the range of 45 to 55% C The forth coming new Mental Health Act in India is
with 95% confidence limit. The minimum sample named as
size required for the study is Mental health care bill
400 C The number of millennium development goals
C True about National Programme for Prevention directly related to health is
and Control of Cancer, Diabetes, CVD and Stroke 3
(NPCDCS) C The study regarding the brain and behavioural
Cardiac care and cancer care facility is to be changes in normal and damaged brain is known as
established at the district level hospital. Neurodevelopmental psychology
C Social pathology describes
C About deworming under national child health
Changes in the natural history of the disease due to
programme, true are
social factors.
It is done for all preschool and school children
C Under vision 2020 the number of vision centres to
yearly, albendazole 400 mg for > 2 years and it is
be setup is
linked with vitamin A supplementation programme.
20,000
C Under NRHM, an ASHA will receive financial C Recommended oral dose of vitamin A in post-
renumeration for the following partum females is
Registration of births, 1st dose of DPT and OPV, 2,00,000 IU
and institutional deliveries. C True regarding the period of isolation/quarantine
C The estimation of prevalence of blindness by blind Chickenpox- 6 days after the onset of rash
school survey in place of population based survey Measles- 3 days after the onset of rash
will lead to Herpes zoster- 6 days after the onset of rash.
Under estimation
C When assessing the efficacy of a newly developed AIIMS NOVEMBER 2012
drug in comparison to placebo, the 95% confidence
interval is used to extract C True about OPV
Both efficacy and non-efficacy of the drug. Useful in epidemics, rapid antibody response and
protective even in presence of maternal antibodies.
C Growth monitoring by anganwadi workers under
C Highest mean and lowest mode indicates
ICDS programme is based on which guidelines
Positive skewing
MGRS standards C Population of district-10 lakh. Under 16 years of
C According to WHO definition, the criteria for age-30%. Prevalence of blindness-0.8/1000 of
considering high endemic area for meningococcal under 16 population. Total number of blind under
meningitis 16 years of age
> 10/1,00,000 population. 240
C The 3 years bachelor of rural health care (BHRC) C If the prevalence is very low as compared to the
was recommended by incidence for a disease. It implies
High level expert group on universal health coverage. Disease is very fatal and/or easily curable.
94 AIIMS and All India PGMEE—Review Questions

C Most common cause of neonatal mortality in India C True about roll back malaria
Prematurity Develop new insecticides, strengthen health
C True regarding tetanus system, train health workers and insecticide treated
Hard immunity is not of much value, incubation bed nets.
period is 6–10 days and the main reservoir is soil C Screening age for trachoma
and intestine of animals and humans. 1–9 years
C New cancer patient in India reported annually
1 million AIIMS MAY 2012
C In a population of 200 people with normal distribu-
tion, people included in 1 standard deviation C In WHO road to health chart, upper and lower limit
136 represents
C True about Rashtriya Swasthya Bima Yojna (RSBY) 50 percentile for boys and 3 percentile for girls.
Applies to BPL families only C In 13–15 years female child, recommended daily
protein intake is
C HIV sentinel surveillance is for
1
Detecting trend of the disease
C Common to both acute and chronic malnutrition
C Mass chemoprophylaxis is given for is
Lymphatic filariasis, plague and vitamin deficiency. Weight for age
C The pattern of change of disease trends of mortality C Rashtriya Swasthya Bima Yojana is
and morbidity where the pandemics of infection are Government run insurance scheme for all poor.
replaced by degenerative and man made disorders
C In vision 2020, the target for secondary service
as the main cause of morbidity and mortality is
centre is for how much population
known as
5 lakh
Epidemiological transition.
C Leprosy is not yet eradicated because
C Current percentage of GDP of India spent on health
No effective vaccine
1.2%
C Which of the following is best suited for the role of
True about Millenium development goals
social worker
C
Reduce by 2/3rd the under 5 mortality rate by year
Health professional involved in coping strate-
1990–2015.
gies, interpersonal skills and adjustment with
C In a study to assess the prevalence of dating among family.
adolescents, schools were selected at random.
C In acute flaccid paralysis surveillance, evaluation
Among the schools, sections were selected at
for residual paralysis is done at
random and among the sections students were
selected at random. This is an example of 60 days
Multistage sampling C Bias can be eliminated by all except
Multivariate analysis
C In a population of 5000, 500 are myopic before 1st
January 2011 and number of new cases till 31st C Food with maximum cholesterol content
December 2011 is 90. Incidence is Egg
2% C Not helpful for elimination of filariasis
C According to blindness assessment, a person with a Larvae are deposited on skin surface where they
best corrected vision of 4/60 in both eyes is having cannot survive.
a disability of C Applications of incubation period
75% To differentiate co-primary cases from secondary
C True about national programme for prevention and cases, to find out time for quarantine and to
control of cancer, diabetes, CVD and stroke prevent infection to the contacts of the infected
(NPCDCS) person.
Cardiac care unit and cancer care facility is to be C True about Indian reference male
established at the district level hospital. Age 18–29 years.
Preventive and Social Medicine/Community Medicine 95

C Natural disaster causing maximum deaths C True about BCG vaccination


Hydrological WHO recommends Danish 1331 strain for vaccine
C According to the Workman’s Compensation Act, production.
1923, which is occupational disease C Fish is deficient in nutrient
Anthrax Iron
C In vision 2020, ophthalmic personnel per population
C True about cluster sampling
ratio is
50,000 Two stage sampling, cheaper and disadvantage of
C A sexually active, long distance truck driver’s wife higher sampling error.
comes with vaginal discharge. Under syndromic C True about breast milk is
approach, drugs given Coefficient of iron absorption is 70%, calcium
Metronidazole, azithromycin and fluconazole. utilization from breast milk is more than cow’s
milk, breast milk protein is reference protein.
AIIMS NOVEMBER 2011 C True about randomized controlled trial
Baseline characteristics are comparable, bias can be
C Founding members of vision 2020
eliminated by double blinding and sample size
WHO, ORBIS and international agency for
depends upon type of study.
prevention of blindness.
C Incidence can be calculated by
C Factors affecting equilibrium in Hardy-Weinberg
Prospective study.
law
C True about purification of water
Gene flow migration, gene mutation and small
Coliforms must not be detectable in any 100 ml
population.
sample of drinking water, sodium thiosulphate is
Fatty acid characteristically present in breast milk
used to neutralize chlorine and coliforms may be
C
and essential for body growth
detected by multiple tube method and indole
Docosahexaenoic acid. production.
C False regarding principles of evidence based C In India, most common cause for maternal mortality
medicine is
EBM depends on clinical model and decision Hemorrhage
analysis to base its recommendation. C True about net protein utilization
It is the ratio of total nitrogen retained by total
AIIMS MAY 2011 nitrogen intake multiplied by 100.
C A 30 years old female patient has sputum positive
TB. Her child is 3 years old. Recommended AIIMS NOVEMBER 2010
chemoprophylaxis is
C Neonatal mortality rate is
INH 5 mg/kg for 6 months
No. of neonatal deaths per thousand live births.
C Late expanding stage of population of India is due to
Cause of neonatal mortality in India
Death declines more than birth rate
C

C True about vaccines Prematurity > infection > birth asphyxia > congenital
anamolies (Mn: PIBC)
Thiomersal is preservative in DPT vaccine,
magnesium chloride as stabilizer is used in OPV C Smallpox eradication was due to
and kanamycin is preservative in measles. Highly effective vaccine, subclinical infection do
C Larval control measures not transmit the disease and life long immunity.
Paris green, Gambusia and intermittent irrigation. C Sputum can be disinfected by
C True about DDT Autoclaving, boiling and cresol.
Pyrethrum has synergistic action, it is a contact C True regarding Clostridium tetani
poison and residual effect lasts for 18 months. The main reservoir is soil and intestine of animals
C Incineration is done for and humans, herd immunity is of not much
Cytotoxic drugs, cotton contaminated with blood importance and the main mode of transmission is
and human tissue. through trauma and contaminated wound.
96 AIIMS and All India PGMEE—Review Questions

C True about Varicella infection C Vitamin A prophylaxis to a child is


SAR is 90%, lesions occur on flexor surfaces and Specific protection
reactivation occur in 10–30% cases. C Savlon contains
C True about Vi polysaccharide vaccine is Cetrimide and chlorhexidine.
Can be administered with yellow fever and C True about crude birth rate
hepatitis A vaccine. It is a measure of fertility, it is independent of age of
C According to a study, incidence of colon carcinoma population and numerator does not include still
is less in pure vegetarians than non-vegetarians so births.
concluded that beta carotene is protective against C A test has high false positive rate in a community.
cancer. This may not be true as vegetarians may be True is
consuming high fibre diet which is protective Low prevalence
against cancer. This is an example of C Weight of Indian reference man in kg
Confounding factor 60
C Mass chemoprophylaxis is given for C Chlorine demand of water is measured by
Lymphatic filariasis, plague and vitamin A Horrock’s apparatus
deficiency. C Included in vision 2020-right to sight immediate goals
C True regarding Spaulding’s criteria Trachoma, cataract and onchocerciasis.
Non-critical items require only decontamination, C True about lepromin test
semicritical items are those which come in contact
Negative in most children in first 6 months of life,
with mucous membrane or non-intact skin and
important aid to classify type of leprosy disease and
cardiac catheters are examples of critical items.
BCG vaccination may convert lepra reaction from
C Bias can be eliminated by negative to positive.
Matching, blinding and randomization. C Diseases transmitted by lice are
C Diseases spread by hard tick are Epidemic typhus, relapsing fever and trench fever.
KFD, tularemia and tick paralysis. C Chikungunya is transmitted by
C In post-operative ward of ICU, 5 patients developed Aedes
infection in the same wound. The best way to C In a study, interpretations obtained are very
prevent MRSA outbreak in indoor patients satisfied, satisfied and dissatisfied. Type of scale is
Practice proper meticulous handwashing. Ordinal
C A person found some correlation between dietary C Treatment of choice for sputum positive pulmonary
factors and a disease by collecting data from food TB detected in first trimester of pregnancy is
manufactures and hospitals, respectively, such a
Start category 1 immediately
study is
C Safe disposal of mercury is
Ecological
Collect carefully and recycle
C You have diagnosed a patient clinically as having
AIIMS MAY 2010 SLE and ordered 6 tests out of which 4 has come
C Leprosy involves positive and 2 negative. To determine the probability
of SLE at this point, you need to know
Nerve, eye and ovary
Prior probability of SLE, sensitivity and specificity
C True regarding global warming is of each test of each test.
CO2 is the major greenhouse gas. C Steps done for
C True about Aedes Surveillance of risk factors of non-communicable
Bites repeatedly, transmits dengue and it takes 7–8 disease.
days to develop the parasite and transmit the C The best graphic representation of frequency
disease. distribution of data gathered of a continuous
C Bacterial indicators of faecal pollution variable is
E. coli, Clostridium perfringens and Streptococcus. Histogram
Preventive and Social Medicine/Community Medicine 97

C Vaccine with maximum efficacy C A temporary provisional view held by the people
Measles on a point of view is
C Arbovirus true is Opinion
Chikungunya is transmitted by Aedes aegypti. C Most common mode of lead poisoning
Inhalation
AIIMS NOVEMBER 2009
C Specificity of a test refers to its ability to detect
True negatives
C Effective leprosy control programme may be C Prevalence of cataract at one point of time can be
indicated by determined by
Decreasing grade 2 disability, low MDR resistant, Cross sectional study
multibacillary cases and high new case detection C License to blood bank is given by
rate.
Drug controller general of India
The vaccines that can be given during pregnancy
Association can be measured by
C
C
is
Correlation coefficient, P value and odds ratio.
Rabies, hepatitis B and diphtheria.
C Targeted intervention for HIV is done for
C Included in NRHM are
Commercial sex worker, migrant laborers and street
Strengthening of JSY, state and district health
children.
mission and recruitment and training of ASHA.
C WHO stage 4 HIV includes
C Impact indicator for evaluation of ASHA’s
performance Toxoplasmosis, Pneumocystis carini and HIV
wasting syndrome.
Infant mortality rate
C About NPCB true are
C True about standard error of mean
Increase cataract surgery rate to 450 operation per
Based on normal distribution, it depends on
lakh population, intraocular lens implantation in
standard deviation of mean and used to estimate
more than 80% cataract surgery cases and
confidence limit.
development of 50 pediatric ophthalmic units.
C A learned behavior which is permanent and
consistent but liable to change C According to CDC recommendations, HIV screening
of pregnant women is
Practice
Opt out testing
C True about Japanese encephalitis
Putting profit ahead of health as a cause of disease
Transmitted by culex mosquito, pigs are ampli-
C
is provided by which theory of sociology
fiers and primary dose of vaccine consists of two
dose. Marxist
C Reliability of a screening test refers to C True about confounding factor
Gives same values even on repeated testing It is itself a risk factor for the disease.
(Note: Reliability is repeatability) C Best contraceptive for the newly married couple
is
C Disposal method of outdated cytotoxic drugs is
Oral contraceptives
Destruction and dumping in a secured landfill.
C Nested case control study is a type of
C Recall bias is most commonly associated with
Prospective study
Case control study
C Approaches to the health education are
C Positive predictive value of a test is Service approach, regulatory approach and health
TP/TP + FP education approach.

AIIMS MAY 2009 AIIMS NOVEMBER 2008


C Inner subjective thought of a person towards an C True about isolation period of enteric fever is
individual or situation is best termed as Till 3 bacteriologically negative stool and urine
Attitude reports.
98 AIIMS and All India PGMEE—Review Questions

C Cyclo developmental lifecycle is seen in C Mean bone density amongst two groups of 50
Filaria people each is compared with each other. The test of
C Post-pasteurized milk quality is tested by significance used would be
Phosphatase test Student t-test
C Best index for measuring contraceptive efficacy is C After applying a statistical test, an investigator gets
the P value as 0.01. It means that
Life table analysis.
The difference is not significant 1% times and
C Goals of national population policy are
significant 99%.
Bring IMR to below 30 per 1000 live births, bring
C If mean is less than the median, than the data is said
MMR to below 100 per 100,000 live births and
to be
achieve 100% registration of births, deaths,
marriage and pregnancy. Negatively skewed
C Shortest incubation period is seen in C Study done on a group of patients showed
coefficient of variance for BP and serum creatinine
Influenza
to be 20% and 15% respectively. Inference is
C In leprosy patient, lepromin test is used for Variation in BP is more than in serum creatinine.
Prognosis C In a study done in UK, association between sale of
C Kaplan Meier method is used for antiarrythmic drugs and increase in death due to
Survival asthma was found. It is example of
C True about Indian reference female Ecological study
Age 20–39 years, weight 55 kg and walking or other C Occupational cancer involves
activities for 2 hours daily and 8 hours of moderate Lung, liver and bladder.
or active work.
C Included in SAFE strategy AIIMS NOVEMBER 2007
Antibiotics, face washing and improvement of the C Kala-azar is transmitted by
environment. Sandfly
C Plastic wrapper of surgical syringe should be C Dietary reference intake recommendations include
discarded in Recommended dietary allowances, adequate intake
Black bag and upper tolerable level.
C Toxins responsible for epidemic dropsy
AIIMS MAY 2008 Sanguinarine
C In epidemiological study, the first case that comes
Problem village is defined in national water supply
to the attention of the investigator is
C
and sanitation programme as all
Index case
Distance of safe water greater than 1.6 km, water
C Staff present at the subcentre level, under primary
exposed to the risk of cholera and water source has
health care
excess iron and heavy metals.
Multipurpose health worker
C Endemic ascites is associated with
C Primary health care, true are
Pyrrolizidine
Essential health care for all, placing people’s health in
C Criteria for normal Indian reference male are people’s hand and sectors like agriculture, animal
60 kg, 8 hours in bed and 20–39 years of age husbandary, food industry and communication are
C Adjuvant used in DPT vaccine is involved.
Aluminium C Under NPCB, screening of school children is first
C Overall survival is increased by screening procedure in done by
Colon carcinoma School teacher
C Best distribution to study the daily admission of C Active search for disease in an apparently healthy
head injury patients in a trauma care centre is individual is known as
Poison distribution Screening
Preventive and Social Medicine/Community Medicine 99

C True regarding fluorosis are C Brucella is transmitted by


It can cause fluorine deposition in bone, deflouri- Aerosol transmission ingestion of raw milk and
dation is done by Nalgonda technique and fluorosis contact with aborted fetuses.
can cause genu valgum. C True about ASHA is
C MMR is maternal mortality per 1000 live births. One per 1000 rural population, mobiliser of
C True about pulse polio immunization antenatal care and female voluntary worker.
Mop up immunization is done in restricted geogra- C Richest source of vitamin D is
phical areas, carried out in all children less than 5 Fish oil
years of age and mopping up is done in areas where
wild polio virus is found. AIIMS NOVEMBER 2006
C Measures of dispersion are
True about communicability of malaria
Range, variance and standard deviation.
C
Gametocytes appear in blood 4–5 days after the
C Disinfectants used for blood spills
appearance of asexual parasite in vivax infection
Sodium hypochlorite and gametocytes appear in blood 10–12 days after
C n-3 PUFA is present in appearance of sexual parasite in falciparum
Mustard oil, fish oil and corn oil. infection and in early stage of infection, their
density may exceed 1000/cmm of blood.
AIIMS MAY 2007 C Eicosapentaenoic acid is present in
Fish oil
C Example of disability limitation is C Revised strategy for NPCB includes
Resting affected limbs in neutral position. Fixed facility surgery, IOL implantation for cataract
C Comprehensive emergency obstetric care includes and uniform distribution.
Manual removal of placenta, blood transfusion and C Any loss or abnormality of psychological, physio-
cesarean section. logical or anatomical structure or function is known as
C What happens when confidence limit of a test is Impairment
increased C Hb estimation was done by a new technique and
Previously insignificant value becomes significant. repeated 10 times. The reports are—9.4, 9.3, 9.5, 9.7,
C IUCD acting for 10 years 9.6, 9.9, 10.1, 10.4, 10.6 and 12.0. The new technique
has
CuT 380A.
Low validity and low reliability
C Common complications of measles are C Socialization of medicine leads to
Diarrhea, pneumonia and otitis media. Free medical care supported by state, eliminates the
C Best method to compare new test and gold standard competition among physicians in search of clients
test is and ensure social equity and universal coverage of
Bland and Altman analysis. health services.
C Accepted methods of randomization are C Causative toxin for endemic ascites is
Computer drawn randomization, lottery and Pyrrolizidine
random number table. C True about ASHA is
C Thiamine deficiency is seen in Deployed 1/1000 population
Chronic diarrhea, chronic alcoholism and food C True about point source epidemics are
faddist. No secondary curves, one single peak only and all
cases occur in one incubation period.
C L-J chart is used for monitoring
C Epidemiological study giving the most accurate
Accuracy result is
C Standard error of mean is calculated by Meta analysis
SD/square root of n C PUFA consumption is associated with
C Characteristics of importance in a screening test Lowering of serum cholesterol and lowering of
Low cost, high safety margin and high sensitivity. LDL cholesterol.
100 AIIMS and All India PGMEE—Review Questions

AIIMS MAY 2006 those with BCG scar. If this is so and an association
is found between TB and not having BCG scar, the
C Brucellosis can be transmitted by association may be due to
Contact with infected placenta, ingestion of raw Interviewer bias
vegetables from infected farms and inhalation of C The risk ratio in a study is
infected dust or aerosol. Incidence among exposed/incidence among non-
C True about leptospirosis exposed.
It is a zoonosis, man is an accidental host and man C Incidence of tuberculin infection in a community is
is the dead end host. most appropriately assessed by
C The mechanism by which cholera may be Identifying new converters to tuberculin test.
maintained during interval between peak cholera C When an intervention is applied to community to
seasons is evaluate its usefulness it is called
Continuous transmission in man. Effectiveness
C Disability limitation C Drug of choice for cholera treatment in pregnancy
Example is resting affected limbs in neutral
Furazolidone
position.
C Most common cause of blindness in India is
C Vitamin A deficiency is considered a public health
problem if prevalence rate of night blindness in Cataract
children between 6 months and 6 years is > 1%. C Under national programme for prevention of
C The highest percentage of PUFA is present in nutritional blindness, a child in the age group of
Soybean oil 6–11 months is given a dose of vitamin A equal of
C Chronic carrier state is found in 1 lakh IU
Typhoid, hepatitis B and gonorrhea C Incubation period for pertussis is
C RCT comparing efficacy of two regimens showed 7–14 days
that difference is statistically significant with P < C Maximum risk of damage of fetus by maternal
0.001 but in reality the two tests do not differ in their rubella is in
efficacy. This is example of Type I (alpha) error 6–13 weeks of pregnancy
C Degree of freedom is C Deficit in weight for height in a 3 years old child
(Row-1) × (Column-1) indicates
C A diagnostic test for a particular disease has a Acute malnutrition
sensitivity of a 0.90 and a specificity of 0.80. A single C A study shows that 85% of cases of lung cancer are
test is applied to each student in the population in due to cigarette smoking. It is a measure of
which the diseased population is 30%. What is the Attributable risk
probability that a person, negative to this test has no
The distribution of random plasma sugar
disease
C
measurements from 100 first year students have
95%
mean of 3.0 mmol/L with a standard deviation of
C Histogram describes 3.0. True about shape of the distribution curve
Quantitative data for a group of patients Nothing can be said conclusively
C If birth weight of each of 10 babies born in a hospital C The age group of children surveyed for assessment
in a day is found to be 0.8 kg, then the standard of primary immunization coverage in WHO
deviation of this sample will be recommended EPI cluster sampling
Zero
12–23 months
C Specificity of a test is the ability of a test to detect
C According to a joint study “Health care in India:
True negatives
Road ahead” by CII and McKinsey and CO in 2002
India’s exisiting bed population ratio is 1.4 : 1000
AIIMS NOVEMBER 2005 C True about childhood mortality rates in India
C It is probable that physician have a higher index of Almost 3/5th of IMR occur in neonatal period,
suspicion for TB in children without BCG scar than almost 3/4th of under five mortality occur in
Preventive and Social Medicine/Community Medicine 101

infancy and about one in ten children die before C National family health survey has successfully
they reach the age of five years. completed
C The best design to study the association between 3 rounds
risk factor and disease is of C The age and sex structure of a population may be
Cohort study described as a
C A known HIV patient is admitted in a ward Population pyramid
following surgery. The resident doctor who C The carrying capacity of any given population is
changed the dressing next day found it to be soaked determined by its limiting resources.
in blood. Right method of discarding the dressing is C True about prevalence and incidence
Put the dressing material directly in an appropriate Incidence is a rate but prevalence is not
bag and send for incineration. C The most common cancer, affecting both male and
C Distribution of FEV in 300 smokers-Median—2.5 L, female of the world is
First quartile—1.5, Third quartile—4.5 L. No. of Lung cancer
persons in the sample expected to have FEV C The most common cause of maternal mortality in
between 1.5 L and 4.5 L is India is
150 Hemorrhage
C If the distribution of IOP seen in 100 glaucoma C The rate adjusted to allow the age distribution of the
patients has an average 30 mm with SD of 1. Lower population is
limit of average IOP that can be expected 95% of Age standardized mortality rate
times C Characteristic of a single exposure common vehicle
28 outbreak
C A test for a disease has sensitivity 0.90 and Explosive
specificity 0.90. Diseased person in the population
C Drug of choice for chemoprophylaxis of cholera
in which single test is applied is 10% probability
that a person positive to this test has the disease is Tetracycline
50% C The national level system that provides annual,
national as well as state level reliable estimates of
fertility and mortality
AIIMS MAY 2005
Sample registration system.
C A bacterium divides every 30 minutes. Number of C Pulse with highest content of iron
bacteria after exponential growth for 3 hours, Soyabean
starting with a single bacterium C True regarding pre-post clinical trial
512 They use the patient as his or her own control
C One day census of inpatients in a mental hospital
could give good information about
The patients in that hospital at that time. AIIMS NOVEMBER 2004
C After applying a statistical test an investigator gets C Indian state with maximum number of AIDS cases
the p value as 0.01. It mean that reported
The difference is not significant 1% times and Tamil Nadu
significant 99% times. C In two studies for association between natural
C For a negatively skewed data nutritional states and birth weight of members, p
Mean will be less than the medium value 0.02 and 0.04 were found. Inference about
C If the prevalence is very low as compared to the magnitude of association
incidence for a disease, it implies The magnitude of association in study with p = 0.02
Disease is very fatal and/or easily curable. is more than that in study with p = 0.04
C Mean and standard deviation can be worked out if C Weakest study design to test the association
data is on between risk factor and disease
Interval/ratio scale Ecological study
102 AIIMS and All India PGMEE—Review Questions

C The most appropriate measure of average for data C Standard error is


250, 350, 250, 5000, 110, 266, 325, 300, 155, the no. of Standard deviation/sample size.
malaria cases reported during test 10 years in a C Highest prevalence of blindness in the world per
town 100,000 population is found is
Median Sub-Saharan Africa
C A total of 5000 patients of glaucoma are identified C True about plague
and surveyed by interviewers regarding family
Bubonic is the most common variety.
history of glaucoma. Study design is
National health policy of India 2002 includes
Case series report
C

C A physician notes down SBP of 50 patients and then Eradicate polio by year 2005, achieve zero level
administers an antihypertensive drug to them. He transmission of HIV/AIDS by 2010 and eliminate
measures the SBP after one week again. Most lymphatic filariasis by 2015.
appropriate statistical test of significance C The incubation period of yellow fever is
Paired T-test 3–6 days
C Population is the unit of study in C Mode of transmission of Q fever is
Ecological study Inhalation of erosol
C A leprosy patient treated with paucibacillary C Characteristic features of staphylococci food
multidrug therapy for 6 months had persistent poisoning
erythema and indurations in the plaque. The next Optimum temperature for toxins formation is 37°C,
step of recommendation by WHO intradietetic toxins are responsible for intestinal
Stop antileprosy treatment symptoms and incubation period is 1–6 hours.
C As compared to routine case control study, nested C Nodal ministry for integrated child development
case control study avoids problem related to Scheme (ICDS) programme centre
Temporal association Ministry for HRD
C The literacy rate of Indian population as per census C The systemic distortion of retrospective studies that
2001 is can be eliminated by a prospective design is one of
65.4% the following
C Arboviral diseases reported in India Recall bias
Japanese encephalitis, Chikungunya fever and C Null hypothesis true and rejected is
Kyasanur Forest disease. Type I error (Mn: NTR)
C Orthotoludine test is used to determine C If a new effective treatment is initiated and all other
Free and combined chloride in water. factors remain the same, true is
C Weight of Indian reference woman Incidence will not change
55 kg C Relationship between different parameters of a
C True about quarantine performance of a test
Absolute quarantine is restriction during the Sensitivity is inversely proportional to specifi-
incubation period, exclusion of children from city.
schools is modified quarantine and quarantine C 20 years old female with category II tuberculosis
should not be longer then the longest incubation (sputum positive relapse). The treatment regimen
period. recommended under DOTS is
C True about mosquito 2 (HRZES)3 + (HRZ)3 + 5 (HR)3
Definitive host in malaria, life cycle is completed in C Sampling method for assessment of immunization
3 weeks and the female can travel up to 3 km. status of children
C True regarding dracunculiasis Cluster sampling
India has eradicated this disease. The disease is C True about break point chlorination
limited to tropical and subtropical regions and no Free chlorine is released in water after breakpoint
animal reservoir has been proved. chlorination, chlorine demand is the amount
Preventive and Social Medicine/Community Medicine 103

needed to kill bacteria, oxidize organic matter and C Entire village was asked to participate in a study to
neutralize ammonia. identify cholera carrier and everyone present at the
C An investigator wants to study the association time submitted to examination due to recent
between maternal intake of iron supplements (yes cholera deaths. The proportion of people who were
or no) and incidence of low birth weight (<2500 or > carriers were computed and compared with regard
2500 gm) in 100 pregnant women. Appropriate to water supply. This is a type of
statistical test of significance is Cross sectional study
Chi-square test C True about ‘New family’
C Medical colleges are classified as eye care centers of All nuclear families of less than 10 years duration.
(under National Programme for Control of Blindness C A new pregnancy test was applied on 100 pregnant
in India) (Known) women and 100 non-pregnant women. 99
Tertiary level and 10 showed positive result in respective groups.
C Taking the definition of blindness as visual acuity Sensitivity of the test is
less than 3/60 in the better eye, the number of blind 99%
persons per 100,000 population in India is estimated C A man with fever and chills 2 weeks after a louse
to be …… bite. There was a maculopapular rash on the trunk
C In a double blind, clinical drug trial which spread peripheraly. The cause of this
The patients do not know which treatment they are infection can be
receiving. Epidemic typhus
(Note: Doctors also do not know which treatment C True regarding Q fever
they are giving) Zoonotic infection, human disease is characteri-
C The most sensitive index of recent transmission of zed by an interstitial pneumonia and no rash is
malaria in a community is seen.
Infant parasite rate C Reservoir of Indian kala-azar is
C Extra calories required during the first six months
Man
for a lactating woman is
Features suggestive of asbestosis
600 Kcal
C

C Bagassosis is most likely caused due to the The disease progresses even after removal of
inhalation of the dust of contact, can lead to neural mesothelioma and
sputum contains asbestos bodies.
Sugarcane (Mn: Both has g)
C In defining general fertility rate, the denominator is C The proportion of false positive results of a
screening test is lower in population A then
Woman between 15 and 49 years of age.
population B because the prevalence of disease is
C Mites are the vectors of
Higher in population A
Scabies, scrub typhus and rickettsial pox
C An investigator wants to study the association
between maternal intake of iron supplements (yes
AIIMS MAY 2003 or no) and weights (in gm) of newborn babies. He
C Most common cause of neonatal mortality in India collects relevant data from 100 pregnant woman
Low birth weight and their newborn. Test of hypothesis advised
C In calculation of dependency ratio, the numerator is Unpaired or independent T test
expressed as C To test the statistical significance of the difference in
Population under 15 and above 65 years of age. heights of school children among three socio-
C An adult male patient with complaints of cough economic groups, test used is
and fever for 3 months and hemoptysis on and off, One way ANOVA
sputum positive for AFB and had already received C For the field diagnosis of trachoma, the WHO
treatment with HRZ for 3 weeks and discontinued. recommends that follicular and intense trachoma
Categorization for management of patient inflammation should be assessed in
Category 1 and start 2 (HRZE)3. Children aged 0–10 years
104 AIIMS and All India PGMEE—Review Questions

AIIMS NOVEMBER 2002 C The number of patients required in a clinical trial to


treat a specific disease increases as the drop out rate
C Drug of choice for chemoprophylaxis in contacts of increases.
a patient of pneumonic plague is C According to WHO, goiter is endemic if its
Tetracycline prevalence in school age children is more than 5%.
C National population policy aims to achieve NRR of C A 46 years old female presented to the eye OPD
1 by with right eye 6/60, and left eye 3/60. Under NPCB,
2010 year she is classified as
C Same reading on repeated testing by a chemical test, Low vision
the measurement is C Cut-off visual acuity in school vision screening
Precise programme in
C True about rheumatic fever/heart disease in India is 6/9
epidemiology C When a diagnostic test is used in series mode than
The prevalence varies between 2 to 11 per 100 Specificity increases but the sensitivity decreases.
children aged 5–16 years, occurs equally in females C The active immunity offered by
and males and rheumatic fever occurs in about 2% TT is effective in nearly 100% patients
of streptococcal sore throats.
C True about DPT vaccine AIIMS MAY 2002
Exposure to direct sunlight, when in use should be C Leprosy eradication programme best evaluated by
avoided, store stocks are needed for three months at Detection of new cases
PHC level and half used vials should not be put C Secondary attack rate is
back into the cold chain after the session. Number of exposed persons developing the disease
C The preferred public health approach to control within the range of incubation period following
non-communicable diseases is exposure to the primary case.
Shift the population curve of risk factors by a C Presumptive treatment of malaria in a chloroquine
population based approach. resistant area
C True for occupational lead poisoning Sulphalene and pyrimethamine
Inhalation is the most common mode of absorption, C HbsAg positivity is found is a pregnant women of
lead in blood and urine provide quantitative 32 weeks of pregnancy. To prevent infection, new
indicator of exposure and basophilic stippling is a born is given
sensitive parameter of hematological response. Hepatitis B vaccine and immunoglobulins
C In community needs assessment approach as part C Schedule for sputum examination after beginning
of RCH the targets for various health activities are the chemotherapy in RNTCP is
set at the level of 2, 4 and 6 months
District C The side effects of DMPA contraceptive
C True about case control study of a suspected Weight gain, irregular bleeding and increased
association between breast cancer and the reproductive tract infection.
contraceptive pill C True morbidity in a population can be calculated by
The control should come from a population that has Active surveillance
the same potential as the cases, control should C Epidemic of plague is controlled by
exclude women taking OCP at time of survey and Isolation of contacts, surveillance and chemopro-
all the control needs to be healthy. phylaxis with tetracycline.
C Prevalence of a disease describes C Best indicator for monitoring of syphilis treatment
The balance between incidence, mortality and VDRL
recovery. C True regarding relative risk and attributable risk in
C The main purpose of randomization in a controlled a study on comparison of a common drug (NSAID)
drug trial is and rare drug (Dypirone) causing disease
To ensure that two groups will be similar in NSAID has higher relative risk and attributable risk
prognostic factors. than Dypirone.
Preventive and Social Medicine/Community Medicine 105

ALL INDIA 2012 C A Cohort study differs from case control study as
Cohort studies require longer time frame (years)
C Intervention trials with individual as a randomiza- but case control studies not.
tion unit C DALY expresses years of life lost to
Drug, surgery and vaccine Premature death and years lived with disability
C The sex ratio for age group 0–6 years as per 2011 adjusted for the severity of disability.
census
914
ALL INDIA 2011
C The concept of design effect is applied to
Cluster sampling C Indicators of HDI
C A state of disease endemicity where the disease Life expectancy at birth, education and GDP.
transmission has been stopped but the agent is C Good index of severity of acute disease
present in the environment is called Case fatality rate
Elimination C Denominator in estimating MMR
C Efficacy of a new drug with an existing drug for Total number of live births
treatment of particular disease is compared in C Chronic carrier state is seen in
Phase 3 trial Diphtheria, typhoid and gonorrhea.
C The p value stands for C Tetracycline is used in prophylaxis of
The probability of concluding that a difference Cholera
exists, while, in reality, such a differentiation does C Vectors may transmit infection by methods
not exist. Regurgitation, rubbing of infected feces and
C Median is a good measure for central tendency of contamination with body fluids.
Health expenditure, incubation period and survival C Non-deliberate cultural practice leading to anti-
time. mosquito effect
C Surveillance summaries are prepared Use of alkaline detergent
To provide information on disease patterns and C Factors responsible for resurgence of malaria
trends. Drug resistance in parasite, insecticide resistance in
C In natural history of the disease, true about vectors and antigenic mutations in parasite.
pathogenesis phase are C Insects that have developed resistance to DDT are
Start after entry of agent into host body, subclinical Musca domestica, Xenopsylla species and Anopheles
phase of an illness is included and tertiary species.
prevention is applicable. C Arboviral disease not found in India
C Aedes/anopheles and Aedes culex are vectors for Yellow fever virus
which disease worldwide C True about Japanes encephalitis
Lymphatic filariasis Two to three cases per village suggest an epidemic,
C True about millennium development goals children are more frequently affected than adults and
Reduce MMR by 3/4th between 1990 and 2015, ratio of inapperent to apperent infection is > 100:1.
reduce under 5 mortality rate by 2/3rd between C True about leptospirosis
1990 and 2015 and have halted by 2015 and Rats are prime reservoirs.
reversed, the spread of HIV/AIDS.
C A sewage worker presents to the emergency
C Vaccination recommended for elderly population department with fever and jaundice. Lab findings
Tetanus, influenza and pneumovax. are elevated BUN and serum creatinine suggestive
C Larvicidal for malaria control of renal failure. Antibiotics recommended is
Abate, copper acetoarsenite and malathion. Penicillin G
C The resource persons for training ASHA under C True about yaws
NRHM are Caused by Treponema pertenue, transmitted non-
AWW and ANM venerally and secondary yaws can involve bone.
106 AIIMS and All India PGMEE—Review Questions

C According to McKeonn’s theory, reduction in C Biological agent carrying the highest potential for use
mortality from tuberculosis was a consequence of as a biological weapon for microbial bioterrorism is
Social and environmental factors. Plague (Yersinia pestis)
C Tests performed to assess the efficiency of C Biological weapon with least potential for use as a
Pasteurization biological weapon
Phosphatase test Brucellosis
C Orthotoluidine test is used for detecting
ALL INDIA 2010
Chlorine
C Causes of indoor air pollution C Movement across socioeconomic level is termed as
Carbon monoxide, nitrogen dioxide and radon. Social mobility
C Diseases with national screening control programme C Associated with emotional valence and most likely
are to be influenced by motivation is
Diabetes mellitus, refractive errors and carcinoma Practice
cervix. C ASHA is posted at
C Reform proposed by World Health Report (2008) Village level
Policy reform, leadership reform, service reform C JSY stands for
and universal reform (Mn: PLUS) Janani Suraksha Yojna
C Direct standardization is done to compare the C Provision of primary health care was done by
mortalty rates between 2 countries. This is done Alma Ata declaration
because of difference in C Essential components of primary health care are
Age distributions Provision of essential drugs, immunization
C A village is divided into 5 relevant subgroups for against major infectious diseases and health
purpose of a survey. Individuals from each education.
subgroup are then selected randomly. This is C Current trend in health care is
Stratified sampling Community participation
C A region is divided into 50 villages for the purpose C Highest level of community participation is
of a survey. 10 villages are then selected randomly Planning of intervention by community.
for the purpose of a study. This is known as C True regarding MMR
Cluster sampling Numerator includes total number of female deaths
C A new test for diabetes mellitus is performed in a within 42 days of delivery, expressed as a rate and
group containing diabetics and non-diabetics. If 120 expressed per 1000.
mg/dl of blood sugar is taken as cut-off, area C Perinatal mortality rate includes
shaded in graph represents Stillborns and death within 7 days of birth.
False negative C IMNCI differs from IMCI in following
C Punett square is used to Malaria and anemia are included, 0–7 days neo-
Predict genotype of offspring nates are included and emphasis on management of
C An investigator observes that 5 independent risk sick neonates over sick older children.
factors influence the occurrence of a disease. C Mass chemoprophylaxis in endemic area is
Statistical test applied to assess the relationship recommended for
between them is Yaws, trachoma and filaria
Multiple logistic regression C Rural and urban difference in prevalence is seen
C Pearson’s coefficient of skewness is defined as in
(Mean-mode)/Standard deviation Lung cancer, mental illness and chronic bronchitis.
C True regarding disposal of biomedical waste C True about TB annual rate of infection(ARI)
Human anatomical waste is disposed in a yellow The average estimated ARI for India is 1.7%, ARI
bag, contents from a red bag may be a source of reflects current trend and effectiveness of control
contamination and black bags are used for disposal measures and ARI represents the percentage new
of ash from incineration. infections.
Preventive and Social Medicine/Community Medicine 107

C True about scrub typhus C True about standard distribution curve is


Caused by O. tsutsugamushi, mites act as reservoirs Mean = median
and tetracycline is the drug of choice. C True about simple random sampling is
C Factors contributing to resurgence of malaria Every element in the population has an equal
Drug resistance in parasite, drug resistance in probability of being included.
vectors and antigenic variations in parasite. C The guidelines according to baby friendly hospital
C A case of acute flaccid paralysis must be observed initiative includes
for residual weakness for Mother and infant to be together for 24 hours a day,
60 days giving newborn infants no food or drinks other than
C India aims to eliminate the following disease by breast milk and encouraging breastfeeding on
2015 demand.
Filariasis C The latest WHO protocol for classification and
C The screening strategy for prevention of blindness measurement of disability is
from diabetic retinopathy according to NPCB ICF
involves C Components of IDEAS include the following
High risk screening Self care, communication and understanding and
C True about confidence limit/interval performing in work.
95% confidence interval will cover 2 standard errors C IMCI initiative was taken to prevent mortality and
around the mean. morbidity from
C A standard Z score is related to Malaria, malnutrition and otitis media.
Normal distribution C Part of targeted intervention in preventive strategy
C The sensitivity of a test is in spread of AIDS
TP/ TP + FN Treating STD, providing condoms and behavior
C Incidence of TB in a community change communication.
Number of cases per mid year population in that C Chemoprophylaxis can be done for
year multiplied by 1000. Diphtheria, cholera and meningitis.
True about red cross emblem
Q fever is transmitted by
C
C
Size of bars in the cross is equal horizontally and
Ticks
vertically, misuse of emblem is punishable offence
Toxic shock syndrome is caused by
under Indian law and was convened in Geneva.
C

C Synthetic pyrethroid compounds are Infected measles vaccine.


Permethrin, proparthin and cypermethrin. C Exposure carrying the maximum risk of transmi-
ssion of HIV
ALL INDIA 2009 Blood transfusion
C Desired end result that is not operationally measu-
C Criteria suggesting causality in non-communicable rable
diseases Goal
Strength of association, specificity of association
C Assessment of final outcome of a predetermined
and dose response relationship.
objective is termed as
C Analytical studies are
Case control study, Cohort study and ecological study. Evaluation
C Incidence can be calculated by
Cohort study ALL INDIA 2008
C Which causes an increase in the prevalence of disease
Longer duration of disease C Iceberg phenomenon differentiates
C The most suitable method for presenting frequency Apparent and inapparent
distribution of data gathered from continuous C Type of bias reduced by allowing equal interview
variables is time
Histogram Interviewer bias
108 AIIMS and All India PGMEE—Review Questions

C Vaccine contraindicated in pregnancy C KAP study in India was first used to study
MMR Family planning
C True regarding live vaccines C Most common occupational cancer in India is
Booster doses are not required when live vaccines Skin cancer
are administered, single dose gives life long C True about Delphi method
immunity and live vaccine contains both major and Method involves formation of a team to undertake
minor antigens. and monitor a Delphi on a given subject, selection
C The completed family size may be estimated by of one or more panels to participate in the exercise,
Total fertility rate customarily the panelists involved are experts
C As per RCH, the community health center is in the area to be investigated and first round
First referral unit of Delphi method involves development of
C True about nutritional surveillance questionnaire.
It is a diagnostic approach, assessment involves C Standard deviation is a measure of
precise measurement of weight and height and Deviation from the mean value.
assessment is done by trained persons.
C True about recommended daily allowance ALL INDIA 2007
RDA is decided by a panel of experts and is based
on scientific research, RDA is often higher than C Indicators included in PQLI are
recommended minimum requirement and RDA is IMR, literacy rate and life expectancy at age one.
based on estimated average requirement. C 85% of lung cancer among smokers was due to their
C Highest amount of saturated fatty acid is seen in smoking. This is an example of
Palm oil Attributable risk
C Psychrometer is used to measure C True regarding herd immunity are
Humidity Mostly due to subclinical infection, can be acquired
C Concurrent list encompasses by immunization and spread of epidemic is
Prevention of extension of communicable disease influenced by it.
from one unit to another, prevention of adulteration C Vaccines included in EPI schedule are
of food stuffs and regulation and development of OPV, BCG and DPT
medical profession. C Under eradication of congenital Rubella syndrome
C Following are zoonosis programme, the first priority group for rubella
Plague, Japanese encephalitis and TB. vaccination is
C Soil forms an important reservoir for infections All non-pregnant woman of 15 to 34 years of age.
Coccoidomycosis, anthrax and tetanus. C Strategies for prevention of neonatal tetanus
C Soft tick transmits includes
Relapsing fever 5 clean practices, 2 doses of TT to all pregnant
woman and immunizing all married woman.
C Isolation is strictly recommended for
Case finding in RNTCP is based on
Pneumonic plague
C
Sputum microscopy
C True about measles
High secondary attack rate, only one strain auses C True regarding endemic typhus
infection and infection confers life long immu- Flea is a vector of the disease
nity. C Common causes of blindness in children in India
C True about yaws are are
Caused by Treponema pertenue, has cross immunity Malnutrition, ophthalmia neonatarum and glaucoma.
with syphilis and cannot be differentiated C BMI to classify obesity should be?
serologically from Treponema pallidum. 30
C Dharmendra’s index and Jopling classification C Impact and efficiency of iodine control programme
deals with can be estimated by
Leprosy Neonatal thyroxine levels
Preventive and Social Medicine/Community Medicine 109

C Compared to cow’s milk, mother’s milk has more C Transplantation of human organs act was passed
Lactose in
C The highest percentage of PUFA are present in 1994
Soyabean oil C The extra-callories allowances needed per day
during pregnancy is
C The major purpose of randomization in clinical trial
is to 350
Reduce selection bias in allocation of treatment. C The premium of community based universal health
insurance scheme launched during 2003–04 is
C A study conducted to test if use of herbal tree
` 1 per day for poor and individual to ` 2 per day for
played any role in prevention of common cold. Data
a family of seven.
was collected on the number of people who
developed cold. The analytical test of choice to test
this study is ALL INDIA 2005
Chi square test C Infant mortality rate includes
Early neonatal mortality, late neonatal mortality
ALL INDIA 2006 and post-neonatal mortality.
C Endemic disease
C True about standardization
Constantly present in a given population group.
It allows comparisons to be made between two
C True regarding herd immunity for infectious
different population, for direct standardization
diseases
age specific death rates of study populations are
used and for indirect standardization, age In case of tetanus, it is affected by the presence and
specific death rates of standard population are distribution of alternate animal hosts and it refers to
used. group protection beyond offered by the protection
of immunized individuals.
C The double blinding in clinical trials is to avoid
observer and subject bias. C A 2 years girl with cough and fever for 4 days,
inability to drink, 5 kg weight and 45/min respi-
C True about randomized controlled trial
ratory rate. The girl is defined as
Baseline characteristic of control group and
Very severe disease
intervention group should be same, sample size
required depends on the hypothesis and C True regarding treatment of class III dog bite
investigator’s bias is minimized by double patient
blinding. Immediately wash wound with soap and water,
C Positive predictive value of diagnostic test in a give ARV and give immunoglobulins for passive
population is immunity.
TP/TP + FP C Drug of choice for cholera treatment in an adult
with a single dose of
C Sensitivity of a diagnostic test is
Doxycycline
TP/TP + FN
C True about congenital rubella syndrome
C Strongly associated with coronary heat disease
Diagnosed when infant IgM antibodies at birth,
Apolipoproteins
diagnosed when IgG antibodies persists for > 6
C Vision screening of school children is conducted months and most common congenital defects are
under National Programme for Control of sensorineural deafness, cardiac malformations and
Blindness by cataract.
School teachers C The most common cancer affecting Indian women
C WHO definition of blindness is in Mumbai and Delhi is
Visual acuity of < 3/60 in the better eye with best Breast cancer
correction. C BMI of a male whose weight is 89 kg and height 172
C Most common cause of ocular morbidity in India is cm is
Refractive error 30 (kg/m2)
110 AIIMS and All India PGMEE—Review Questions

C True regarding influence of smoking or risk of C Percent prevalence of disease = Total diseased/
coronary heart disease total population × 100
Smoking is independent risk factor, smoking is C The best indicator for monitoring the impact of
synergistic to other risk factors and influence of iodide deficiency disorders control programme is
smoking is directly related to number of cigarettes Neonatal hypothyroidism
per day. C Under the WHO vision 2020 programme, SAFE
C Viral infection transmitted by tick is strategy is adopted for
KFD Trachoma
C Anti-larval measures are C WHO definition of adolescent age is between
Intermittent irrigation, Paris green and Gambusia 10–19 yrs of age
affinis (and Malathion though as antilarval not C Information Technology Act was passed by
recommended).
Government of India in 2000
C Tools for checking efficiency of pasteurized milk are
C Source of manager’s power is
Phosphatase test, standard plate count and
Reward, coercive and legitimate
coliform count (Mn: PSC).
C The RDA of energy intake for an adult woman with
heavy work is ALL INDIA 2004
2790 kcal C A study beginning in 1975 with a group of 5000
C Color of bag for human anatomical waste adults asking about their alcohol consumption and
Yellow occurrence of cancer was studied in 1995–2000. This
C True about standard normal distribution study is
Has variance 1.0 Concurrent Cohort study
C The PEFR of a group of young girl following normal C Proxy measures for incubation period are
distribution has mean 300 L/min and standard Latent period, serial interval and generation time.
deviation 20 L/min C A 3 1/2 years old child has not received primary
True is 95% of the girls have PEFR between 260 and immunization. Best vaccination advice to such a
340 l/min. child is
C Z score is BCG, DPT, OPV, measles, vitamin A
Observed value - mean value/standard deviation. C True about DPT vaccine
C The events A and B are mutually exclusive, Aluminum salt has an adjuvant effect, whole cell
therefore, prob (A or B) = Prob (A) + Prob (B). killed bacteria of Bordetella pertussis has an adjuvant
C The diagnostic power of a test to correctly exclude effect and presence of a cellular pertussis component
the disease is reflected by increases its immunogenecity.
Negative predictivity C A 37 weeks pregnant woman attends an antenatal
C Highly significant correlation coefficient (r = 0.90, p clinic at PHC. No antenatal visit prior to it. Tetanus
= 0.01) between the systolic blood pressure values immunization in this case can be best approached
and serum cholesterol values of patients were by
found. Correct interpretations are Give a dose of TT and explain to her that it will not
A patient with high level of SBP is also likely to protect the newborn and she should take the second
have a high level of serum cholestrol, a patient dose after four weeks even if she has delivered in
with low level of SBP is also likely to have a low the meantime.
level of serum cholesterol and about 80% of the C A 4 months old child had cough, fever, chest
variation in SBP among his patients can be indrawing and respiratory rate 45 per minute. Best
explained by their serum cholesterol values and way to manage the child is
vice versa. Refer urgently to hospital after giving the first dose
C Total cholesterol level = a + b (calorie intake) + c of an antibiotic.
(physical activity) + d (body mass index) is an C The most appropriate test to assess the prevalence
example of of TB infection in a community
Multiple linear regression Tuberculin test
Preventive and Social Medicine/Community Medicine 111

C According to international health regulations, there C Dietary changes advocated by WHO for prevention
is no risk of spread of yellow fever if the of heart diseases include
Aedes aegypti index remains below 1%. Reduction in fat intake to 20–30 percent of caloric
C Mode of transmission of leprosy are intake, consumption of saturated fats to be limited
to less them 10% of total energy intake and
Breast milk, insect bite and droplet infection.
consumption of cholesterol to below 100 mg per
C True regarding leprosy 1000 kcal per day.
Multi-bacillary leprosy is diagnosed when there are C Multipurpose worker scheme in India was
> 5 skin patches, new case detection rate is an introduced following the recommendation of
indicator for incidence of leprosy and the target for Kartar Singh committee.
elimination of leprosy is to reduce the prevalence to
C If each value of a given group of observations is
less than 1 per 10,000 population.
multiplied by 10, standard deviation of new
C A 10 months old child weighing 8 kg has Bitot’s observation is original standard deviation × 10.
spots in both eyes. Most appropriate schedule to C If SBP in a population has mean = 120 mm Hg and
prescribe vitamin a to this child is median 125 mm Hg, the distribution is said to be
1 lakh units IM on day 0, 1 and 14 Negatively skewed.
C The current recommendation for breastfeeding is C It prevalence of diabetes mellitus is 10% the
that probability that three people selected of random
Exclusive breastfeeding should be continued till from the population will have diabates is
6 months of age followed by supplementation with 0.001
additional foods. C The usefulness/utility of a screening test in a
C True for breast milk community depends on
The coefficient of uptake of iron in breast milk is Sensitivity
70%, calcium absorption of human milk is better
than that of cow’s milk and it provides about 65 kcal ALL INDIA 2003
per 100 ml.
C Denominator in secondary attack rate is
True about intrauterine devices
All susceptibles amongst close contact.
C
The pregnancy rate of lippes loop and Cu 250 are
C Necessary for calculation of sample size for a
similar, IUD can be used for emergency
prevalence study
contraception within 5 days and LNG releasing
Prevalence of disease in population, significance
IUD has an effective life of 5 years.
level and desired precision.
Goals of the national population policy of India
The parameters of sensitivity and specificity are
C
C
includes
used for assessing
To reduce IMR to 30 per thousand live births, to Criterion validity
reduce MMR to 100 per 100000 live births and 100 C True about meningococcal meningitis
percent registration of births, deaths marriages and The disease is more common in dry and cold
pregnancies. months, chemoprophylaxis of close contacts of
C Essential component of RCH programme in India cases is recommended and the vaccine is not
includes effective in children below 2 years of age.
Prevention and management of unwanted C A 24 months old child with cough, fever for 2 days,
pregnancies, maternal care including antenatal 11 kg weights, respiratory rate 38/min, and chest
delivery and postnatal services and management of indrawing present. Management is
reproductive tract infection and sexually transmitted Classify as severe pneumonia, start antibiotics and
infections. refer urgently.
C Five clean practices under strategies for elimination C For controlling an outbreak of cholera, measures
of neonatal tetanus includes recommended are
Clean surface for delivery, clean hand of the Proper disposal of excreta, chlorination of water
attendant and new blade for cutting the cord. and early detection and management of cases.
112 AIIMS and All India PGMEE—Review Questions

C The organism most commonly causing genital ALL INDIA 2002


filariasis in most parts of Bihar and Eastern UP is
Wuchereria bancrofti. C Iron and folic acid supplementation forms
C Class II exposure in animal bites includes Specific protection.
The most important function of sentinel surveillance is
Licks on a fresh wound.
C
To find out the total amount of disease in a popula-
C True about endemic typhus
tion.
Is transmitted by the bite of fleas.
C Serial interval is
Leprosy is considered a public health problem if the
Time gap between primary and secondary case
C
prevalence of leprosy is more than 1 per 10,000.
C Advantages of case control studies are
C Lepromin test is most useful for
Useful in rare diseases, odds ratio can be calculated
Prognosis and cost effective and inexpensive.
C Scope of family planning services include C Odds ratio is
Screening for cervical cancer, providing services for ad/bc
unmarried mothers and providing adoption services. C In a prospective study of 1000 subjects, 600 person
C Most important epidemiological tool used for were put on beta carotene and 400 were not. 3 out of
assessing disability in children is first 600 developed lung cancer and 2 out of other
Wing’s handicaps, behavior and skills schedule 400 developed lung cancer. Interpretation is
(HBS). Beta carotene and lung cancer have no relation to
C The vitamin A supplement administered in each other.
prevention of nutritional blindness in children C True about direct standardization
programme contain 1 laks IU/ml. Standard population is needed, population should
C The protein efficiency ratio is defined as be comparable and two populations are compared.
The gain in weight of young animals per unit C Vaccine contraindicated in pregnancy
weight of protein consumed. Rubella
C Under national programme for anemia prophy- C True regarding pertussis
laxis, elemental iron and folic acid contents of iron Erythromycin prevents spread of disease between
and folic acid tablets are children.
100 mg elemental iron and 500 μg of folic acid. C The infectivity of chickenpox lasts for
C Under RCH programme, elemental iron and folic 6 days after onset of rash
acid contents of pediatric iron C Carrier’s are important in
Folic acid tablets are 20 mg iron and 100 μg folic acid. Polio, typhoid and diphtheria
C A measure of location which divide the distribution C Acute flaccid paralysis is reported in a child aged
in the ratio of 3:1 is 0–15 years.
Third quartile C A 2 years old child with cough, fever, difficulty in
C The P value of RCT comparing operation A (new breathing, RR 50/min, bilateral crepitations but no
procedure) and operation B (Gold standard) is 0.04. chest indrawing
Conclusion is the probability of a false positive Diagnosis is pneumonia
conclusion that operation A is better than operation C Active and passive immunity should be given
B, when in truth it is not is 4% together in
C Chi-square test is used to measure Tetanus, rabies and hepatitis B
The degree of association between two variables. C Cereals and pulses are considered complementary
C Elements of primary health care includes because cereals are deficit in lysine and pulses are
Adequate supply of safe water and basic sanitation, deficient in metheonine.
providing essential drugs and health education. C For a reference Indian man minimum daily protein
C The response graded on agree or disagree conti- requirement is 40 g (mean) and standard deviation
nuum is based on is 10. The RDA of protein is
Likert scale 60 g/day
Preventive and Social Medicine/Community Medicine 113

C The best method to show the association between C In a study, variation in cholesterol was seen before
height and weight of children in a class is by and after giving a drug. The test of significance is
Scatter diagram Paired t test
C The correlation between two variable was found to C Ram and Shyam stay in the same hostel. Ram
be 1.1 in a study. This is developed infection with group B meningococci
and Shyam developed infection due to group C
Computational mistake in calculating correlation.
meningococci. True statements are
The biological oxygen demand indicates
Educate students about meningococcal transmission
C
Organic matter and take preventive measures, chemoprophylaxis
C In a surveillance centre for hepatitis B, in a low to all against both group B and group C and vaccine
prevalence area, method for testing for hepatitis prophylaxis of contacts of Shyam.
B changed from single ELISA to double testing C Common causes of post-neonatal infant mortality
in series. The parameters of the test affected in India are
are Malnutrition, diarrheal diseases and acute respira-
Specificity and positive predictive value tory infection.
114 AIIMS and All India PGMEE—Review Questions

Ophthalmology

AIIMS NOVEMBER 2015 C High molecular weight protein in cataractous lens


seen only in humans
C Most common ocular fungal infection in HIV patient HM 3 and 4
Candidiasis C Painful unilateral dimness of vision with after
C Keyhole visual lesion seen in images and flashes of light
Occipital lobe lesion Optic neuritis
C Most common mode of spread of retinoblastoma
AIIMS MAY 2015 Optic nerve
C Corneal sensation is lost in lesion of C Multifocal ERG is used for
Nasolacrimal nerve. (Nasociliary nerve is a better Assessing macular cone cells.
answer if given in the option. It is a branch of
ophthalmic branch of trigeminal nerve). AIIMS NOVEMBER 2014
C True about direct ophthalmoscope C Used for self tonometry
Self illuminating, vertical and erect and field of Rebound tonometer
vision 2 disc diameter. C Diagnostic procedure not done in a dilated pupil
(Note: Magnification is 15 times) Gonioscopy
C Evisecration of eye is not done in C The normal ratio of light peak and dark trough on
Malignancy an EOG is
C Cherry red spot and Hollenhorst plaque is seen in More than 185%
CRAO C A 35 years old man complains of vision distortion
C Stain used for granular dystrophy of cornea increasing progressively in both the meridians on
wearing his spectacles. True are
Masson trichrome
It is aniseikonia, it is due to wearing of cylindrical
C Vitreous humour is rich in
glasses and it is due to asymmetrical convex lenses
Ascorbate in both the eyes.
(Note: Aqueous humour is rich in potassium) C Substance strongly implicated as a cause of corneal
C What is seen in right trochlear nerve palsy? edema under hypoxic conditions
Diplopia on upward gaze and adduction, exotropia Lactate
and hyperopia C A diagnosis of fungal corneal ulcer is not made if
(Note: Head tilt is seen in opposite side) there is presence of
C Cherry red spot after blunt trauma in eye Marked photophobia and pain compared to signs.
Berlin’s edema. C A patient presents to the OPD and a diagnosis of
C Limbal stem cell marker fungal keratomycosis is made. Drug effective for him
ABCG2 Silver sulfadiazine

114
Ophthalmology 115

C Absolute contraindications for corneal transplanta- C Express glaucoma implant is made up of


tion Stainless steel
Death due to unknown cause, rabies and subacute C Ascorbate and ? tocopherol are maintained in a
sclerosing panencephalitis. reduced state in the lens by
C A young male patient presents with painless Glutathione
diminution of vision with an intraocular pressure of C A 7 years old male child presents with normal
60 mm Hg. Most likely diagnosis vision 6/6 in the right eye and hand movement
Glaucomatocyclitic crisis perception close to the face in the left eye. On
C Laser trabeculoplasty is done for fundoscopy, his right eye was normal and left eye
Pseudoexfoliative glaucoma. showed retinal detachment, subretinal yellowish
C Congenital cataract occurs due to abnormalities in exudates and telangiectatic vessels. The most likely
protein formation in gene diagnosis
cryGS3 Coats’ disease
C True about rhegmatogenos retinal detachment C A young male patient presents with central
There is abnormal pull on the retina due to changes scotoma in left eye. His right vision showed 6/6
in the vitreous, anterior extent is up to ora serrata vision. On examination, in the left eye, there was
and management is usually surgical. focal foveal detachment. Next done
C The developement of subretinal demarcation line Inquire about the use of steroids
(high water marks) in the retina is indicative of (Note: Central serous retinopathy).
Long standing retinal detachment. C A 28 weeks baby suffered from respiratory distress
C A man presents with iron foreign body in the eye. syndrome at birth. On day 14 of life, he developed
Best investigation to monitor the vision sepsis. No other morbidity was present. Retinopathy
Serial electroretinogram. of prematurity should be evaluated at post natal age
of
AIIMS MAY 2014 4 weeks
C The critical angle of cornea-air interface is C A person is diagnosed to be a diabetic on his 45th
46° birthday. You will recommend a dilated fundoscopic
examination
C Diagnostic procedure not done in a dilated pupil
Immediately
Gonioscopy
C A 60 years old male patient presents with dimness C Microaneurysms are the earliest manifestation of
of near vision. On examination, the media was clear diabetic retinopathy. Layer of retina involved in
in both the eyes. Next done diabetic retinopathy
Refraction with near add Outer plexiform layer
C Infection of organism that is highly virulent and C Pneumoretinopexy is an outpatient procedure in
may cause corneal ulcer perforation within 48 hours which retinal detachment is sealed with air
insufflation. Gas used in pneumoretinopexy
Pseudomonas
Sulfur hexafluoride
C Circumcorneal vascularization is observed in
deficiency of C Cells affected in glaucomatous optic neuropathy
are
Riboflavin
Ganglion cells
C A patient gives history of chuna falling into his eyes.
Immediate management C Lipogranulomatous inflammation is seen in
Irrigation of both eyes with copious amounts of Chalazion
normal saline, frequent sodium citrate drops and AIIMS NOVEMBER 2013
removal of chuna particles by double eversion of
upper eyelids. C True about degenerative myopia
C Drugs acting on trabecular meshwork and affecting Myopic degeneration can lead to retinal detachment.
the aqueous outflow C Most common cause of bilateral proptosis in children
Pilocarpine Neuroblastoma
116 AIIMS and All India PGMEE—Review Questions

C Optic atrophy is seen in C A patient presents with an upper eyelid swelling.


Retinitis pigmentosa, methanol poisoning and Chalazion is diagnosed. Histopathology reveals
CRAO. Lipogranulomatous inflammation.
C Band shaped keratopathy is formed due to deposi- C An infant weighing less than 1000 gm is more likely
tion of to develop
Calcium Retinopathy of prematurity
C Cells most commonly affected in glaucomatous optic C Most common intraocular metastasis in females
atrophy occur from which of the following primary tumor
Ganglion cells Breast
C True about indirect ophthalmoscopy C Ocriplasmin is the new drug used in
Convex lens is used, magnification is 4–5 times and Vitreomacular adhesion
it is so bright that regular haziness is penetrated. C The risk of rhegmatogenous retinal detachment is
C A 36 years old lady on prone dark room test increased in
develops pain in the eyes. Drug avoided Pseudophakia, lattice degeneration and trauma.
Atropine C The most common presentation of retinoblastoma
C Concentration of tropicamide is
0.01 Leucocoria and strabismus
C True about concentration of proteins in senile C True about acute conjunctivitis
cataract Topical antibiotic is the antibiotic of choice, pupil is
More insoluble protein, less soluble protein. not affected and vision is not affected.
C Changes seen in conjunctiva in vitamin A deficiency C True about degenerative myopia
Hyperkeratosis of squamous epithelium. Myopic degeneration can lead to retinal detach-
C Drug avoided in patient of narrow angle glaucoma ment.
Homatropine
C True about pterygium AIIMS NOVEMBER 2012
Bare sclera technique has 30–80% recurrence. C Bilateral proptosis in children is the most common
C Long spaced collagen is present in presentation of
Cornea Chloroma
C Tonometer used for irregular surface of the cornea
AIIMS MAY 2013 MacKay-Marg tonometer
C Pair of drug correctly matched with its mechanism C Tonometry with variation in applanation surface is
of action Maklakov tonometer
Brimonidine-decreased aqueous formation. C Variable range tonometer
C Salt and pepper retinopathy is observed in Rebound tonometer
After healing of macular edema C Ciliary muscles are embryologically derived from
C Calcification is seen in Neural ectoderm
Retinoblastoma, choroidal osteoma and optic nerve C Most common complication after lens extraction in
head drusen. persistent hyperplastic primary vitreous
C In a district hospital in India, an ophthalmologist is Retinal detachment
expected to perform which surgery most C Protozoan affecting the eye
commonly Toxoplasma
Phacoemulsification C Given as eye drops in corneal ulcer
C A 55 years old diabetic patient presents with Olaptadine, methylcellulose and chloremphenicol.
unilateral transient obscuration for 2–3 days C The combination of fixed dilated pupil with iris
followed by sudden diminution of vision. Next atrophy and secondary glaucoma following
done penetrating keratoplasty is known as
Serum homocysteine level Urrets- Zavalia syndrome
Ophthalmology 117

AIIMS MAY 2012 C Percentage of endothelial cell loss during


Descemet’s membrane stripping in automated
C Corneal dystrophies are usually penetrating keratoplasty
Primarily bilateral 30–40%
C Weakness of both adduction and abduction is seen C Topical administration of 1% pilocarpine failed to
in produce constriction in a patient who had a large
Duane’s retraction syndrome type 3. dilated pupil. Cause
C Area of fundus seen with direct ophthalmoscope Pharmacological blockade
2 DD
C Gyrate atrophy patient with defective ornithine AIIMS MAY 2011
aminotransferase will be benefitted by
C A patient with clinically significant diabetic
Arginine free diet
macular edema with non-progressive diabetic
C In patients with anterior uveitis, decrease in vision retinopathy was treated with macular grid photo-
due to posterior segment involvement can occur coagulation. The patient still has vitreomacular
because of traction. Preferred treatment is
CME Pars plana vitrectomy
C Cause of a patient hit on the eye by tennis ball with C Baby born prematurely at 29 weeks on examination
fundoscopy showing red dot at 42 weeks with ROP both eyes shows stage 2 zone
Berlin’s edema 1 plus disease. Management is
C Ocular dendritic cells have Laser photocoagulation of both eyes
HLA 2
C A patient presented with unilateral proptosis , AIIMS NOVEMBER 2010
compressible and increasing on bending forward,
but no thrill or bruit and MRI showing a retro- C Retinitis pigmentosa is a feature of
orbital mass with enhancement. Diagnosis Refsum disease, NARP and abetalipoproteinemia.
Orbitall varix C 25 years old presenting with painless sudden loss of
vision but ocular and systemic examination is not
C A 40 years old male patient with diabetes presents
contributory. Diagnosis is
with vitreous hemorrhage. Cause
Eales disease
Neovascularization at disc
C Stocker’s line is seen in
C A young adult presented with diminished vision.
Pterygium.
On examination he has anterior uveitis, vitritis,
focal necrotizing granuloma, macular spot. C A 60 years old man with history of DM and HTN for
Diagnosis is 10 years. Reduced vision in one eye. Central bleed
on fundus examination. Other eye is normal. Most
Ocular toxoplasmosis
probably it is a case of
Diabetic retinopathy
AIIMS NOVEMBER 2011 C Treatment of threshold ROP
Laser photocoagulation
C Seen in 3rd cranial nerve palsy
Diplopia, ptosis and outward eye deviation.
C Dilator pupillae is supplied by AIIMS MAY 2010
Post-ganglionic sympathetic fibers from cervical C Disease with autosomal dominant inheritance is
sympathetic chain. Best disease
C Mizou phenomenon is seen in C A patient presenting with unilateral proptosis,
Oguchi’s disease compressible and increasing on bending forward
C Drug currently used for the prophylaxis of non- but no thrill or brui present. MRI shows a retroorbital
infectious uveitis in LUMINATE programme mass with enhancement. Diagnosis is
Voclosporin Orbital varix
118 AIIMS and All India PGMEE—Review Questions

C Free radical scavengers in lens AIIMS NOVEMBER 2008


Vitamin C, vitamin E and catalase
C Highest refractive index in part of eye C A 20 years old female presents with proptosis and
abducens nerve palsy. On MRI scan, hyperintense
Centre of lens
lesion were seen on T2 weighted images which
Mucopolysaccharide hyaluronic acid is present
showed intense homogenous contrast enhancement.
C
in
Diagnosis is
Vitreous humor
Schwannoma
C Features of Horner’s syndrome
AIIMS NOVEMBER 2009 Ptosis and miosis in ipsilateral eye, anhydrosis of
C Most common malignant tumor of eyelid ipsilateral face and heterochromia irides.
Basal cell carcinoma C Inclusion conjunctivitis is caused by
C Blowout fracture of orbit leads to fracture of Chlamydia trachomatis.
Posteromedial floor of orbit. C Left sided sixth nerve palsy would lead to
C Sclera is thinnest at Diplopia in left gaze.
Posterior to attachment of superior rectus.
C Adult male with restriction of eye movements in all AIIMS MAY 2008
directions and moderate ptosis but with no diplopia C Topical antifungal in keratomycosis is
or squint. Diagnosis is Natamycin
Chronic progressive external ophthalmoplegia. C Retinitis pigmentosa is associated with
C Aniridia is associated with Usher syndrome, Kornzweig syndrome and Kearn-
Nephroblastoma Sayre syndrome.
C Headache with bitemporal hemianopia with 6/6 C Choroidal neovascularization is seen in
vision is seen in Myopia, angioid streak and trauma.
Chiasmal lesion C Iridocorneal endothelial syndrome is associated with
C Ipsilateral third nerve palsy with pupillary sparing Progressive atrophy of iris stroma.
is seen in C Pterygium is
Diabetes mellitus. Connective tissue degeneration.
C Chalazion of lid is
AIIMS MAY 2009 Chronic lipogranulomatous inflammation.
C Corneal endothelium is derived from C An orbital tumor has following characteristics-
Retrobulbar location within muscle cone, well-
Neural crest
defined capsule, slowly progressive proptosis,
C A patient presents with diplopia with limitation of easily resectable, most common in 2nd to 4th
adduction in the left eye and abducting saccades in decade. It is a case of
right eye. Convergence is preserved. Cause is
Cavernous hemangioma
Internuclear ophthalmoplegia. C An 11 years old patient presents with left sided head
C Most common cause of fracture, roof of orbit is tilt, on correcting that he has right sided hypertropia.
Blow on the forehead The hypertropia increases on left gaze and tilting
C A young man with blurring of vision in right eye the head towards right. Muscle paralyzed is
followed by left eye after 3 months, showing disc Right superior oblique.
hyperemia, edema, circumpapillary telangiectasia, C Factors involved in pathogenesis of macular edema
with normal pupillary response with centrocecal in diabetic retinopathy
scotoma on perimetry. Cause is Oxidative retinopathy, VEGF and increased protein
Leber’s hereditary optic neuropathy. kinase C.
C Anteroposterior stability of eyeball is provided C Telecanthus is
by Increase in intercanthal distance with normal
Superior oblique, superior rectus and orbital fat. interpupillary distance.
Ophthalmology 119

C Reciprocal inhibition of antagonistic muscle upon C The afferent pathway for light pupillary reflex is
lateral gaze is explained by Optic nerve
Sherrington law. C True about intraocular retinoblastoma
94% of cases are sporadic, calcification in the tumor
AIIMS NOVEMBER 2007 can be detected on ultrasound scan and Reese
Ellsworth classification is useful in predicting
C Ionic pump in corneal endothelium is necessary for visual prognosis following radiotherapy.
maintaining deturgescence of the cornea and thus
C Factors of prognostic significance in choroidal
transparency. It can be blocked by
melanoma
Inhibition of anaerobic glycolysis.
Size of tumor, cytology of tumor cells and presence
C SAFE strategy is used for of extra-ocular extension.
Trachoma
C Corneal transparency is maintained by AIIMS NOVEMBER 2005
Relative dehydration, unmyelinated nerve fibers
and uniform spacing of collagen fibrils. C Drug of choice in a 50 years old lady presenting
C Dangerous zone of eye is with history of severe eye pain, redness, diminution
Ciliary body of vision, on examination-visual acuity 6/60,
circumcorneal congestion, corneal edema, shallow
anterior chamber
AIIMS NOVEMBER 2006
IV mannitol
C Oculomotor nerve palsy causes C A 30 years old male patient presenting with history
Ptosis, diplopia and outward eye deviation. (Mn: of recurrent headaches, on fundus examination,
OPD) generalized arterial attenuation, multiple cotton
wool spots and flame shaped hemorrhages in both
AIIMS MAY 2006 eye. Diagnosis
Hypertensive retinopathy.
C Ocular structures derived from surface ectoderm
C A female patient presents with loss of vision in both
Corneal epithelium, crystalline lens and epithelium
eyes, on examination-normal pupillary responses,
of lacrimal glands.
normal fundus, and visually evoked response
C True stereopsis is perceived due to examination showing extinguished response.
Binasal disparity. Diagnosis
C True regarding PHPV Cortical blindness
It is generally unilateral C The most common retrobulbar orbital mass in
C Mucin layer tear film deficiency occurs in adults is
Keratoconjunctivitis sicca. Cavernous hemangioma
C The laser procedure, commonly used for treatment C Drug contraindicated in patient with history of
of iris neovascularization is sulfa allergy presenting with acute attack of angle
Panretinal photocoagulation. closure glaucoma
C Most important factor in prevention of endophthal- Acetazolamide
mitis in cataract surgery C The most common systemic association of scleritis is
Preoperative preparation with povidone iodine. Rheumatoid arthritis
C The crystalline lens derives its nourishment from C Angioid streaks in the eyes are seen in
Aqueous and vitreous. Pseudoxanthoma elasticum.
C Histopathology of recurrent chalazion is done to C The corneal transparency is maintained by
exclude- Sebaceous cell carcinoma. Endothelium
C Band shaped keratopathy is caused by C A 50 years old patient complains of decreased
Calcium distance vision now does not need his near glasses
C Bitemporal heminopic field defect is characteristic of for near work. Diagnosis is
Pituitary tumor Nuclear sclerosis
120 AIIMS and All India PGMEE—Review Questions

C A patient presenting with right homonymous C On refraction testing using a plane mirror in a
hemianopia with saccadic pursuit movements and patient with refractive error of 3D sphere and 2D
defective optokinetic nystagmus has lesion in cylinder at 900 from 1 metre distance under no
Parietal lobe cycloplegia, the reflex is seen to move
C Idiopathic nyctalopia is due to Against the movement in both the axes.
Hereditary absence of rod function. C A 25 years old male patient presents with history of
redness, pain and mild diminution of vision in one
AIIMS MAY 2005 eye for past 3 days, history of low backache for past
one year, on examination- circumcorneal
C Donor tissue in human cornea transplantation is congestion, cornea is clear apart from a few fine
Donation human cadaver eyes. keratic precipitates on the corneal endothelium, 2 +
C Berlin’s edema is seen cells in the anterior chamber, IOP within normal
After concussional trauma limits. Diagnosis is
C Saccade is defined as HLA B-27 related anterior uveitis.
Abrupt involuntary rapid eye movements. (Mn. C Initial management of a soft contact lens user
AIR). presenting with pain, watering, photophobia and a
C Painless sudden visual loss is seen in white spot in the centre of the cornea
CRAO, retinal detachment and vitreous Start frequent antibiotic eye drops after disconti-
hemorrhage. nuing the contact lens.
C The conversion of CO2 and H2O into carbonic acid C Appropriate management of a 3 years old child
during the formation of aqueous humor is catalysed presenting with right convergent squint of 6
by months duration
Carbonic anhydrase Proper refractive correction, amblyopia therapy
C Laser used for treating aftercataract followed by surgical correction.
Nd-YAG (Mn: AND). C A 30 years old immunocompetent farmer from a
rural community presents with difficulty in vision
AIIMS NOVEMBER 2004 in the left eye for last 10 days and history of trauma
to his left eye with vegetative matter 10–15 days
C Best refractive surgery for a 20 years old girl student
back, on examination ulcerative lesion in the
presenting with myopia of-2.0 D and post-
cornea, whose base has raised soft creamy infiltrate,
traumatic nebular corneal opacity in right eye
ulcer margin is feathery and hyphate and a few
Photorefractive keratectomy. satellite lesion. Etiology is
C A 50 years old man undergoing chemotherapy for Fusarium
non-Hodgkin’s lymphoma one year back, now
C Cranial nerve lesion in a patient presenting with
presents with painful weeping rashes over the right
normal eye sight absence of direct and consensual
upper eyelid and forehead for last 2 days along with
light reflexes
ipsilateral acute punctate keratopathy. Diagnosis is
Oculomotor nerve.
Herpes zoster
C A 20 years old girl student using spectacles for last
10 years presents with history of photopsia and AIIMS NOVEMBER 2003
sudden loss of vision in right eye. Clinical C Old age patient presents with history of increasing
examination performed to clinch the diagnosis pain and diminution of vision after an initial
Indirect ophthalmoscopy improvement, coming to eye emergency after 3
C A 30 years old IDDM patient on insulin for the past days of cataract surgery. Cause is
10 years presents with gradually progressive Endophthalmitis
painless loss of vision. Diagnosis C Signs resulting from infection within the right
Cataract cavernous sinus are
C Drugs used in glaucoma Loss of pupillary light reflex, ptosis and right
Dipivefrine, physostigmine and timolol ophthalmoplegia.
Ophthalmology 121

C Condition likely to be worsened in a case of POAG, C Types of glaucoma commonly seen in the orbit
when treated with timolol maleate 0.5% eye drops Burkitt’s lymphoma, NHL (mixed lymphocytic and
are histiocyctic) and NHL (lymphocytic poorly
Bronchial asthma, depression and hypercholeste- differentiated).
rolemia.
C Management of a 25 years old executive presenting AIIMS NOVEMBER 2002
with metamorphosis in right eye, on examination
fundus showing shallow detachment at the macula, C Most visually handicapping type of cataract
fluorescein angiography showing a smoke stack Posterior subcapsular
sign C A 2 years old child presents with leucocoria in left
Just wait and watch for spontaneous recovery. eye for 2 months. On examination—Total retinal
C A patient presents with sudden loss of vision in the detachment in same eye, US/B scan showing
right eye following injury to right brow due to fall heterogeneous subretinal mass with calcification.
from scooter, on examination the pupil shows Diagnosis
absent direct light reflex but normal consensual Retinoblastoma
pupillary reflex present, fundus is normal. C A premature infant was delivered at 27 weeks of
Treatment is gestation with birth weight 1500 gm. Fundus
Intensive IV corticosteroids as prescribed for spinal examination is requested by an ophthalmologist
injuries to be instituted within six hours. At 34 weeks gestation age.
C A patient with spectacle correction of 6.0 and 8.0
AIIMS MAY 2003 one morning complains of seeing some opacities
floating in front of his eye with history of slight
C Anisokonia means decrease in vision over last few days. Investigation
Difference in the size of images formed by the two advised
eyes. Indirect ophthalmoscopy.
C Common ocular manifestation in Trisomy 13 is C According to NPCB surgery (1938–1989), the
Bilateral microphthalmos. highest prevalence of blindness in India is in
C Optic nerve injury results in J and K
Loss of vision in that eye, loss of light reflex and C In trachoma grading, Trachomatous inflammation-
dilation of pupil. follicular is defined as the presence of 5 or more
C Enucleation of eyeball is contraindicated in follicles in the upper tarsal conjunctiva.
Panophthalmitis C An 8 years old boy presents with swelling in the left
C The differential diagnosis of retinoblastoma eye for 3 months duration, on examination-proptosis
includes of left eye with preserved vision. Right eye is
Coat’s disease, PHPV, and retinal astrocytoma. normal. CT- intraorbital extraconal mass lesion,
biopsy embryonal rhabdomyosarcoma, metastatic
C Management of congenital glaucoma is
work up normal. The standard line of treatment is
Trabeculotomy with trabeculectomy.
Chemotherapy and radiation therapy.
C Under school eye screening progamme in India,
C Risk factor for acanthamoeba keratitis in 18 years
initial vision screening of school children is done by
old girl presenting with keratitis and severe pain in
School teachers. the eye are
C World bank provided assistance to NPCB (1924– Extended near contact lens, exposure to dirty water
2001) for and corneal trauma.
Cataract C Significant loss of vision in a patient with hyper-
C Disease under vision 2020 goals,WHO are tension can occur due to
Refractive errors, cataract and trachoma. Anterior ischemic optic neuropathy, occipital
C Gene mutation that may give rise to hereditary infarct and retinal hemorrhage.
glaucoma C Diagnosis of a 30 years old male patient presenting
Optineurin with a history of injury to the eye with a leaf 5 days
122 AIIMS and All India PGMEE—Review Questions

ago and pain, photophobia and redness of eye for 2 blurred vision, history of trauma with vegetative
days matter, on examination dentiritic ulcer in cornea,
Fungal corneal ulcer microscopic examination of corneal scraping shows
C A 60 years old man present with watering from his macrophage like cells on culturing the corneal
left eye since 1 year, syringing revealing patent scraping over a non-nutrient agar enriched with E.
drainage system. Rest of ocular examination coli, there were plaque formations. Organisms most
normal. A provisional diagnosis of lacrimal pump likely is
failure was made. Confirmed by Acanthamoeba trophozoites
Dacryoscintigraphy
C Incision in a standard sutureless cataract surgery ALL INDIA 2012
done with phaco-emulsification and foldable job is
3 mm–3.5 mm C A malnourished child from poor socioeconomic
C Most common cause of ocular morbidity in the status, living in overcrowded dirty areas presented
community with a nodule around limbus with hyperemia of
Refractive error conjunctiva and axillary and cervical lymphadeno-
pathy. Diagnosis
Phlyctenular conjunctivitis.
AIIMS MAY 2002
C Cause of CRAO
C Uveitis is most commonly associated with Orbital mucormycosis
Pauciarticular JRA C Fundoscopy of a patient shows chalky white optic
C Fluorescein angiography is used to identify lesion disc, the rest of retina absolutely normal. Diagnosis
in Primary optic atrophy.
Iris, optic nerve and retina C A 20 years male patient presents with night blindness,
C If axial length of eyeball change by 1mm, then the tubular vision, IOP 18 mm Hg, fundoscopy-
power changes by attenuation of arteries and waxy pallor of optic disc,
3D perimetry—Ring scotoma, ERG subnormal.
C Lens contains the oldest cells in Diagnosis
Nucleus Pigmentary retinal dystrophy.
C Recurrent chalazion is predisposed to develop C Most common mode of spread retinoblastoma
Adenocarcinoma of Meibomian gland Optic nerve
C Optic radiation lesion involving Meyer’s loop causes C A patient complains diminished vision, 3 weeks
Superior quadrantanopia after IOL implantation and fundus fluorescein
C Horner’s syndrome consists of angiography—Flower petal hyperfluorescence of
Miosis and ptosis > Miosis and exophthalmos. macula. Diagnosis
C Internuclear ophthalmoplegia is caused due to Aphakia
lesion in C A patient presents with diplopia in one eye. On
Medial longitudinal fasciculus examination with oblique illumination- Golden
C The best irrigating fluid during ECCE crescents and on axial illumination- Black/dark
Balanced salt solution and glutathione. crescents. Diagnosis
C A 30 years old male patient presenting with history Ectopia lentis. (Mn: Ectopic Ab Go)
of proptosis in right eye and pain on eye movement C A child presents with bilateral white pupillary
for last 15 days with difficulty in upward and down reflex. On slit lamp examination, there is zone of
ward gaze movement with CT scan showing cystic opacity around fetal nucleus with spokes of wheel
lesion with a hyperdense opacity with it, located in like arrangement toward centre. Diagnosis
superior oblique muscle. Diagnosis Lamellar cataract
Cysticercosis cellulosae. C Following 2 days of successful emulsification and
C A young adult male patient presenting with red IOL placement, a diabetic patient presents with
eye, complains pain, photophobia watering and redness, pain and grey white pupillary reflex.
Ophthalmology 123

Patient also shows hypopyon, retrolental flare and C Young patient with loss of central vision. No
posterior synachiae. Diagnosis obvious family history. ERG and EOG normal
Postoperative endophthalmitis. Stargardt’s disease.
C A 35 years old male patient on slit lamp shows C Young patient with loss of central vision. ERG is
keratic precipitates and aqueous flare in right eye, normal but EOG is abnormal
no synachiae but complicated cataract seen. Best’s vitelline dystrophy.
Diagnosis C Drug used in treatment of diabetic macular edema/
Fuch’s heterochromic iridocyclitis. diabetic retinopathy
C A 35 years old female patient presents with recurrent Ruboxistaurin, pyridazinones and benfotiamine.
chalazion of the upper eyelid and curettage subjec- C The most common cause of bilateral proptosis in
ted to histopathological examination to rule out children
Sebaceous cell carcinoma Chloroma
C An elderly diabetic male patient presents with C The most common malignant orbital tumor in
severe panophthalmitis with orbital cellulitis, children is
sample collected from periorbital region on Gram Rhabdomyosarcoma
staining shows irregularly branching, aseptate and
C A 5 years old boy presents with leucocoria in right
broad hyphae. Etiological agent
eye due to diffuse retinoblastoma involving entire
Apophysomyces elegans.
globe. 2–3 mm tumor in left eye in the periphery
C A regular contact lens user presents with complaints ideal management
of redness, photophobia, blurring of vision in one eye
Enucleation of right eye and focal therapy of left eye.
for more than 2 weeks, not responding to treatment,
Young boy presenting with decreased vision, six
on examination- cornea shows ring shaped lesion along
C
months after being hit by a ball in the eye. Finding
with some overlying epithelial defect. Diagnosis
on optical examination suggestive of blunt injury to
Acanthamoeba keratitis.
the eye
C Wavelength of laser used for shaping cornea in
Vitreous base detachment.
refractive surgery
193 nm (Hint: Excimer laser- UV laser)
ALL INDIA 2010
ALL INDIA 2011
C Endophthalmitis involves inflammation of
C Late onset endophthalmitis after cataract surgery is Uvea, retina and vitreous.
most often caused by C Mechanism of antimicrobial resistance in isolated
Propionibacterium acne Pseudomonas strains in a patient of keratitis after
C A patient presents with history of running nose and prolonged usage of contact lenses
pain over medial aspect of eye, sudden onset fever, Ability of Pseudomonas to produce biofilms.
prostration, chemosis, proptosis and diplopia on
C Least common corneal dystrophy
lateral gaze with congestion of optic disc. It is a case of
Macular dystrophy
Cavernous sinus thrombosis.
C Ophthalmoplegic migraine is best defined by C True regarding corneal transplantation
Recurrent transient 3rd, 4th and 6th nerve palsy Specular microscopy analysis is used to assess
associated with headache. endothelial cell count.
C Diplopia in superior oblique palsy is described as C Relative afferent pupillary defect is characteristically
Vertical on looking down. seen in damage to
C True about Argyll Robertson pupil Optic nerve
Near reflex normal, visual acuity normal and direct C Ptosis with retraction of ptotic eye lid on chewing is
light reflex absent. Marcus Gunn jaw winking syndrome.
C Patient has flashing light, sudden floaters and C A person with known mutation in Rb gene is
perception of a curtain falling in front of eye. Most disease free from retinoblastoma. He is at the
likely diagnosis is highest risk of developing
Rhegmatogenous retinal detachment. Osteosarcoma
124 AIIMS and All India PGMEE—Review Questions

C Arden index is related to C Intractable secondary glaucoma is found in


EOG Diffuse iris melanoma
C Persistent hyperplastic primary vitreous is associa-
ALL INDIA 2009 ted with
Patau syndrome
C Factors stimulating angiogenesis in a fetus
VEGF, bFGF and IL-8
ALL INDIA 2007
C Increased LDH in aqueous humor is suggestive of
Retinoblastoma C Transport of ascorbic acid to lens is done by
C Indocyanin green angiography is most useful in Na+/K+ ATPase
detecting C Which handle oxidative damage in lens
Occult choroidal neovascularization. Vitamin C, glutathione and catalase.
C Features differentiating CRVO from ocular C True about papilloedema
ischemic syndrome due to carotid artery stenosis Extracellular edema, stasis of axoplasmic flow and
Tortuous retinal vein, retinl artery pressure and axonal swelling.
ophthalmodynamometry. C Cherry red spot is not seen in
C Treatment of advanced proliferative diabetic Krabbe’s disease.
retinopathy with extensive vitreoretinal fibrosis C Lamina cribrosa is absent in
and tractional retinal detachment involves Morning glory syndrome.
Vitrectomy, removal of epiretinal membrane and
reattachment of detached or torn retina.
ALL INDIA 2006
C True about juxtafoveal retinal telangiectasia
Variant of Coat’s disease, associated with C Afferent component of corneal reflex is meditated by
telangiectasia in the macula and associated with Trigeminal nerve
structurally abnormal vessels. C The average distance of fovea from temporal
C Uveal effusion syndrome may be associated with margin of optic disc is
Nanophthalmos, ciliochoroidal detachment and 2 disc diameter
structural defect in sclera. C Blood supply of retina are
Retinal arteries, central retinal artery, and plexus of
ALL INDIA 2008 Zinn and Haller arteries.
C The magnification obtained by direct ophthalmo-
C The WHO definition of low vision in children scopy
Visual acuity 6/18 to 6/60 in better eye with best
15 times
correction.
C SAFE strategy
C The most common cause of ocular morbidity in
Trachoma
India is
Refractive error C Drug of choice for corneal ulcers caused by
C The major function of MIP 26 is filamentous fungi
Transport of water in lens. Natamycin
C Seen in interphotoreceptor matrix C Antiglaucoma medications causing drowsiness
Metalloproteinase, SPARC and TIMP. Brimonidine
C Final common pathway for horizontal pathway C Drugs used topically in open angle glaucoma
involves Dorzolamide, timolol and latanoprost.
Abducent nucleus C Drug causing macular toxicity when given intra-
C von Recklinghausen disease is associated with vitreally
Glaucoma, optic nerve glioma and neurofibroma of Gentamicin
the lids. C A case of NIDDM with history of diabetes for 1 year
C Sarcoidosis is associated with should have ophthalmic examination
Band keratopathy As early as feasible
Ophthalmology 125

C The most common cause of vitreous hemorrhage in C Diagnosis of 2 years old child presenting with white
adults is reflex in one eye for past 1 month with CT finding
Diabetes mellitus of calcification within the globe is
C Typically bilateral inferior lens subluxations of the Retinoblastoma
lens is seen in C Retinal vascular tumors are often associated with
Homocystinuria intracranial hemangioblastoma in VHL syndrome.
C Uveitic conditions contraindicated for intraocular Site of such vascular abnormalities
lens implantation after cataract surgery Cerebellum
JRA C A 20 years old boy presenting with history of injury
C Autosomal recessive stromal dystrophy with tennis ball, on examination—No perforation
Macular dystrophy but hyphema present. Source of bleeding
Circulus iridis major
C True about pthisis bulbi
Calcification of the lens is common, sclera is
ALL INDIA 2004
thickened and size of globe is reduced.
C The most common retro-bulbar orbital mass in C Maximum density of goblet cells in seen in
adults is Nasal conjunctiva
Cavernous hemangioma C Diagnosis of a 10 years old boy presenting with
C True about optic glioma severe itching of eye and ropy discharge with
Peak incidence in first decade, arises from symptoms aggravating in summer season
oligodendrocytes and associated with type 1 NF. Vernal keratoconjunctivitis.
C The most common second malignancy in survival C Arlt’s line is found in
of retinoblastoma Trachoma
Osteosarcoma C Most important adjuvant therapy in fungal corneal
ulcer
ALL INDIA 2005 Atropine sulphate eye ointment
C Polychromatic lustre is seen in
C Horner’s syndrome is characterized by Complicated cataract
Miosis, enophthalmos and ptosis. C Best management for a two weeks old child
C A 25 years old man presenting with 6/5 vision each presenting with unilateral cataract
eye, unaided, cycloplegic retinoscopy + 1.0 D sph at The best age to operate him to get the best visual
1 metre with complains of blurring of new sprints at results is four weeks.
30 cm that clears up in 30 second. Diagnosis (Note: as early as possible)
Accomdative inertia C Prominent ocular condition associated with
C A 30 years old male presenting with sudden Marfan’s syndrome
painless loss of vision in one eye for past 2 weeks, no Ectopia lentis
history of trauma, on examination normal anterior C Diagnosis of a two months old child presenting
segment but no fundal glow. Diagnosis with epiphora and regurgitation is
Vitreous hemorrhage Congenital dacryocystitis
C True about Acanthameba keratitis C Photodynamic therapy is used in the eye for
Acanthameba does not depend upon a human host Wet AMD (Age related macular degeneration).
for completion of its life cycle. C Ocular emergencies are
C Contact lens have deleterious effect on corneal Angle closure glaucoma, central retinal arterial
physiology. True statements are occlusion and retinal detachment.
Reduction in glucose utilization by corneal epithe- C The most common cause of inherited blindness due
lium, increased production of CO2 in the epithelium to mitochondrial chromosomal anomaly is
and reduction in hemidesmosome density. Leber’s hereditary optic neuropathy.
C Enlarged corneal nerves may be seen in C Differential diagnosis of white pupillary reflex are
Kerotoconus, leprosy and NF. Cataract, retinoblastoma and retrolental fibroplasia.
126 AIIMS and All India PGMEE—Review Questions

ALL INDIA 2003 C Overnight storage of a sample of vitreous aspirate


collected in an emergency at 9 pm by
C A young adult man presents with suddenly The sample should be kept at 4°C.
developed double vision, on examination-right eye, C Vortex vein invasion is commonly seen in
when at rest, turned medially. Structure involved
Malignant melanoma
are
Lateral rectus and abducent nerve.
C The commonest cause of low vision in India ALL INDIA 2002
Uncorrected refractive errors. C In the normal human right eye, peripheral field of
C Summer onset recurrent bilateral conjunctivitis in a vision is least in
young boy presenting with burning, itching, and Upward direction
lacrimation with polygonal raised areas in the C Tonography helps to determine
palpebral conjunctiva is The facility of outflow of aqueous.
Vernal keratoconjunctivitis. C If a person needs more one of the color for matching
C Type IV hypersensitivity to Mycobacterium than a normal person, then he has a color anomaly.
tuberculosis antigen may present as More red color is needed in the case of
Phylectenuler conjunctivitis. Protanomaly
C Drug treatment of a contact lens wearer patient C The colors best appreciated by the central cones of
developing corneal infection wih acanthamoeba our foveomacular area are
keratitis Red and green
Polyhexa methylene biguanide. C Epiphora is defined as
C Chorioretinitis in AIDS patient is typically caused An abnormal overflow of tears due to obstruction of
by lacrimal duct.
Cytomegalovirus C Oculomoter nerve palsy affects
C A man sustains a gunshot injury in his left eye. Inferiooblique, medial rectus and levator palpabrae
Three weeks after, he complains of difficulty in superioris.
reading the newspaper with his right eye. Diagnosis C Diagnosis of a patient presenting with acute painful
Sympathetic ophthalmitis. red eye and mild dilated vertically oval pupil
C A child presents with congenital cataract involving Acute angle closure glaucoma.
visual axis, detected right at birth. He should be C Ocular finding helpful to rule out open angle
operated glaucoma in middle aged patient
Immediately Cupping of the disc
C Factor suitable for performing LASIK surgery in a girl C Most useful drug to reduce intraocular pressure in
Corneal thickness of 600 μ, myopia of 4D and stable hypertensive uveitis
refraction for 1 year. Latanoprost
C After enucleation of painful blind eye, a proper C Blepharoconjunctivitis is side effect of antiglaucoma
sized prosthetic eye is fitted after a post-operative drugs
period of Timolol
About 21 days. C A 10 years old child presents with complaints of
C Management of a one year old child presenting headache and decreased vision, on examination
with leucocoria, detected to be having a unilateral visual acuity of 6/36 in right eye 6/6 in left eye, on
large retinoblastoma filling half the globe retinoscopy at 66 cm-correction of 5 D in right eye
Enucleation and 1.5 D in left eye, anterior chamber and fundus
C Fasanella servan operation is specifically indicated in of eye normal. Cause of depressed vision in right eye
Horner’s syndrome Amblyopia
C The operation of plication of inferior lid retractors is C Wernick’s hemianopic pupillary response is seen in
indicated in lesion of
Senile entropion Optic tract
10

Ear, Nose and Throat

AIIMS NOVEMBER 2015 C Kashima operation is done for


Bilateral vocal cord palsy
C Screening for hearing in neonates by C Test for eustachian tube malfunction
Transient otoacoustic emissions. Tympanometry
C Nerve supply of ear lobule
Greater auricular nerve. AIIMS NOVEMBER 2014
C A patient presents to you with difficulty in hearing.
True about malignant otitis externa
Rinnie’s test shows air conduction greater than
C

bone conduction on both sides with Weber’s test Diabetic and immnocompromised people are more
lateralized to right ear. Next done susceptible, granulation tissue is seen on the floor of
the external auditory canal on otoscopy and ESR is
Schwabach’s test
used for monitoring the disease.
(Note: To confirm the findings of Rinnie’s and
C True about Meniere’s disease
Weber’s test.
Electrocochleography is the gold standard inves-
Positive Rinnie’s test is AC > BC in normal persons
tigation and Semont maneuver decreases giddi-
and sensorineural deafness. Mn: PoRi AB normally
ness.
sensed.
C A child has retained disc battery in the nose. The
Weber’s test-Lateralised in conductive deafness to most appropriate statement
diseased ear. Mn: LCD)
Battery contents might leak resulting into chemical
C Most common type of congenital ossicular damage of the surrounding tissue.
dysfunction
C Condition in which topical steroid is not indicated
Stapes defect with fixation of foot plate and After endoscopic surgery for antrochoanal polyp.
lenticular process involvement. (Main etiology is the infection)
C Auditory neuropathy is an effective modality of C A patient presents with traumatic head injury and
treatment for CSF leak. Next step
Meniere’s disease Wait and watch for 3–5 days.
C The complications of adenoidectomy include
AIIMS MAY 2015 Retropharyngeal abscess, cervical spine injury and
velopharyngeal insufficiency.
C Most common source of bleeding in a case of (Note: also hypernasality)
nasopharyngeal angiofibroma C The most common cause of vocal cord palsy
Internal maxillary artery Malignant disease
C Topical treatment for recurrent respiratory papillo- C An elderly man presents with T3N0 laryngeal
matosis carcinoma. Managed by
Cidofovir Concurrent chemoradiotherapy

127
128 AIIMS and All India PGMEE—Review Questions

AIIMS MAY 2014 C A patient after eating peanut presented with


laryngeal edema, stridor, hoarseness of voice and
C Which of the following show negative Rinne’s test swelling of tongue. Most likely diagnosis
in the right ear Angioneurotic edema
Profound hearing loss, conductive hearing loss of C A 36 years old obese man, who is a known smoker,
40 dB in both ears and conductive hearing loss of 40 snores and has hypertension. In sleep test, 5 apnea/
dB in right ear and left ear normal. hyperapnea per hour was noted. He is put on
C The screening test of high risk neonates in ICU for antihypertensive and advised to quit smoking.
suspected hearing loss is Next to be done
Otoacoustic emissions Weight reduction and diet plan.
C The wave V in BERA represents activity in C A 35 years old pregnant lady complained of hearing
Lateral lemniscus loss, which aggravated during pregnancy, was sent
C Resected in stapedectomy for tympanometry. Graph seen
Anterior crus of stapes, posterior crus of stapes and As
stapedial ligament. (Note: Otosclerosis)
C Part of cochlear implant implanted during surgery
Receiver stimulator AIIMS MAY 2013
C A 75 years old diabetic patient presents with severe C True about Bells palsy
ear pain and granulation tissue at external auditory Steroids are used, unilateral facial weakness and
canal with facial nerve involvement. The most role of herpes simplex.
likely diagnosis C Bone anchored hearing aid(BAHA) can be used in
Skull base osteomyelitis A 7 years old child with bilateral microtia and canal
(Note: Malignant otitis external) atresia with conductive hearing loss.
C A 14 years old boy presents with nasal bleeding. His C A 40 years old man presents with left ear discharge
Hb was found to be 6.4 g/dl and normocytic and mild ear pain for the past 7 years but no history
hypochromic anemia on peripheral smear. The of deafness. On examination, the tympanic membrane
most likely diagnosis is intact and discharge is seen coming from the
Juvenile nasopharyngeal angiofibroma. posterior superior wall. The left ear canal and the
C A patient presents with antrochoanal polyp arising tympanic membrane are normal. Most likely diagnosis
from the medial wall of the maxilla. Best management Chronic otitis media
FESS with polypectomy. C A girl underwent mastoidectomy for chronic ear
C An elderly male patient presents with T3N0 discharge and retroorbital pain. But there was no
laryngeal carcinoma. Patient is managed with relief postoperatively and continuos ear discharge
Concurrent chemoradiotherapy was seen. Most probable diagnosis
(Note: T3 laryngeal cancer is 3P—Palate, pharyngeal Apical petrositis
muscle and paralysis of vocal cord) C Boundaries of the facial recess are
Chorda tympani, facial nerve and short process of
incus. (Mn: CSF)
AIIMS NOVEMBER 2013
C Father of neurology
C Topical steroids are recommended in House
Post-surgery for ethmoidal polyps, chronic C A 70 years old gentleman with diabetes mellitus
rhinosinusitis and allergic fungal sinusitis. presents with persistent ear discharge with fever
C True about tonsillectomy in children are and headache and pain out of proportion. On
Sleep apnea is an indication, adenoids should also examination, granulation tissue is observed in the
be removed if significantly involved and cricothyroid external auditory canal along with facial nerve
region is high and anterior in children than adults. palsy. He is not responding to antibiotics. Most
C What is placed during surgery for cochlear implant probable diagnosis
Receiver stimulator Malignant otitis externa
Ear, Nose and Throat 129

C A 58 years old gentleman presents with persistent C In right handed person, direct laryngoscope is held by
fullness of left ear and hearing loss for 3 months. On Left hand
examination, there is fluid behind the tympanic C A 6 years old child presented with history of
membrane. Impedence audiometry shows a type B recurrent upper respiratory tract infections,
audiogram. Next done mouth breathing, nasal obstruction, hearing
Endoscopic examination for any nasopharyngeal impairment with high arched palate; management
pathology. will be
C An elderly lady presents with nasal blockade, nasal Adenoidectomy with grommet insertion.
discharge, diplopia and facial swelling. On
examination, there is blackish discharge from the
nasal cavity with necrosis of nasal mucosa, septum AIIMS MAY 2011
and hard palate. There is elevated blood sugar and C According to European laryngeal society, subliga-
urinary ketones are positive. Best drug for this patient mental cordectomy is classified as
Amphotericin B. Type 2
C Topical mitomycin is used for
AIIMS NOVEMBER 2012 Treatment for laryngeal stenosis.
C Third window effect is seen in C Infection of inner ear spreads through
Superior semicircular canal dehiscence. Cochlear aqueduct
C Laryngeal pseudosulcus is seen in C Microwick and microcatheter sustained release
Laryngopharyngeal reflux. devices are used in
C A necessary criterian for successful cochlear implant Delivering drug to the round membrane.
Presence of auditory nerve
C Initial mechanism of action of intra-tympanic AIIMS NOVEMBER 2010
gentamycin microwick catheter in treatment of
Meniere’s disease C True regarding retropharyngeal abscess
Inactivates Na+/K+ ATPase channels of hair cells. Confined to one side of midline, can be palpable per
C High frequency audiometry is used in orally by pressing the finger on posterior pharyn-
geal wall and presents with dysphagia and
Ototoxicity
difficulty in breathing.
C Initial screening test for newborn hearing disorder
C True about cochlear implant
Otoacoustic emissions
Not contraindicated in cochlear malformation.
C Laser uvulo pharyngo-palatoplasty is the surgery
for C Laparoscopic intranasal approach is used for
Snoring Lacrimal sac, pituitary gland and optic nerve.
C Frontal sinus can be best visualized by
Caldwell’s view
AIIMS MAY 2012
C CSF rhinorrhoea, immediate treatment
C Initial screening test for newborn hearing disorder Antibiotics and wait and watch for 7 days.
Otoacoustic emissions (OAE)
C Vestibular evoked myogenic potential (VEMP) AIIMS MAY 2010
detects lesion of
Inferior vestibular nerve. C Most common site for CSF rhinorrhoea is
C Second primary tumor of head and neck is most Cribriform plate > Ethmoid sinuses
commonly seen in malignancy of C Endolymph in the ear
Oral cavity Is secreted by stria vascularis.
C In electrocohelography C Bell’s palsy patient presenting on day 3. Treatment
Evoked potential generated in cochlea and auditory given is
nerve. Oral steroid and acyclovir
130 AIIMS and All India PGMEE—Review Questions

AIIMS NOVEMBER 2009 C A patient presents with carcinoma of larynx


involving left false cords, left arytenoid and left
C Radiotherapy is treatment of choice for aryepiglottic folds with bilateral mobile true cords.
Nasopharyngeal carcinoma. T3N1. Treatment of choice is
Horizontal partial hemilaryngectomy.
AIIMS MAY 2009 C A 32 years old male patient presents with nasal
discharge, facial pain and fever which subsided
C High frequency audiometry is used in with antibiotics and antihistaminics, but recurred
Ototoxicity again after 2 months. On examination mucopuru-
C Positive Romberg test with eyes closed detects lent discharge from middle meatus and inflammed
defect in sinus opening, investigation of choice
Propioceptive pathway NCCT PNS
C Higher auditory centre determines
Sound localization AIIMS MAY 2007
C A lady with bilateral hearing loss for 4 years.
AIIMS NOVEMBER 2008 Worsen during pregnancy. Type of impedence
audiogram will be
C True about nasolabial cyst As
Present submucosally in anterior nasal floor,
C Treatment of choice for glue ear is
bilateral and usually seen in adults.
Myringotomy with ventilation tube insertion.
C A patient of carcinoma tongue of right side has
AIIMS MAY 2008
lymph node of 4 cm size on its lateral border of
C True about facial nerve palsy in temporal bone anterior 2/3rd on left side of neck. Stage of disease
fractures is
More common with transverse fractures. N2
C Laryngocele arises from C A 5 years old boy with recurrent URTI with mouth
Saccule of the ventricle breathing and failure to grow with high arched
C An elderly diabetic with excruciating pain in ear, palate and impaired hearing is
appearance of granulation in meatus, skull bone Adenoidectomy with grommet insertion.
infection with facial paralysis should be treated
with
AIIMS MAY 2006
Penicillin
C Most common etiology of paranasal sinus mycoses
Aspergillus spp
AIIMS NOVEMBER 2007
C Hyperacusis in Bell’s palsy is due to paralysis of
C A female patient on 2nd postoperative day of Stapedius
appendicectomy bumped her nose on a table and
developed difficulty in breathing and examination C Perichondritis of thyroid cartilage developed in a
showing a swelling in anterior part of nasal septum. case of carcinoma larynx with anterior commissure
Next step is and right vocal cord involved. Management is
Incision and drainage Laryngectomy followed by radiotherapy.
C A child has left ear hearing loss for last 3 months. On C Typical features of Meniere’s disease
examination-foul smelling purulent discharge with Fluctuating deafness, sensorineural deafness and
perforation in pars flaccida. Most appropriate vertigo.
management is C Features of malignant otitis externa
Tympanomastoid exploration Caused by Pseudomonas aeruginosa, old patient and
(Note: Unsafe CSOM) immunocompromised patients.
Ear, Nose and Throat 131

C A 30 years old lady developed hearing loss during C Route of spread of infection from middle ear
pregnancy with family history from maternal side. Bony invasion, directly through oval and round
Bilateral slowly progressive hearing loss with window and osteothrombotic route.
bilateral tinnitus and pure tone audiometry C An elderly diabetic patient presents with painful
showing conductive hearing loss with Carhart’s ear discharge and edema of external auditory canal
notch. Diagnosis with facial palsy not responding to antibiotics. An
Otosclerosis increased uptake on technetium bone scan is noted.
Diagnosis is
Malignant otitis externa.
AIIMS NOVEMBER 2005
C True about after effects of cochlear implantation
C The treatment of choice for paralysed vocal cord in Improved sound and speech perception, better
a 10 years old boy with hoarseness of voice after chances of developing normal or almost normal
diphtheria auditory and verbal skills and improved scholastic
Wait for spontaneous recovery of vocal cord. performance as compared to profoundly hearing
C Management of large antrochoanal polyp in a 28 impaired patients without the use of a cochlear
years old man implant.
Endoscopic sinus surgery. C Nasal tumors originating from the olfactory
mucosa
C Laryngocele arises from
Esthesioneuroblastoma
Saccule of ventricle.
C True about laryngomalacia
C Objective piece of microscope for ear surgery
Most common congenital anamoly of the larynx,
250 mm.
omega shaped epiglottis and stridor increases on
crying, but decreases on placing the child in prone
AIIMS NOVEMBER 2004 position.
C Packing most useful to prevent synachiae forma- C True about spasmodic dysphoria
tion after nasal surgery Botulinum toxin is the standard treatment, it affects
Mitomycin the muscles of the larynx and multiple sittings of
C Techniques used to control bleeding from bone botulinum toxin A is required for its treatment.
during mastoid surgery
Bone wax, bipolar cautery and diamond drill. ALL INDIA 2011
C True about sodium in otosclerosis
AIIMS MAY 2004 Acts by inhibiting proteolytic enzymes in cochlea, is
C Gray white membrane on tonsil due to contraindicated in chronic nephritis and is
Infectious mononucleosis, streptococcal tonsilitis indicated in patients with a positive Schwartz sign.
and diphtheria. C Vestibular schwannoma arise most commonly
from
ALL INDIA 2012 Superior vestibular nerve
C Species of Vibrio most often associated with ear C Endolymphatic hydrops is seen in
infections Meniere’s disease
V. alginolyticus C Young boy while eating suddenly develops
C Sensory supply of the auricle aphonia and respiratory distress. Next step
Greater auricular nerve, auricular branch of vagus Heimlich’s maneuver
nerve and tympanic branch of glossopharyngeal C Gold standard test for laryngopharyngeal reflux
nerve. 24 hours double probe pH monitoring.
C Conductive hearing loss is seen in C Secondary hemorrhage after tonsillectomy usually
Otosclerosis, otitis media with effusion and presents at
suppurative otitis media. 6 days
132 AIIMS and All India PGMEE—Review Questions

C Pain sensations from ethmoidal sinus are carried by 2 weeks with deafness in affected ear and negative
Nasociliary nerve. fistula test. Rinne’s test will be
C Onodi cells and Haller cells are seen in relation to False negative
structures, respectively (Note: Dead labyrinth)
Optic nerve and floor of orbit. C Troter’s triad includes
Deafness, mandibular neuralgia and palatal palsy.
C Laryngeal pseudosulcus is seen secondary to
ALL INDIA 2010
Laryngopharyngeal sulcus
C Otoacoustic emissions arise from C True about recurrent laryngeal papillamatosis
Outer hair cells Caused by HPV, HPV 6 and 11 are most commonly
(Note: Screening tests) implicated and transmission to neonate occurs
C True about nasopharyngeal carcinoma through contact with mother during vaginal
Bimodal age distribution, IgA antibody to EBV is delivery.
found and squamous cell carcinoma is most
common histologic subtype. ALL INDIA 2008
C True about CSF leak C A newborn presents with bilateral microtia and
Most common site of leak is fovea ethmoidalis, β external auditory atresia. Corrective surgery is
transferrin estimation is highly specific for its performed at
diagnosis and fluorescein dye can be used 5–7 years of age
intrathecally for diagnosis of leak. C In cochlear implants electrodes are most commonly
C Extrinsic laryngeal membranes are placed at
Hyoepiglottic, cricotracheal and thyrohyoid. Cochlea
C Structures seen on bronchoscopy C A 70 years old male patient presents with loss of
Trachea, voal cords and first segmental division of sensation of external acoustic meatus (Histelberg’s
bronchi. sign positive). It is a case of
C True about a child presenting with stridor, barking Vestibular schwannoma
cough, and difficulty in breathing for 2–3 days. He C A 75 years old male patient presents with tinnitus,
has fever and elevated leukocyte count conductive deafness, dull tympanic membrane on
Subglottic stenosis and hypopharyngeal dilatation right side, enlarged lymph node (3 × 3 cm) in
may be seen on X-ray, boys are more commonly posterior triangle of neck and B wave on
affected than girls and symptoms are predomi- tympanogram. It is most likely a case of
nantly caused by involvement of subglottis. Nasopharyngeal malignancy
C Drug of choice in laryngeal stenosis is C Types of joints between the ossicles of ear
Mitomycin C Synovial joint
C Lasers most commonly used in laryngeal surgery C Diagnostic criteria of allergic fungal sinusitis
CO2 laser Area of high attenuation on CT scan, allergic
eosinophilic mucin and type 1 hypersensitivity.
C An adult female patient presents with history of
ALL INDIA 2009
singing, vocal abuse, gastroesophageal reflux and
C Young man presenting with an accident leading to has developed nodules at junction of anterior
loss of hearing in right ear. On otoscopic examina- 1/3rd and middle 3rd of her vocal cords.
tion, TM was intact. Pure tone audiometry shows Treatment is
an air bone gap of 55 dB in right ear with normal Speech therapy and PPI
cochlear reserve. Likely tympanometric finding
Ad type tympanogram
ALL INDIA 2007
C A patient of suspected diagnosis of suppurative
labyrinthitis with positive Rinne’s test and positive C Rhinolalia clausa is associated with
fistula test refused treatment and returned after Allergic rhinitis, adenoids and nasal polyps.
Ear, Nose and Throat 133

C Rhinophyma is associated with ALL INDIA 2004


Hypertrophy of sebaceous glands.
(Note: Potato tumor) C In right middle ear pathology, Weber’s test is
C Common site for CSF rhinorrhoea is Lateralized to right side (Mn: LDC-lateralized to
diseased ear in conductive deafness).
Cribriform plate > ethmoid sinus.
C Triad of Meniere’s disease
C CSF rhinorrhoea is diagnosed by
Tinnitus, vertigo and sensorineural hearing lass
β2 transferrin.
(Mn: TVS).
C Earliest cranial nerve involved in acoustic neuroma is
C Fordyce’s spots in oral cavity arise from
5th cranial nerve.
Sebaceous glands
C Brown sign is seen in
C Space showing collection of pus in quinsy
Glomus tumor.
Peritonsillar space.
C Bell’s palsy is
ALL INDIA 2006 Idiophatic ipsilateral paralysis of facial nerve.
C Progression of posterosuperior retraction pocket
may lead to ALL INDIA 2003
Sensorinerual hearing loss. C Management of posterosuperior retraction pocket
(Note: Cholesteatoma) cholesteatoma includes
C Surgery of choice in attic cholesteatoma of ear with Mastoid exploration, audiometry and tympanoplasty.
lateral sinus thrombophlebitis in a 30 years old C A 30 years old woman presents with bilateral
patient is hearing loss for last 5 years, TM normal, bilateral
Canal wall down mastoidectomy. conductive deafness on audiogram, as type of curve
C Investigation of choice for assessment of hearing and absent acoustic reflex on impedance audiometry.
loss in neonates Management includes
BERA Hearing aid, sodium fluoride and stapedectomy.
C Maximum hearing loss in C Carhartz notch occurs at
Ossicular disruption with intact tympanic mem- 2 kHz
brane. C Type 1 thryoplasty
C Sensorineural deafness is features of Vocal cord medialization.
Alport syndrome, Bartter’s syndrome and distal C Laryngeal examination showing reddish area of
renal tubular acidosis (Mn: BAD) mucosal irregularity overlying a portion of both
cords in a 40 years old male patient with hoarseness
C During inspiration the main current of airflow in a
of voice for last 2 years and history of smoking for
normal nasal cavity is through
22 years. Management includes
Middle part of the cavity in middle meatus in a
Cessation of smoking, microlaryngeal surgery for
parabolic curve.
biopsy and regular follow-up.
C Laryngocele arises as a herniation of laryngeal (Note: Polyp)
mycosa through
Thyrohyoid ALL INDIA 2002
C Weber test is best elicited by
ALL INDIA 2005 Placing the tuning fork on forehead and asking him
C Most common site of leak in CSF rhinorrhoea to report in which ear he hears it better.
Cribiform plate C Positive fistula test is found in
C Earliest manifestation of glottis carcinoma After fenestration surgery, hyper mobile stapes foot
Hoarseness of voice. plate and labyrinthine fistula.
C Androphonia corrected by C Most common cause of acute otitis media in children
Type 4 thyroplasty. Pneumococcus
134 AIIMS and All India PGMEE—Review Questions

C True about Ramsay Hunt syndrome C A 50 years old male smoker presents with
Viral etiology, 7th cranial nerve involvement and hoarseness of voice and keratosis of larynx on
8th cranial nerve involvement. microlaryngoscopic biopsy. Management includes
C An infant presents with failure of gaining weight Cessation of smoking, laser vaporizer and stripping
and noisy breathing which becomes worse when of vocal cord.
child cries with laryngoscopy finding of reddish (Note: Laryngeal keratosis)
mass in subglottis. Management includes C Treatment of a choice of a patient presenting with
CO2 laser treatment, steroid and tracheostomy. T1N0M0 glottic carcinoma
(Note: Subglottic hemangioma) External beam radiotherapy.
Medicine 135

11

Medicine

AIIMS NOVEMBER 2015 C False about Meckel’s diverticulum


Remnant of distal part of vitellointestinal duct.
C Benign intracranial hypertension may cause-
(Note: Meckel’s diverticulum is remnant of
Normal sized ventricles, papilloedema and cranial
proximal part of vitellointestinal duct)
nerve palsy.
C A 27 years old man presents with back pain
specially in the morning and relieved by bathing in AIIMS MAY 2015
warm water. Additional feature
C True about H1N1
Chest expansion decreased.
Pregnant woman with sore throat can be started
(Note: Diagnosis-Ankylosing spondylitis)
immediately on oseltamivir without diagnostic
C Most characteristic sign of metabolic encephalopathy testing under category B.
Asterixis
C High cardiac output heart failure occurs in
C Glucocorticoid deficiency causes
Anemia, beriberi and AV fistula.
Fever, hypotension and hyperkalemia.
C Most common site of hypertensive intracranial bleed is
(Note: It also causes weight loss)
Basal ganglia
C A 65 years old patient presents with chronic
sinusitis, nasopharyngeal ulcers, cavitatory lung C A girl presents with symptoms suggestive of
nodules and renal failure. Next done Sydenham’s chorea and is suspected of acute
rheumatic fever although other major criteria are
ANCA and evaluation for vasculitis.
absent and there is no evidence of sore throat. Best
(Note: Diagnosis-Wegener’s granulomatosis)
investigation to prove rheumatic etiology
C Serology of a patient showed HBsAg negative, anti
ASO titre
HBsAg antibody positive, HBV DNA negative,
HBeAg negative and antiHbC IgG positive. Likely C Most typical feature of alcohol withdrawal
diagnosis- Tremor
Chronic hepatitis B in recovery state. C Sweating is not seen in
C True about microalbuminuria are: Heat stroke
It is the erliest marker of diabetic nephropathy C Duke’s criteria for infective endocarditis includes
It is not detected by routine dipstick method IV drug abuser, fever > 100°C and blood culture.
It is an independent risk factor for cardiovascular C A young boy presents with primary malignancy of
morbidity in diabetics. pre-aortic lymph node. Histopathology shows
(Note: Microalbuminuria is defined as 30–299 mg/ starry sky pattern. Most likely mutation present in
day in a 24 hour collection) the boy is
C Naming and fluency is impaired in Translocation involving C-myc.
Broca’s aphasia (Note: Burkitt’s lymphoma)

135
136 AIIMS and All India PGMEE—Review Questions

C A patient suffering from multiple clotting factor C Rheumatic fever is least likely to be associated with
deficiency presents with active bleeding. Type of Pulmonic stenosis.
blood product best for the transfusion in such C True on auscultation in a case of severe MS
situation Loud S1, opening snap and mid diastolic murmur.
Fresh frozen plasma. C Angina, dyspnea and syncope are most commonly
C True about TRALI (transfusion associated acute associated with
lung injury) Aortic stenosis (Mn: SAD)
Mostly seen after sepsis and cardiac surgeries, it’s C A child, known case of bronchial asthma, presents
a cause of non-cardiogenic pulmonary edema with respiratory rate of 48/minute, occasional
and plasma is more likely to cause it than whole wheeze and inability to speak two words. His SpO2
blood. is 95%. He is treated with salbutamol nebulization
(Note: It develops within 6 hours) 3 times. He can now complete a sentence but SpO2
C Window period for post-transfusion potential falls to 85%. Drop in SpO2 can be explained by
exposure is defined as V/Q mismatch
From the receipt of infection to laboratory detection C The feature of idiopathic non-specific interstitial
of disease. pneumonia (NIA) include
C Disease not causing reduced DLCO Good prognosis
Bronchial asthma. C False about obstructive sleep apnea
C Osteoporosis, true is More common in females.
Normal calcium, normal alkaline phosphatase. C Excess calcium intake can lead to
C Most common nephropathy associated with Milk alkali syndrome
malignancy C A patient presents to the emergency with a crush
Membranous nephropathy injury. ECG shows diminished/absent P waves,
wide QRS complexes and tall, wide and symme-
(Note: Particularly carcinoma of the lung and colon
trical T waves. Useful in management
and melanoma)
Calcium gluconate, dialysis and insulin + glucose.
C Surgery was done in a child for EHPVO with
(Note: Hyperkalemia)
hepatojejunal anastomosis. Post-operative bilirubin
level after 2 weeks was 6 mg/dl from a pre-operative C The marker for diagnosing acute hepatitis B
level 12 mg/dl. Cause between 4 and 8 weeks is
Delta bilirubin IgM anti-HBc
A nurse is seropositive for both HBsAg and HBeAg.
A young woman presents with symptoms of
C
C
Most likely diagnosis
hyperthyroidism with elevated T4 and TSH and
bitemporal hemianopia. Next done Acute infectious hepatitis B.
Start antithyroid drugs, and do urgent MRI brain. C A 3 years old child took aspirin for fever after which
he developed raised liver enzymes, elevated
(Note: Pituitary adenoma-best treated by surgery)
bilirubin and microvascular steatosis on liver
biopsy. The molecular level change in this disorder
AIIMS NOVEMBER 2014 is due to
Impaired β oxidation of fatty acids.
C Matched correctly are
C For treatment of ascites, the Le Veen shunt is placed
Tachypnea-acidosis, wheeze-CCF and chest pain- between the peritoneum and
pericarditis.
Superior vena cava.
C Inability to work without any discomfort is (Note: Internal jugular vein)
NYHA class 3 C 90% association with HLA B27 is seen in
(Note: Discomfort at rest is grade 4) Ankylosing spondylitis.
C Marker of heart failure C A person is waiting for a train on a railway platform
Brain natriuretic peptide, CRP and troponin T. starts having seizures. He has a band showing him
Medicine 137

to be an epileptic on medication and his medications C Diagnostic criteria for declaring brainstem death
are in his pocket. Next done Absence of brainstem reflexes, a positive apnea test
Take the person away from the train, make sure he and lack of cerebromotor response to pain in all
does not fall, meanwhile call for medical help and extremities.
transfer him to hospital. C A vascular lesion in the midbrain stem at the level
C The first neural disorder to be associated with the of lateral part of medulla can cause
complete deficiency of a neurotransmitter is Contralateral loss of pain and temperature below
Parkinson’s disease. the neck, nystagmus and paralysis of soft palate.
C Two persons A and B were told to draw a square
AIIMS MAY 2014
from the blank and filled circle, respectively. The
square drawn by A is irregular. Possible causes
C Characteristic features of multiple myeloma Lesion in cerebellum.
Increased Ig levels in serum, plasmacytosis and M C Drugs used in acute bacterial meningitis
spike on electrophoresis.
Ceftriaxone
C A 60 years old obese male patient with a history of
chronic smoking since childhood presents with
pelvic fracture due to fall from height. On 4th day of AIIMS NOVEMBER 2013
the ICU stay, he developed sudden tachypnea, fall
in SpO2 and hypotension. On 2D echo, there was a C A 55 years old diabetic patient presents with
dilation of right ventricle and tricuspid regurgitation. transient obscuration in vision for 2–3 days
Next immediate step followed by sudden loss of vision. Best test to
evaluate him
Systemic thrombolysis.
Serum creatinine levels.
C A 4 years old girl presents with severe vomiting
after a viral fever of 5 days. She is hospitalized and C A drug that can replace valproate as montherapy
develops cerebral edema. Liver biopsy is most likely being taken by a female patient of child bearing age
to show for juvenile myoclonic epilepsy
Marked microvesicular steatosis. Levetiracetam.
C A 40 years old obese female patient presents with C A patient on amphotericin B develops hypokalemia
fullness of right upper quadrant of abdomen. Past of 2.3 mEq/L, supplementation required
medical history shows type 2 DM and hyperlipi- 120–160 mEq/L over 24 hours.
demia. Liver biopsy suggestive of diagnosis C Investigation not done in patient with tubercular
Non-alcoholic steatohepatitis. meningitis on regular ATT for 1 month presenting
C A young female patient presents with jaundice and with altered sensorium
elevated liver enzymes with 2 similar episodes in CSF examination.
the past and on investigation, serum ANA 1:40, and C Recently approved drug by USFDA for treatment of
IgG 2400 IU, serum copper level normal and viral Lennox-Gestaut syndrome
marker negative. Based on liver biopsy report Rufinamide.
immunosuppressant was started and condition
C A patient with native aortic valve disease came with
improved. Most likely diagnosis
right hemiparesisdone.
Autoimmune hepatitis.
Done to prevent further stroke antiplatelet only
C Laxative abuse is associated with
C True about CML in children
Hypokalemia
Protein tyrosine kinase inhibitors are used in
C A 27 years old male patient presents with low
treatment.
backache, that occurs early in the morning, associa-
ted with stiffness, and persists for more than C Coarse tremors of tongue seen in
30 minutes. On examination, his chest expansion is Parkinsonism, alcohol and general paresis.
also restricted. The most likely diagnosis C A lady developed breathlessness, pruritus,
Ankylosing spondylitis. urticaria 1 hour after eating NSAIDS for headache.
138 AIIMS and All India PGMEE—Review Questions

Chest showed rales and BP 80/50 mm Hg. Used for C True about aspiration pneumonia
initial management Aspiration of 20–30 ml of contents with pH < 2.5 is
Early respiratory support and oxygenation, required, posterior segment of the right upper lobe
crystalloids infusion and adrenaline. is most commonly affected in the recombinant
C Which indicates renal impairment in contrast position and aspiration is responsible for 5–15% of
induced nephropathy community acquired pneumonia.
Increase in serum creatinine. C The most common type of Non-Hodgkin’s lym-
phoma in India is
C A 50 years old patient presented with progressive
jaundice. LFT showed total bilirubin—6.7%, direct Diffuse large B-cell lymphoma.
serum bilirubin—4.8%, alkaline phosphatase—550 C True about hemoptysis
IU, SGOT—50, SGPT—65. Most probable diagnosis Massive hemoptysis is bleeding > 600 ml in 24
Jaundice due to choledocholithiasis. hours, in 90% cases, bleeding is from bronchial
arteries and in an unstable patient, rigid broncho-
C A 60 years old male patient with alcoholic liver
scopy is done to identify the lesion.
disease presented to emergency with a history of
Acquired cause of pure red cell aplasia are
hemetemesis. True about his management
C
ABO incompatability in bone marrow transplanta-
Upper GI endoscopy is done initially, somatostatin
tion, lymphoma and drug induced NSAID.
infusion is indicated and negative nasogastric
Criteria for admission in upper GI bleed are
aspirate does not exclude variceal bleed.
C
Frank blood in nasogastric aspirate, BP > 100 mm Hg
C A 50 years old male patient came to the emergency
and shock index > 1.5%.
with right hemiparesis and loss of speech for 2.5
(Note: Shock index = HR/SBP, normal = 0.5 to 0.7)
hours. BP is 180/100. Next best step
C True about iron deficiency anemia
Go for NCCT.
Transferrin saturation < 16%, detected by serum
A 30 years old women from West Bengal presents
ferritin levels even in earlier states and latent
C
with fever for 48 hours with history of irrelevant
anemia is most prevalent in India.
talk and 2 episodes of tonic clonic seizures on
C Drugs given in patients of primary pulmonary
succession on the way to the hospital. Drug given
hypertension
while investigation is being carried out
Bosentan
IV acyclovir.
C A 50 years old lady on junk food diet presented with
(Note: Viral encephalitis due to HSV-Empirical point hemorrhages in scalp, bleeding in joints and
treatment) erythematous lesions in the skin, X-ray of knee joint
C A 14 years old boy presents with abnormal body was suggestive of hemarthrosis. Mechanism defec-
movements with MRI showing signal changes in tive
corpus striatum, thalami, pons, medulla, centrum Hydroxylation of proline and lysine.
semi-ovale and asymmetric diffuse white matter (Note: Vitamin C deficiency)
involvement. Most likely diagnosis
C True about severe malaria
Wilson’s disease. Hypoglycemia, blood sugar < 40 mg/dl, creatinine
C Glucocorticoid is used in more than 3 mg/dl and LDH > 750 U/L.
Severe typhoid. C Old age male patient, hypertensive, presents with
C True about critical illness myoneuropathy sudden onset headache, vomiting, neck rigidity,
Diaphragmatic atony due to prolonged intubation but no focal neurological deficit. Most likely
may cause it. diagnosis
C A patient developed sudden severe headache Sub-arachnoid hemorrhage.
2 hours ago and became unconscious. Upon regai-
ning conscious, patient developed photophobia AIIMS MAY 2013
and neck rigidity. Next done C Division of the chromosome perpendicular to the
NCCT scan. usual axis of division
(Note: Subarachnoid hemorrhage) Isochromosome
Medicine 139

C Cyanosis does not develop in severe anemia because C An asymptomatic boy on ECG is detected to have a
of short PR interval and delta waves. Effective
It requires a critical concentration of reduced measures would be
hemoglobin in blood. Reassurance, Holter monitoring and beta-blockers.
C True about ESR in tuberculosis (Note: WPW syndrome)
ESR values are increased due to formation of larger C A 8 years old boy complains of muscle weakness.
aggregates of RBCs. On examination, his calves are bulky and show
C Most likely probability of a 16 years old girl muscle tightening. His serum creatine kinase levels
presenting with abdominal pain, seizure and are decreasing with age. Most likely diagnosis
incoherent state Dystrophin deficiency
Acute intermittent porphyria. C A person presents with-pCO2: 30 mm Hg, pO2: 105
C Trans-esophageal echocardiography (TEE) is supe- mm Hg, pH: 7.45, he is having partially compensated
rior to trans-thoracic echocardiogram because of Respiratory alkalosis
It is more sensitive in picking up atheromatous C A boy presents with fever, vomiting and headache
plaques in ascending thoracic aorta. for last 3 days, disorientation for 1 day. Neck
C Left bundle branch block can develop suddenly in rigidity is also present. There is 1 episode of GTC
Acute MI, and hyperkalemia. and then he became unconscious. CECT is normal.
C A 55 years old male patient presents with renal CSF shows cell count: 300 cell/mm3 (polymorphs
failure, gives history of mild bone pain for last 7 50–70%), protein 70 mg/dl, sugar 50 mg/dl (blood
years, X-ray pelvis showing osteolytic lesions, M sugar 95 mg/dl). Most likely diagnosis
spike on serum electrophoresis, rouleax formation Pyogenic meningitis
of RBCs with 35% plasma cells on peripheral blood, C Characteristic features of Kluver-Bucy syndrome in
bone marrow showing increase in plasma cells with children
aberrant antigen expression. Most likely diagnosis Visual agnosia, hypermetamorphosis and hyper-
Plasma cell leukemia. sexuality.
C An elderly lady presents with acute onset of C A person is found to be seropositive for both HBsAg
confusion and bumping into things. On exami- and HBeAg. He is suffering from
nation, she is alert, oriented with fluent speech and Acute infection hepatitis B infection.
normal comprehension but impaired writing, right C Vasopressor preferred in a patient of aortic stenosis
left orientation, arithmetic abilities and finger Phenylephrine
identification. MRI shows several foci of cortical C Features helping in distinguishing seizures from
and subcortical increased T2 signals and various syncope
areas of leptomeningeal enhancement. The most Physical weakness with clear sensorium.
probable diagnosis C True about iron deficiency anemia
Gerstmann’s syndrome. Low serum ferritin (<15 mg/dl), elevated RDW
C Modality of treatment showing the best seizure free (>14.5%) and low serum iron levels (< 75 mg/ml).
interval in medically intractable seizures C Bilateral Babinski’s sign is characteristic of
Epilepsy surgery. Pontine hemorrhage.
C Drug effective as monotherapy as a replacement for C A 50 years old male patient presents with ptosis,
sodium valproate in a female patient of child difficulty in chewing and occasional difficulty in
bearing age group for treatment of juvenile swallowing but no history of diplopia or visual loss.
myoclonic epilepsy On examination, there is asymmetric ptosis and
Levetiracetam mild restriction of extraocular muscle movement
C True about treatment of CLL with finger abduction test 60°. Nerve conduction
Can be withheld in asymptomatic patients. study shows decremental response in orbicularis
C The mutation seen in most common type of only. ERG revealed a myopathic pattern. Anti-
maturity onset diabetes of young AChR radioimmunoassay was negative. The most
Hepatocyte nuclear factor 1. probable diagnosis
(Note: MODY-Type 3) Generalized myasthenia gravis.
140 AIIMS and All India PGMEE—Review Questions

AIIMS NOVEMBER 2012 C True about polymyositis


Limb girdle weakness, paraneoplastic syndrome
C A 28 years old man has anterior lenticonus and and spontanous discharge in EMG.
ESRD now with death of his maternal uncle due to
Gait apraxia is seen in thromboembolic episode
similar illness
C
involving
Alport’s disease
C A patient with hepatitis C, shows hypocomple- Anterior cerebral artery.
mentemia, with 2 gram/day proteinuria and C A patient on amphotericin B develops hypokale-
hematuria. The most probable diagnosis is mia 2.3 mg/dl. Potassium supplement required
MPGN 120–160 mEq over 24 hours.
C Which of the following does not need treatment C An 1 year old girl born to Punjabi parents developed
Neuroblastoma pallor since 3 months of age. 1 unit of blood trans-
C Blink reflex is used for fused at 5 months of age. At present, pallor present,
Mid pontine lesions hepatosplenomegaly, Hb 3.8 gm/dl, MCV 68,
C An elderly female patient presents with the nasal MCH 19, peripheral smear showing- schistocytes,
blockade and nasal discharge with black debris. bone marrow shows erythroid hyperplasia.
Blood sugar is raised and urine is positive for Diagnosis
ketones. Best treatment for the patient would be β thalassemia.
Amphotericin B C True about type 1 renal tubular acidosis
(Note: Mucormycosis)
Renal stones, hypokalemia and urine pH < 5.5.
C Advice given to 35 years old lady with recurrent
renal stone C A 78 years old male patient presents with headache,
Increase water, restrict protein and restrict salt. visual disturbance and tenderness over his right
C All are indicated in a patient with cystinuria with temporal region. Biopsy from right temporal artery
multiple renal stones will show
Increased fluid intake, alkalinization of urine and Granulomatous inflammation with giant cells.
penicillamine. C A patient of liver abscess with abscess of size 8 × 8 cm
C A patient presents with pain in MTP joint. He is a in right lobe was aspirated thrice and given
known case of chronic renal failure. This is due to systemic amoebicide. Now cavity remain but
accumulation of empty. How will you follow up
Uric acid. USG weekly for 1 month followed by monthly USG
C A patient of rheumatoid arthritis presents with pro- till one year.
gressive quadriparesis and weakness, Babinski’s C True about Wilson’s disease
sign positive, increased muscle tone of limbs with Low serum copper.
exaggerated tendon jerks and worsening of gait C Causative agent of SARS
with no sensory, sphincter involvement. Best initial Corona virus
investigation is
Seen in cervical syringomyelia
X-ray of cervical area of neck in flexion and extension.
C

C With ageing, a slight decrease in cognitive impair- Burning sensation in hands, plantar extensor and
ment is seen due to increase in level of absent biceps reflex.
Homocysteine C A patient of tubercular meningitis on regular ATT
C True about Grisel syndrome for last 1 month developed deterioration in
Post-adenoidectomy, conservative treatment and sensorium. Investigations required for emergency
inflammation of cervical spine ligaments. evaluation
C Pinna calcification is seen in MRI, NCCT and liver function test.
Ochronosis, frost bite and Addison’s disease. C A 60 years old retired army officer, with good
C In a patient with implanted cardiac pacemaker health, physically quite active and no addictions
(defibrillator) investigation to know the position of presents with complaints of bitemporal throbbing
the misplaced implant headache, worsening on lying flat with feverish but
X-ray never documented and suppressed appetite. He
Medicine 141

feels better on sitting upright and on pressing side C Correctly matched-Neurofibromatosis- renal artery
of his neck. stenosis, Marfan’s disease-dural ectasia and
Giant cell arteritis Mulibrey nanism- constrictive pericarditis.
(Note: GCA can present as PUO) C The most common form of spinocerebellar ataxia
(SCA) in India is
AIIMS MAY 2012 SCA2
C Type of wave in metabolic encephalopathy
C Corticospinal injury is associated with Delta
Babinski’s sign present, superficial abdominal C In 2 patients with atherosclerosis, one is diabetic
reflex absent and clasp knife rigidity. and other is non-diabetic. In relation to non-
C In porphyrias, enzyme defect not leading to diabetic, diabetic patient has 100 times increased
photosensitivity risk of
HMB synthase Lower limb ischemia
C A 50 years old male patient presents in the C In pneumonia severity scale, most important factor is
emergency with unilateral headache, diplopia and Age
difficulty in chewing food with MRI normal and
(Note: CURB-65. Confusion blood urea nitrogen,
ESR persistently high. The most appropriate drug
respiratory rate > 30/min, BP < 90/60 and age > 65
for treatment
yrs)
Prednisolone
C Pseudotumor cerebri is seen in
A 20 years old male patient presents with jaundice
Obese women in the age group 20–40 years
C
for 2 weeks, LFT showing serum bilirubin 0.9 mg/
C Most prominent feature among the following in
dl, SGOT/SGPT-1240/1450 IU, HBsAg and anti-
immunoproliferative small intestinal disease
HEV antibodies positive and IgM anti-HBc negative.
(IPSID)
Most likely, it is a case of
Abdominal pain
Superinfection of hepatitis E virus with chronic
hepatitis B infection. C A 25 years old patient with history of repeated
episodes of rheumatic fever is hypersensitive to
C A patient with previously normal Hb suffered a
penicillin. Drug prescribed
sudden massive acute hemorrhage. He is most likey
to show Sulfisoxazole
High reticulocyte count and high neutrophil count. C A person with following parameters pCO2—30 mm
C Thrombolytic therapy is given in a patient of Hg, pO2—105 mm Hg, pH—7.45 is having partially
myocardial infarction within compensated
24 hours of chest pain Respiratory alkalosis
C True regarding small vessel disease (SVD), cerebral C Post-transplant lymphoma is most associated with
amyloid angiography (CAA) and Alzheimer’s EBV
disease (AD) C Which most significantly increases the risk of
SVD is related to CAA, CAA is associated with AD hepatocellular carcinoma
and SVD is not correlated to AD. Hep B
C A 40 years old lady presented with dyspnea on C A 65 years old suddenly fell in the toilet, BP 90/50
exertion grade 3 and palpitations, ECG- atrial mm Hg, pulse 100/min, stool dark/black in color,
fibrillation with fast ventricular rate, echocardio- past history of hypertension and coronary artery
graphy-severe mitral stenosis with left atrial disease and was regularly taking aspirin, atenolol
appendage clot. She is advised and sorbitrate. The most likely diagnosis is
Diltiazem to control ventricular rate, start warfarin Gastric ulcer with bleeding.
and follow-up with repeat echocardiography and C A study of skin biopsy of SLE patient using FITC
open mitral commissurotomy with clot removal. labeled human IgG antiserum shows deposition of
C Most common clinical presentation of juvenile irregular particles at dermoepidermal junction. It
myoclonic epilepsy shows the presence of
Myoclonus Immune complex deposits.
142 AIIMS and All India PGMEE—Review Questions

C Associated with an intrinsic defect in the RBC C A 25 years old female patient presented with
membrane history of recurrent abortions. The most relevant
Hereditary spherocytosis. investigation to identify the cause
C Fever blisters can occur due to Dilute Russel viper venom test.
Reactivation of HSV 1
C A 40 years old female patient presents with AIIMS NOVEMBER 2011
intolerance to cold, constipation and hoarseness of C Diagnostic criteria for primary hyperaldosteronism
voice with cardiomegaly on chest X-ray. Best
Diastolic hypertension without edema, low plasma
investigation to determine the cause of her
renin activity that is not stimulated by volume
cardiomegaly
depletion and hyperaldosteronism, i.e. increased
Echocardiography aldosterone secretion which is not suppressed by
C An elderly male patient presented with headache, volume expansion.
fever and hemiparesis of right side. On further C Drug of choice for prenatal treatment of CAH due
investigation and examination, a diagnosis of brain to 21 α hydroxylase deficiency
abscess was made. Antibacterial agent effective in Dexamethasone
this condition
C Limb girdle muscle dystrophy includes
A combination of cephalosporin and metronidazole.
Calpainopathy, dysferinopathy and sarcoglycano-
C A 45 years old lady is diagnosed with pneumococcal pathy.
meningitis, blood sample sent for culture and C Exclusively involving neurons
sensitivity. Best drug to start empirically
Corticobasilar degeneration.
Vancomycin + ceftriaxone.
C Physical examination finding in a young male
C Most likely to be affected in an aneurysm of patient presenting with LDL 600 mg/dl and
posterior cerebral artery triglycerides 140 mg/dl
Oculomotor nerve Tendon xenthoma
C An adult hypertensive male patient presents with C True about universal definition of Myocardial
sudden onset severe headache and vomiting, on infarction
examination-marked neck rigidity but no focal Sudden death is MI, new regional wall motion with
neurological deficit. Most probable diagnosis increased biochemical marker is MI and reinfarc-
Subarachnoid hemorrhage tion can be diagnosed if elevation in troponin level
C An elderly female patient presents to the emer- by 20% on serial sampling.
gency and was found to have a transmural C Systemic eosinophiluria with renal failure is
myocardial infarction. Based on her ECG, she was associated with
started on thrombolytic therapy with strepto- Drug induced interstitial nephritis, contrast
kinase. Finding that is risky and for which nephropathy and atheroembolic renal failure.
thrombolysis should be stopped C True statements are
Significant pericardial effusion on echocardiogra- PTH-rP is responsible for hypercalcemia in cancer
phy. patients, Mg2+ influences PTH secretion in the same
C A 60 years old lady presents with decreased direction as Ca2+ but is a less potent secretagogue
movement for 2 years with rigidity and vertical and Ca 2+ ion influences PTH secretion by acting
large square wave jerks. Probable diagnosis is on a calcium sensor G protein coupled receptor
Progressive supranuclear palsy. located in the parathyroid gland.
C True for hepatitis B C Diagnosis of 60 years old male patient of RA with
Age of onset determines prognosis, period of Hb 4.5 gm/dl, platelet count 2 lac/mm3 WBC 6000/
communicability lasts several months and virus can mm3, serum ferritin 250 μg/dl, serum iron 30 mg/
be found in blood 1 month before jaundice. dl, TIBC 270 ng/L
C A boy is suffering from acute pyelonephritis. Most Anaemia of chronic disease.
specific urinary finding (Note: Normal value-serum iron- 50–150 μg/dl,
WBC casts serum ferritin– 50–150 ng/ml, TIBC-300 μg/dl)
Medicine 143

C Investigation needed to find out the cause in a with history of fall while partying with friends.
woman with complaints of abdominal pain and Additional problems he is facing
CECT finding of B/L papillary necrosis are Paralysis during sleep wake transition with
Culture for bacteria, sickling test and urine hallucinations.
acidification. (Note: Narcolepsy)
C Diabetes is diagnosed when
The level of fasting glucose is > 125 mg/dl and that AIIMS MAY 2011
of post-prandial is > 199 mg/dl.
C Reperfusion is useful for
Features seen in myopathies are
Hibernating myocardium
C
Facial sensory impairment, brisk pectoral jerk and (Note: Also known as reversible ischaemia or
urgency and incontinency of micturition. chronic stable angina)
C Diagnosis of 18 years old girl presenting with short C True about atherosclerosis
height, enlarged pituitary gland, low T4 and
Intake of PUFA is associated with decreased risk.
increased TSH
C Most rapid way to decrease serum K+ level in a girl
Primary hypothyroidism. presenting with severe hyperkalemia and peaked T
C Most common cause of Addison’s disease in India waves on ECG
Tuberculous adrenalitis. Insulin + glucose.
C True about Alzheimer’s disease C A 10 years old female patient presents with difficulty
Number of senile plaques correlate with age, in combing hairs and climbing upstairs for 6
underlying tau protein suggest neurodegeneration months, Gower’s sign positive and maculopapular
and number of neurofibrillary tangles is associated rash over MCP joints. Next appropriate investigation
with the severity of dementia. Creatine kinase
C An aortic stenosis patient could perform exercise C A 14 years female patient on exposure to cold
for 10 minutes (stopped due to fatigue) in Bruce develops pallor of extremities followed by pain and
protocol with peak systolic gradient of 60 mm Hg cyanosis. She is prone to develop
across the aortic valve at rest. The best management Scleroderma
is C Defect in Berry’s aneurysm
Medical management Degeneration/defect of media/muscle cell layer.
C Clinical features of demyelinating myelopathy C Pulsatile liver and ascites is found in
suggesting progression to multiple sclerosis TR (Hepatomegaly with liver pulsation, All India
Bilateral visual loss, complete cord transaction and 2009).
poor prognosis. C Positive hepatojugular reflux is found in
C A 30 years old, 4 months pregnant primigravida Increased capillary bed pressure, RHF and TR and
with history of juvenile myoclonic epilepsy is on PS (A/09).
sodium valproate regularly. Opinion C Thrombotic event is seen in
Continue valproate with monitoring of drug level. DIC, HIT (Heparin induced thrombocytopenia)
C Best drug to manage a 70 years old patient presen- and PNH.
ting with new onset focal seizures with normal C True about Horner’s syndrome
renal function Anhydrosis, hyperchromatic iris and miosis.
Oxcarbazapine C Congenital myopathies are
C True about SIADH Central core myopathy, centronuclear myopathy
Vaptans are approved by FDA for its treatment, and Nemalin myopathy.
water loading test can be used for its diagnosis and C Cause of alpha-thalassemia
serum sodium may be as low as 125 mEq/L in these Deletion of alpha gene.
patients. C A patient develops sudden palpitation with heart
C An adult man complains of falling asleep at work rate 150/min, regular cause is
frequently attributing to disturbed sleep at night, PSVT
144 AIIMS and All India PGMEE—Review Questions

AIIMS NOVEMBER 2010 C Hyperkalemia without ECG changes may be


treated with
C A bone marrow transplant recipient patient presents Salbutamol, sodium bicarbonate and insulin with
with chest infection, tree in bud appearance on dextrose and resins (M/09).
HRCT. Most likely causative agent is C Horner’s syndrome is seen in
Pneumocystis
Carotid artery aneurysm, can occur following
C A 10 years old boy complains of pain in left surgery for Raynaud’s syndrome and multiple
hypochondrium for 2 days, Hb 9.69 g/dl, history of sclerosis.
passing block colored stools for 7 days and 2–3
occasions for 2 years with fatigability during True about hepatitis B management
episodes. Clinical finding suggestive of diagnosis Supportive care—acute viral hepatitis B.
Palpable spleen Supportive care—chronic viral hepatitis B.
(Note: EHPVO) (1st and 2nd decade–EHPVO, 3rd Anti-viral drugs—chronic viral hepatitis B.
and 4th decade–NCPF) True about LKM
C Respiratory centre depression is caused by LKM1—autoimmune hepatitis.
Opium, barbiturates and gelsemium. LKM1—chronic hepatitis C
C Potassium present in RL solution LKM2—drug induced.
4 mEq/L C Tests for DM
C Secondary hyperparathyroidism is seen in Fasting blood sample, random blood sample and
Rickets, osteomalacia and renal failure. OGTT.
C Splenomegaly is associated with C Included in Jone’s major criteria
CML, polycythemia rubra vera and idiopathic Pancarditis, chorea and subcutaneous nodule.
myelofibrosis. (Note: Migratory polyarthritis and erythema
C A 10 years old boy with short stature presents with marginatum are also included in major diagnostic
polyuria and polydipsia. Lab values—pH 7.34, CO2 criteria)
32 mm Hg, HCO3 16, Na+ –140, K+ –4.8 Cl 114, BUN C A 25 years old gentleman presents with renal failure.
140. Acid base picture His uncle died of renal failure a few years ago.
None-anionic gap metabolic acidosis. Keratoconus is found on slit lamp examination.
C Used to prevent contrast nephropathy Diagnosis
N-acetylcysteine, infusion of half normal saline and Alport’s syndrome
hemodialysis. C Minor criteria for multiple myeloma are
C Diet restrictions in a patient passing stones Plasmacytosis 20%, multiple lytic lesion and IgA <
recurrently in urine for past few years are 100 mg/dl and Ig G < 600 mg/dl.
Protein restriction, salt restriction and phosphate
C A patient presented with thunder clap headache
restriction.
followed by unconsciousness with progressive 3rd
C A 75 years old lady with fracture neck of femur cranial nerve palsy. Diagnosis
2 months back presents with 2 days history of
Aneurysmal subarachnoid hemorrhage.
altered sensorium and decreased urine output.
Urea is 140 mg/dl, creatinine is 2 mg/dl and Ca2+ is C A patient presents with prosthetic valve replace-
15.5 mg/dl. Useful in immediate treatment ment and he develops endocarditis 8 months later.
Organism most likely
Normal saline, hemodialysis and bisphosphonates.
Staphylococcal epidermidis
C Congenital causes of hypercoagulable states are
(Hint: >12 months-Streptococcus viridans).
MHTFR mutation, protein C deficiency and protein
S deficiency. C Murmur increasing with valsalva maneuver
C A 20 years old man with history of primary syphilis HOCM
(genital ulcers and mucocutaneous lesions) C Following can cause SLE like syndrome
presents with neurosyphilis in the form of INH, hydralazine and sulphonamide (Mn: SHIP).
meningitis. Monitoring of treatment by C True about starting of beta-blocker therapy in a case
VDRL of CHF
Medicine 145

Dose gradually increased over weeks, special C Likely cause of young man back from leisure trip
precaution taken in case of NYHA class III and IV has swollen knee joints and foreign body sensation
and carvedilol and metoprolol are preferred drugs. in eyes
C A 35 years old lady presents with normal PT and Reiter’s syndrome
increased aPTT. 2 years back, she was operated C Cause of vasodilatation in spider nevi
for cholecystectomy and did not have any Estrogen
bleading episode. Next investigation for clinical C A patient presented with arthritis and purpura, lab
diagnosis examination showing monoclonal and polyclonal
Antiviper venom assay cryoglobulins. Histopathology shows deposits of
(Hint: APLA) cryoglobulins around the vessels. Patient should be
C Bad prognosis in AML is indicated by tested for
Monosomy 7 HCV
C Cause of nephrocalcinosis in granulomatous C Hepatomegaly is the essential feature of
disease Niemann Pick disease, von Gierke’s disease and
Increased conversion to 1,25 OH. Hurler syndrome.
C True for hepatitis B C Rapid infusion of insulin causes
Age of onset determines prognosis, period of Hypokalemia
communicability lasts several months and virus can C Digitalis toxicity enhanced by
be found in blood 1 month before jaundice. Renal failure, hypercalcemia and hypomagnesemia.
C A 20 years lady complains of headache while C True about hemochromatosis
studying with normal vision. Further evaluation Is genetically heterogenous
Family history of headache, menstrual history and C Strawberry gingivitis seen in
her interest in studies. Wegener’s granulomatosis.
C A child presenting with recurrent episodes of lip
AIIMS MAY 2010 and laryngeal edema and abdominal pain
C Radiological features of left ventricular heart failure associated with stress. Level reduced
are C1 esterase inhibitor.
Kerley B lines, cardiomegaly and increased flow in C Best test for intestinal malabsorption
upper lobe veins. D-xylose test.
C Most common childhood CNS tumor to metastasize C True about temporal arteritis
extraneuronally Can lead to sudden bilateral blindness, more
Medulloblastoma common in females and mostly affects elderly.
C Chang staging is used for C True about a patient presenting with acute
Medulloblastoma rheumatic carditis with fever
C Hypersensitivity vasculitis affects Valve replacement will ameliorate CCF.
Post-capillary venules C Cardiovascular complications of HIV infection
C Hyperextensibility of joint with normal elastic includes
recoil of skin is a feature of Pericardial effusion, cardiac tamponade and
Ehlers Danlos syndrome cardiomypathy.
C True statement about atherosclerosis is
Intake of PUFA is associated with decreased risk. AIIMS NOVEMBER 2009
C A girl presenting with severe hyperkalemia and
peaked T waves on ECG. Fastest way of shifting C Dissociated sensory loss in a case of tumor of central
potassium intracullarly spinal cord is due to lesion of
Insulin + glucose Decussating fibers of lateral spinothalamic tract.
C Features of hypocalcemia are C Rat tissue used in immunofluorescene method to
Numbness and tingling, circumoral paresthesias detect antinuclear antibody
and skin irritability and sensitivity. Liver > kidney
146 AIIMS and All India PGMEE—Review Questions

C PFT in asthma C Cross matching is required for transfusion of


Increased FRC and increased RV Single donor platelets, FFP transfusion and platelet
C A 21 years old female patient presents with history rich plasma transfusion.
of mild bilateral ptosis, proximal muscle weakness C A 15 years old girl presenting with intermittent
and easy fatiguability. Best in diagnosing the headache, tinnitus, vertigo, hearing loss and family
condition history of similar episodes in mother. Diagnosis
Edrophonium test Basilar migraine
C Alkaline phosphatase is decreased in C Type of anemia caused by pulmonary TB
Hypophosphatasia Microcytic hypochromic anemia.
C A patient with limited systemic sclerosis for past 10
years complains of shortness of breath for past one AIIMS MAY 2009
year. PFT-FVC—2.63/2.82, FEV %-88%/80%,
DLCO—5.2/16.3 (Observed/predicted). Diagnosis C OSHA guidelines for needle and other sharp
Pulmonary HTN injuries requires
C Vasculitis seen in adults Record maintaining of all injuries, information and
HSP, temporal arteritis and PAN. training and post-exposure prophylaxis.
C A 28 years gentleman met with an accident and C Radiological features of LVHF are
sustained crush injury (severe). He may develop Kerley B lines, cardiomegaly and increased flow in
Acute renal failure upper lobe veins.
Treponema pallidum can be found in CSF in
Diagnosis of 20 years male patient dying while
C
C
playing. Autopsy of heart showing myocyte Primary syphilis and secondary syphilis.
hypertrophy C Drug not used for treatment of acute hyperkalemia
HOCM Potassium exchange resins.
Seen in primary S j̈ogren syndrome
Diagnosis of elderly woman presenting with
C
C
features of fever, headache, diplopia, biopsy of Lymphoma
artery-panarteritis. Most probable diagnosis C Transmitted by blood
Temporal arteritis CMV, parvovirus B19, hepatitis G.
C A 21 years lady presenting with aortic arch C Left ventricular hypertrophy is caused by
aneurysm underwent resection and specimen sent MR, AR and AS
to histopathology showing all three layers involved C Gene therapy is given in
and giant cell present. Diagnosis Cystic fibrosis, sickle cell anemia and SCID.
Nonspecific aortoarteritis C Hypersensitivity vasculitis involves
C ABO antigens are seen in Postcapillary venules
Saliva, semen and sweat C Beevor sign is seen in
Abdominal muscle
C Necrotising lymphadenitis is seen with
C Most important predictor of coronary artery
Kikuchi disease.
disease among options
C A case of jaundice with 50% direct bilirubin, other
HDL > LDL
LFT normal. Diagnosis is
C Causes of congenital hypercoagulability
Rotor syndrome
Protein C deficiency, protein S deficiency and
C Used in congestive heart failure MTHFR gene mutations.
Spironolactone, nitrates and nesiritide. C Most common cause of acute RVF
C HLA B27 is associated with Massive pulmonary embolism
Ankylosing spondylitis (Hint: Acute cor pulmonale).
C Drugs used in visceral leishmaniasis C True about lupus anticoagulant
Pentamidine, paromomycin and miltefosine. Prolongation of only aPTT, recurrent second
C Not used in hyperkalemia trimester abortion and can be associated without
50 ml of 50% dextrose. any clinical symptoms.
Medicine 147

C Complication of blood transfusion C Framingham major criteria for diagnosis of heart


Hyperkalemia, citrate toxicity and hypothermia. failure
C A 56 years female patient presents with severe PND, cardiomegaly and S3 gallop
headache and neck stiffness of abrupt onset. Such a C Antibody found in myositis-Anti Jo1.
headache she had never seen before. Also nausea C A 40 years old lady presents with polyarthritis with
and photophobia, diagnosis increased RF and increased ANA levels. Feature
Subarachnoid hemorrhage. differentiating RA from SLE
C Serum fructosamine can be used in Articular erosions on X-ray
Rapid change in DM treatment, monitoring short term C True about fluorescent antibody detection test in
control of DM and screening of DM in pregnancy. diagnosis of Plasmodium falciparum
C Tumor lysis syndrome, true are It is an immunochromatographic test, detection of
Hyperkalemia, hyperuricemia and hyperphospha- LDH antigen and detection of glutamate dehydro-
temia. genase antigen.
C True about prions disease C Best investigation for lung volume measurement in
Caused by infectious proteins, biopsy of brain an emphysematous patient with bullous lesion
diagnostic and neurodegenerative disease.
Body plethysmography
C Nodular regenerating hyperplasia is associated with
C True statements
Drug
Selenium deficiency causes cardiomyopathy,
C Treatment of choice for Kawasaki disease
increased calcium intake cause iron deficiency and
IV immunoglobulin vitamin A deficiency occurs after 6 months to 1 year
C True about Wilson’s disease of low vitamin A diet.
AR, serum ceruloplasmin level < 20 mg/dl and zinc C Diagnosis of a 54 years old smoker man comes with
acetate is used as maintenance therapy. fever, hemoptysis, weight loss, oligoarthritis and
C Test for reversible cardiac ischaemia fleeting opacities on serial skiagram
Thallium scan Wegener’s granulomatosis.
C Conditions causing subcortical dementia C Spontaneous CSF leak occurs in
Parkinson’s disease, HIV encephalopathy and
Partial or complete empty sella syndrome, pseudo-
progressive supranuclear palsy.
tumor cerebri and low risk encephalocele.
Nobel prize in medicine in 2007 was given for
Most common cause of mononeuritis multiplex in
C
C
Principles for introduction of specific gene modifi- India
cations in mice by use of embryonic stem cells.
Hansen’s disorder
C Neurophysiological defects present in right lobe are
Visuospatial defect, anosognosia and dysgraphia.
C A 80 years old women presents with progressive AIIMS MAY 2008
loss of memory, difficulty in recalling names, C Most common organism associated with reactive
difficulty in speech and irritability along with arthritis
neurofibrillary tangles, found in brain Chlamydia
Beta-amyloid C Nerve compressed by aneurysm of posterior
communicating artery is
AIIMS NOVEMBER 2008 Oculomotor nerve
C True about Wiskott-Aldrich syndrome C Feature of high altitude pulmonary edema
Failure of aggregation of platelets in response to Associated with pulmonary HTN
agonists, thrombocytopenia is seen and patient C Non-noxious stimulus perceived as pain in
presents with eczema. Allodynia
C Anti-TNF alpha drugs are used for the treatment of C All are seen in cystinosis
Seronegative arthritis, psoriatic arthritis and Crohn’s Photophobia and blonde hair, presence of cystine
disease. within leukocytes and Fanconi’s syndrome.
148 AIIMS and All India PGMEE—Review Questions

C True statements are C Alzheimer’s disease, involved is


1 alpha hydroxylation occurs in kidney, 25 alpha Cortical atrophy of temporoparietal cortex.
hydroxylation occurs in liver and in absence of C Rasmussen’s aneurysm involves
sunlight daily requirement of vitamin D is 400–600 IU. Pulmonary aneurysm
C Secondary structure of prion proteins in prion C Damage to categorical hemisphere leads to -
disease like Creutzfeldt-Jakob disease ( CJD) is Senseless, fluent speech.
Beta sheets C Drug of choice in absence seizure
C Post-transplant lymphoma is Valproate
B-Cell
C Diagnosis of a man with painful ophthalmoplegia, AIIMS NOVEMBER 2007
cavernous sinus dilatation
Tolosa Hunt syndrome C True about linkage analysis in familial gene
C Lhermitte Duclos disease, true statement disorders
Thickened cerebellar folli Characteristic DNA polymorphism in a family is
associated with disorders.
C A male patient presenting with ST segment
depression in lead V1–V4, after one hour treated C Causes of pulmonary HTN
with Morbid obesity, high altitude and fenfluramine.
Aspirin, beta-blocker and morphine. C An 18 years old girl presents with rheumatic
C Most common primary lymphoma of spleen carditis and mitral insufficiency. PFT findings
Small lymphocytic lymphoma. Increaseded RV and decreased FRC.
C Rituximab is used in C A 40 years old male patient, chronic smoker with
acute epigastric discomfort for past one hour. ECG
NHL, RA and SLE
shows ST segment elevation in inferior leads.
C Kasabach-Merritt syndrome is associated with
Immediate treatment
Giant hemangioma
Aspirin
Causes of increased anion gap
Best investigation where there is clinical suspicion
C
C
DKA, starvation and ethylene glycol poisoning. of pulmonary embolism in a patient is
C Valve affected in rheumatic fever Multi-detector CT angiography.
Mitral valve, aortic valve and tricuspid valve. C Causes of HTN with hypokalemia
C Drug used to perform stress echo Bilateral renal artery stenosis, primary hyper-
Dobutamine aldosteronism and Cushing’s disease.
C Torsade de pointes causes C Features of Cushing’s disease
Wide QT Central obesity, easy bruising and glucose intole-
C True about autonomic neuropathy rance.
Resting tachycardia, silent MI and orthostatic C Intensive management of diabetes is needed in
hypotension. Pregnancy, post-kidney transplant in diabetic
C True in Millard Gubler syndrome–6th cranial nerve nephropathy and DM with acute MI.
palsy, 7th cranial nerve palsy and contralateral C An 18 years old girl presents with amenorrhea,
hemiplegia. anorexia, weight loss, and with milk discharge from
C Differentiating test between insulinoma and nipples. Diagnosis
sulfonylurea related hypoglycemia Anorexia nervosa
Plasma sulfonylurea level C All are seen in DKA
C Drugs used for treating pulmonary HTN Tachypnea, dehydration and abdominal pain/
Calcium channel blockers, endothelin receptor tenderness.
antagonists and phosphodiesterase inhibitors. C Hypercalcemia is seen in
C Goodpasture’s disease is characterized by Lithium, multiple myeloma and hypervitaminosis D.
Glomerulonephritis, diffuse alveolar hemorrhage C A 36 years old female patient with symptoms of
and presence of antibodies to BM. hyperparathyroidism, pancreatic tumor, adrenal
Medicine 149

cortical hyperplasia, pituitary adenoma, islet cell C Associated with pituitary apoplexy
tumor with cutaneous angiofibroma. Diagnosis is DM, HTN and sickle cell anemia.
MENI C HUS is associated with
C Senile cardiac amyloidosis is due to deposition of EHEC, Shigella and Campylobacter.
Transthyretin C Infective endocarditis due to Pseudomonas is most
C Fractional excretion of Na+ < 1 is seen in commonly seen with
Pre-renal azotemia Intravenous drug abuse of pentazocin.
C Features of SIADH C Causes of raised JVP with hypotension are
Decreased sodium, maintaining the concentrating Cardiac tamponade, RV MI and heart failure.
ability of urine osmolality>100 mosm, normal C Water hammer pulse seen in
sodium balance maintained indicating excess AR
urinary sodium is due to efficient sodium intake C Useful to decrease mortality and renal failure in
and hypouricemia. acute liver disease due to alcoholism
C Metabolic alkalosis is seen in Pentoxyfylline
Primary mineralocorticoid excess. C In primary pulmonary HTN basic abnormality in
C Micronodular cirrhosis is seen in gene lies in
Alcoholic liver disease, hemochromatosis and Bone morphogenetic protein receptor-II.
chronic extrahepatic biliary obstruction. C Life threatening complications of DM are
C Seen in chronic active hepatitis B Malignant otitis externa, rhinocerebral mucormy-
Total core antibody, HBsAg, HBeAg. cosis and emphysematous pyelonephritis.
C Marker of acute hepatitis B infection C Digital clubbing is seen in
DNA polymerase Endocarditis, pulmonary arteriovenous fistula and
C Window period in HIV tricuspid atresia.
Period between onset of infection and clinically C Best method to monitor intracranial pressure is
detectable level of antibodies. Intraventricular catheter
C Cryoprecipitate does not contain C Most common site of histiocytosis
Factor IX Bone
C Absence of corpus callosum leads to C Atkin’s diet is
No neurological manifestations. Carbohydrate restricted low calorie diet.
C Most common site of subependymal astrocytoma C In uremic patient nail and half nail sign seen in
(giant cell) Increased capillary density at the distal half of nails.
Foramen of Munro C Earliest phenotypic manifestation of idiopathic
C von Hippel Lindau is associated with hereditary hemochromatosis is
Endolymphatic sac tumors, pheochromocytoma Increased transferrin saturation.
and islet cell tumors. C Reticulocytosis is seen in
C NFI is most commonly associated with PNH, hemolysis and dyserythropoietic syndrome.
Optic pathway glioma C Dietary intervention reducing further risk in MI
patients
AIIMS MAY 2007 n3PUFA
C Arsenic is used in treatment of C Addisons’s disease is associated with
Acute promyelocytic leukemia. Cardiac atrophy, decreased diastolic BP and serum
C Plasmapheresis (plasma exchange therapy) is used cortisol < 8.
in treatment of C Associated with MEN II
TTP Pheochromocytoma, MTC and parathyroid
C Most common catheter induced blood infection is adenoma.
due to C Increased anion gap metabolic acidosic is seen in
Coagulase negative staphylococci. Starvation, salicylate poisoning and lactic acidssis.
150 AIIMS and All India PGMEE—Review Questions

C Ratio of AST/ALT > 1 C Deleterious effect likely to occur in hypothermia


Alcoholic hepatitis. Cardiac arrhythmia, reversible coagulation and
C A 16 years old girl with primary amenorrhoea renal failure.
attends OPD with normal sexual development, C Charcoat’s joint includes
normal breast but with absent pubic and axillary Syringomyelia, leprosy and diabetes.
hair, on examination-bilateral inguinal hernia, C Persistently low C3 complement levels is found in
USG-absent uterus and blind vagina. Diagnosis Mesangiocapillary glomerulonephritis, cryoglobu-
Androgen insensitivity syndrome. linemia and SLE.
C A female child with virilization, hypertension and C Likely to cause constrictive pericarditis
low plasma renin. Diagnosis Tuberculous pericardial effusion, staphylococcal
11 β hydroxylase deficiency. effusion and post-cardiac surgery.
C Features seen in hemolytic anemia C Treatment of multiple sclerosis by drug
Decreased heptoglobin, reticulocytosis and Interferon β
hemoglobinuria. C Feature of high altitude pulmonary edema
C Nephrocalcinosis is seen in Associated with pulmonary hypertension.
Sarcoidosis, distal RTA and milk alkali syndrome. C Limb girdle dystrophies are
C Interstitial nephritis is seen with Sarcoglycan dystrophy, dysferlin dystrophy and
β lactam inhibitors, diuretics and allopurinol. calpain dystrophy.
C A 30 years male patient presents with chronic C Most specific test for rheumatoid arthritis
diarrhea, anemia, raised liver enzymes. Most likely Anti-ccp antibody.
associated with C Lipid profile after treatment with ω3 PUFA
Antiendomysial antibody Decreased LDL and decreased total cholesterol.
C Most common site of intracranial hemorrhage in C Most common gene mutation in β-thalassemia
HTN is Intron 1 inversion.
Basal ganglia C Exclusively involving neurons
(Note: Putamen) Spinocerebellar ataxia.
C Most specific urinary finding in acute pyelonephritis C A male patient presenting with increased serum
WBC cast ferritin, decreased TIBC, increased serum iron,
C A 20 years male patient presents with anemia, and increased % saturation. Diagnosis
mild hepatosplenomegaly. His Hb is 5 gm/dl, Sideroblastic anemia
history of single blood transfusion till date. C A 15 years old boy presents with LDL 600 mg/dl,
Diagnosis is TG 160 mg/dl. Finding on physical examination
Autoimmune hemolytic anemia. Tendon xenthoma
C Investigations done in neurocardiogenic syncope C Tests useful in a patient with history of syncopal
Tilt table, carotid sinus massage and orthostatic attack
blood pressure monitoring. Electrophysiological test, table tilt test and Holter
monitoring.
AIIMS NOVEMBER 2006 C HAM test is done for
C Mutation in α 5 chain of collagen 4. Diagnosis is GPI anchor protein
Alport’s syndrome
(Note: Mutation in α 3 chain of collagen 4–Good- ALL INDIA 2012
pasture syndrome. Mn: 3G) C Pituitary macroadenoma refers to size of tumor
C True about Carey Coombs’ murmur > 1 cm
Delayed diastolic murmur, seen in rheumatic fever C A 5 years old child of severe nephrotic syndrome on
and low pitched murmur. treatment with tacrolimus, frusemide and predni-
C Reflux disease causing proteinuria of nephrotic range solone developed seizures, lab finding—serum
FSGS sodium–135 mEq/L, blood urea–78 mg/dl, serum
Medicine 151

creatinine–0.5 mg/dl, serum albumin–1.5 g/dl, C A child presents with ambiguous genitalia without
serum total calcium–7.5 mg/dl and urine albumin– hyperpigmentation, and normal BP, 2.5 cm phallus
2 g. Most likely cause of symptoms in this patient with no opening at tip , labia developed. Gonads are
Tacrolimus toxicity. not seen in inguinal region and Mullerian structures
C The least common presentation of multiple myeloma are present on USG. Diagnosis
is Maternal virilizing tumor.
Hyperviscosity C Neurofibrillary tangles are seen in case of
C True about fibromyalgia Alzheimer’s disease. Areas of brain resistant to
Associated with EEG abnormalities, increased neurofibrillary tangle formation
cortisol and decreased blood flow to brain. Lateral geniculate nucleus.
C The treatment of acute pulmonary edema includes C The most preferred area of deep brain stimulation
Frusemide, morphine and positive ventilation. in a patient with Parkinson’s disease with intractable
C RTA type 1 tremors
Hypokalemia, nephrocalcinosis and cannot acidify Subthalamic nuclei.
urine to pH < 5.5. C Small fiber neuropathy is seen in
C Most sensitive and specific tests for diagnosis of HIV, Hansen’s and amyloidosis
myocardial infarction in an athelete presenting C Episodic muscle weakness can be caused by
with chest pain are Channel opathies, Lambert Eaton myasthenic
Troponin I and T. syndrome and hyperphosphatemia
C Example of small vessel vasculitis C The triad of strabismus, diplopia and ptosis due to
Microscopic polyangitis damage of
C A young girl is admitted with joint pains and butterfly Oculomotor nerve.
rash and positive urine proteinuria. Test for diagnosis C Sign of brain stem death
Anti-ds-DNA antibody. Fixed non-reactive pupil.
C Diseases that may progress to lymphoma C A patient presents with ataxia, ankle and knee jerk
S j̈ogren syndrome, ataxia telengiectasia and lynch absent and plantar extensor. Diagnosis
2 syndrome.
Friedreich’s ataxia
C True about angioneurotic edema
C A 14 years female patient presents with quadriparesis,
It is due to C1 esterase inhibitor deficiency, occurs facial palsy, winging of scapula and ptosis and
in extremes of temperature and it is due to ACE history of similar but less severe illness in father and
inhibitors. brother, CPK level raised (500 IU/L). Diagnosis
Secondary hemochromatosis occurs in
Scapulofacia humeral dystrophy.
C
β thalassemia, repeated blood transfusions and
C Conduction velocity of nerve affected in
sideroblastic anemia.
Leprosy, hereditary neuropathy and AIDP.
C The occurrence of hyperthyroidism following
administration of supplemental iodine to subjects in C Prader Willi syndrome is associated with
endemic multinodular goitre Ghrelin
Jod Basedow effect. C Age related deterioration of cognitive function is
C True about central hypothyroidism due to increase in
Most common cause is craniopharyngioma, before Homocysteine
starting treatment, checkout for adrenal insufficiency C A patient presents with subarachnoid hemorrhage,
and TSH is not a good marker for diagnosis. NCCT- blood in 4th ventricle. The blood is most
C The third generation TSH detection tests can detect likely to occur from an aneurysm of
TSH at a minimum level of PICA
0.01–0.02 C True about von Hippel Lindau syndrome
C Culture CSF showing Pneumococcus is most likely Hemangiopericytomas are seen in the craniospinal
to show axis, supratentorial lesions are common and tumors
Pleocytosis, high protein and reduced sugar. of Schwann cells are common.
152 AIIMS and All India PGMEE—Review Questions

ALL INDIA 2011 C Drug recommended for medical treatment of


variceal bleed
C Dietary goals for patient with high risk of coronary Octreotide
heart disease are
C True regarding DM
LDL cholesterol<100 mg/dl, saturated fat <7% of
total calories and salt restriction < 6 gm/day. Insulin level may be increased in patients with Type
II DM, insulin is essential to reverse diabetic
C ECG is poor in detecting ischemia supplied by
ketoacidosis and intravenous insulin is administered
Left circumflex coronary artery.
as a sliding scale in hospital setting.
C True about treatment of hypothyroidism in a
C True about diabetic ketoacidosis
patient with ischemic heart disease
Decreased bicarbonate
Low dose of levothyroxine.
C Coarctation of aorta is most commonly associated with C Causes of fasting hypoglycemia are
Bicuspid aortic valve Glucose 6-phosphatase def., glycogen synthase
C True about digoxin induced arrhythmia deficiency and uremia.
PAT with variable AV block, ventricular bigeminy C Tests used in diagnosis of insulinoma
and atrial fibrillation. Fasting blood glucose, C-peptide levels and
C Best inotrope agent for right heart failure secondary insulin/glucose ratio.
to pulmonary hypertension C True about Nesidioblastosis
Milrinone Hypoglycemia episodes may be seen, histopatho-
C Poor prognostic factor in ALL logy shows hyperplasia of islet cells and diazoxide
t(9,22), t(4,11) may be used for treatment.
C Cavitatory lesions in lung are seen in C True about diabeties insipidus
Staphylococcal pneumonia. Water deprivation test is required.
C Tumor most commonly associated with SVC C Drug of choice for central diabetes insipidus
syndrome
Desmopressin
Small cell carcinoma lung
C Conditions associated with hyperthyroidism are
C Pulmonary fibrosis is associated with the use of
Bleomycin > Methotrexate. Grave’s disease, toxic multinodular goitre and
struma ovary.
C The acid base status of a patient is pH–7.48 and
pCO2–30 mm Hg. The pH has a partially compesated C Diagnosis of a patient with lab finding decreased T4
primary and increased TSH
Respiratory alkalosis Hashimoto’s disease
C Sterile pyuria is seen in C True about pseudohypoparathyroidism
Renal tuberculosis. Decreased formation of cAMP is seen.
C A young lady presents with symptoms of UTI. C A patient presenting with low serum Ca, high
Findings of urine sample supporting diagnosis of phosphate and elevated PTH. Investigations done
uncomplicated acute cystitis are to establish a diagnosis are
Positive nitrite tests, detection of one bacteria/field Serum creatinine, cAMP response to PTH and
on Gram stain and >10 WBC/HPF. urinary myoglobin.
C Contraindication for medical management of gall- C True about pheochromocytoma
stones
Arise from sympathetic ganglions, secrete catecho-
Radiopaque stones
lamine and 95% occur in abdomen.
C A young patient presents with fever, right upper
quadrant pain, hepatosplenomegaly but no C A 15 years old girl presenting with repeated episodes
jaundice, USG-solitary homogenous hypoechoic of throbbing occipital headache, ataxia and vertigo
lesion in right lobe measuring 5 cm × 5 cm × 4 cm, tests with similar history in mother, diagnosis
for hydatid disease negative. Best initial treatement Basilar migraine.
Antiamoebic/antibiotics alone. C Drugs used in prophylaxis of migrane are
(Note: Liver abscess) Propranolol, flunarizine and topiramate.
Medicine 153

C True about anterior choroidal artery syndrome C Streptokinase and urokinase are contraindicated in
Hemiparesis, hemisensory loss, and homonymous Intracranial malignancy
hemianopia. C A young male patient presents with pH–7.5, pCO2–
C Sites involved in posterior cerebral artery infarct 30 mm Hg, pO2–102 mm Hg, and HCO3–16 mEq/L.
Midbrain, thalamus and temporal lobe. Compensatory mechanism
C True about delirium tremens are Metabolic acidosis
Visual hallucinations, tremor and clouding of C A 29 years lady with history of progressive
consciousness. breathlessness and exercise intolerance for four
C A 60 years old man with progressive dementia of months, FVC 90%, FEV/FVC 86%. Oxygen
recent onset presenting with intermittent irregular saturation dropped from 92% to 86%. Diagnosis
jerky movements with EEG showing periodic sharp Primary pulmonary hypertension.
biphasic wave. Diagnosis is C A truck driver presented with history of fever for 4
Creutzfeld Jakob disease weeks and dry cough. He also gives history of
C A 30 years old woman presenting with proximal weight loss of about 10 kg with X-ray showing
weakness of muscle, ptosis and easy fatigubility. bilateral reticulonodular infiltrates. Diagnosis
The most sensitive test for diagnosis is Pneumocystis carinii pneumonia.
Single fibre EMG > Edrophonium test. C Cavitary lesions in lung are seen in
C NARP syndrome is a type of Staphylococcal pneumonia.
Mitochondrial function disorder C True about lung carcinoma
Oat cell variant is typically associated with hilar
adenopathy.
ALL INDIA 2010
C Pre-renal azotemia is characterized by
C A 16 years old girl presents with history of fatigua- Fractional excretion of Na+ <1%, urinary osmolality
bility, weakness and lethargy, CBC-Hb–7.0 gm% > 500 mOsm/kg and reversible with replacement
MCV 70, MCH 20 pg/cell and RDW–20. Diagnosis fluids.
Iron deficiency anemia C Plasma urea creatinine ratio of 20:1 may be seen in -
C Major criteria for diagnosis of polycythemia vera Pre-renal failure.
JAK-2 mutations C A woman presents with non-progressive dysphagia
C True about Fanconi’s anemia only for solids. Barium study showing proximal
Congenital anomalies, hypercellular bone marrow esophageal dilatation with distal constriction.
and usually normocytic/macrocytic cell morphology. Diagnosis is
C Investigation done immediately to best confirm a Lower esophageal ring.
non-matched blood transfusion reaction C A young girl presents with abdominal pain and
Direct Coombs’ test recent change in bowel habit, with passage of
C Bence Jones proteinuria may be seen in mucous in stool. There is no associated blood in
μ heavy chain disease. stool and symptoms are increased with stress.
C Bence Jones proteins are derived from Diagnosis
Gamma globulins Irritable bowel syndrome.
C True about third heart sound (S3) C Investigation of choice for an elderly patient
Seen in constrictive pericaraditis, ASD and VSD presenting with a prolonged history of weakness
C A 20 years lady asymptomatic on routine examination and lethargy, on examination-anaemic and stool
is observed to have midsystolic click. Valves may show positive for occult blood
Myxomatous degeneration. (Mn: M for M) Colonoscopy
C Beck’s triad is seen in C A patient presents with positive HBsAg on routine
Cardiac tamponade testing. Other serological test negative. Clinically
C Accelerated idioventricular rhythm (AIVR) is most asymptomatic and liver enzymes within normal
common arrhythmia associated with range. Diagnosis
Myocardial reperfusion Inactive HBV carrier
154 AIIMS and All India PGMEE—Review Questions

C A male patient is observed to have HBsAg +ve, C Associated with hypergonadotrophic hypogonodism
HBeAg-ve, anti-HBeAg antibody +ve. HBV DNA in males
copies 100,000/ml and AST and ALT elevated to 5 Klinefelters’s syndrome, Noonan syndrome and
times upper limit of normal value. Diagnosis viral orchitis.
HBV precore mutant C Associated with peripheral artery disease, coronary
C A patient presents with unconjugated hyperbiliru- heart disease and stroke
binemia and elevated urobilinogen levels in urine. Insulin deficiency
Diagnosis C True about hyponatremia
Hemolytic jaundice Hyponatremia associated with hyperglycemia has
C A patient presents with unconjugated hyperbiliru- high plasma osmolality, hyponatremia associated
binemia and presence of urobilinogen in urine. with SIADH is normovolemic and NSAIDS increase
Least likely diagnosis the potency of vasopressin.
Dubin Johnson syndrome (Mn: UGC-unconjugated C Diagnosis of a patient presenting with dementia,
in Gilbert and Criggler-Najjar syndrome) urinary incontinence and ataxia
C True about Wilson’s disease Normal pressure hydrocephalus.
Low serum ceruloplasmin and high urinary copper. (Note: DUA)
C Gout is a disorder of C Diagnosis of a patient known to have mitral
Purine metabolism stenosis, atrial fibrillation, presents with acute onset
C True about primary gouty arthritis of weakness in left upper limb recovering completely
Uric acid levels may be normal at the time of an in two weeks
acute attack, male > female and definitive diagnosis Ischemic stroke
requires aspiration of synovial fluid. C A 25 years old patient presents with acute onset of
C APLA syndrome is associated with fever and focal seizures. MRI scan showing
Thrombotic disorders, coagulation disorders and hyperintensity in temporal lobe and frontal lobe
recurrent fetal loss. with enhancement. Diagnosis
C True about APLA Herpes simplex encephalitis.
Commonly presents with recurrent fetal loss, may C Brain damage in head injury patient aggrivated by
cause pulmonary hypertension and warfarin given Hyperglycemia, hypercapnia.
as treatment. C Site of lesion in motor neuron disease
C Treatment recommended in a women with APLA Anterior horn cells.
and history of prior abortion/stillbirth C True about GB syndrome
Aspirin + LMW heparin. Ascending paralysis, flaccid paralysis and albumino-
C Associated with thymoma cytological dissociation.
Myasthenia gravis, hypogammaglobulinemia and C Kayser-Fleishner rings (KF rings) are seen in
Cushing’s syndrome. Wilson’s disease
C Plasmapharesis is used in
Myasthenic crisis, GB syndrome and polymyositis. ALL INDIA 2009
C Low calcium and high phosphate is seen in
Hypoparathyroidism C A 20 years female patient asymptomatic, MCV–70,
C True about pseudohypoparathyroidism Ferritin–100 g/L, Hb–10 gm%. Diagnosis
Low serum calcium, high serum phosphate and Thalassemia trait
Albright hereditary osteodystrophy. C Associated with PNH
C A patient presents with symptoms of hypogly- Cerebral thrombosis, Budd-Chiari syndrome and
cemia. Investigation reveals decreased blood pancytopenia.
glucose and increased insulin levels. C-peptide C A 7 years female child presents with repeated
assay is done showing normal levels of C-peptide. episodes of bleeding into joints, APTT prolonged,
Most likely diagnosis PT normal. Diagnosis
Accidental exogenous insulin administration. von Willebrand disease
Medicine 155

C True about coagulation factor VII C Test for mucosal function of GIT
AR, deficiency treated by FFP, and shorter half life D-Xylose test
as compared to Hageman factor (XII). C Non-invasive diarrhea caused by
C True about lupus anticoagulant Bacillus cereus
May present with isolated prolongation of APTT, C Features of secretory diarrhea includes
may present with recurrent abortions and may Stool volume > 1L/day, normal osmotic anion gap
occur with minimal clinical manifestation. and painless.
C Acquired causes of hypercoagulability includes C True about cystic fibrosis
IBD, myeloproliferative disorders and prolonged AR, predisposition to pulmonary infection with
surgery> 1 hour. Pseudomonas and cirrhosis is an established
C Causes of deep vein thrombosis complication of CF.
OCP, PNH and prolonged surgery. C Hepatomegaly is a feature of
C Investigation to clinch the diagnosis in older man von Gierke’s disease, Hurler’s disease and Niemann-
presents with headache, recurrent infections, Pick’s disease.
multiple punched out lytic lesions of X-ray skull C Increased aldosterone leads to
Protein electrophoresis
Hypernatremia, hypokalemia and hypertension.
International prognostic index for lymphoma
A 30 years old male known diabetic patient on oral
C
C
includes
hypoglycemic drugs for last few years has lost
Stage of disease, number of extra-lymphatic sites weight and never had DKA. His grandfather is
involved and LDH. diabetic but his father is non-diabetic. Diagnosis
C wave in JVP is due to
Type II DM (Note: If both father and grand father
C
Bulging of tricuspid valve into the right atrium. diabetic, diagnosis-MODY).
C Features of right sided heart failure
C Change occurring in a patient with severe hyper-
Pulsatile liver, increased JVP and positive hepato-
glycemia given IV insulin
jugular reflux.
Hypokalemia
C Systolic thrill in left 2nd and 3rd intercostal space
may be seen in C Most likely cause of bilateral superior temporal
quadrantopia and galactorrhea in a women
Subpulmonic VSD, PS and Ebstein anomaly.
C Best predictor for risk of cardiovascular events in Pituitary macroadenoma
future C Hypercalcemia is associated with
CRP Hyperparathyroidism, sarcoidosis and milk alkali
C Cardiovascular complications of HIV infection syndrome.
includes C Endocranial causes of carpal-tunnel syndrome
Pericardial effusion, cardiac tamponade and cardio- includes
myopathy. DM, hypothyroidism and acromegaly.
C Seen in idiopathic pulmonary hemosiderosis C Hirsutism is caused by
Iron deficiency anemia, diffuse alveolar hemorrhage Cushing’s syndrome, hyperprolactinemia and
and hemoptysis. acromegaly.
C A male patient presents with swelling of face and C True about temporal arteritis
lips, respiratory distress, intense pruritus, hypoten- More common in females seen in elderly women
sion and feeling of impending doom. Diagnosis and can lead to sudden bilateral blindness.
Anaphylaxis C Recurrent bilateral hypopyon formation associated
C True about small cell carcinoma with thrombophlebitis is consistent with
Chemosensitive tumor. Behçet’s syndrome
C Restless leg syndrome is seen in C False positive rheumatoid factor associated with
CRF HbsAg, VDRL and Coombs’ test
C Positive urinary anion gap helps establish diagnosis of C True about hemochromatosis
Renal tubular acidosis AR
156 AIIMS and All India PGMEE—Review Questions

C Spastic paraplegia caused by C Cold agglutinin


Vit. B12 deficiency, cervical spondylosis and motor IgM antibody and Donath Landsteiner antibody.
neuron disease. C Treatment with erythropoietin in a patient of renal
C True about Lambert Eaten mysthenic syndrome cell carcinoma may be associated with
Inceremental response to repeated electrical Increased Hb and eliminates need for transfusion.
stimulation. C Most common blood transfusion reaction
C Associated with von Hippel-Lindau syndrome Febrile non-hemolytic reaction.
Retinal and cerebellar hemangioblastoma, Pheo- C DIC is seen in AML
chromocytoma and RCC. M3
C Triad of tuberous sclerosis include C All transretinoic acid is used in treatment of
Epilepsy, adenoma sebacium and low intelligence. Acute promyelocytic leukemia (M3).
C True about NF C Drug of choice for CML
Cataract, scoliosis and cutaneous neurofibroma. Imatinib
C SIRS includes C Torsades des pointes may be caused by
Leucocytosis, hyperthermia and hypothermia. Quinidine
C Correct pair C Drug of choice for supraventricular tachycardia
Selenium deficiency—Cardiomyopathy Verapamil
Iodine toxicity—Acne like eruptions C Investigation of choice for diffuse esophageal
Calcium toxicity—Milk alkali syndrome spasm
C In a uremic patient, dialysis can reverse Manometry
Peripheral neuropathy, seizures and pericarditis. C A 30 years lady presents with non-progressive
C True about TTP dysphagia for both solids and liquids. Finding on
Microangiopathic hemolytic anemia, thrombo- barium swallow that confirm the diagnosis is
cytopenia and normal complement level. Multiple sacculations and pseudodiverticulae
C Disease showing abnormal folding of proteins (Corkscrew esophagus).
Creutzfeld Jakob disease, prion disease and C A 20 years old male patient presents with HBsAg
amyloidosis. +ve, HBeAg -ve and liver enzyme normal. Next
done is
Serial monitoring
ALL INDIA 2008
C Acute pyelonephritis and uncomplicated UTI may
C Investigation needed in a patient presenting with be differentiated by
macroglossia and loss of tongue papilla; Hb 11.5 WBC cast, concentrating defect and antibody to
and MCV 100. Next step Tomm Horsfall protein.
B12 estimation C Best method of urine collection associated with
C Megaloblastic anemia should be treated with both least contamination
folic acid and vitamin B12 because Suprapubic aspiration
Folic acid alone causes improvement of hematologic C Colored urine is seen in
symptoms but worsening of neurological symptoms. Pyridium, rifampicin and nitrofurantoin.
C Most appropriate test for 40 years lady presenting C MENI syndrome is most commonly associated with
with anemia, jaundice and spherocytosis Gastrinoma
Coomb’s test C Cushing’s disease is associated with
C Ring sideroblasts are characteristically seen in Increased ACTH and increased cortisol.
Myelodysplastic syndrome C Conn’s syndrome is most commonly associated
C Pancytopenia with cellular marrow is seen in with
Megaloblastic anemia, myelodysplasia and PNH Cortical adenoma
C Hemolysis in G6PD may be caused by C Causative agent for rheumatoid arthritis is
Primaquine, chloroquine and quinine Mycoplasma
Medicine 157

C APLA is associated with ALL INDIA 2007


Recurrent abortions, venous thrombosis and
pulmonary HTN. C Diagnostic of iron deficiency anemia
C True about hemochromatosis Increased TIBC, decreased serum ferritin.
Arthropathy involving small joints of hands may be C A normotensive patient with normal Hb suffered
seen, skin pigmentation is a frequent presentation massive blood loss. Findings present in him
and hypogonadism may be seen. Reticulocytosis, thrombocytosis and increased
C A 24 years male patient presents with abdominal MCV.
pain, rash, palpable purpura and arthritis. Diagnosis C Autoimmune hemolytic anemia is associated with
is malignancy of cell lineage
HSP B-cell
C Thymoma is associated with C Pancytopenia with cellular marrow is seen in
Myasthenia gravis PNH
C Chron’s disease may be caused by C Pancytopenia with hypercellular bone marrow is
Clostridium difficile seen in
C A 25 years male patient presents with history of Megaloblastic anemia
chronic diarrhea. Pathological examination reveals C Bone marrow biopsy is absolutely indicated in
cryptitis and crypt abscess. Diagnosis is Hairy cell leukemia
Ulcerative colitis C Following are seen in PML
C Histopathological findings in Whipple’s disease Retinoic acid is used in treatment, 15/17 transloca-
includes tion may be seen and associated with DIC.
Marked increase in number of macrophages in the C A 80 years male patient, asymptomatic, presents
mucosa, dilatation of lymphatic in the mucosa and with total leukocyte count of 1 lakh with 80%
lipid deposition in mucosa. lymphocytes and 20% PMC. Most probable
C Dying back neuropathy is seen in diagnosis is
Diabetic neuropathy; arsenic neuropathy and CLL
porphyria. C True about mycosis fungoides
C Agents likely to cause meningitis in the elderly Most common form of cutaneous lymphoma,
Listeria monocytogenes, Streptococcus pneumonia and Pautrier’s microabscess and erythroedema seen
Gram-negative bacteria. and spreads to peripheral circulation.
C Pick’s body in pick’s disease is C Erythropoietin is increased in
Tau protein HCC, RCC and cerebellar hemangioblastoma.
C Biondi ring tangles are found in C A patient on aspirin will have
Choroidal plexus cells. Prolonged BT
C Feature of extramedullary tumor C Set of findings in DIC
Early corticospinal sign and paralysis, root pain or Increased FDP, prolonged PT and reduced
midline back pain and abnormal CSF. platelets.
C Clinical feature of conus medullaris syndrome C True regarding Bernard-Soulier syndrome
includes Ristocetin aggregation is normal.
Plantar extensor, sacral anesthesia and lower sacral C ECG findings in hypokalemia
and coccygeal involvement. Increased PR interval with ST depression.
C True about diffuse axonal injuries C CCF is associated with increase in
Caused by shearing force, most common at junction Right atrial mean pressure, serum urea and serum
of gray and white matter and predominant white epinephrine.
matter hemorrhages in basal ganglion and corpus C A person with MR and atrial fibrillation presents
callosum. with syncope. On examination person’s heart rate is
C Seen in chronic regional pain syndrome 55. Most probable cause is
Pain, swelling and osteoporosis. Digitalis toxicity
158 AIIMS and All India PGMEE—Review Questions

C A patient presents with acute anterior wall C C-ANCA indicates antibody formed against
infarction and hypotension. Immediate treatment Proteinase 3
modality for this patient (Note: P-ANCA-antibody against M protein)
Angiography and primary angioplasty. C Deposition of Anti-ds DNA Ab in kidney, skin,
C A 70 years male patient develops pneumonia choroid plexus and joints is seen in
and septicemia. Patient goes into renal failure and SLE
has BP of 70/50 mm Hg. Drug used to maintain BP C Best marker for drug induced lupus
is Anti-histone ab
Norepinephrine C True about Raynaud’s disease
C A patient presents with decreased vital capacity Positive antinuclear antibodies (ANA)
and decreased total lung capacity. Most likely C Components of Kawasaki disease
diagnosis
Pedal edema, truncal rash and pharyngeal conges-
Sarcoidosis tion.
C Drugs commonly used in regimens against H. pylori C Increased ICT is associated with
Amoxicillin, bismuth subciatrate and omeprazole. Abducens paralysis, headache and visual blurring.
C A patient with H. pylori infection is treated with C Pontine stroke is associated with
drug. The best method to detect presence of
Bilateral pinpoint pupil, pyrexia and quadriparesis.
residual H. pylori infection in this person is
C True about Benedikt’s syndrome
Urea breath test
Contralateral tremor, 3rd nerve palsy and
C A patient presents with chronic small bowel
involvement of penetrating branch of basilar
diarrhea, duodenal biopsy showing villous
artery.
atrophy. Antiendomysial ab and IgA TTG ab are
Components of Brown-Sequards syndrome
also positive. Treatment of choice
C
Ipsilateral extensor plantar response, ipsilateral
Gluten free diet
pyramidal tract involvement and contralateral
C The test used to diagnose Dubin Johnson syndrome
spinothalamic tract involvement.
is
(Note: Spinothalamic tract crosses more or less at
Bromosulphalein test (BSP) (Mn: DJ in BSP)
the point of entry it self. Dorsal column is also
C Most common cause of nephrotic range proteinuria ascending tract and cross over in medulla.
in an adult is Corticospinal tract/pyramidal tract is descending
DM tract and crosses at medulla and its injury at spinal
C A 30 years male patient presents with generalized cord is ipsilateral)
edema and hypertension. Urine examination shows C Classical CSF finding seen in TBM
subnephrotic proteinuria (<2gm) and microscopic Increased protein, decreased sugar and increased
hematuria. Serum complement levels are decreased lymphocytes.
and he is positive for ANCA. Diagnosis
C Organs involved in leprosy
MPGN > mixed cryoglobulinemia (elderly female Eye, testes and ovary
with skin ulceration favors essential mixed
C Treatment of severe ulnar neuritis in borderline
cryoglobulinemia)
tuberculoid leprosy
Endocrine tumor most commonly seen in MEN I
MDT + steroid
C
Pancreatic polypeptide noma > Gastrinoma
C Treatment of choice for Zollinger-Ellison syndrome
is ALL INDIA 2006
PPI C Megaloblastic anemia due to folic acid deficiency is
C SIADH is associated with commonly due to
Vincristine Inadequate dietary intake
C Hypophosphatemia is seen in C Myelodysplastic syndromes are most common in
Resolving phase of DKA, chronic alcoholism and age group
COPD/respiratory alkalosis. > 50 years of age.
Medicine 159

C A 60 years old lady presenting with backache and C Mycotic abscesses occur due to
recurrent chest infections for last 6 months, Fungal infection.
developing urinary retention and sudden weakness C Smoking is a risk factor for
of legs and Hb–7.3 gm/dl, serum Ca–12.6 mg/dl, Small cell carcinoma, emphysema and respiratory
phosphate–2.5 mg/dl, alkaline phosphatase–95 bronchiolitis.
mcg/L, serum albumin–3.2 mg/dl, serum globulin– C Common features of anorexia nervosa
7.0 gm/dl and urea–180 mg/dl. Diagnosis
Amenorrhoea, self perception of being fat and
Multiple myeloma underweight.
C Major criteria for multiple myeloma C Most common presenting symptom of non-
Plasmacytoma on tissue biopsy, bone marrow cirrhotic portal HTN
plasmacytosis > 30% and M spike > 3 g% for IgG, > Upper GI bleeding
2 g% for IgA.
C Pre-malignant colonic polyps are
B-cell prolymphocytic leukemia patient have a
Villous adenoma, tubular adenoma and hamar-
C
difference from those with B-cell CLL in
tomatous polyps associated with Peutz-Jeghers
Have a shorter survival syndrome.
C True about primary effusion lymphoma C Drug induced lupus
It generally presents in elderly patients, patients are Anti-histone antibodies
commonly HIV positive and there is often an
C Anticentromere antibodies are most commonly
association with HHV-8.
associated with
C A patient with leukemia on chemotherapy develops
CREST syndrome
acute right lower abdominal pain associated with
ASCA is a surrogate marker of
anemia, thrombocytopenia and leukopenia.
C

Diagnosis Crohn’s disease


Neutropenic colitis. C Renal artery stenosis occurs in
C Major complications of massive transfusion Atherosclerosis, fibromuscular dysplasia and
Hypothermia, hypomagnesemia and hypocalce- Takayasu’s arteritis.
mia. C Joint erosions are feature of
C Risk factors for atherosclerosis RA, psoriasis and multicentric reticulohistiocytosis.
Increased waist hip ratio, hyperhomocysteinemia (Note: Non-erosive arthritis in SLE)
and decreased HDL levels. C Known causes of osteoporosis
C ECG changes in hyperkalemia Hypogonadism, hyperthyroidism and hyperpara-
Prolonged PR interval, prolonged QRS interval and thyroidism.
ventricular asystole. C Feature of hypercalcemia
C Seen in cardiac tamponade Polyuria, depression and vomiting.
Pulsus paradoxus, electrical alternans and RVDC C Most common location of hypertensive intracranial
on echocardiography. hemorrhage
C True for mitral valve prolapse Basal ganglia
AD, valve leaflets show myxomatous degeneration C Common cause of subarachnoid hemorrhage
and is common cardiovascular complication of Aneurysm
Marfan’s syndrome (Mn: M for M for M). C Drug associated with hemorrhagic stroke
C Sudden cardiac death may occur in Phenylpropanolamine
Dilated cardiomyopathy, HOCM, and Eisenmenger’s C Most common type of glial tumor
syndrome. Astrocytoma
C Agents causing infective endocarditis C Brain abscess in cyanotic heart disease is commonly
Staphylococcus albus, streptococcal faecalis and located in
Pseudomonas aeruginosa. Parietal lobe.
C Mycotic aneurysm is an aneurysm infected because of C Clinical feature that excludes polymyositis
Both intravascular and extravascular infections. Extraocular muscle involvement.
160 AIIMS and All India PGMEE—Review Questions

C Young girl having consumed barium carbonate C A 32 years male patient presenting with complaints
with suicidal intent complains of generalized of weakness in right upper and both lower limbs of
muscle weakness. Electrolyte disturbance that may last 4 months, developing digital infracts involving
occur 2nd and 3rd fingers on right side and 5th finger on
Hypokalemia left side, on examination—BP—160/140 mm Hg,
peripheral pulses palbable, asymmetrical neuro-
pathy, Hb—12 gm/dl, TLC—11500/cmm, platelet—
ALL INDIA 2005
9,30,000, ESR—49 mm, urine- proteinuria, RBC—
C Splenectomy is indicated in 10–15/hpf, no casts most likely diagnosis is
Hereditary spherocytosis, thalassemia and sickle Polyarteritis nodosa
cell disease with large spleen. C Conditions causing osteoporosis
C Causes of relative polycythemia are Hyperparathyroidism, steroid use and thyrotoxicosis.
Dehydration, dengue hemorrhagic fever and C Pancreatitis, pituitary tumor and phacochromo-
Gaisböck’s syndrome. cytoma may be associated with
C Palpable purpura may occur in Medullary carcinoma of thyroid.
Small vessel vasculitis, disseminated gonococcal C Conditions known to cause DI
infection and acute meningococcemia. Head injury, histiocytosis and viral encephalitis.
C Conditions causing ST segment elevation on ECG C Cluster headache is characterized by
Early repolarization variant, ventricular aneurysm Unilateral headache, onset typically in 20–25 years
and prinzmetal angina. of life and associated conjunctival congestion.
C A 60 years male patient with severe myxomatous C Drug useful in prophylaxis of migraine
MR, asymptomatic, LVEF 45% and end systolic Propranolol
diameter index of 2.9 cm/ml. Treatment appropriate is C LMN lesion is associated with
Mitral valve replacement. Flaccid paralysis
C A 55 years male alcoholic patient (with smoking C Most common site of hypertensive hemorrhage in
history to) presents with 3 hour history of the brain
increasing shortness of breath, pain started while Putamen/external capsule.
eating that is constant and radiates to the back and C Normal CSF glucose level in normol gycemic adult
interscapular region, known hypertensive, on 40–70 mg/dl
examination-cold and clammy skin with heart rate C CSF findings present in tubercular meningitis
130/min, BP 80/40 mm Hg, JVP normal, peripheral
Raised protein levels, low chloride levels and cob
pulses normal and equal, breath sounds decreased
web formation.
at left lung base and chest-X-ray-left pleural
Cause of reversible dementia
effusion. Diagnosis
C
Subacute combined degeneration.
Acute aortic dissection
A 40 years male patient heavy alcohol drinker for
Abnormal preoperative PFT in a patient with severe
C
C
last many years presents with visual hallucination,
kyphoscoliosis includes
not recognizing family members, violent behavior
Increased RV/TLC
and tremulousness for few hours, after missing
C Serum ACE may be raised in alcohol for last 2 days, O/E increased BP, tremor,
Sarcoidosis, silicosis and berylliosis. increased psychomotor activity fearful effect,
C Disease commonly predisposing to colonic hallucinatory behavior, disorientation, impaired
carcinoma judgement and insight
Ulcerative colitis Delirium tremens
C 5’-nucleotidase enzyme activity increased in C A 40 years male patient with history of alcohol
Cholestatic jaundice. dependence presents with confusion, nystagmus,
C Serum levels of that help distinguishing acute liver ataxia, on examination—6th cranial nerve palsy.
disease from chronic liver disease Diagnosis is
Albumin Wernick’s encephalopathy.
Medicine 161

C EEG is usually abnormal in of cells with clear cytoplasm and frequent areas of
SSPE, Creutzfeldt Jacob disease and hepatic hemorrhage and necrosis. Cytogenetic abnormality
encephalopathy. may reveal
C Neurologic channel opathies are Chromosome 3
Hypokalemic periodic paralysis, episodic ataxia C First drug of choice for non-gonococcal urethritis
type 1 and familial hemiplegic migraine. Doxycycline
C Vitamin B12 deficiency can give rise to C An HIV patient complains of visual disturbances,
Myelopathy, optic atrophy and peripheral neuro- fundal examination showing bilateral retinal
pathy. exudates and perivascular hemorrhages. Virus
C True about vitamin E deficiency manifestations most likely responsible for retinitis
Hemolytic anemia, posterior column abnormalities Cytomegalovirus
and cerebellar ataxia. C True about HIV infection
C Most sensitive test for diagnosis of myasthenia gravis Following needle stick injury infectivity is reduced
Single fibre EMG > edrophonium test. by administration of nucleoside analogues.
C Nucleoside reverse transcriptase inhibitors are
Zalcitabine, lamivudine and didanosine.
ALL INDIA 2004
C Antiretroviral drug that is avoided in a known
C DIC differs from TTP in the aspect sputum positive pulmonary TB patient undergoing
Decreased coagulation factor levels. treatment with isoniazide, rifampicin, pyrazinamide
C A 5 years old girl presents with history of progressively and ethambutol is HIV positive, CD 4 count 100 and
increasing pallor since birth and hepatospleno- viral load more than 50,000 copies/ml
megaly. Most appropriate test Ritonavir
Hb electrophoresis C Serum HbA1c level explains
C Test picture of elevated serum ferritin, serum iron Long term status of blood sugar
and percent transferrin saturation is consistent with (Note: last 8 week).
the diagnosis of C A 40 years female patient presents with complains
Hemochromatosis of increased appetite and thirst and increased
C Bone marrow transplant is used in treatment of frequency of micturition, impalpable pulses in the
Osteopetrosis, adrenoleukodystrophy and Hurler’s feet and gangrene. OGTT finding showing ketone
syndrome. bodies negative, urinary glucose negative in fasting
C True about sickle cell disease but positive after 1 hour (+++) and 2 hours (++) and
serum glucose respectively 155, 270 and 205 mg/dl
Patient may require frequent blood transfusion,
(fasting, 1 hr and 2 hrs). True statements are
acute infection is the most common cause of
She suffers from NIDDM, treated with oral
mortality before 3 years of age and positive
hypoglycemic drugs only when diet and exercise
correlation between conc. HBS and polymerization
could not control it and family history of DM
of HBS.
predicts the nature of diabetes.
C Features seen in cardiac tamponade
C The best marker to diagnose thyroid related
Pulsus paradoxus, RVDC and electrical alternans. disorders is
C In hematuria of glomerular origin, urine is TSH
characterized by the presence of C The occurrence of hyperthyroidism after supple-
Red cell casts, crenated red cells and dysmorphic mental iodine to patients with endemic thyroid
red cells. deficiency goiter is called as
C Polycystic kidney disease may have cysts in the Jod Basedow effect
Liver, pancreas and spleen. C True about amiodarone induced thyroid dysfunction
C A 40 years male patient presenting with painless Hyperthyroidism is common in iodine deficient
hematuria, bimanual examination revealing areas, amiodarone inhibits deiodinase activity and
ballotable mass over right flank, subsequently amiodarone therapy is associated with initial
undergoing right nephrectomy and mass composed reduction of serum T4 levels.
162 AIIMS and All India PGMEE—Review Questions

C A 28 years female patient has put on 10 kg weight C Features of Alzheimer’s disease


over 3 years and has oligomenorrhea followed by Amyloid angiopathy, senile neuritic plaques and
amenorrhea for last 8 months. BP–160/100 mm Hg. neurofibrillary tangles.
Investigation most appropriate C Conditions causing neuropathy with predominant
Plasma cortisol motor involvement
C Earliest manifestation of Cushing’s syndrome Acute inflammatory demyelinating polyneuropathy,
Loss of diurnal variation. acute intermittent porphyria and lead intoxication.
C Retardation of skeletal maturity can be caused by C Common causes of viral encephalitis
CRF, PEM and hypothyroidism CMV, nipah virus and Japanese encephalitis virus.
C A 20 years female patient presents with primary C A 20 years old boy presents with fever, headache,
amenorrhea, height 153 cm, weight 51 kg, well confusional state, focal seizures and right
developed breasts, no pubic or axillary hair and no hemiparesis and MRI-bilateral frontotemporal
hirsutism. Most probable diagnosis hyperintense lesion. Diagnosis
Complete androgen insensitivity syndrome. Herpes simplex encephalitis.
C A 30 years female patient presents with secondary C Brain tumors that spread via CSF
amenorrhea for 3 years along with galactorrhea. Germ cell tumors, CNS lymphoma and medullo-
Most likely diagnosis blastoma.
Prolactinoma C A 15 years old boy with epilepsy on treatment with
C A 55 years male patient on bed rest for last 10 days, valproate and phenyltoin have good control of
complains of chest pain and breathlessness, X-ray seizures-levels of both drugs in therapeutic range.
chest normal. Next step Side effects attributed to valprote
Pulmonary ventilation perfusion scan. Weight gain of 5 kg, serum alanine aminotrans-
C A 18 years female patient presents with bilateral aminase 150 IU/L and rise in serum ammonia level
conductive deafness, palpable purpura on legs, by 20 mg/dl.
hemoptysis, chest X-ray-thin walled cavity in left C Persistent vomiting most likely causes
lower zone, TLC-12000/cmm, red cell casts in urine Acidic urine excretion and hypochloremia.
and serum creatinine 3 mg/dl. Most likely diagnosis C True about idiopathic edema of women
Wegener’s granulomatosis. Not related to menstrual cycle, increased water
C A 20 years male patient presenting with digital retention in upright position and ACE inhibitors
gangrene in 3rd and 4th fingers, on examination-BP can be useful in some cases.
170/100 mm Hg, peripheral pulses palpable, blood C Site of obstruction in portal system in patient with
and urine normal and ANA, anti-dsDNA and cirrhosis
ANCA are negative. Diagnosis Sinusoids
PAN C Site of venous thrombosis in Budd-Chiari syndrome
C A lady presents with complains of low grade fever Hepatic veins
of 6 weeks duration, chest X-ray-bihilar adenopathy C The short bowel syndrome is characterized by
with clear lung fields. Investigations needed for Diarrhea, weight loss and steatorrhea.
differential diagnosis C Hormone estimation useful in investigating
Serum ACE levels, CECT of chest and gallium scan. gynecomastia
C A 80 years old lady, asymptomatic patient, presented Testosterone, estradiol and LH
with lab findings of monoclonal spike on serum
electrophoresis (IgG level 1.5 gm/dl) and bone
ALL INDIA 2003
marrow showing 8% plasma cells. Diagnosis
Monocolonal gammapathy of unknown significance. C Normal anion gap metabolic acidosis is caused by
C A 12 years old girl with tremors and emotional Cholera
lability has a golden brown discoloration in C Causes of metabolic alkalosis include
Descemet’s membrane. Diagnosis Bartterr’s syndrome, thiazide diuretic therapy and
Wilson’s disease recurrent therapy.
Medicine 163

C A 50 kg man presents with severe metabolic acidosis, C During CPR, IV calcium gluconate is indicated in
pH-7.05, pCO 2 12 mm Hg, pO 2-108 mEq/L, base Hypocalcemia, calcium channel blocker toxicity
excess-30 mEq/L. Approximate quantity of sodium and electromechanical dissociation.
bicarbonate that he should receive in half hour-½ × C A postoperative cardiac surgery patient developed
body wt (kg) × [Desired HCO2- measured HCO2] sudden hypotension, raised CVP, pulsus
Half of the quantity given within first half hour. paradoxus at 4th postoperative hour. Diagnosis
C Most sensitive and specific test for diagnosis of iron Cardiac tamponade.
deficiency anemia is C The blood gas parameters-pH—7.58, pCO 2—23
Serum ferritin mm Hg, pO2—300? mm Hg, and oxygen saturation
C Leukoerythroblastic picture may be seen in 60% is consistent with
Myelofibrosis, metastatic carcinoma and Gaucher’s Ventilatory malfunction
disease. C DLCO is decreased in-ILD, emphysema and
C Poor prognostic factors for acute myeloid leukemia primary pulmonary HTN.
Age more than 60 years, leukocyte count more than C Diagnostic criteria of ABPA includes
1,00,000/μl and secondary leukemias. Changing pulmonary infiltrates, peripheral eosino-
C A 60 years old gentleman presents with massive philia and serum precipitins against Aspergillosis
splenomegaly, lymphadenopathy, total leukocyte fumigants.
count of 17,000/cmm, flowcytometry-CD19 and C Conditions leading to exudative pleural effusion
CD5 positive, CD 23 negative, monoclonal B-cells Bronchogenic carcinoma
with bright Kappa positivity comprising 80% of C Pulmonary HTN may occur in
peripheral blood lymphoid cells. Diagnosis Toxic oil syndrome, progressive systemic sclerosis
Mantle cell lymphoma. and sickle cell anemia.
C True about ECG in acute pericarditis C A 60 years old gentleman presented with breath-
Global ST elevation in early pericarditis, sinus lessness, facial swelling and dilated veins on the
tachycardia is common and PR segment depression chest wall. Most common cause is
is present in majority. SVC obstruction
C ECG features of severe hyperkalemia C Virus having significant perinatal transmission
Peaked T waves, sine wave pattern and loss of P Hepatitis B
waves. C SIADH is characterized by
C Hypocalcemia is characterized by Hyponatremia and urinary sodium excretion > 20
Numbness and tingling of circumoral region mEq/L.
hyperactive tendon reflexes and carpopedal spasm. C A 23 years old lady presented with episodes of
C Heart sounds occurring shortly after S2 myalgias, pleural effusion, pericarditis, arthralgia
Tumor polyp, opening snap and pericardial knock. without joint deformity over course of several
C An early systolic murmur may be caused by years. Best laboratory test to diagnose
Small VSD, papillary muscle dysfunction and TR. ANA
C Exercise testing is absolutely contraindicated in C Characteristic EEG feature of absence seizure
Aortic stenosis 3 Hz spike and wave.
C Most common cause of TR is secondary to C Most common presentation of neurocysticercosis
Dilatation of right ventricle. Seizures
C Heart valve most likely to be involved by infective C Most common cause of sporadic encephalitis
endocarditis following septic abortion HSV
Tricuspid valve. C Serum total LDH is raised in
C Osler’s node found in Muscle crush injury, MI and hemolysis.
Acute staphylococcal endocarditis. C Thiamine deficiency is known to occur in
C Troponin-T is preferable to CPK-MB in diagnosis of Food faddist, chronic alcoholic and CHF patient on
acute MI in conditions diuretics.
Bedside diagnosis of MI, postoperatively (after C A 30 years old male patient, HIV positive, presents
CABG) and small infarct. with fever, dyspnea, non-productive cough,
164 AIIMS and All India PGMEE—Review Questions

cyanosed patient, chest X-ray revealing bilateral C PNH is associated with


symmetrical interstitial infiltrates. Diagnosis Aplastic anemia, venous thrombosis and iron
Pneumocystis carini pneumonia. deficiency anemia.
C A 60 years gentleman with DM presents with C A 20 years old boy presents with severe hypoplastic
painless swollen right ankle joint X-ray showing anemia. Most effective treatment
destroyed ankle joint with large number of loose Bone marrow transplant.
bodies. Diagnosis C Commonly seen in polycythemia vera
Charcot’s joint Thrombosis, hyperuricemia and prone for acute
C CNS manifestation in CRF as a result of leukemia.
Acidosis, hyponatremia and hyperosmolarity. C Conditions associated with APLA syndrome
C A 25 years old male patient presents with fever, Venous thrombosis, recurrent fetal loss and
cough, expectoration and breathlessness of 2 neurological manifestations.
months duration, CECT chest-bilateral upper lobe
C Hypergastrinemia with hypochlorhydria is seen in
fibrotic lesions and mediastinum had enlarged
necrotic nodes with peripheral rim enhancement. Pernicious anemia
Most probable diagnosis C Correctly matched phases of jugular venous pulse.
Tuberculosis a-x descent-atrial relaxation (Mn: X for relaxation)
C Cardiac output measured by thermodilution v-y emptying- emptying of blood from right
technique is unreliable in atrium into right ventricle.
TR, low cardiac output and PR. y-a ascent-filling of right atrium from vena cava
(Mn: Y for empty).
C True about JVP finding-cannon wave
ALL INDIA 2002
Complete heart block
C Conditions associated with increased anion gap C Cause of respiratory distress in a patient while
type of metabolic acidosis inserting central venous catheters
COPD Pneumothorax
C Acute metabolic acidosis C Clinical features of myxoma
Decreased urinary K+ excretion Fever, clubbing and embolic phenomenon.
C Urinary anion gap is an indication of excretion of C Renal vein thrombosis is most commonly associated
NH4+ with
C Most common mode of inheritance of congenital Membranous glomerulopathy
heart disease C Characteristic of HSP
Multifactorial
Blood in stool
Autosomal dominant disorder
Renal dystrophy differs from nutritional and
C
C
Hereditary spherocytosis. genetic forms of osteomalacia in having
C HLA associated diabetes Hyperphosphatemia
Type 1 diabetes C Medullary cystic disease of kidney is best diagnosed
C Sexually transmitted diseases are by
Candida albicans, Molluscum contagiosum, and Group Biopsy
B Streptococcus. C A 15 years old boy with nephrotic syndrome on
C Infections transmitted by blood transfusion long-standing corticosteroid therapy may develop
Parvovirus B19, CMV and hepatitis G virus. Hyperglycemia, neuropsychiatric symptoms and
C Hypoglycemia is a recognized factor of suppression of piturtary adrenal axis.
Uremia, Addision’s disease and hepatocellular C A 40 years old gentleman presents with repeated
failure. episodes of bronchospasm and hemoptysis chest-
C Features seen in TTP X-ray-perihilar bronchiectasis. Diagnosis
Fever, hemolysis, low platelet count. ABPA
Medicine 165

C Characteristically associated with development of glucose decreasing to 100 mg/dl after giving
ILD insulin. Changes in blood level expected
Coal dust, sulphur dioxide and thermophilic Increase in Na+ level
actinomycetes. C A 20 years old boy presents with exertional
C A 38 years male patient presents with HBsAg and dyspnea, headache and giddiness, on examination-
HBeAg positive during screening of blood hypertension and LVH, X-ray-showing notching of
donation and SGPT and SGOT normal. the anterior ends of the ribs. Diagnosis
Next done HBV DNA estimation. Coarctation of aorta
C Rheumatoid factor in RA is important because of
C A 25 years old lady presents with bloody diarrhea
RA factor is associated with bad prognosis.
diagnosed as a case of ulcerative colitis. Conditions
Conn’s syndrome is associated with
associated are
C
Hypertension, muscle weakness and hypokalemia.
Sclerosing cholangitis, iritis and ankylosing
Characteristic triad of Zollinger-Ellison syndrome
spondylitis.
C
Peptic ulceration, gastric hypersecretion and non-β
C Investigation of choice for invasive amoebiasis cell tumor.
ELISA C Feature of phacochromocytoma
C A diabetic patient with blood glucose of 600 mg/dl Hypertensive paroxysm, headache and orthostatic
and Na+ 122 mEq/L treated with insulin with blood hypertension.
166 AIIMS and All India PGMEE—Review Questions

12

Surgery

AIIMS NOVEMBER 2015 C Minimum and maximum score of Glasgow coma


scale
C Best guide for fluid resuscitation in shock 3 and 15
Central venous pressure (Note: GCS- E4V5M6- Eye opening-4
C In massive bleeding, intravenous catheter used Verbal response-5
16G size > 18G size Motor response-6
C A 60 years old chronic smoker presents with Eye opening- Mn: SVP
progressive jaundice, pruritus, clay coloured stool Spontaneous-4
fot 1 month. History of waxing and waning present. To verbal response-3
on CT scan, dilated main pancreatic duct and CBD To painful stimuli-2
is seen. Diagnosis No response-1
Verbal response- Mn: OC InAC
Ampullary carcinoma
Oriented-5
(Note: Double duct sign) Confused-4
C A child presents with a swelling in the right groin Inappropriate words-3
region. While reducing the swelling, gurgling Incomprehensible sound-2
sound is heard. True statements are No response-1
The hernia lies above and medial to pubic tubercle Motor response- Mn: OLWFEN
Patent processus vaginalis Obeys command-6
This type of hernia is most common in children. Localises pain-5
Withdraws to pain-4
(Note: Content can be omentum or intestine.)
Flexion (abnormal)-3
C Cardiac tamponade and tension pneumothorax can Extension (abnormal)-2
be best differentiated in a patient presenting with No response-1)
breathlessness after RTA by
Breath sound AIIMS MAY 2015
C Peau d’ orange appearance of breast is due to
C Screening is helpful in
Obstruction of subdermal lymphatics
Breast carcinoma, colon carcinoma and prostate
C A tack was accidentally placed during laparoscopic carcinoma.
inguinal hernia repair below and lateral to iliopubic C Most commonly performed and effective method of
tract. Patient complains of pain in the thigh post bariatric surgery is
operatively. This occurred due to injury to
Roux-en-Y procedure.
Lateral cutaneous nerve of thigh C Lymphatic spread seen most commonly in type of
C Thyroid storm in thyroid surgery occurs due to thyroid carcinoma
Inadequate preoperative preparation Papillary carcinoma.

166
Surgery 167

C Mallory Weiss tear mainly affects C A head injury patient is brought to the emergency
Left gastric artery. in unconscious state. In his pocket antidiabetic drug
C Physiological changes in laparoscopy includes all is found. What should be done
except Take the blood sample, urgent CT scan, measure the
Increased pH. blood glucose, if less than 70, start dextrose
C Most commonly performed shunt in hydrocephalus infusion.
Ventriculo-peritoneal shunt. C True about functional division of liver
C Best incision preferred for surgery in diaphragm- Divided into 8 segments, 4 sectors and there are
Circumferential. three major and minor fissures.
(Note: Most commonly used incision is radial).
C Prognosis of esophageal cancer depends on one of AIIMS NOVEMBER 2014
the following option
C You are a qualified general surgeon in a community
T stage. health centre and receive a patient with severe head
C Treatment of choice for hydrocele of hernia sac in 3 injury, rapidly deteriorating GCS, hemiparesis, and
years old children pupils becoming dilated and fixed. CT scan and a
Herniotomy. neurosurgeon are unavailable. You decide to go for
C A patient undergoes right sided laparoscopic exploratory burr hole. Most appropriate statement
hernia surgery and a tack is accidentally placed if no localizing signs are present, place the burr hole
below and lateral to the iliopubic tract and develops on the left temporal side.
pain in the right thigh. Most like nerve injured is C False about traumatic brain injury
Lateral cutaneous nerve of thigh. Lactate increase is associated with good prognosis.
Oncotype Dx test is for
Cushing’s triad include
C
C
Chemotherapy for hormone receptor positive Rise in BP, bradycardia and irregular respiration.
breast cancer.
C On examination of a patient with RTA respiratory
(Note: Other similar genomic tests are Mamma Print, distress and reduced SpO2 was found. Stratosphere
Mammostrat and Prosigna. These tests help determine sign was seen on M mode USG of right anterior
the risk benefit ratio of chemotherapy in early stage chest. Most likely diagnosis
hormone receptor positive breast cancer patients. It
Pneumothorax
helps determine whether patient also needs chemo-
The immediate management for tension pneumo-
therapy along with surgery and hormone therapy.)
C
thorax is
C A female patient undergoes splenectomy for
resistant ITP. He presents with fever and chills on Needle insertion in the second intercostals space in
3rd postoperative day. Most likely cause the midclavicular line.
Left lower zone consolidation. C Panniculus adiposus is found at
C A pregnant woman presents with stab injury in the Orbit
right side of chest. She is shouting for the help. On C The best time for repair of cleft palate is
examination she has pulse 110/minutes and BP 90/ 9 to 12 months
60 mm Hg. Her breath sound is decreased on the C A 15 years old girl presents with pain and swelling
right side. First done over posterior aspect of left thigh that is soft on
Needle decompression on the right side. examination. Color Doppler ultrasonography
C A 2 months pregnant woman presents with a breast showed multiple dilated veins without any major
lump in the upper outer quadrant. Lump is not arterial feeder, indicating a venous malformation.
visible in USG. Next to be done Sclerosing agent that can be used in this patient
Palpation guided core biopsy. Sodium tetradecyl sulphate.
C A woman presents with bloody nipple discharge C A 12 years old girl presents with a nodule in the
from a single duct with family history of breast right lobe of thyroid. Resection revealed a small
cancer. Initial investigation of choice nodule of 2 × 2 cm. Histopathology showed Orphan
MRI Annie eye nuclei. Most likely diagnosis
(Note: Mammogram was not in the choice.) Papillary carcinoma
168 AIIMS and All India PGMEE—Review Questions

C FNAC cannot diagnose C During resuscitation, artefacts of fractured ribs


Folliclar carcinoma thyroid most commonly involve
C A 15 years old psychiatric girl was brought with the 5th–7th ribs
complaints of pain in the epigastrium and C A 12 years old girl presents with nodular goitre.
vomiting. Clinical examination revealed a palpable True about evaluation and management
lump in the epigastrium. On further examination, 99 m Tc scan should be performed to determine
there were crepitus in epigastrium and focal whether the nodules are hypofunctioning or
alopecia. Most likely diagnosis hyperfunctioning, functional thyroid nodules are
Trichobezoar usually benign and FNAC should be performed for
C A 2 months old child presents with white colored all nodules > 1 cm in diameter.
stool for 1 month and yellowish discoloration of C Best incision taken for diaphragmatic surgery
eyes and urine. She was started on steroids and Circumferential
ursodeoxycholic acid for 5 days. Best investigation C Etiological factor for pancreatitis
for the diagnosis Abdominal trauma, hyperlipidemia and germ line
HIDA scan mutations in the cationic trypsinogen gene.
(Note: Biliary atresia)
C A 6 months old child got up crying with abdominal AIIMS NOVEMBER 2013
pain in the middle of the night. He then passed red
colored stools. After sometime, the pain subsided and C Mortality associated with emergency abdominal
the child became comfortable. Most likely diagnosis aortic surgery
Intussusception > 50%
C A 55 years old man presents with a lump in the right C A 35 years old lactating mother presents with a
iliac fossa, with gross pallor and chronic dull aching painful breast lump. Initial investigation done
pain. Most likely diagnosis USG
Carcinoma ascending colon. C Part of duodenum examined in a patient of burn for
C A 65 years old man became dizzy while passing curling ulcer
stools and noticed fresh blood filling the entire pan. 1st part
His previous stool Guiaic test was negative and C Investigation to confirm the anorchia
screening tests for colon cancer were also negative. Laparoscopy
Most likely cause of the bleed C In a couple with normal female partner, male
Dilated submucosal veins and venules in the colon. partner has 0.8 ml semen volume per ejaculate on
C True about GIST repeated samples and absent fructose and no sperm
It is the most common mesenchymal tumor of the noted on microscopy. Next done
GIT, they arise from the interstitial cells of Cajal and Transrectal ultrasound to detect duct obstruction.
tumor size is an important prognostic factor. C Most likely reason of hypotension in a patient of
C A 50 years old man with history of right ureteric road traffic accident with spine, chest and
calculi presented with pain in the abdomen. abdominal injury
Resection of right kidney was done. The macro- Neurogenic shock
scopic structure is shown. Most likely diagnosis C FAST stands for
Hydronephrosis with chronic pyelonephritis. Focussed assessment with sonography for trauma.
C First line of management for a patient of road traffic
AIIMS MAY 2014 accident with tension pneumothorax
C The first priority in management of a case of head Insert wide bore needle in 2nd intercostals space.
injury with open fracture of shaft of femur is C A 60 years old man presents with fever, chills and
Intubation dysuria and was hospitalised in emergency for 5
C Neurogenic shock is characterized by days. PSA level was 7.4. Next done
Hypotension and bradycardia. Antibiotics and admit.
Surgery 169

C Contaminated wound with necrotic material is best C The mortality rate of emergency operation for
managed with abdominal aortic aneurysm is
Debridement > 50%
C Color of triage given highest priority C True about celiac plexus block is
Red Diarrhea and hypotension are the common side
C False about endemic bladder stones effects.
Always associated with recurrence. C True about damage control surgery
C In GCS, withdrawal to pain stimulus comes under Minimal intervention done to stabilize the patient
M4 and do the definitive surgery later.
C Tumor marker for seminoma C In acute pyelonephritis, USG findings are
PLAP Renal enlargement, compression of the renal
C Time interval in a 60 years old patient of RTA sinuses and increased echogenicity.
presenting in unconscious state but becoming C Most common cause of fresh bleeding per rectum in
conscious and then again unconscious is called a 5 years old child is
Lucid interval Juvenile rectal polyp
C A young male patient previously healthy presented C Osteoblastic secondaries are most commonly seen
with abdominal pain and history of altered bowel from which primary malignancy
habits for last 6 months. On CT scan, there was Prostate
distal dilated ileum, thickened ileocecal junction C The most serious complication of a pelvic fracture is
with thickened cecum with presence of sacculations Hypovolemic shock
on the antimesenteric border. The vascularity of C Structures pierced during pleural tapping are
adjoining mesentery is also increased and there is Endothoracic fascia, skin and intercostal muscle.
surrounding mesentery fat. Differential diagnosis
C A 49 years old male patient with a 35 pack year
includes
history of smoking presents with a painless left
Ulcerative colitis, Crohn’s disease and tuberculosis. scrotal mass. Examination revealed microscopic
C Neo adjuvant chemotherapy is used in hematuria, absence of AFP and LDH. The most
Osteosarcoma, chest wall PNET, ovarian cancer probable cause
stage 3. Renal cell carcinoma.
C A patient underwent sentinel node biopsy for
AIIMS MAY 2013 treatment of breast carcinoma. Nerves likely to be
C A 45 years old patient presents with a 4 × 5 cm mass injured during this procedure
in the neck. Histopathology showed metastasis of Intercostobranchial nerve.
squamous cell origin. Diagnosis of carcinoma of C Innovation of refrigeration leading to better food
unknown primary (CUP) was made after no preservation has lead to decreased prevalence of
primary was found despite thoroughly examining Stomach cancer
laryngoscope, whole body CT and PET CT. TNM C Done in investigation for malabsorption
staging as per AJCC is 13-C triclosan assay
T0N2MO C Parotid gland tumor showing hot spot on Tc 99
C A 5 years old girl spills boiling water accidentally pertechnate scan
over her face and trunk. Methods most accurate to Adenolymphoma
estimate the body surface area involved in burns C An elderly male patient with a history 60 pack year
Lund and Browder chart of smoking is diagnosed with carcinoma lung.
C A 3 years old child suffers from burn injury with the Histology shows small highly mitotic cells, scant
following body parts involved: face including cytoplasm with hyperchromatic nuclei. Clinical
scalp, both buttocks and circumferentially around presentation that might occur in the patient during
both thighs. How much is TBSA involved the course of his illness
0.35 Thin extremities and central obesity.
170 AIIMS and All India PGMEE—Review Questions

AIIMS NOVEMBER 2012 incompetence and dilatation of GSV but deep veins
normal. Management includes
C A 70 years old man diagnosed with prostate cancer Endovascular stripping, stab avulsion and saphe-
was treated with radiotherapy. The recurrence of nofemoral flush ligation with stripping.
the cancer is monitored clinically by
C True about Buerger’s disease
Prostate specific antigen only
Neural involvement
C True regarding nerve injury
C True about peripheral vascular disease
In all cases of open wound with clinical signs of
Buerger’s disease also involves nerves.
nerve injury, nerve exploration should always be
done.
C BIRADS stands for AIIMS MAY 2012
Breast imaging reporting and data system. C Most important physiological cause of GERD is
C Organism associated with fish consumption and Transient LES relaxation.
also causing carcinoma gallbladder C After parotidectomy, patient sweats on cheeks
Clonorchis sinensis while eating, because auriculotemporal nerve
C A child was operated for intussusceptions by resec- (containing parasympathetic secretomotor fibres to
tion of the affected ileal segment revealing tumor parotid gland) is fused with
Villous adenoma Great auricular nerve.
C Causes of acute anal pain C True about epigastric hernia
Perianal abscess, thrombosed hemorrhoids and Located above the umbilicus and on either side.
acute anal fissure. C Pre-malignant jaw cyst
C A 30 years old lactating mother presented with a Odontogenic keratocyst
painful, palpable lump in her left breast. The most C Female patient of meningioma with inflammatory
appropriate investigation to diagnose her condition edematous lesion undergoing surgery, not done
would be Stop steroids
USG C Toddler presenting with few drops of blood coming
C True about inguinal hernia surgery out of rectum. Diagnosis
Hernia in children treated with herniotomy, Juvenile rectal polyp
absorbable mesh should not be used for surgery C Endoscopy of a female patient presenting with dys-
and surgery can be done using laparoscopy. phagia and intermittent epigastric pain shows
C True about radiological features of the intestinal dilated above and narrow at the bottom. Treatment is
obstruction Heller’s cardiomyotomy
The small intestines are said to be dilated if the (Note: Achalasia cardia–Bird’s beak appearance–AB)
loops of bowel have a diameter of > 3 cm, the large C Prognosis of which cancer has become better due to
bowels are said to be dilated if they are > 5 cm for advances in cancer treatment
distal bowel and > 9 cm for proximal bowel and the
ALL in children
absence of air-fluid level on plane supine X-ray does
not rule out obstruction. C Barrett’s esophagus is diagnosed by
C All of the following can be detected on USG in a Intestinal metaplasia
patient with surgical cause of jaundice (Note: Columner metaplasia)
Biliary tree obstruction, gallbladder stones and C Carcinoma breast stage T4b involves
ascites. Skin ulcer over the swelling, dermal edema and
C On the basis of anatomical knowledge of the pelvis, satellite nodule.
a rupture of the urethra above the deep perineal C Postoperative patient of Crohn’s disease with
pouch will lead to extravasation of urine in resection anastomosis presents on 7th post-
Deep pelvis operative day with anastomotic site leak from a
C A patient presents with varicose vein with color fistula with every day leakage adding up to 150–200 ml.
Doppler showing saphenofemoral junction There is no intra-abdominal collection and the
Surgery 171

patient is stable without any complaints. Next line C Complications of ileoanal patch anastomosis in an
of management ulcerative colitis patient most likey leads to
Do conservative treatment and leave him and wait Small bowel obstruction
for spontaneous resolution. C Following RTA, a patient with vague abdominal
C A 42 years old male patient presents with pain was undertaken for emergency laparotomy.
hematemesis. On examination—BP–90/60, HR– On examination- large, contained, stable, non-
120/min splenomegaly present. Most probable pulsatile retroperitoneal hematoma on right side.
cause of bleeding One shot IVU shows a barely discernable nephrogram
Portal hypertension on right side and prompt uptake and excretion on
C External hemorrhoids below the dentate line are left. Next step
Painful Isolate proximal renal vessels , open Gerota’s fascia
and explore kidney.
C Early complication of ileostomy in the postopera-
tive period C Part of capsular plate system of liver
Cystic plate, hilar plate and umbilical plate.
Necrosis
C Enzymes elevated in a child presenting with
C A patient presents with pain and tenderness in left
jaundice, icterus and clay colored stools
iliac fossa. USG shows a 3 cm stone in the renal
Gammaglutamyl transferase, alkaline phosphatase
pelvis without any hydronephrosis. Most appropriate
and 5’ nucleotidase.
treatment
C Correlating with severity of acute pancreatitis
PCNL
AST, serum calcium and serum glucose.
C Drug used in estrogen dependent breast cancer
C A 20 years old man presenting with proptosis and
Tamoxifen
pain in the right eye 2 days after trauma to the eye.
C After doing a graft repair of a thoracoabdominal On examination-bruise on the right eye and
aneurysm, the patient developed weaknesses in forehead. Diagnosis
both legs. Most probable cause Cortico cavernous fistula.
Discontinuation of arteria radicularis magna. C A surgeon while performing a laparoscopic chole-
C Due to decelerations, aorta can be ruptured at cystectomy finds a stone in CBD but is not experienced
places where it is fixed in laparoscopic CBD exploration. Next done
At ligamentum arteriosum, behind the crura of Convert to open cholecystectomy and CBD stone
diaphragm and aortic valve. extraction.
C A 40 years old male patient presented with mild C A patient presents to the emergency with history of
abdominal pain, mild constipation with a feeling of fall from a tree within 2 hours of injury. On
incomplete evacuation and mucus in the stools for examination-he is unable to move both his lower
the past 4 years on examination, tenderness is limbs, has absent sensations at umbilicus and bowel
present in left iliac fossa. Most likely diagnosis and bladder involvement. Spinal tenderness is
Irritable bowel syndrome elicited at the level of D10- L2. The loading dose of
C Characteristic of basal cell carcinoma is IV methylprednisolone given
Nuclear palisading 30 mg/kg within 3 hours

AIIMS MAY 2011


AIIMS NOVEMBER 2011
C True about hepatic duct
C Injury to which artery is least likely to result in
significant damage Left hepatic duct formed in umbilical fissure, right
hepatic duct formed by V to VIII and left hepatic
Inferior mesenteric artery.
duct crosses IV segment.
C Postoperative paralytic ileus most commonly seen at C True about femoral artery cannulation
Ileum Common femoral artery is cannulated, femoral
C Structure immediately lateral to sac of hernia in a artery is catheterized at medial third of femoral
patient of femoral hernia head and Seldinger technique is used both for
Femoral vein femoral artery and vein.
172 AIIMS and All India PGMEE—Review Questions

C True statement is C Risk factors for malignant change in an asymp-


Goitre more than 5 percent of population is termed tomatic patient with a gallbladder polyp on USG
endemic goitre. includes
C True about mesothelioma Age > 60 years, rapid increase in size of polyps and
Occurs in late middle age associated with asbestos associated gallstones.
exposure and biphasic pattern on histopathology. C True about pancreatic carcinoma
C True about Hashimoto’s thyroiditis Mutation in p53 gene is associated in 75% of cases,
Follicular destruction, increase in lymphocytes and hereditary pancreatitis significantly increases the
oncocytic metaplasia. risk and five years survival after pancreaticoduo-
C Elderly male patient presents with obstructive denectomy is 15–20%.
symptoms. Biopsy of stomach revealing GIST. Most C Treatment of medullary carcinoma thyroid
appropriate marker Surgery only
CD117 C Most common cause of acute mesenteric ischemia
C Genes involved in development of carcinoma colon Embolism
APC, K-ras and mismatch repair genes. C True about Gleason score
C Gold standard test for insulinoma Used for grading prostate cancer, higher the score,
72 hours fasting test poorer the prognosis and helps in planning
C True about FAP management.
Screening by sigmoidoscopy, polyps develop in C Most common site of chalangiocarcinoma
late adulthood and epidermal cysts and osteomas Hilum
may occur. C Best test to diagnose GERD and quantify acid output is
C BRCA 1 gene is located on 24 hours pH monitoring.
Chromosome 17 C In a male patient after laparoscopic cholecystec-
(Note: BRCA2-chromosome 13) tomy, specimen is sent for histopathology showing
C Most protective against carcinoma colon carcinoma gallbladder stage 1a. Management
High fibre diet Excision of all port sites.
C Terminal group of LN for colon (Note: In open cholecystectomy, simple cholecys-
Preaortic tectomy is done in stage 1a).
C Gold standard test for recurrent GIST C True about mucinous cystadenoma pancreas
PET CT Premalignant, lined by columnar epithelium and
C On abdominal USG, gallbladder shows diffuse wall focus of ovarian stoma in it.
thickening with hyperechoic nodules at neck and C The technique of laparoscopic cholecystectomy was
comet fail artifacts. Diagnosis first described by
Adenomyomatasis Erich Muhe
C A 3 weeks old infant presenting with vomiting, (Note: Kurt Semm-Laparoscopic appendicectomy).
failure to thrive is suffering from pyloric stenosis. C Mangled extremity severity score includes
Next step of management Shock, ischemia and energy of injury.
Correction of electrolyte disturbance. C Survival in unresectable GB carcinoma
C Most important prognostic factor for colorectal 4–6 months
carcinoma C Associated with renal cell carcinoma
Stage of lesion > lymph node status. Polycythemia, amyloidosis and HTN.
C Best treatment for a 2 cm thyroid nodule in a 50 years
old man with FNAC finding papillary carcinoma AIIMS NOVEMBER 2010
Total thyroidectomy with left sided modified neck C Stone resistant to lithotripsy
dissection > near total thyroidectomy with Cystine stone
modified neck dissection. C True about GIST
C Non-progressive contraction of esophagus are Necrosis and ulceration present, PET is used to
Tertiary assess response to therapy and well-circumscribed.
Surgery 173

C Charcot’s triad includes C A 24 years old man presented with retroperitoneal,


Fever, pain and jaundice. necrotic heterogenous enhancing mass on CT scan
C Cock’s peculiar tumor near the hilum of left kidney. Diagnosis
Ulcerated sebaceous cyst. Metastatic germ cell tumor.
C Fogarty’s catheter is used for
Uretheral catherterization. AIIMS MAY 2010
C Brilliantly transilluminant swellings are C Commonest organ injured in blast injury
Hydrocele, cystic hygroma and meningocele. Tympanic membrane > lung > bowel.
C Tumor with perineural invasion C Approach for laparotomy in blunt trauma
Adenoid cystic carcinoma. abdomen
C Organism associated with fish consumption and Always midline incision
causing carcinoma gallbladder C True about brachial anomaly
Clonorchis sinensis. Most commonly due to 2nd branchial remnant.
C Most important indication of surgery in follow-up C Best prognostic factor for head injury
of BPH is GCS
Bilateral hydronephrosis. C Most common site of Morgagni hernia
C A 60 years old man with poor stream of urine, post- Right anterior
void residual urine 400 ml, bilateral hydronephrosis (Note: RAM-Right anterior-Morgagni)
and prostate weighing 70 gm. His urea is 120 mg/
C True about congenital torticollis
dl and creatinine 3.5 mg/dl. Next done best
Spontaneous resolution in most cases, 2/3rd cases
Catheterize with Foley’s catheter.
have palpable neck mass at birth and uncorrected
C CT feature of adrenal adenoma cases develop plagiocephaly.
Low attenuation, rare calcification and homoge- C Psammoma bodies seen in
nous density with well-defined borders.
Papillary carcinoma of thyroid, meningioma and
C Done first during reconstruction of an amputated serous cyst adenocarcinoma of ovary.
limb
C Barrett’s esophagus shows
Bone fixation
Intestinal metaplasia
True about cranial trauma
A 50 years old male with positive family history of
C
C
Depressed fracture is associated with brain injury at prostate cancer has come for a screening test. The
the immediate area of impact, carotid cavernous most sensitive screening test is
fistula occurs in skull base fracture and post-
DRE + PSA.
traumatic epilepsy seen in 15%.
C Orchidopexy for cryptorchidism is done at the age of
C A child presents with recurrent abdominal pain and
1 to 2 years.
bilious vomiting. Barrium meal follow through
diagnosed the condition. Surgery was done- (Note: Idealy around 6 months)
Mesenteric widening appendicectomy and cutting C Colonic polyp with no risk for malignancy
the ladd’s band. Diagnosis Juvenile polyps.
Malrotation C Peutz-Jeghers polyps present most commonly in
C The cecum is found to be placed below the stomach Jejunum
and in the midline. Abnormality having taken place C Congenital hypertrophic pyloric stenosis is
while rotation of the gut associated with
Malrotation Hypokalemic hypochloremic alkalosis with
C A young officer, on tour, presents with bright red paradoxical aciduria.
painless bleed since 7 days and abdominal pain. C Sister Mary Joseph nodule seen most commonly in
External and PR examinations are normal. Next Stomach carcinoma (Mn: Stomach carcinoma,
step as attending general practitioner colon carcinoma, ovarian carcinoma and pancreatic
Proctoscopy carcinoma (SCOP))
174 AIIMS and All India PGMEE—Review Questions

C To expose the celiac axis, left renal artery, SMA and C True about flail chest
abdominal aorta in case of trauma abdomen, Fracture of at least 3 ribs. if overlapping fractured
procedure done is ribs with severe displacement is seen then patients
Left medial visceral rotation. are treated surgically with open reduction and
C Wilm’s tumor associated with fixation and paO2 < 40 and FiO2 > 60 treated with
Aniridia, HTN and hemihypertrophy. intubation and PEEP.
C Early postoperative complication of ileostomy in C Full thickness grafts can be obtained from
postoperative period Elbow, groin and supraclavicular area.
Necrosis C A patient, hemodynamically stable, presents with
blunt injury abdomen. Next investigation
C Smoking associated with
FAST
Carcinoma larynx, carcinoma bladder and carcinoma
esophagus. C AFP increased in
C A 60 years old chronic smoker presents with Hepatoblastoma
painless gross hematuria of 1 day duration. C Treatment of desmoid tumor
Investigation of choice Wide excision
Urine microscopy for malignant cytology cells. C A male patient with azoospermia have normal FSH
C A man with blunt abdominal trauma with history of and testosterone levels and normal sized testes
pelvic fracture has passed only few drops of blood cause
per meatus and no urine in the past 8 hours. His Vas obstruction
bladder is palpable per abdomen. True is C Presence of nephroblastomatosis in biopsy of
Urethral injury Wilm’s tumor of left kidney is indicative of
Increased risk of tumor in right kidney.
C Tumors associated with infective etiology
AIIMS NOVEMBER 2009 Gastric carcinoma, hepatocellular carcinoma, and
C Most important prognostic factor in congenital nasopharyngeal carcinoma.
diaphragmatic hernia C True about cryptorchidism
Pulmonary HTN Cryptorchidism is a risk factor for testicular tumor,
C Risk factor for cholangiocarcinoma seminoma is most common tumor, contralateral
testis is also at risk.
Chronic typhoid carrier, chronic ulcerative colitis
and parasitic infestation (Clonorchis sinensis). C Laparoscopic cholecystectomy was done in a
patient with cholelithiasis with biopsy report of
C Hirschsprung’s disease is due to
adenocarcinoma with invasion of muscular layer,
Failure of migration of neural crest cells from CT normal. Further treatment is
cranial to caudal direction.
Wedge hepatic resection with lymph node dissec-
C A robust male baby, with vigorous feeding and tion.
immediate vomiting at 2 months of age. Diagnosis
Congenital hypertrophic pyloric stenosis. AIIMS MAY 2009
A 65 years old patient of coronary artery disease on
A 45 years old male patient presents with abrupt
C
C
aspirin for 24 years complains of black stools,
onset pain, weakness, loss of contour of shoulder
abdominal examination normal. Diagnosis
and muscle wasting on 5th day of tetanus toxoid
Duodenal ulder immunization. Likely cause is
C Ileocaecal TB is associated with Brachial plexus neuritis.
Megaloblastic anemia. C Double bubble sign is seen with
(Note: Vitamin B12 is absorbed from ileum) Duodenal atresia
C Lynch syndrome is associated with C Indications for penile angiography
Colon carcinoma, endometrial carcinoma and Peyronie’s disease, erectile dysfunction and arterio
carcinoma ovary. (Mn: CEO) venous malformation.
Surgery 175

C Right hepatic duct drains C Treatment of medullary carcinoma thyroid


Segment I, V and VI. Surgery only
(Note: and VII and VIII) C The Hunterian ligature performed for
C Dye used in chromoendoscopy for detection of Aneurysm
cancer C Glomus tumor is seen in
Methylene blue> Toluidine blue. Finger
C Most common cause of esophagitis is C Bariatric surgical procedure includes
Gastric banding, gastric bypass and biliopancreatic
Esophageal reflux
diversion.
C Predisposing factor for carcinoma esophagus
C Complications of TPN
Caustic ingestion, diverticula and HPV (Mn: DCH). Essential fatty acid deficiency, hypophosphatemia
C Criteria used to define penetrating neck injury and metabolic bone disease.
Breach of platysma. C A women presents with mass and bloody discharge
C True about choledochal cyst from her left breast, histopathology revealing
Associated with anomalous junction of the …………
pancreatic and biliary duct, surgical removal is Hadfield operation
treatment of choice and if ruptures can cause biliary C Triangle of doom bounded by
peritonitis. Gonadal vessels, peritoneal reflection and vas
C Allopurinol is used in organ preservation as deferens. (Mn: Vas go Paris for Doom)
Free radical scavanger. C Central dot sign is seen in
C Most common cause of death in Crohn’s disease is Caroli’s disease (Mn: C for C)
due to C In post-radical neck dissection shoulder syndrome,
Malignancy all are seen
C Skip lesions are seen in Pain, restricted range of movement and shoulder
Crohn’s disease drooping. (Mn: PRS)
C Most important prognostic indicator for renal cell C Nicoladoni sign is also known as
carcinoma Branham sign
Pathological staging
C True about testicular tumor AIIMS MAY 2008
Seminoma is radiosensitive, seminoma can be C Auxillary orthotopic liver transplant is indicated for
treated by orchidectomy with radiotherapy and Acute fulminant liver failure for any cause.
seminoma spreads commonly through lymphatics. C Among head and neck cancers, lymph node
metastasis is least common in
AIIMS NOVEMBER 2008 Hard palate
C Most common testicular tumor in prepubertal
C Concomitant chemoradiotherapy is indicated in adults is
Cervix carcinoma (Stage IIIb), anal cancer Yolk sac tumor
(T2NoMo) and nasopharyngeal cancer (T1N2MO). C Metabolic abnormality seen in large colorectal
C Most common site of Curling’s ulcer villous adenoma
Duodenum Hypokalemic metabolic acidosis. (Mn: In villous
C Most common site of peripheral aneurysm adenoma, ac is ok).
Popliteal artery C Abbey Estender flap is based on
C A man presents with stab injury to left flank with Labial artery
vitals stable. Next step in management C Chronic burrowing ulcer is caused by
CECT Microaerophilic streptococci.
C Ipsilateral supraclavicular lymph node are positive C True about denaturation of proteins
in a patient of carcinoma breast. Stage is Unfolding occurs, disruption of secondary structure
IIIC occurs and sequence of amino acids remain the same.
176 AIIMS and All India PGMEE—Review Questions

C True about gastric lymphoma painless mobile, cystic and just below hyoid bone.
Stomach is most common site, associated with Ultrasound showed a thick walled cystic lesion.
H.pylori infection and 5 years survival rate after Management includes
treatment is 60%. Surgical removal
C A 6 years old girl presents with constipation,
urinary retention and on examination- presacral AIIMS MAY 2007
mass. Diagnosis
C A 18 years old girl presenting with thyrotoxic
Anterior sacral meningocele. symptoms and 9 months history of neck swelling.
C True about treatment of early breast carcinoma On investigation, increased T4, decreased TSH and
Postmastectomy radiation therapy is given when 4 palpable 2 cm nodule was found. Next done
or more lymph nodes are positive. Thyroid scan
C Most probable diagnosis of altered sensorium and
AIIMS NOVEMBER 2007 drowsiness after 3 days after TURP in a old age
C Most common location of spinal tumors patient
Extradural Hyponatremia
C Movement aggrevating pain in a case of retrocecal C Most common complication after ERCP
appendicitis Acute pancreatitis
Extension C A 32 years old lady, presenting with unilateral
C In couinaud classification, segment IV of liver is breast cancer with axillary lymphadenopathy. Post
Quadrate lobe modified radical mastectomy is done
C Shoulder pain post-laparoscopy is due to Adriamycin based chemotherapy followed by
CO2 retention tamoxifen depending on estrogen/progesterone
C True about Ogilive’s syndrome receptor status.
Involves entire/part of the large colon, occurs after C Colonic diverticulosis best diagnosed by
previous surgery and occurs commonly after Barium enema
narcotic use. C Complications of surgery for thoracic outlet
C FNAC needle size syndrome
22–26 Pneumothorax, brachial plexus injury and long
C Congenital urogenital abnormality associated with thoracic nerve injury
increased risk of bladder carcinoma C Stone resistant to lithotripsy
Bladder exostrophy Cystine stone
C A 24 years old male, otherwise normal, compains of C A patient, presenting with hematemesis and
mild pain in right iliac fossa in a waveform pattern malena, under goes upper GI endoscopy but there
which increases during the night and he becomes is no significant finding. The patient rebleeds after
exhausted and is admitted to the hospital. On 2 days. Next done
examination-Mild hematuria. Urine- Plenty of RBC, Laparotomy
50 WBC/hpf and pH-5.5 . Diagnosis C True about solitary rectal ulcer syndrome
Ureteral calculus Increased muscle layer proliferation, crypt distortion
C Persistent fetal lobulation of adult kidney is due to and subepithelial fibrosis
Is a normal variant C CEA increased in
C Meiosis occurs in Lung cancer, breast cancer, colon cancer
Primary to secondary spermatocyte
C Transhiatal esophagectomy was planned for
AIIMS NOVEMBER 2006
adenocarcinoma of lower end of esophagus.
Appropriate approach C True about congenital torticollis
Abdomen neck Spontaneous resolution in most cases, 2/3rd cases
C A central midline neck swelling is noted in a 4 years have palpable neck mass at birth and uncorrected
old girl posted for tonsillectomy. The swelling is cases develop plagiocephaly.
Surgery 177

C Adjuvant chemotherapy is of definite value in C A 40 years old woman presents with dysphagia to
Carcinoma colon both liquids and solids and regurgitation for 3
C Lowest risk of carcinoma breast is seen in months. The dysphagia is non-progressive. Diagnosis
Ataxia telangiectasia Achalasia cardia.
(Note: Achalasia cardia may be slowly progressive
After genetic counselling in a family for familial
and may present as non-progressive).
C
polyposis coli, next screening test is
C A 45 years old male patient presents with long
APC gene history of cigarette smoking, gangrene of left foot,
C A patient with carcinoma tongue in right lateral amputation of left foot was done. Biopsy specimen
aspect with lymph node of 4 cm size in level 3 on left showing presence of arterial thrombus with
side of neck. Stage is neutrophilic infiltration in the arterial wall with
N2 inflammation also extending into the neighbouring
C Fruit juice preventing UTI veins and nerves. Diagnosis is
Cranberry Thromboangiitis obliterans.
C Enzymes elevated in a child presenting with C RET proto-oncogene mutation is a hall mark of
jaundice icterus, pruritus and clay colored stools are Medullery carcinoma thyroid.
Gamma glutamyl transpeptidase, alkaline phos- C A 65 years old miner has last 7 kg weight within 2
photase and 5’ nuclestidase. months, presenting with cough, blood streaked
C A 50 years old male presenting with history of sputum, treated for pulmonary TB 10 years ago,
hematomesis-500 ml of blood and on examination- drooping of left eyelid for one month, on
BP-90/60, pulse rate 110/min and splenomegaly 5 examination-Ptosis of left eye and pupillary miosis
cm below lower costal margin. Diagnosis chest X-ray revealing-round opacification in the left
upper apical lobe. Diagnosis
Portal HTN
Squamous cell carcinoma
C A 45 years old female patient complains of
progressive lower limb weakness, spasticity, (Hint: Pancoast tumor).
urinary hesitancy. MRI-Intradural enhancing mass C Acquired/secondary megacolon is seen in
lesion. Diagnosis Rectal malignancy
Meningioma C True about papillary carcinoma of thyroid
C Prognostic factors for acute pancreatitis Can be reliably diagnosed using FNAC, typically
Hypocalcemia, hyperglycemia and AST elevation. spreads to the cervical lymph nodes and requires
total thyroidectomy for large tumors.
C Hypotension in acute spinal injury is due to
C True statements about stones in gallbladder
Loss of sympathetic tone
Pigment stones are due to increased excretion of
conjugated bilirubin, are considered a risk factor for
AIIMS MAY 2006 development of gallbladder carcinoma and 10%
gallstones are radio-opaque.
True about radiological evaluation of a patient with
A 45 years old gentleman has undergone truncal
C
C
Cushing’s syndrome
vagotomy and pyloroplasty for bleeding duodenal
Petrous sinus sampling is the best way to ulcer seven years ago, now presenting with intra-
distinguish a pituitary tumor from ectopic ACTH ctable recurrent symptoms of peptic ulcer. Features
producing tumor, MRI of adrenals may distinguish suggesting Zollinger-Ellison syndrome are
adrenal adenoma from carcinoma and adrenal CT Basal acid out put of 15 meq/hour, serum gastrin
scan distinguished adrenal cortical hyperplasia value of 500 pg/ml and ulcers in proximal jejunum
from an adrenal tumor. and lower end of esophagus.
C Barrett’s esophagus commonly associated with C Defect in migration of neural crest cells result in
Adenocarcinoma Albinism and congenital megacolon.
C Brain tumor highly vascular in nature C True about primary grade IV-V VUR in young children
Glioblastoma > meningioma. Postnatal scarring may occur even in absence of UTI.
178 AIIMS and All India PGMEE—Review Questions

C Best way of preventing development of DVT in C Features of Zollinger-Ellison syndrome


post-operative period Intractable peptic ulcers, severe diarrhea and very
Prophylactic heparin high acid output.
C Middle aged sports photographer presents with C Modalities of therapy for hCC
small nodule over the upper lip from four months, Percutaneus acetic acid, radiofrequency ablation
histopathology-nodule is pearly white with central and transarterial catheter embolization.
necrosis telangiectasia. Diagnosis is C True about subclavian steal syndrome
Basal cell carcinoma Reversal of blood flow in the ipsilateral vertebral
C True about Meckel’s diverticulum artery.
Is present on anti-mesenteric border of the small C Features evaluated for histological grading of
intestine, heterotropic gastric mucosa can create breast carcinoma include
and causes a brick gastrointestinal bleed and a Nuclear pleomorphism, mitotic count and tubule
fibrous band between the apex and umbilius can formation.
cause intestinal obstruction. C Factor used to predict severe acute pancreatitis
C A 40 years old premenopausal patient has recently Glasgow score 3, APACHE II score 9 and CT severity
developed a 1.5 cm sized pigmented lesion on her score 6.
back. Investigation recommended are C A patient presented with massive upper GI bleed,
Excision biopsy on examination-mild splenomegaly. Most appro-
priate treatment at this stage in absence of any other
AIIMS NOVEMBER 2005
information
C True about fat embolism IV pantoprazole
Most patients with major trauma involving long C True about bile duct injuries following cholecystec-
bones have urinary flat globules. tomy
C Confirmation of diagnosis of congenital megacolon Incidence following open cholecystectomy is in
Rectal biopsy range of 0.2% to 0.3%, incidence rate following
C Treatment of choice for anaplastic carcinoma of laparoscopic cholecystectomy is three times higher
thyroid infiltrating trachea and sternum will be than rates following open cholecystectomy and
Palliative treatment untreated cases may develop secondary biliary
C True about treatment of a 16 years old boy presenting cirrhosis.
with acute onset pain in the left testis C Platelets can be stored at
Color flow Doppler will be very useful in diagnosis, 20–24°C for 5 days.
scrotal exploration should be done without delay if C Most common cause of peripheral limb ischemia in
Doppler is not available and if left testis is not viable India is
on exploration, patient should undergo left Atherosclerosis
orchidectomy and right orchidopexy. C True about management of a 45 years old erythroid
C Extraintestinal manifestations of ulcerative colitis male patient presenting with 4 × 4 cm, mobile right
responding to colectomy are solitary thyroid nodule of 5 months duration are
Pyoderma gangrenosum, peripheral arthralgia and FNAC is investigation of choice, patient should
episcleritis. undergo hemithyroidectomy if FNAC report is
C Most effective intravesical therapy for superficial inconclusive and indirect laryngoscopy should be
bladder cancer done in the preoperative period to assess mobility
BCG of vocal cords.
C Most common facial abnormality seen in Gardner C In carcinoma breast case, chest wall involvement
syndrome is means involvement of
Multiple osteoma Serratus anterior, intercostal muscles and ribs. (Mn:
C Soft tissue sarcoma with propensity for lymphatic SAI)
spread are C Indications for surgery in gastric lymphoma
Synovial SA, rhabdomyosarcoma and epitheloid Bleeding, perforation and residual disease following
sarcoma. chemotherapy.
Surgery 179

C Head injury with lucid interval, most likely it is a C True about GI bleeding
case of brain hemorrhage of The sensitivity of angiography for detecting GI
Extradural bleeding is about 10–20% as compared to nuclear
C AFP is elevated in imaging, 99 mTc-RBC scan will image bleeding at
Hepatocellular carcinoma rates as low as 0.05–0.1 ml/min and angiography
C Acalculous cholecystitis can be seen in will detect bleeding only if extravasation has
DHF, Enteric fever and leptospirosis. occurred during contrast injection.
C Most beneficial technique of using chemotherapy
AIIMS MAY 2005 with a course of radiotherapy in head and neck
C A warthin’s tumor is malignancies
An adenolymphoma of parotid gland. (Mn: WAL) Concurrent chemotherapy.
C Features seen more in seminoma than non- C Hypoparathyroidism following thyroid surgery
seminomatous germ cell tumors of the testis are occurs within
Tumors remain localized to testis for a long time, 2–5 days
they are radiosensitive and they metastasize C A 55 years male patient presents with features of
predominantly by lymphatics. obstructive jaundice, loss of 7 kg weight in last 2
C Complications of total thyroidectomy include months, CT scan-CBD dilated till the lower end and
Airway obstruction, hemorrhage and hoarseness. main pancreatic duct is also dilated. Pancreas is
C The initial investigation of choice for a post- normal. Diagnosis
cholecystectomy biliary stricture is Periampullary carcinoma
CT scan (Hint: Double duct sign).
C Treatment of choice for an 8 mm retained CBD stone is C Investigation of choice for assessment of depth of
Endoscopic stone extraction. penetration and perirectal nodes in rectal cancer
C Treatment of choice for medullary carcinoma thyroid Transrectal ultrasound.
Total thyroidectomy. C Following resuscitation, a patient with bleeding
C Indications for cholecystectomy are: esophageal varices should be treated initially with
70/M with symptomatic gallstones. Sclerotherapy
20/M with sickel cell anemia and symptomatic C A 20 years female patient presented with a thyroid
gallstones and swelling. FNAC will not diagnose
60/F with large gallbladder polyp. Follicular carcinoma of thyroid
C True about Ludwig’s angina C A 13 years old boy presents with acute onset right
An infection of the cellular tissues around submandi- scrotal pain. The pain is not relieved on elevation of
bular salivary gland. scrotum and he has no fever or dysuria. The testis is
enlarged and tender. Routine urine examination is
AIIMS NOVEMBER 2004 normal. No history of trauma. Most appropriate
management
A 25 years female patient presents with spontaneous
Immediate exploration.
C
nipple discharge of 3 months duration, on examina-
C A 40 years female patient has undergone cholecys-
tion-discharge is bloody and from a single duct.
tectomy. Histopathology reveals that she has a 3 cm
True about management
adenocarcinoma in the body of gallbladder
Ultrasound can be a useful investigation, galacto-
infiltrating up to the serosa. Further management
gram is not essential and most of the blood stained
nipple discharges are due to papilloma or other Radical cholecystectomy
bengin condition. C Migratory thrombophlebitis is associated with
C A 25 years male patient presents with history of Lung carcinoma, pancreas carcinoma, and GI carci-
RTA two hours ago, hemodynamically stable, noma.
abdomen-soft and hematuria noted on catheteri- C During bilateral adrenolactomy, intraoperative
zation of the bladder. Next step dose of hydrocortisone should be given after
CECT abdomen Excision of both adrenal glands.
180 AIIMS and All India PGMEE—Review Questions

C Treatment of choice for a 4 cm retroperitoneal C Most common cause of superficial thrombophlelbitis


lymph node mass in a patient with non-seminoma- Intravenous influsion
tous germ cell tumor of the testis C A man rushed to casuality, nearly dying after a
High orchidectomy + RPLND. massive blood loss in an accident. Blood group that
C True about repair of groin hernias can be transfused if there is no time for matching
Lichenstein tension free repair has a low recurrence blood group
rate, TEPP repair is an extra-peritoneal approach to O negative
laparoscopic repair of groin hernia and the surgery C Risk factors for carcinoma gallbladder
can be done under LA in selected cases.
Typhoid carriers, adenomatous gallbladder polyps
C A 40 years male patient suffers from carotid body and choledochal cysts.
tumor. Treatment of choice
Excision of tumor
C True about renal transplantation AIIMS NOVEMBER 2003
Renal transplant is heterotopic, cyclosporine is the C The intra-abdominal pressure during laparoscopy
mainstay of immunosuppression and kidney after should be set between
removal is flushed with cold perfusion solution. 10–15 mm Hg
C Hepatic lesion diagnosed with high accuracy by C Young adult man weight 65 kg was admitted to the
using nuclear imaging hospital with severe burns in a severe catabolic
Focal nodular hyperplasia. state. An individual in this state requires 40 K cal/
kg/day, 2 gm of protein/kg/day and he was given
AIIMS MAY 2004 a solution containing 20% glucose and 4.25%
protein. If 3000 ml of solution is infused per day,
C A 60 years male smoker presents with hoarseness, both protein and calories would be adequate.
hemoptysis and hard painless lump in the left C Features of a malignant tumor on mammogram
supraclavicular fossa. Next investigation most are
appropriately done Irregular mass, microcalcification and specula-
Undertake an open biopsy of the neck lump. tion.
C During surgery of hernia, the sac of a strangulated C A 55 years old post-menopausal woman on HRT
inguinal hernia should be opened at the presents with heaviness in both breasts. A
Fundus screening mammogram reveals a high density
C Most common site of a bengin peptic ulcer speculated mass with cluster of pleomorphic
Lesser culvature near incisura angularis. microcalcification and ipsilateral large axillary
C Most common endocrine tumor of pancreas lymph node. The mass described is
Insulinoma Carcinoma
C Cock’s peculiar tumor C During repair of indirect inguinal hernia, while
An infected sebaceous cyst. releasing the constriction of the deep inguinal ring,
C A 40 years female patient presents with a 5 cm lump surgeon takes care not to damage
in right breast with a 3 cm node in supraclavicular Inferior epigastric artery
fossa. According to AJCC staging C Recurrent laryngeal nerve is in close association with
T2N3MO Inferior thyroid artery.
C True about Holmium: YAG laser C A 50 years male smoker patient presents with pain
Has a wavelength of 2100 nm, effective against the along the left arm and ptosis. His chest radiograph
hardest urinary stones and it can even cut the wire shows a soft tissue opacity at the left lung apex with
of stone baskets. destruction of adjacent ribs. Diagnosis suggestive
C Liver tumor with propensity to invade the portal or Pancoast tumor
hepatic vein C A 50 years male alcoholic and smoker presents with
Hepatocellular carcinoma a 3 hours history of severe retrosternal chest pain
C Treatment of choice for a mucocele of gallbladder is and increasing shortness of breath. He started
Cholecystectomy having this pain while eating, which was constant
Surgery 181

and radiated to back and in interscapular region. ultrasound abdomen showing-stone in the CBD.
He is a known hypertensive. On examination-cold Best treatment option
and clammy, HR—130/min, BP—80/40 mm Hg, JVP ERCP and bile duct stone extraction.
normal, peripheral pulses present and equal. Breath C A 14 years female patient of normal height and
sounds decreased at the left lung base and chest weight for age, complains of right breast size twice
X-ray showing left pleural effusion. Diagnosis that of left breast since onset of puberty 2 years ago.
Acute aortic dissection. On examination-both breasts have a similar
C Breast conservation surgery for breast cancer in consistency on palpation with normal nipples and
indicated in areola. Diagnosis
T1 breast cancer and extensive in situ cancer. Virginal hypertrophy
Treatment of acute upper GI bleed possibly avoided
A 65 years male patient diagnosed with prostate
C
C
cancer three years back was treated by surgery and Intravenous β blockers.
hormone therapy. Presently he has developed C True about varicose veins
urinary symptoms and progressive backache. 5% oily phenol is an appropriate sclerosant for
Tumor marker indicative of disease relapse venous sclerotherapy.
Dysphagia lusoria is due to
PSA
C
Compression by aberrant blood vessel.
C Malignant disease of childhood with best progno-
C Least malignant thyroid cancer
sis
Papillary carcinoma.
Wilm’s tumor. C Blood stained discharge from the nipple indicates
C A 3 years male child presents with history of Duct papilloma.
constipation and abdominal distension for last 2 C Most common site of carcinoma esophagus in India
years, plain radiograph of abdomen revealing, Middle 1/3rd.
faecal matter containing distended bowel loops. A C Substance not used as an irrigant during TURP
barium enema study done subsequently shows a
Normal saline.
transition zone at the rectosigmoid junction with
reversal of rectosigmoid ratio. Diagnosis AIIMS MAY 2003
Hirschprung’s disease
C Most likely cause of a 3 years old boy presenting
The tendency of colonic carcinoma to metastasize is
with poor urinary stream
C
best assessed by
Posterior urethral valve.
Depth of penetration of bowel wall.
C There is a high risk of renal dysplasia in
C A 40 years female patient has undergone an open Posterior urethral valve.
cholecystectomy, procedure uneventful. She has
C Post dural puncture is typically
100 ml of bile output from the drain kepts in the
Bifrontal or occipital
gallbladder bed on first postoperative day. On
examination afebrile and an icteric. The abdomen is C A young patient presents with history of dysphagia
soft and bowel sounds are normal. Advice more to liquid than to solids. First investigation
Clinical observation Barium swallow
C True about hepatocellular carcinoma C True of gas gangrene
High incidence in East Africa and South-East Asia, Caused by Clostridium perfringens, gas gangrene is
worldwide incidence parallels the prevalence of characterized by severe local pain, crepitus and sign
Hepatitis B and liver transplantation offer the of toxemia and high dose penicillin and aggressive
only chance of cure in those with irresectable debridement of affected tissue is treatment of
disease. established infection.
C A 50 years female patient presents with history of C In blast injury, organs vulnerable to blast wave
recurrent episodes of right upper abdominal pain Ear drum, GIT and lung.
for last one year, history of jaundice and fever C Contraindication for ESWL for renal calculi
for 4 days, on examination- patient is toxic, BP 90/ Uncorrected bleeding diasthesis, pregnancy and
60 mm Hg, started on intravenous- antibiotics, ureteric strictures.
182 AIIMS and All India PGMEE—Review Questions

C Investigations done for anterior urethral stricture C Indications for surgery in gastric lymphoma
Retrograde urethrogram, micturating cystoure- Bleeding, perforation and residual disease following
throgram and high frequency USG. radiotherapy.
C Treatment of choice for prepucial adhesions pro- C A middle age male patient presents with stab injury
ducing ballooning of prepuce during micturition in to chest, on examination-hemodynamically stable,
a 2 years old boy neck veins engorged and heart sound muffled. True
Circumcision. statements are
(Hint: Phimosis) Diagnosis is cardiac tamponade, echocardiogram
C Cells from the neural crest are involved in is done to confirm pericardial blood and the entry
Hirschprung’s disease, neuroblastoma and PNET. wound should be sealed with an occlusive
C A warthin’s tumor is dressing.
An adenolymphoma of parotid gland. (Mn: WAL) C A young adult male patient presents with pain on
C True about testicular tumors the right flank and hematuria, CECT abdomen-
Seminomas are radio sensitive. Large 8 × 8 cm solid mass in right kidney and 3 × 3 cm
C Upper GI endoscopy and biopsy from lower solid mass occupying upper pole of left kidney.
esphagus in a 48 years old woman with chronic Surgery of choice
heart burn shows presence of columner epithelium Right radical nephrectomy and left partial
with goblet cells. It is consistent with nephrectomy.
Metaplasia C True about germ cell tumors of testis
(Note: Normally squamous). They constitute 90–95% of all primary testicular
C Oncological emergencies are tumors, seminoma is most common tumor
Spinal cord compression, SVC syndrome and developing in the patients with cryptorchid testis
tumor lysis syndrome. and high inguinal orchidectomy is the initial
C Side effect of tamoxifen in carcinoma of breast patients surgical procedure.
Cataract, endometrial carcinoma and thromboem- C A 25 years male patient presents after road traffic
bolic events. accident, on examination-pelvic fracture and blood
C Common causes of respiratory difficulty in the at urethral meatus. True about this patient
immediate postoperative period are Retrograde urethrography should be done after the
Mycardial infarction, overdose of narcotic analgesic patient is stabilized, Foley’s catheter may be
and residual effect of muscle relaxant. carefully passed if the RGU is normal and rectal
C A patient undergoing surgery suddenly develops examination may reveal a large pelvic hematoma
hypotension with et CO2 decreasing by 15 mm Hg with the prostate displaced superiorly.
suddenly. Diagnosis C True about peyronie’s disease
Pulmonary embolism Painful erection, associated with Dupuytren’s
contracture of the tendon of the hand and sponta-
AIIMS NOVEMBER 2002 neous regresssion occurs in 50% of the cases.
C The Gold standard investigation for extrahepatic C Post traumatic epilepsy refers to
biliary atresia Seizure occurring within several weeks to months
Preoperative cholangiography. after injury.
C True about malignant potential of colorectal polyps C Fetal hydronephrosis is diagnosis in a mother at 34
Polyps of FPC invariably undergoes malignant weeks gestation. The aminotic fluid is normal most
change, villous adenoma is associated with high appropriate management
risk of malignancy and juvenile polyps have little or Delivery at term followed by radiological evaluation.
no risk. C The length of the feeding tube for transpyloric
C Treatment of squamous cell carcinoma of anal canal feeding is measured
Chemoradiation From tip of ear lobe to the umbilicus.
(Note: Cisplatin based chemotherapy and radical C Preservative for packing catgut suture
radiotherapy). Isopropyl alcohol.
Surgery 183

AIIMS MAY 2002 C A patient presents with 1 × 1.5 cm growth on the


lateral border of the tongue. Treatment indicated
C Rupture of the pituitary stalk in a accident may led to Interstitial brachytherapy.
Hyperprolactinemia, diabetes insipidus and hypo-
thyroidism.
C Part of lung commonly affected in foreign body ALL INDIA 2012
aspiration in supine position C On GCS, inappropriate words, scoring is
Apical part of right lower lobe. 3
C True about hydatid cyst of lung C Most common cause of pyogenic liver abscess
More common in lower lobes.
Biliary tract disease.
C Treatment of choice in a patient with compensated
Surgery for rectal prolapse in an adult patient
liver cirrhosis presenting with history of variceal
C
associated with least rate of recurrence
bleed
Endoscopic sclerotherapy Rectopaxy
C A patient presented with headache and flushing C Carcinoma with highest chances of metastasis to
and family history of death due to thyroid tumor. spleen
Investigation required Ovary
Measurement of catecholamine C A 40 years male patient presents with hematomesis,
(Hint: MEN2) on examination-BP 90/60 mm Hg, heart rate 120/
C Lateral aberrant thyroid refers to min and splenomegaly. Cause of bleeding
Metastatic foci from primary in thyroid. Portal hypertension
C Parathyroid adenoma most commonly involves C The absorption of which is affected in case of short
Inferier parathyroid lobe. bowel syndrome
C True about Warthin’s tumor Vitamin B12
Commonly involves the parotid glands, they arise C A child presents with Wilm’s tumor stage 1 in one
from the epithelial and the lymphoid cell and 10% kidney with contralateral kidney appearing to be
bilateral. normal. First modality of treatment
C Features to differentiate parathyroid adenoma Open nephroureterectomy.
from hyperplasia includes C An elderly male patient presents with complaints of
Identifying hyperplasia of all 4 glands at surgery in choking while sleeping with history of non-
parathyroid hyperplasia. progressive dysphagia and halitosis. Diagnosis
C Insulinoma is most commonly located in parts of Zenker’s diverticulum.
pancreas C The expanded criteria for liver donation includes
Equally distributed
HbsAg positive without any evidence of chronic
C In video assisted thoracoscopic surgery for better disease progression, age can be more than 70 years
vision, the space in the operative field is created by and donor with mild hepatic steatosis.
Collapse of ipsilateral lung.
C Most common complication of TPN
C Most common tumor to produce metastasis to
Catheter infection
cervical lymph node
Nasopharyngeal carcinoma. C True about non-small cell carcinoma lung
C A patient operated for thyroid surgery for thyroid Contralateral mediastinal lymphadenopathy is
swelling later in the evening developed difficulty in suggestive of inoperable disease, Geftinib is
breathing. There was swelling in the neck. Immediate effective in adenocarcinoma histology and single
management is agent chemotherapy can be used in elderly patients.
Open the wound sutures in the ward. C True about small bowel carcinoids
C A female patient presents with thyroid swelling, Multiple tumors lead to increased risk of lung cancer.
radionuclide scan showing cold nodule and ultra- C A patient with thoracic spine T3 injured presents
sound showing non-cystic solid mass. Management is with headache, BP 200/120 and HR 54. Management
Hemithyroidectomy Oral nifedipine
184 AIIMS and All India PGMEE—Review Questions

C A 9 years male patient underwent near total C A patient of abdominal injury presents with signs of
thyroidectomy followed by radical neck dissection peritonitis and shock. Airway and breathing were
for cervical lymphadenopathy (needle biopsy of lymph secured and IV fluid started with 2 large bore
node- secondaries from papillary carcinoma thyroid) cannulas. Next line of management
and biopsy confirming the diagnosis. Next done Exploratory laparotomy under GA.
Start thyroxine suppression therapy C Surgeon excises a portion of liver to the left of the
C Left varicocele in old aged male patient arises suspi- attachment of the falciform ligament. Segments
cion of resected are
Left renal tumor Segment 2 and 3.
C Renal stone associated with Proteus infection
Triple phosphate
ALL INDIA 2011
C Most common cause of delayed urinary tract
C True about ranula obstructive symptoms after TURP
Cystic swelling in floor of mouth. Bladder neck stenosis
C Follicular carcinoma of thyroid can be best C During TURP, surgeon takes care to dissect above
differentiated from a follicular adenoma by the verumontanum to prevent injury to
Vascular invasion External urethral sphincter.
C Multiple cutaneous sebaceous adenoma are seen in C A 55 years old smoker presents with history of five
Muir-Torry syndrome. episodes of macroscopic hematuria each lasting for
C A 30 years old male patient presents with about 4–5 days in past 5 years. Investigation
symptoms of epigastric pain, radiating to back that performed to evaluate the suspected diagnosis
makes him wake-up at night and is relieved by Urine microscopy and cytology.
consuming food. There is history of similar pain in C Best recommended management option for a 40
the past diagnosed as perforated duodenal ulcer years old patient with a single kidney presenting
and treated with omental patch surgery on two with a solitary exophytic mass of 4 cm localized at
occasions. Pain before and after surgery has been its lower pole
controlled with PPI and analysis. Diagnosis Partial nephrectomy
Duodenal ulcer C A woman presented with hematuria was found to
C True about gastric carcinoma have stage II transitional cell carcinoma of bladder.
Often associated with hypochlorhydria/achlorhy- True is
dria. 70% chance of recurrence 5 years after TURP.
C A woman presents with 3 day history of epigastric C Surgery for undescended testis performed at 6
pain radiating to back, normal serum amylase months.
levels and gallbladder stones and enlarged C A 5 years old child presenting with balloning of
pancreas on USG abdomen. Diagnosis is prepuce after micturition. O/E prepucial adhesions
Acute pancreatitis in penis. Best management
C Middle aged woman presents with 2 years history Circumcision
of recurrent abdominal pain with radiation to her C Nutrient not included in TPN
back, pain severe in intensity and refractory to Fibres
simple analgesics, USG and CECT confirming C A patient with head injury opens eyes to painful
diagnosis and showing dilated pancreatic duct. stimulus uses inappropriate words and localizes
Operation of choice pain. GCS score
Longitudinal pancreaticojejunostomy. 10
C A patient with stab injury to anterior abdomen
ALL INDIA 2010
presents with a tag of omentum providing through
the abdominal wall near the umbilicus. He is C Lord’s plication is done for
hemodynamically stable and shows no signs of Hydrocele
peritonitis Next done C Stones hard to break by ESWL
CECT abdomen Calcium oxalate monohydrate.
Surgery 185

C A 50 years old woman presented with abdominal C In orthotropic liver transplantation, best way to get
pain, anuria, radiological finding of bilateral bile drainage in donor liver
impacted ureteric stones with hydronephrosis, Donor bile duct with recipient bile duct or Roux en
urinalysis showing RBC with pus cells in urine, Y choledochojejunostomy.
serum creatine 16 mg/dl serum urea 200 mmol/dl. C Most common cyst and other spleen are
Immediately required Hydatid cyst
J stent drainage C A male patient presents with fever for 3 weeks,
C Complication of PCNL done through 11th splenomegaly hypoechoic shadow in spleen near
intercostals space the hilum on USG, and gram-negative bacili isolated
Hydrothorax on blood culture. Most likely caustive organism is
C Most common renal vascular anomaly Salmonella
Supernumeray renal artery. C A patient with ITP has a platelet count of 50,000
C First autologous renal transplant was done by being planned for splenectomy. Best time for
Hardy platelet infusion in this patient
C Most common site of urethral carcinoma in men is After ligating the splenic artery.
Bulbomembranous urethra. C Most common cause of abdominal aortic aneurysm
C The Grayhack shunt is established between Atherosclerosis
Corpora cavernosa and saphenous vein. C Primarily restrictive operations for morbid obesity
C An adult presented with hemetemesis, and upper are
abdominal pain, endoscopy showed growth at Vertical band gastroplasty, laparoscopic adjustable
pyloric antrum of stomach, CT scan-growth gastric banding and Roux en Y operation.
involving pyloric antrum without infiltration or C TRISS includes
invasion into surrounding structures and no RTS + ISS +Age (Mn: MIRA–M is mechanism of
evidence of distant metastasis. At laparotomy injury)
neoplastic growth was observed to involve the C A 30 years old patient presents with left sided abdo-
posterior wall of stomach and pancreas extending minal pain, 6 hours after RTA, hemodynamically
6 cm up to tail of pancreas. Most appropriate surgery. stable, FAST positive, CECT showing grade II
Partial gastrectomy + distal pancreatectomy. splenic laceration. Most appropriate management
C True about Gastrointestinal carcinoid tumors Conservative management
Small intestine and appendix account for almost (Note: If on CECT contrast blush with grade III is
60% of all GI carcinoid, 5 years survival for seen splenic artery embolization is most
carcinoid tumors is 76% and appendicial carcinoids appropriate treatment).
are more common in females than males. C A woman presented with stab injury to left side of
C Surgery of choice in a young male patient the abdomen, hemodynamically stable. CECT
presenting with complete rectal prolapse revealing laceration in spleen. Laparoscopy was
Abdominal rectopexy. planned but patient’s pO2 suddenly dropped as
C Treatment of choice for annular pancreas soon as pneumoperitoneum was created. Most
Duodenojejunostomy likely cause is
Gaseous embolism through splenic vessels.
C A lady presents with recurrent attacks of giddiness
and abdominal pain since 3 months, normal C Scale to evaluate prognosis after subarachnoid
endoscopy, fasting blood glucose level 40 mg% and hemorrhage
elevated insulin levels, CT abdomen showing well Hess and Hunt Scale
defined 8 mm enhancing lesion in the head of pan-
creas with no other abnormal findings. Treatment ALL INDIA 2009
Enucleation C A man with blunt injury abdomen after RTA has BP
C According to Bismuth/Strasberg classification, 100/80 mm Hg, PR 120/mn, airway established
cystic blowout is classified as and respiration stabilized. Next step
Type A Blood for cross matching and IV fluids.
186 AIIMS and All India PGMEE—Review Questions

C True about necrotizing fascitis C Parastromal hernia is most frequently seen with
Infection of fascia and subcutaneous tissue, most End colostomy
commonly caused by group A beta hemolytic C The advantage of bladder drainage over enteric
Streptococcus and surgical debridement is manda- drainage after pancreatic transplantation
tory. Better monitoring of amylase levels.
C True about lymphoepithelioma of the parotid gland C Post-chemotherapy based staging system in Wilm’s
Associated with EBV infection, highly radiosen- tumor
sitive and type of squamous cell carcinoma SIOP (International Society of Paediatric Oncology)
C Triple assessment for carcinoma breast includes
Clinical examination, mammogram and FNAC. ALL INDIA 2008
(Mn: FMC)
C Moderately increased risk of invasive breast C Non-absorbable sature
carcinoma is associated with Polypropylene
Atypical lobular hyperplasia C Decision about surgery in case of hemothorax due
C Thoracic outlet syndrome is primarily diagnosed by to blunt trauma chest is based on
Clinical evaluation. Hemodynamic status > nature of chest tube output.
C Popliteal artery pulsation are difficult to feel C A male patient with blunt trauma abdomen is
beacause hemodynamically stable. Next done
Further imaging of abdomen.
It is not superficial and does not cross prominent
bone. C A young patient presents with a massive injury to
proximal duodenum, head of pancreas and distal
C Intermittent claudication is defined as
common bile duct. Surgery of choice
Pain in muscle on excercise only.
Pancreaticoduodenectomy (Whipple’s operation).
A patient with critical lower limb ischemia presents
Landmark for facial nerve during parotid surgery
C
C
with
Digastric muscle, tragal pointer and retrograde
Rest pain and ischemic, ulcers. dissection of distal branch.
C Lumbar sympathectomy is of value in the manage- C Abbe-Estlander flap is used for
ment of Lip
Distal ischemia affecting the skin of the toes. C Most common histological type of thyroid carcinoma
(Note: Sympathectomy is not indicated in anhydro- Papillary carcinoma thyroid.
sis). C Complications of hemithyroidectomy includes
C Pseudoclaudication is caused by External branch of superior laryngeal nerve palsy,
Lumbar canal stenosis recurrent laryngeal nerve palsy and wound
C Best material for below inguinal arterial graft is hematoma.
Saphenous vein graft (upside-down). C A 25 years old women complains of discharge of
C Most common part of subclavian artery affected by blood from a single duct in her breast. Most appro-
stenosis priate treatment is
First part Microdochectomy
C The most common site of rupture of abdominal C Most common tumor in the posterior mediastinum
aortic aneurysm Neurofibroma
Laterally into the left peritoneum. C A 45 years old woman presents with symptoms of
C Most common cause of superficial thrombophlebitis acute cholecystitis. On USG, there is solitary gall-
Intravenous catheters/infusion. stone of size 1.5 cm. Symptoms are controlled with
C SEPS is a procedure used for medical management. Next most appropriate test
Veins Laparoscopic cholecystectomy immediately.
C True about Zenker’s diverticulum C An otherwise normal woman presents with
Acquired diverticulum, false diverticulum and symptoms of flatulent dyspepsia. She was started
lateral X-rays on barium swallow are often diag- on PPI which controlled her symptoms. The next step
nostic. Wait and watch
Surgery 187

C A chronic alcoholic presents with abdominal pain, ALL INDIA 2007


radiating to the back that responds to analgesics.
On evaluation, the pancreatic duct was found to be C Histological variants of thyroid neoplasm
dilated and stones were noted in the tail of pancreas- Follicular, insular and anaplastic.
most appropriate managmement C True about lymphona of the thyroid
Medical management More common in females, clinically confused with
(Note: If pain becomes intractable in this patient, undifferentiated tumors and may present with
treatment is pancreaticojejunostomy). respiratory distress and dysphagia.
C A patient undergoes a prolonged and complicated C Marker of Paget’s disease of the mammary gland
pancreatic surgery for chronic pancreatitis. Most CEA
preferred route for supplementary nutrition C True about gynecomastia
Feeding jejunostomy Can be drug induced, seen in liver disease and there
may be estrogen/testosterone imbalance.
C True about volvulus
C True about gastric injury
More common in psychiatric patients, sigmoid
Mostly related to penetrating trauma, treatment is
volvulus is more common than cecal volvulus and
simple debridement and suturing and heals well
volvulus of cecum is managed by conservative
and fast.
methods.
C Least malignant potential in
C Anal fissure is best diagnosed by
Hamartomatous polyps in Peutz-Jaghers synd-
History and superficial clinical examination. rome.
C True for Nigro’s regimen C True about external hemorrhoids
It is a regimen for anal canal neoplasm, it incorpora- Painful, below the dentate line.
tes chemotherapy with radiation as an alternative C PR—True about pilonidal sinus
to surgery and has the advantage of preserving More common in females, mostly congenital and
continence. prognosis after surgery is poor.
C An adult male patient presents with sudden onset C Investigations recommended in an elderly male
swelling and pain over the right hemiscrotum. On presenting with one episode of gross hematuria
examination scrotum is redened and tender. True is IVP, cystoscopy and urine microscopy for malig-
Probable diagnosis is torsion, the right testis is nant cells.
likely to ride high in the scrotal compartment and if C A 50 years old diabetic presents with poor urinary
torsion confirmed, treat with antibiotics and stream, hesitancy, difficulty in micturition and
analgesics and perform corrective surgery failure to void completely. Diagnosis is
immediately. Autonomic neuropathy
C Ureteric colic due to stone is caused by (Note: Increased frequency of micturition-BPH).
Increased peristalsis of ureter to overcome the
obstruction. ALL INDIA 2006
C Epispadias is associated with
C The most common malignancy found in Marjolin’s
Chordee
ulcer
C Transrectal ultrasonogram in evaluation of Squamous cell carcinoma.
carcinoma prostate is most useful for C Prognosis of rhabdomyosarcoma is poor if the site
Taking guided biopsy of the tumor is
C Diagnostic procedure of choice in a newborn who Extremity
has not passed meconium for 48 hours. C FNAC not suitable for
Contrast enema (Lower gastrointestinal contrast Aneurysmal bone cyst bone cyst. (Mn: FNAC-
study). FNAC Not for AC-Aneursymal bone cyst).
C A newborn baby presents with absent anal orifice C True about thyroglossal cyst
and meconuria. Most appropriate management is Frequent cause of anterior midline neck masses in
Transverse colostomy the first decade of life, the cyst is located within 2 cm
188 AIIMS and All India PGMEE—Review Questions

of the midline and the swelling moves upward on C Allen’s test is useful in evaluating
protrusion of tongue. Integrity of palmar arch.
C Mixed tumors of the salivary glands C Most common cause of AV fistula
Most common in parotid gland. Penetrating trauma.
C The most important prognostic factor in breast C Dacron vascular graft
carcinoma Textile synthetic (Mn: STD).
Stage of tumor at the time of diagnosis. C Preferred sites for planning vascular access for
C Biopsy of opposite breast is indicated in type of maintenance hemodialysis
breast carcinoma Non-dominant extremity, upper limb and radio-
Lobular carcinoma cephalic AV fistula.
C The tumor that may occur in the residual breast C Most preferred approach for pituitary surgery is
overlying skin following wide local excision and Trans-sphenoidal
radiotherapy for mammany carcinoma is C Components of Glasgow Coma Scale
Angiosarcoma Eye opening, Verbal response and motor response.
C Referred pain is felt along the inner side of right (Mn: EVM)
thigh C Total score in GCS
In inflamed pelvic appendix inflamed ovaries and 15
stone in pelvic ureter. C Blood stained discharge from nipple indicates
After E.coli, next most common organism impli-
Duct papilloma
C
cated in acute suppurative bacterial peritonitis is
C Perineural invasion in head and neck cancer is seen
Bacteriodes
in
C A patient of post-cholecystectomy biliary structure
Adenoid cystic carcinoma
has undergone ERCP three days ago. Following this
she has developed acute cholangitis. The most C Sialography is contraindicated in
likely organism Acute parotitis
E. coli. C Most common complication of hiatus hernia
C True about stones in CBD Esophagitis
Can present with charcoat’s triad, suggested by a C Full thickness biopsy of rectum of a neonate
bile duct diameter > 6 mm of USG and ERCP, presenting with bloated abdomen shortly after
sphincterotomy and balloon clearance is now the birth with passing of less meconium is most likely
standard treatment. to show
C Biliary stricture after laparoscopic cholecystectomy Lack of ganglion cells
usual occurs at (Note: Hirschsprung disease)
Upper part of CBD C Gardener’s syndrome (a rare hereditary disorder of
C Modalities used for in situ ablation of liver colon) is characterized by
secondaries Polyposis colon, osteoma, epidermal cysts and
Alcohol, cryotherapy and radiofrequency. (Mn: CAR) fibroma.
C The most common cause of gastric outlet obstruction C Carcinoid tumor is most common in
in India Small bowel > Lung
Stomach carcinoma C Overall survival increases by screening in
C Treatment for anal canal carcinoma Colon cancer
Chemoradiotherapy C True about patients of ulcerative colitis associated
C Part of triple therapy immunosuppression for post- with primary sclerosing cholangitis (PSC)
renal transplant patients May develop biliary cirrhosis, may have raised
Cyclosporine, azathioprine and prednisolone (Mn: alkaline phosphatase and increased risks of
CAP). cholangiocarcinoma.
C True about testicular tumors C Ideal for treatment with sclerosing agent injection
Bilateral in up to 10% cases. Internal hemorrhoids.
Surgery 189

C Catheter material most suitable for long term use C A 20 years old male patient presents with chronic
is constipation, headache, palpitation, on examination
Silicone Marfanoid habitus, neuromas of tongue, medullated
C Most trouble some source of bleeding during a corneal nerve fibres and nodule of 2 × 2 cm size in
radical retropubic prostatectomy left lobe of thyroid gland. Diagnosis is
Dorsal venous complex MEN IIB
C True about finasteride C Most common site of oral cancer among Indian
Used in medical treatment of BPH, impotence is population
well documented side effect and it is a 5 alpha Alveolobuccal complex
reductase inhibitor. C Most common malignant tumor of adult males in
C According to GCS, score of 1 indicates India is
No response Oropharyngeal carcinoma.
C Earliest sign/symptom of increased intracranial C Most common site of intestinal obstruction in gall-
pressure following head injury is stone illeus is
Altered mental status Ileum
C Soft tissue sarcoma frequently metastasing to the C Type of pancreatitis with best prognosis
lymph nodes Gallstone pancreatitis.
Embryonal rhabdomyosarcoma C Surgery of choice for a rectal carcinoma at 5 cm from
the anal verge is
ALL INDIA 2004 Abdominoperineal resection.
Significant risk factors for colonic carcinoma in an
Biopsy from a mole on foot shows cytologic atypia
C
C
adenomatous polyp are
of melanocytes and diffuse epidermal infiltration
by anaplastic cells, also present in the papillary and Villous histology size > 2 cm and atypia.
reticular dermis C A 50 years old male patient presents with painless
Melanoma Clark level IV terminal hematuria. Cystoscopic examination
revealed a solitary papillary tumor. Histopatho-
C Chronically lymphadenopathy limb is predisposed
logy of completely resected tumor is suggestive of
to
grade III transitional cell carcinoma with no muscle
Thickening of the skin, recurrent soft tissue infec-
invasion. Management
tions and sarcoma.
Intravesical BCG
C Initial treatment of rupture of varicose veins at the
C Unilateral undescended testis ideally operated
ankle should be
around
Direct pressure and elevation.
6 months
A 20 years old man suffering from pulmonary TB
Important causes of hyponatremia
C
C
presents with massive recurrent hemoptysis—
Gastric fistula, excessive vomiting and prolonged.
Vascular structure first evaluated for angiographic
Ryle’s tube aspiration.
treatment is
C Preservative used while packing catgut suture
Bronchial artery
Isopropyl alcohol
C CECT chest of an elderly male with carcinoma right
C Indication of radiotherapy in pleomorphic adenoma
lung shows a tumor of 5 × 5 cm in upper lobe and
of parotid
3 × 3 cm in middle lobe. Primary treatment
Second histological benign recurrence.
Surgery
C Thoracic extension of cervical goitre is usually
ALL INDIA 2003
approached through
Neck C Most characteristic of congenital hypertrophic
C Pancreatitis, pituitary tumor and Pheochromocy- pyloric stenosis
toma may be associated with The pyloric tumor is best felt during feeding.
Medullary carcinoma thyroid. C Marjolin’s ulcer is
(Hint: MEN syndrome) Malignant ulcer found on the scar of burn.
190 AIIMS and All India PGMEE—Review Questions

C Bed sore is an example of C Increased incidence of cholangiocarcinoma is seen


Trophic ulcer in
C Early stage of trauma is characterized by Polycystic disease of liver, sclerosing cholangitis
Catabolism and liver flukes.
C One unit of fresh blood arises Hb% concentration by C The Hunterian ligature operation is performed for
1 gm% Aneurysm
C A 10 mm calculus in right lower ureter associated C Pseudoclaudication is due to the compression of
with proximal hydronephrosis is best treated by Cauda equina
Ureteroscopic reversal. C Strong correlation with colorectal cancer is seen in
C The best treatment for cystic hygroma is Familial polyposis coli
Surgical excision C Sympathectomy is indicated in
C Laser used for treatment of benign prostatic Acrocyanosis, ischemic ulcers and intermittent
hyperplasia as well as urinary calculi claudication.
Ho: YAG laser C A 50 years old male hotel cook with four dependent
C Semen analysis of a young man with primary family members have early stage squamous cell
infertility shows low volume, fructose negative cancer of anal canal with more than 60% chances of
ejuclate with azoospermia. Most useful imaging cure. Best treatement
modality Chemoradiation
Transcrotal ultrasonography. C In postoperative ICU, five patients developed post-
C A 65 years old male smoker presents with gross operative wound infection on the same wound. The
total painless hematuria. Diagnosis best method to prevent cross infection is
Carcinoma of urinary bladder. Practice proper hand washing.
C A posteriorly perforating ulcer in the pyloric antum C During routine check up of a 70 years old male
of stomach is most likely to produce initial localized patient with history of chest pain diagnosed as
peritonitis or abscess formation in coronary artery disease, ultrasonography shows
Oriental bursa (lesser sac). gallbladder stones. There is no past history of
C A 25 years old man presents with left testicular biliary colic or jaundice. Best treatment advice
tumor with a 10 cm retroperitoneal lymph node No surgery for gallbladder stones.
mass. Treatment C The nerve commonly damaged during McBurney’s
Left high inguinal orchidectomy plus chemothe- incision is
rapy. Iliohypogastric
C Absolute indication for surgery in cases of BPH C A 70 years old patient with BHP underwent transure-
Bilateral hydroureteronephrosis. thral resection of prostate under spinal anesthesia.
C Most suggestive of neonatal small bowel obstruction One hour later, he developed vomiting and altered
Bilious vomiting sensorium. Cause is
C Conditions predisposing to pulmonary embolism Water intoxication
Protein S deficiency, obesity and malignancy. C Most appropriate surgery for a solitary nodule in
C Risk factors for deep vein thrombosis one lobe of thyroid
Duration of surgery more than 30 minutes, obesity Hemithyroidectomy
and use of estrogen progesterone pills C The most common cause of an obliterative stricture
C If a patient with Raynaud’s disease immersed his of membranous urethra is
hand in cold water, the hand will RTA with fracture pelvis and rupture urethra.
Turn white
ALL INDIA 2002
C Referred pain from ureteric colic is felt in the groin
due to involvement of C A patient suddenly experienced pain radiating
Genitofemoral nerve along the medial border of the dorsum of foot.
C Radiation exposure during infancy linked to Nerve most likely to be accidently ligated
Thyroid Saphenous nerve
Surgery 191

C In an adult patient with pleural effusion, most C Most common tumor of salivary gland
appropriate site for pleurocentesis done by Pleomorphic adenoma.
inserting a needle in C Premalignant condition with highest probability of
7th intercostal space in mid-axillary line. progression to malignancy is
C Measurement of intravascular pressure by a Erythroplakia
pulmonary artery catheter should be done C An edentulous old man develops squamous cell
At end expiration. carcinoma in buccal mucosa that has infiltrated to
C A 24 years old man falls on the ground when struck the alveolus. Following may be indicated in
in the right temple by a base ball. On way to treatment
hospital, lapses into coma. He is unresponsive with Radiotherapy, segmental mandibulectomy and
dilated right pupil at hospital emergency. Most marginal mandibulectomy involving removal of
important step in initial management upper half of mandible.
Cranitomy C Cork screw esophagus is seen in
C A 75 years old woman presents with post-myocardial Diffuse esophageal spasm.(Mn: CD)
infaction after 6 weeks with mild CHF. There is past C Achalasia cardia treatment associated with high
history of neck surgery for parathyroid adenoma rate of recurrence
5 years ago. EKG showing slow atrial fibrillation. Botulinum toxin
Serum Ca2+ 13 mg/dl and urinary Ca2+ 300 mg/24 h. C Barrett’s esophagus is
On examination-small mass in the paratracheal Lower esophagus lined by columner epithelium.
position behind the right clavicle. Appropriate C The adenocarcinoma of esophagus developed in
management
Barrett’s esophagus.
Ultrasound guided alcohol injection of the mass.
C The lowest recurrence of peptic ulcer is associated
C Feature of de Quervain’s disease with
Increased ESR, tends to regress spontaneously and Vagotomy + antrectomy.
painful and associated with enlargement of
C Risk factor for development of gastric carcinoma
thyroid.
Intestinal metaplasia III.
C A 35 years old woman with recurrent episodes of
headache and sweating, on examination-thyroid C Early gastric cancer indicates
nodule and ipsilateral large cervical lymph node Involvement of mucosa, submucosa with/without
and family history of renal calculi and thyroid lymph node.
cancer to mother. Investigations necessary before C Site of obstruction in gastric outlet obstruction in a
thyroid surgery peptic ulcer patient
Estimation of urinary metanephrines, VMA and Duodenum
catecholamines (Mn: CMV). (Hint: Misnomer).
C Associated with thyroid storm C The most suitable route for protein and calorie
Surgery for thyrotoxicosis, stressful illness in administration in a patient in coma for last 15 days
thyrotoxicosis and I131 therapy for thyrotoxicosis. following RTA
C Needle biopsy of solitary thyroid nodule in a young Jejunostomy tube feeding
woman with palpable cervical lymph node on the C An infant presents with acute intestinal obstruction,
same side demonstrating amyloid in stroma. contrast enema showing intussceptions. Likely
Diagnosis is cause is
Medullary carcinoma of thyroid. Payer’s patch hypertrophy.
C Treatment of choice of 25 years old woman C A patient develops single liver metastasis of 2 cm
presenting with palpable thyroid nodule, ipsilateral after undergoing operation for carcinoma colon.
palpable cervical lymph node and needle biopsy Next done
demonstrating amyloid in stroma Resection
Total thyroidectomy and modified radical neck C Sentinel lymph node is important in
dissection on the side of enlarged lymph node. Breast carcinoma
192 AIIMS and All India PGMEE—Review Questions

C Clinical feature of thromboangiitis obliterans C True about adrenal insufficiency


Raynaud’s phenomenon, claudication of extremities Acute adrenal insufficiency is usually secondary to
and absence of popliteal pulse. exogenous glucocorticoid administration acute
A 16 years old girl presents with non-pitting edema
adrenal insufficiency presents with weakness,
C
of recent onset affecting her right leg but no other
symptoms. True is vomiting fever and hypotension and hyponatremia
A lymphangiogram will show hypoplasia of the occurs because of impaired renal tubule sodium
lymphatics. reabsorption.
Obstetrics and Gynecology 193

13

Obstetrics and Gynecology

AIIMS NOVEMBER 2015 C 38 weeks pregnant patient with placenta previa


presents with bleeding PV, pulse 140/min, BP 80/
C Earliest change in puberty in girls 50 mmHg. Anaesthesia for the management of this
Onset of growth spurt. patient
(Note: Adrenarche was not in the option. GA with ketamine and succinylcholine.
Order of sequence- C A patient of Swyer syndrome can conceive by
Adrenarche IVF with donor ovum
Onset of growth spurt C Menopause is diagnosed by
Thelarche FSH > 40 Iu/L
Pubarche C Dose of misoprostol for the treatment of PPH
Peak height velocity 600 μg.
Menarche) C Anti Rh immunoglobulin is given at period of
C Modified biophysical profile gestation
Non stress test and amniotic fluid index. 28 weeks
C Shortest anteroposterior diameter of pelvis C Ulipristal acetate is
Selective progesterone receptor modulator.
Obstetric conjugate diameter
C Folic acid prophylaxis started
3 months before pregnancy AIIMS MAY 2015
C Third gravid with P2L2 with history of 1 thalassemic C According to WHO 2010 criteria, normal semen
baby now presents at 18 weeks period of gestation. should have parameters
Investigation done at this stage to exclude thalassemia 15 million/ml, 1.5 ml in volume, 4% normal
Amniocentesis morphology and 32% motility.
C Asherman syndrome most commonly presents C Supports of uterus are
with Perineal body, urogenital diaphragm and pelvic
Hypomenorrhoea diaphragm.
C Levonorgestrel should not be used for C Antihypertensive drug not given in pregnancy
Pre menstrual tension Enalapril
C In first 2 months of pregnancy, estrogen and C A 6 weeks pregnant woman is found to be HBsAg
progesterone are produced by positive but anti-HBC negative. Next investigation
Corpus luteum that should be done
C Most common cause for beaded fallopian tube and HBeAg
clubbing of fimbriae on HSG is C Antihypertensive drug of choice in pregnancy
Tuberculosis Methyldopa

193
194 AIIMS and All India PGMEE—Review Questions

C A young lady presents with nausea and vomiting. C Fistula test is conducted in a woman who underwent
She is 6 weeks amenorrhea and urine test for hysterectomy for carcinoma cervix by introducing
pregnancy is positive. BP is 100/60 and heart rate is a catheter into the bladder through the urethra and
120/min. USG shows adnexa of size 5 × 5 cm. Next vaginal cavity is packed with 3 sterile swabs. Dye is
best done injected and it is noticed that dye stains uppermost
Laparoscopy immediately swab. Most likely diagnosis
C A woman presents with bilateral ovarian mass with Ureteovaginal fistula
smooth surface. On histopathology, mucin secreting C A woman presents with BP–100/60, pulse–120/
cells are found with signet ring appearance min, 6 week amenorrhea, urinary pregnancy test
Krukenberg tumor positive. Next done
C Dose of dexamethasone given for fetal lung maturity Laparoscopy.
6 mg, 4 dose, 12 hours apart (Note: Suspected ruptured ectopic pregnancy)
C Dose of carbetocin in PPH C Level of proteinuria to diagnose severe pre-eclampsia
100 μg IV 2000 mg
(Note: Carbetocin is the long acting analogue oxytocin (Note: Old diagnostic criteria. According to new
introduced recently for prevention of uterine atony diagnostic criteria, proteinuria is not needed for
after cesarean section and to control PPH.) diagnosis of severe pre-eclampsia)
C A 10 years old girl presents with huge mobile mass C A 32 years old lady presents 4 days after unprotected
in pelvis that is present in hypogastric region also. sexual intercourse. Contraceptive advised
On examination, doctor is unable to insinuate the Copper IUCD
fingers between mass and pelvis. Most likely
diagnosis in her case is
Ovarian mass AIIMS NOVEMBER 2014
C Galactorrhea-amenorrhea syndrome-hormone that C A woman with antiphospholipid syndrome should
is tested in addition to prolactin is be treated with to prevent miscarriage
TSH. Aspirin and low dose heparin.
C A 16 years old girl, who plays football daily, presents C Expected date of delivery is arbitrary. True about
with primary amenorrhea and was treated with EDD
OCP in the past. On examination she has breast Less than 5% of woman deliver on the exact date
Tanner stage 5 and pubic hair Tanner stage 1. She which is 40 weeks of gestation, 50% of females
may have deliver within 1 week of EDD and 80% of females
Androgen insensitivity syndrome. deliver within 2 weeks of EDD.
C Two hour oral glucose tolerance test is done in a C True about augmentation of labor
pregnant woman. What are the cut off values Amniotomy enhances the progress of labor in the
Fasting 95 mg/dl, 1 hour 180 mg/dl and 2 hours 155 first stage and negates the need for oxytocin
mg/dl. administration, there is no increased risk of
C Drugs given in PCOS unfavourable maternal and neonatal adverse
OCP, metformin and clomiphene outcomes and intermittent auscultation is equivalent
C Dose of folic acid in a lady for prevention of neural to continuous electrical fetal monitoring when done
tube defects in subsequent babies when she has in a 1:1 doctor/nurse to patient ratio.
history of a baby with neural tube defects C Elective cesarean section even at term can lead to an
4000 μg increased incidence of medical complications and
C A lady presents with history of recurrent abortions adverse outcomes. The universal recommendation
(5 times) around 14–18 weeks. Investigations done for elective C.S. is at how many weeks
in current pregnancy are 39
Fetal karyotyping, anticardiolipin antibody and C Methods of abdominal tubal ligation
lupus anticoagulant. Irwing’s method, Pomeroy technique and Parkland
(Note: Syphilis does not cause recurrent abortion.) method.
Obstetrics and Gynecology 195

C If lifestyle factors are identified, the changes leading C Basis for sex chromatin testing
to improved chances of fertility are Barr body
Lose weight, gain weight and less or more exer- C Hysteroscopic myomectomy scores over open
cise. myomectomty in the following except
C A 40 years old male patient presents with primary Less bleeding
infertility, testes normal in shape but vas not
palpable, semen analysis showing azoospermia, AIIMS NOVEMBER 2013
low volume, low pH, high viscosity and high
liquefaction time. Most likely diagnosis C Seen in pregnancy with heart disease but not in
CFTR mutation. normal pregnancy
C Normal stature with minimal or absent pubertal Distended neck veins.
development is seen in C Complications more common in ventouse delivery
Kallmann syndrome than forceps
C First line treatment in the management of a 13 years Subgaleal hemorrhage, cephalohematoma and
old girl with abnormal uterine bleeding (anovulatory) intracranial hemorrhage.
are C Risk factors of cervical cancer
Mefenamic acid, tranexamic acid and estrogen + Multiple partners, early sexual partners and
progesterone pill. smoking.
C A 12 years old girl presents with history of child C Fertilized ovum reaches the uterine cavity by
abuse. She is bleeding profusely from the genitalia, 4–5 days
with severe perineal injuries and fractured pelvis. C A 16 years old girl presents with primary amenorr-
The most appropriate management hea, hirsutism, irregular bleeding and infertility
Urgent blood transfusion. and diagnosed as PCOS. Drugs that can be given
C The screening tests for breast and genital cancer in Spironolactone, OCPs and clomiphene citrate.
females include C Maximum cardic output seen in which week of
CA 125 for ovarian cancer, pap smear for cervical pregnancy among the following
smear and mammography for breast cancer. 32 weeks > 28 weeks.
C A pregnant woman presents with bleeding per
vaginum at 20 weeks of pregnancy. On examination
AIIMS MAY 2014 the os is open but no product has come out. Most
C Following indicate superimposed pre-eclampsia in likely diagnosis
a pregnant female of chronic hypertension Inevitable abortion.
New onset proteinuria, platelet count < 75,000 and C A 26 years old lady got pregnant for the first time,
fresh retinal hypertensive changes. LSCS was done for fetal distress. During preg-
C Antihypertensive drug not to be given in pregnancy nancy, mild headache was reported. 2 days after
Enalapril delivery, she presents with headache and seizure
but proteinuria is absent. CT scan is done and
C Administered in acute hypertension during labor
shows 2 × 3 cm parasagittal hematoma. Most likely
IV labetalol, IV nitroprusside and IV hydralazine. diagnosis
C Part of HELLP syndrome Sagittal sinus thrombosis.
Hemolysis, elevated liver enzymes and thrombocy- C If untreated, percentage of mother to child
topenia. transmission of HIV during delivery without
C Indications for elective cesarean section in a patient intervention in a non-breastfed child is
with past history of ceserean section 15–30%
Breech, macrosomia and post-term. C Radiation dose for early and locally advanced
C Responsible for pubertal growth in females cancer cervix at point A during brachytherapy
Pulsatile release of GnRH during sleep. 80–85 gray and 85–90 gray.
196 AIIMS and All India PGMEE—Review Questions

C True about latent phase of labor C Following constitutes the active management of
According to ACOG, it starts after 3–4 cm cervical third stage of labor for the prevention of post-
dilatation but they are planning to increase it to partum hemorrhage
5 cm, begins at the end of active phase and is a part Direct injection of oxytocin when the baby’s
of 1st stage of labor and starts with contraction of shoulders are delivered, constant controlled cord
labor. traction and misoprost.
C A 16 years old girl presents with primary amenorrhea C Perinatal transmission of HIV can be prevented by
and raised FSH level. Her height is 58 inches. Histo- Zidovudine administration to both the mother and
pathology of ovary shows baby, avoidance of breastfeeding and administra-
Absence of oocytes in the ovaries (streak ovaries). tion of vitamin A.
C Most accurate and safest method to diagnose viable C A young girl presents with short stature, widely
pregnancy at 6 weeks spaced nipples and amenorrhea. Genetic constitution
USG for fetal cardiac activity. 45 XO
C Most common site of primary for intraocular (Note: Turner syndrome)
metastasis is from
C The most accurate and safe method to diagnose
Breast
viable pregnancy at 6 weeks of gestation is
C Correct order of lochia
Ultrasound visualization of fetal cardiac activity.
Rubra, serosa and alba.
C True about fetal breathing in utero
C Tests used for screening of cancers in females-CA-
125- ovarian cancer, mammography Increases risk of amniotic fluid aspiration, causes
Breast cancer and pap smear-cervical cancer. conditioning of respiratory muscles and has a
higher risk of respiratory distress syndrome.
C Type of abnormality in sexual development with
best prognosis among the following C A 30 years old woman from poor background from
Congenital adrenal hyperplasia. hilly area presents with complaints of frequency,
dysuria, hematuria, loss of appetite, mild fever and
amenorrhea. The most likely cause
AIIMS MAY 2013
Genitourinary tuberculosis.
C True regarding time of ovulation
It occurs after ripening of follicles by FSH. AIIMS NOVEMBER 2012
C Patient develops shock just after a normal delivery.
C Feature seen in pregnancy with heart disease which
Most probable cause
is not seen in normal pregnancy
Uterine inversion
Distended neck veins.
C The test used to differentiate between maternal and
fetal blood in a given sample is C A 36 years old multigravida at 34 weeks, with
previous 2 normal vaginal delivery now presented
Apt test
with unstable lie. The most likely diagnosis in this
C A 19 years old patient presents with primary case is
amenorrhea, well developed breast and pubic hair
Placenta previa
but absence of vagina and on USG her uterus was
absent. Diagnosis C A young lady presents with complaints of copious
vaginal discharge, but there is no cervical discharge on
Müllerian agenesis
per speculum examination. Drugs for the treatment
C HRT can be used to provide benefit in post-
menopausal females for all of the conditions except Metronidazole and fluconazole.
Cardiovascular disease C Shock index (HR/SBP) is used to know the severity
C True about Duncan’s method of placental separation of the bleeding in cases of hemorrhage. Value most
indicative for significant PPH
There is peripheral separation of placenta, maternal
surface of placenta presents at the vulva and blood 0.9–1.1
collects between the placenta and fetal membranes C Treatment of carcinoma cervix stage 3b include
and escapes from the vagina. Concurrent chemoradiation.
Obstetrics and Gynecology 197

C A young female patient with 2 months amenorrhoea AIIMS MAY 2012


presents with sudden abdominal pain with adnexal
mass, urine pregnancy test is positive. Most likely C A 25 years old primigavida with severe rheumatic
diagnosis heart disease (mitral stenosis with mitral regurgi-
Ectopic pregnancy tation) is in early labor. Ideal intervention for labor
analgesia for normal labor
C Test useful in case of tubal pregnancy
Neuraxial analgesia
Pelvic examination, USG and hCG levels.
C A 24 years old primigravida at 32 weeks of gestation
Helpful in diagnosis in case of tubal mass in ectopic
experienced faintness and nausea in lying down
C
pregnancy
and recovers after turning on her side or getting up.
β hCG estimation, transvaginal ultrasound and These symptoms can be attributed to
pelvic examination. Inferior vena cava compression.
C A woman presents with 8 weeks amenorrhoea with C Test used to differentiate between maternal and
pain in lower abdomen. On USG, there was thick fetal blood
endometrium with mass in lateral adnexa. Most Apt test
probable diagnosis C Treatment of carcinoma cervix 3B include
Ectopic pregnancy Intracavity brachytherapy followed by external
C Least seen in uterine fibroid beam radiotherapy.
Sarcomatous change C Management of 55 years old woman with carcinoma
C Advice to the couple to prevent a child being born cervix , FIGO stage 2–3, locally advanced
with Down syndrome when one of the parent has a Radiotherapy plus chemotherapy.
balanced translocation between chromosomes 15 C Endometrial cancer involving > 50 % of myome-
and 21 trium, extending to vagina, no pelvic extension, no
Prenatal diagnosis and advice abortion. pre- and para-aortic lymph nodes and peritoneal
C A 24 years gravid female at 36 weeks gestation comes lavage positive. Stage is
with visual disturbance, headache and elevated BP 3b
of 180/110 mm Hg and 176/104 mm Hg in 2 readings C A 20 years old girl presents with average weight,
taken 20 minutes apart. Best management is oligomenorrhea, facial hair, raised free testosterone
Start antihypertensive, magnesium sulphate and levels and USG pelvis showing ovary normal
terminate pregnancy. morphology. Likely cause
C A young female patient presents with cyclical pain, PCOD
dysmenorrhea and complaints of infertility. C After a full term normal delivery patient went into
Investigation of choice shock. Most probable cause is
Inversion of uterus.
Diagnostic laparoscopy.
C The highest volume overload in a parturient due to
A lady with previous cesarean section presents in
maximum cardiac output is seen
C
labor. Trial of normal vaginal delivery is contrain-
Immediately after delivery.
dicated in
Previous classical cesarean section.
C Best agent for premenstrual syndrome management AIIMS NOVEMBER 2011
SSRI C A 20 years old female patient presents with an
C A 19 years old girl comes with complaints of primary ovarian mass 6 × 6 × 6 cm with solid structures in
amenorrhea. She had well developed breast and pubic mass on USG. AFP, β hCG and CA 125 normal but
hair, normal ovaries but absent vagina and uterus serum alkaline phosphatase increased. Diagnosis
Müllerian agenesis Dysgerminoma
C In a study it is observed that the right ovary ovulates C Ovaries function normally in
more than the left, possible explanations Rokitansky-Kuster-Hauser syndrome.
Anatomical asymmetry, difference in blood supply C Complications of hypertensive pregnancy in renal
to both sides and some embryological basis. failure are
198 AIIMS and All India PGMEE—Review Questions

Eclampsia, HELLP syndrome and renal failure. C Ideal contraceptive for a couple living in different
C Oral glucose tolerance test is performed to diagnose cities meeting occasionaly
gestational diabetes mellitus in cases of Barrier method
Congenital anomalies in fetus, polyhydramnios C A 36 weeks pregnant diabetic female with NST non-
and unexplained fetal loss. reactive Next done
C True for an elective forcep delivery Proceed to biophysical profile.
The fetal head should be at station ‘0’. C Drug used in pregnancy
Prophylthiouracil
AIIMS MAY 2011
AIIMS NOVEMBER 2010
C Seen in ovulatory phase
Stimulation of continuation of reduction division of C Treatment of carcinoma cervix III-B include
oocytes. Intracavitatory brachytherapy followed by external
C True about prolactinoma in pregnancy beam radiotherapy.
Most common pituitary tumor but rarely sympto- C Risk factors for vaginal candidiasis are
matic, regular visual check up and macroadenoma DM, HIV and pregnancy.
> 1 cm associated with bad prognosis. C The regimen used for expectant management of
C Best treatment for a 30 years old woman presenting placenta previa
with hypertension and menorrhagia Mac Afee and Johnson regimen.
MIRENA (LNG IUCD) C Sugar present normally in the urine of a pregnant
C Diagnosis of 35 years old woman presenting with woman in the third trimester
primary infertility and palpable pelvic mass, Ca- Glucose
125 level-90 u/ml diagnosis is C Classification for abruptio placenta
Endometrioma Pages and Sher classification.
C Ideal contraceptive for lactating mothers C Best regimen for eclampsia
Lactational amenorrhoea. MgSO4.
Test for ovarian reserve
Seen in corpus cancer syndrome
C
C
FSH
HTN, Obesity and DM, (Mn: HOD).
C True about clomiphene citrate
C Endometrial cancer involving 50% of endome-
Enclomiphere is antiestrogenic.
trium, extending to vagina, lymph nodes negative
C Seen in gestational diabetes with positive peritoneal cytology. Stage
Previous macrosomic baby, obesity and polyhy-
3b
dramnios.
A 30 years old mother has a boy with Down’s
Vaginal delivery can be allowed in
C
C
syndrome from previous pregnancy. She is now
Mentoanterior, extended breech and dichorionic
9 weeks pregnant. What would you advise to
twins with first vertex and second breech.
her
C Pregnant lady acquires chickenpox 3 days prior to
Chorionic villous biopsy at this stage will confirm
delivery. She delivers by normal vaginal route. True is
presence or absence of Down’s syndrome.
Baby will develop neonatal varicella syndrome.
C A 30 years old pregnant mother presents with C A 20 years old patient presents with primary
obstructed labor, febrile, dehydrated with IUFD amenorrhoea, well developed breasts and pubic
and cephalic presentation. Best management option hair, but absence of vagina and uterus. Diagnosis
Caesarean section. Müllerian agenesis
C Antiphospholipid antibody syndrome is associated C Test differentiating maternal and fetal blood cells
with Apt test
Antibody to lupus, recurrent abortion and venons C A 20 years old average weight girl presents with
thrombosis. oligomenorrhoea, facial hair, raised free testosterone
C Ideal contraceptive for newly married couple levels, USG- ovary normal. Diagnosis
Combined OCP. Adrenal hyperplasia.
Obstetrics and Gynecology 199

C Used in emergency contraception C True about MRKH


Oral LNG, oral mifepristone and Cu T IUD. Absent uterus, absent cervix and absent vagina.
C A 37 years old second gravida, previous LSCS at 37 C Most sensitive test for ectopic pregnancy
weeks of pregnancy presents with BP 150/100, Trans vaginal USG.
urinary albumin ++, PV-cervix soft, 50% effaced, C A pregnant woman, previous LSCS with hematuria.
station-3, pelvis adequate and cervical os closed. Diagnosis is
Most appropriate management Prolonged labor.
Do caesarean section C A 20 years girl presenting with primary ameno-
C A/C to a study, right ovary ovulates more than the rrhoea, absent breast and hypoplastic uterus.
left, possible explanations likely Diagnosis
Anatomic asymmetry, difference in blood supply to Turner’s syndrome.
both sides and embryological basis. C Investigation for a 35 years old female with post-
C Retinitis pigmentosa is a feature of coital bleeding
Abetalipoprotienemia, NARP and Refsum’s Pap smear with clinical examination.
disease. C Management of a patient with occipito posterior
position
AIIMS MAY 2010 Wait and watch for progress of labor.
C A women with placenta previa delivered a baby. C A pregnant patient with prosthetic valve should be
She had excessive bleeding and shock. Most likely switched to heparin in
complication after resuscitation 36 weeks.
Loss of menstruation C Call exner bodies are seen in
C Best treatment for a 35 years old woman P3 to having Granulosa cell tumor.
CIN grade II on colposcopic biopsy C Uterine blood flow at term is
LEEP 500–750 ml/min.
C A women undergoes total radical hysterectomy for C Drugs interfering with effectiveness of OCP
stage Ib carcinoma carvix. It was observed that Phenytoin, rifampicin and tetracycline.
carcinoma extends to lower part of body of uterus C Tocolytic of choice in heart disease
and upper part of cervix. Next done Atosiban.
Follow-up
C Cholestasis of pregnancy should be terminated at
gestational age AIIMS MAY 2009
38 weeks C MRKH syndrome, true
C Pain of ovarian carcinoma is referred to Uterus absent, fallopion tubes ovaries present.
Medial surface of thigh. C Transabdominal CVS can be done in
11–13 weeks
AIIMS NOVEMBER 2009 C DOC for a woman presenting with creamy white
C Progesterone of choice in emergency contracep- vaginal discharge with fishy odour
tion Metronidazole
Levonorgestrone. (Hint: Bacterial vaginosis).
C Pre labor pains are mediated through C True about vasa previa
T11–12. Incidence is 1:1500, associated with low lying
(Note: Labour pains are mediated through T10–L1) placenta and cesarean section indicated.
C Treatment of genital warts in pregnancy C True about congenital diseases in DM
Cryotherapy Results due to free radical injury, 6–10% cases are
C HPV vaccine is associated with major congenital abnormality and
Both bivalent and quadrivalent. insulin can be given.
200 AIIMS and All India PGMEE—Review Questions

C True about clomiphene citrate C True about AFP


Enclomiphene has antiestrogenic effect, risk of Half-life 5–7 days.
multiple pregnancy is 6–10% and it can also be used C True about diabetes in pregnancy
for male infertility with oligozoospermia. Glucose challenge test is done between 24–28
C Indications for cesarean section in pregnancy are weeks, 50 gm of sugar given as screening test and
MR, AR and AS diabetic control before conception is important to
C Normal pregnancy can be continued in case of prevent malformation.
Wolf- Parkinson C Marker for granulosa cell tumor
White syndrome Inhibin.

AIIMS NOVEMBER 2008 AIIMS NOVEMBER 2007


C Sudden hyperflexion of thigh over abdomen C Complication you would expect during internal
(McRobert’s procedure) commonly involves podalic version in case of transverse lie
Lateral cutaneous nerve of thigh. Uterine rupture
C Management of a 25 years old married nullipara C MIRENA is
undergoing laparoscopic cystectomy for ovarian Progesterone IUCD
cyst which on histopathology reveals ovarian C Side effect of clomiphene citrate includes
serous. Cisadenocarcinoma Multiple pregnancy, increased risk of ovarian
Unilateral salpingo-ophorectomy. carcinoma and multiple polycystic ovary.
C Vasopressor of choice in pregnancy C Drug given to reduce the uterine contraction during
Ephedrine preterm labor with least side effect
C Investigations used for FIGO staging of carcioma Progesterone
cervix C Drug contraindicated in patient with rheumatic
IVP, cystoscopy and proctoscopy. heart disease in PPH
C True about PCOD Methylergo metrine
Persistently elevated LH, increased LH/FSH ratio C Indication for radiotherapy in endometrial
and increased dheas. carcinoma includes
C A woman presents with history of 6 weeks ameno- Pelvic node involvement, deep myometrial
rrhoea, USG-no sac, serum beta hCG 1000 IU/L. involvement and poor differentiation.
Further management
C Patient with squamous cell intraepithelial lesion
Repeat hCG after 48 hours. with highest risk for progression to carcinona
High grade squamous intraepithelial neoplasia.
AIIMS MAY 2008
C Polyhydramnois associated with
C Decrease in incidence of neural tube defect by Anencephaly, open spina bifida and tracheoeso-
Preconceptional intake of folic acid. phageal fistula.
C Most commonly implicated in genital (vulval) C Increased AFP level seen in
warts Congenital nephrotic syndrome.
HPV 6. C Essential criteria according to WHO for normal
C Fetal death in ectopic pregnancy is caused by semen analysis
Vascular accident. Sperm count > 15 million/ml, sperm with normal
C A woman with ovary carcinoma in follow up has morphology (Strict criteria) > 15% and motility >
raised CA125 level. Next done is 32% with rapidly progressive motility.
PET C Highest maternal mortality seen among congenital
C Blood in urine in a patient in labor. Diagnosis is heart disease in
Obstructed labor Eisenmenger’s syndrome
C True about androgen insensitivity syndrome C G3 with previous second trimester abortion
Phenotype may be completely female. presents with 22 weeks of gestation, abdominal
Obstetrics and Gynecology 201

pain, USG-funneling of internal os. Ideal manage- AIIMS NOVEMBER 2006


ment
McDonald stitch C During pregnancy HIV transmission occurs mostly
C Cervical smear fixation done by during
Ethyl alcohol Labour
Drug given to prevent HIV transmission from
Asherman’s syndrome diagnosed by
C
C
mother to child
Hysterosalpingography, hysteroscopy and saline
Nevirapine
sonography.
After coming head of breech have difficulty in
Goniometer is used for
C
C
delivery in conditions
Urethrovesical angle
Extension of head, hydrocephalus and incomplete
dilatation of cervix.
AIIMS MAY 2007
C The most common congenital anomaly in baby
C Birth weight of a baby increased by born to IDDM mother is
Cessation of smoking. Cardiovascular anomalies.
C A woman with recurrent abortion and isolated C Lady with infertility with bilateral tubal block at
prolonged APTT is most likely associated with cornua. Best method of management
Lupus anticoagulant. Laparoscopy and hysteroscopy.
C HRT is useful in C Lactational amenorrhoea is due to
Flushing, osteoporosis and vaginal atrophy. Prolactin induced inhibition of GnRH.
C Endodermal sinus tumor is associated with C Late hyperglycemia in pregnancy is associated with
Schiller duval body. Macrosomia
C Bilateral ovarian tumor C Antitubercular drug given in pregnancy
Dysgerminoma Ethambutol, INH and rifampicin.
C A 9 months pregnant lady presents with jaundice, C Red degeneration most commonly occurs in
distension, pedal oedema after delivering normal
2nd trimester.
baby. Clinical condition worsening with increasing
C Physiological changes in pregnancy are
abdominal distension and severe ascites. Lab
finding-Bilirubin 5 mg/dl, serum alkaline Increase in intravascular volume, increase in
phosphatase-450 IU/L and ALT-345 IU/dl with cardiac output and increase in stroke volume.
tender hepatomegaly 5 cm below costal margin and C Pap smear showing carcinoma in situ–Next best done
ascitic fluid showing protein less than 2 mg%. Colposcopy and biopsy
Diagnosis
Budd Chiari syndrome. AIIMS MAY 2006
C Nile blue sulphatase test in amniotic fluid is for
C The risk of endometrial carcinoma is highest with
Skin immaturity
histological pattern of endometrial hyperplasia
C Most common cause of postmenopausal bleeding
Complex hyperplasia with atypia.
in women in India
CA 125 is a marker antigen for diagnosis of
Cancer cervix
C

C The most common site of ligation in female sterili- Ovarian carcinoma


zation C Risk of Asherman syndrome is highest if D and C is
Isthmus done for
C Amniotic fluid contains acetyl cholinesterase PPH
enzyme. Diagnosis C Highest maternal mortality in type of viral hepatitis
Open spina bifida infection in pregnancy
C Premature baby of 34 weeks was delivered. Baby Hepatitis E
had bullous lesion on the body. X-ray showing C Pseudomyxoma peritoni occurs as a complication
periostitis. Next investigation of ovarian tumor
VDRL for mother and baby Mucinous cyst adenoma.
202 AIIMS and All India PGMEE—Review Questions

C Parameters in the Manning scoring system of C The most important indication for surgical repair of
biophysical profile for fetal monitoring are a bicornuate uterus is
Fetal tone, fetal gross body movements and Habitual abortion
nonstress test C Treatment recommended to reduce the hereditary
C A 27 years old primigravida presents with PIH with risk for ovarian cancer in woman with BRCA I and
BP 150/100 mm Hg at 32 weeks of gestation with no BRCA II mutation
other complication. If BP is controlled on treatment Prophylactic oophorectomy
and no other complication developed, pregnancy C Antihypertensive safe in pregnancy
should be best terminated at
α methyldopa, amlodipine and clonidine.
37 completed weeks
C A 13 years old girl presents with acute pain in the
lower abdomen with history of cycical pain for last AIIMS MAY 2005
6 months and not attaining menarche yet. On local C Biochemical markers of triple tests are
genital examination, a tense bulge in the region of β hCG, unconjugated estriol and AFP.
hymen was seen. Diagnosis C Causes of DIC during pregnancy
Imperforate hymen Abruptio placenta, amniotic fluid embolism and
C The shortest diameter in fetal head Intrauterine death.
Bitemporal diameter C At 28 weeks gestation, amniocentesis reveals a delta
C Embryo implantation occurs after how many days OD of 0.20 that is at the top of third zone of liley
of ovulation curve. The most appropriate management
7–9 days Repeat amniocentesis after 1 week.
C hCG is secreted by C Characteristics of an ideal candidate for copper-T
Trophoblast cells insertion includes
C Appropriate material for antenatal diagnosis of Has normal menstrual periods, has borne at least
genetic disorders includes one child and is willing to check IUD tail.
Fetal blood, amniotic fluid and chorionic villi. C During laparoscopy the preferred site for obtaining
C Estrogen administration in a menopausal woman cultures in a patient with acute PID is
increases the Fallopian tubes
Bone mass C Absolute contraindication of OCP in a young
C The sensitivity of uterine masculature is female of reproductive age group
Enhanced by estrogen and inhibited by progeste- Impaired liver function
rone. (Mn: e for e). C In a young female of reproductive age with regular
C In those mammals, seasonal breader, females are cycles of 28 days, ovulation occurs around 14th day
receptive only once a year. The cycle is of cycle. First polar body is extruded
Estrous Accompanied by ovulation.
C In the perspective of the busy life schedule in the
AIIMS NOVEMBER 2005 modern society, the accepted minimum period of
sexual cohabitation resulting in no offspring for a
C Markers for malignant germ cell tumors of ovary couple to be declared infertile is
AFP, β-hCG and LDH. One year
C Highest level of MSAFP is found in C The best time to do chorionic villous sampling
Gastroschisis 11–13 weeks
C Measurement of fetal maturity not affected by a C Indications for postoperative radiotherapy in
bloody tap during amniocentesis carcinoma endometrium
Phosphatidylglycerol Endocervical involvement, myometrial invasion
C DOC of typhoid fever in pregnancy of more than half thickness and positive lymph
Ceftriaxone nodes.
Obstetrics and Gynecology 203

C Risk of complex hyperplasia of endometrium with AIIMS NOVEMBER 2003


atypia progressing to malignancy in a post-
menopausal woman is about 28%. C A non-stress test is considered reactive when
(Hint: 1, 3, 8 and 28). Two fetal heart rate accelerations are noted in 20
minutes.
C Use of OCP decreases the incidence of
C Most common cause of primary amenorrhoea
Ectopic pregnancy, epithelial ovarian malignancy
and PID. Ovarian dysgenesis
Drugs indicated in ectopic pregnancy
Drug contraindicated in hypertension with
C
C
pregnancy Actinomycin D, methotrexate and potassium
chloride.
Enalapril
C Risk factors for development of ovarian carcinoma
Family history of ovarian carcinoma, use of clomi-
AIIMS NOVEMBER 2004
phene and BRCA-1 positive individual.
C Test normal in DIC C A pregnant woman with fibroid uterus develops
Clot solubility acute pain abdomen, low grade fever, mild leuco-
cytosis at 28 weeks. Diagnosis
AIIMS MAY 2004 Red degeneration of fibroid.
C Formula for calculation of Pearl Index
Drugs used in emergency contraception
No. of accidental pregnancies × 1200/No. of patients
C
LNG, danazol and mifepristone. observed X months of use.
C After unprotected intercourse, emergency contra- C Polyhydromiosis in a pregnant woman is likely to
ceptives are effective if administered within be associated with
72 hours Esophageal atresia, micro anencephaly and
C Minimum effective dose of ethinyl estradiol in twinning.
combined OCP C Folic acid administration to prevent congenital
20 μg malformation is best initiated
C The regeneration of endometrium takes place at Before conception
Zona basalis C Features on second trimester ultrasound as a
C Pelvic structures supporting the vagina marker of Down syndrome are
Pelvic diaphragm, levator ani muscle and perineal Duodenal atresia, diaphragmatic hernia and single
body. umbilical artery.
C A patient of GTN presents with lung metastasis. Stage C Associated with chorioamnionitis
III Abruptio placenta, endometritis and preterm
C Indications for myomectomy in a case of fibroid labor.
uterus C Apoptosis can occur by change in hormone levels in
Associated infertility, pressure symptoms and the ovarian cycle. When there is no fertilization of
recurrent pregnancy loss. ovum, endometrial cells die becuase
C Used for induction of labor The involution of corpus luteum causes estradiol
PGE1 tablet, PGE2 gel and misoprostol. and progesterone levels to fall dramatically.
C Medical treatment of ectopic pregnancy is contrain- C OCP used is associated with an increased incidence
dicated in of
Presence of fetal heart activity Chlamydial endocevicitis
C DOC for bacterial vaginosis during pregnancy
Metronidazole
AIIMS MAY 2003
C Conservative management is contraindicated in a
case of placenta previa in situations C Low dose progestational contraceptives primarily
Evidence of fetal distress, fetal malformation and act on
women in labor. Cervix
204 AIIMS and All India PGMEE—Review Questions

C Most common cause of breech presentation C A 20 years old woman presents with BP 70/40 mm
Prematurity Hg, PR 120/min and positive urine pregnancy test.
C According to WHO criterian, minimum normal Managed by
sperm count Immediate laparotomy
15 millions/ml (Hint: Ruptured ectopic pregnancy).
(Note: According to WHO 2010 criteria) C The production of cervical mucus is stimulated by
C Known side effects of tocolytic therapy Estradiol
Hypotension, hyperglycemia and tachycardia. C Risk factors for postoperative infection after hyste-
C Drugs used for medical abortion rectomy
Mifepristone, misoprostol and methotrexate. Surgery for malignancy, urinary catheterization > 7
C True about ventouse (Vaccum extractor) days and use of blood transfusion.
Minor scalp abrasions and subgaleal hematomas in C Investigation required in a case of recurrent
newborn are more frequent than forceps. spontaneous abortion
C A 20 years old primigravida presents with painless Thyroid function tests, hysteroscopy and tests for
APH at 38 weeks of gestation. O/E-uterus soft and APLA.
non-tender, head engaged. Appropriate management C High risk factors for development of choriocarci-
is noma in a case of vesicular mole
Pelvic examination in OT Serum hCG levels> 100000 miu/ml, features of
C The most common cause of occipitoposterior thyrotoxicosis and presence of bilateral theca lutin
position of fetal head during labor cysts of ovary.
Android pelvis
C Most common congenital anomaly seen in preg-
nancy with DM is AIIMS MAY 2002
Neural tube defects C A pregnant lady develops chickenpox. Highest
C A drop in fetal heart rate typically lasting less than chance of neonatal infection is in
2 minutes and usually associated with umbilical Last 5 days
cord compression is called C Twin discordance is defined as difference in two
Variable declaration twins
C Renal disorder associated with worst pregnancy 25% with the larger twin as index.
outcome
C CVS done before 10 weeks may result in
Scleroderma
C A perimenopausal woman with well differentiated Oromandibular limb defects.
adenocarcinoma of uterus has more than half of C True about red degeneration of fibroid
myometrial invasion, vaginal metastasis and Occurs commonly during pregnancy, due to
inguinal lymph nodes metastasis. Staging is interference with blood supply and treated with
IV B analgesics.
C A woman has 2 children and presents with galac-
AIIMS NOVEMBER 2002 torrhoea and amenorrhoea for 1 year. Diagnosis
C Use of LNG releasing IUCD in helpful/preventive in Sheehan’s syndrome.
Dysmenorrhoea, menorrhagia and PID. C Pap smear is useful in diagnosis of
C Use of OCP are known to protect against HPV, inflammatory changes and Trichomonas
Ovarian carcinoma, endometrial carcinoma and vaginalis.
uterine sarcoma. C A woman presenting with 2nd stage of labor with
C Best parameter for fetal age estimation by USG in PV finding is anterior fontanelle and supraorbital
third trimester is ridge felt. The presentation is
Femur length Brow presentation.
C Most common causes of VVF in India is C Used as post coital contraception
Obstructed labor. Danazol, ethinyl estradiol and LNG.
Obstetrics and Gynecology 205

C Sterilization procedure with maximum chance of C A 55 years old female patient presents with
reversal abdominal pain, distension, ascites and dyspnea,
Laparoscopic tubal ligation with clips. CA 125 levels elevated. Diagnosis
C Used in treament of PPH Carcinoma ovary
Misoprostol, carboprost and methyl ergometrine. C True about mother to child transmission of HIV
C Estrogen replacement for postmenopausal sympton Risk of infection decreases if elective caesarean
causes an increase in section is done, HIV may be transmitted to fetus
Triglycerides during gestation and risk of infection in fetus
increases as duration of infection in mother
C Danazol is used in treatment of
increases.
Cyclical mastalgia
C A pregnant mother presents with fever, Hb 7
C Acetic acid staining of cervix shows
gm%, TLC normal, platelet decreased and frag-
Squamous meteplasia, cervical carcinoma in situ
mented RBC (shistocytes). Differential diagnosis
and cervical dysplasia.
are
(Note: Acetic acid staining is done in colposcopsy).
DIC, TTP and HELLP syndrome.
C Associated with gynaecomastia
ALL INDIA 2012
Estrogen secreting tumor, hCG secreting tumor and
C Seen in a normal pregnancy TSH secreting adenoma.
Increase in blood volume, increase in cardiac C True about twins delivery with preterm labor
output and increase in heart rate. Second has more chance of polycythemia.
C A woman has just given birth to a normal baby. The C True about GFR in pre- eclampsia
most appropriate time for starting Kegel’s exercises is It decreases
She should have started in 3rd trimester itself.
IUCD not requiring to be changed after 3–5 years is
ALL INDIA 2011
C
CuT 380A.
(Note: 10 years) C Factors leading to weight gain in pregnancy
C True about HBsAg positive pregnant woman- Ethnicity, pre-pregnancy weight and socio-
HBsAg Ig should be given to baby within 12 economic status.
hours. C Congenital malformation of fetus diagnosed
C A young male patient presents with delayed earliest in a first trimester ultrasound
puberty with decreased LH, FSH and testosterone. Anencephaly
Differential diagnosis are C Most specific marker for neural tube defects
Kallman syndrome, constitutional delay and Dax 1 Acetylcholinesterase
gene defect. C Gold standard test for assessing β-hCG in maternal
C Pregnant woman G3P1A1 on full term pregnancy serum
presents. On examination- Uterine contractions 2 Radioimmunoassay
per 10 minutes, lasting 30–35 seconds, cervix 4 cm C Included in expectant management of placenta
dilated, membranes intact and 3/5th of head previa
palpable per abdomen. On repeat examinations 4 Blood transfusion, corticosteroids and anti-D
hours later, cervix 5 cm dilated and cervicograph administration.
remain to the right of alert line. True is C A lady presents at 37 weeks of gestation with
On repeat examination, her cervicograph should uterine contraction and pain suggestive of labor for
have touched the action line. 10 hrs. O/E-cervix is persistently 1 cm dilated and
C LH surge occurs due to uneffaced. Next line of treatment
Markedly increased estrogen level. Sedation and wait
C Germ cell tumor of the ovary are C Maneuvers used in shoulder dystocia
Choriocarcinoma, dysgerminoma and endodermal Macroberts maneuver, suprapubic pressure and
sinus tumor. woods corkscrew maneuver.
206 AIIMS and All India PGMEE—Review Questions

C Drug contraindicated in conduct of labor in a C Motility of fallopian tube is impaired in


woman with rheumatic heart disease Kartagener’s syndrome.
Methylergometrine C True about obstructive azoospermia
C Earliest sign of magnesium toxicity in a patient with Normal FSH and normal LH.
eclampsia on treatment with magnesium sulphate C A woman presents to GOPD with history of stress
Loss of deep tendon reflexes. incontinence. TOC for genuine stress incontinence
C Best marker for intrahepatic cholestasis of pregnancy Tension free vaginal taping.
Bile acids C Blood chimerism is most likely to occur in
(Note: IOC) Monochorionic dizygotic twins.
C Drugs that can be given to mother with lupus after
35th week of gestation ALL INDIA 2010
Chloroquine, sulphadiazine/sulphasalazine and
Method best predicting the timing of ovulation for
prenisolone.
C
assessment of female fertility during a menstrual cycle
C Treatment for a primigravida in first trimester
Hormonal study
sputum positive for AFB but no previous history of TB
C Primary amenorrhoea with absent uterus, normal
Category I DOTS.
breasts and scant pubic hair is seen in
C Virus least likely to cross placenta
Androgen insensitivity syndrome.
HSV
C Condition presenting with absence of both Müllerian
C Clue cells are seen in and Wolffian duct structures
Bacterial vaginosis. (Mn: BC) Androgen insensitivity syndrome.
C Causes of primary amenorrhoea C A woman presented with secondary amenorrhoea
Turner’s syndrome, Rokitansky syndrome and 6 months after having an abortion with FSH level-6
Kallman’s syndrome. U/ml. Diagnosis
C Most common cause of a female with 45 XO geno- Uterine synechiae
type with primary amenorrhoea C Microbial agent most commonly associated with
Gonadal dysgenesis. carcinoma cervix
C Treatment for menorrhagia supported by evidence HPV 16
Combined OCP, progesterone and tranexamic acid. C Among gynaecological malignancies, sentinel
C A 45 years old post-menopausal woman presents lymph node biopsy is most useful for
with vaginal bleeding. TVS reveals endometrial Carcinoma vulva
thickness of 8.0 mm. Next step in management. C A 40 years old woman presents with abnormal
Histopathological examination of curettage. cervical cytology on PAP smear suggestive of CIN
C Mifepristone may be used for III (HSIL). Next best done
Ectopic pregnancy, fibroids and molar pregnancy. Colposcopy and LEEP.
C True about PCOD C A 50 years old woman presents with constant
High LH, high LH/FSH ratio and high androgens. leakage of urine and dysuria two weeks after a
C Drugs used in treatment of hot flushes complicated total abdominal hysterectomy. Most
important test for diagnosis of suspected VVF
Gabapentin, paroxetine and venlafaxine.
Triple swab test (After confirmation of VVF, cysto-
C Progesterone preferred in combination with scopy is most important for management).
estrogen in low dose OCP
C True about partial mole
Desogesterel Usually associated with triploidy, rarely causes
C Methods of contraception avoided in women with persistent GTN and usually present as missed
epilepsy abortions.
OCP C Conversion of complete hydatiform mole into
C True about Chlamydia trachomatis choricarcinoma is indicated by
Use of OCP is associated with increased risk of Plateau hCG levels, enlarged uterine size and
asymptomatic Chlamydial infection. persistence of theca lutein cysts.
Obstetrics and Gynecology 207

C Associated with PCOD C Surgical staging of a patient diagnosed with ovarian


Insulin resistance, ovarian carcinoma and endome- cancer and planned for staging laparotomy involves
trial Ca. Peritoneal washing, omental resection and
C AFP is raised in peritoneal biopsy (Mn: Peritoneal POP).
Yolk sac tumor C Tumor marker raised in case of dysgerminoma of
C Increased nuchal translucency at 14 weeks gestation ovary
is suggestive of Serum LDH
Down’s syndrome. (Note: and PLAP also).
C Non-immune hydrops fetalis is caused by C True about DMPA
Parvovirus Failure rate 0.3 HWY, reduce menorrhagia and can
C True about multiple pregnancies be used in seizures.
Twin peak sign is seen in dichorionicity. C Most effective contraceptive method recommended
C TOC for intrahepatic cholestesis in pregnancy is in lactating mothers
Ursodiol. (Urso deoxycholic acid). Progesterone only pill
C Cardiac C/I to pregnancy C Contraindication of laparoscopic sterilization
Eisenmenger’s syndrome, pulmonary HTN and Cardiac failure
Coarctation of aorta. C Mifepristone used in
C DOC to treat Chlamydia infection in pregnancy Ectopic pregnancy, fibroids and molar pregnancy.
Azithromycin > erythromycin C Features that may be seen in severe pre-eclampsia/
C Condition associated with polyhydramnios eclampsia
Cleft palate ARF, cerebral hemorrhage and pulmonary oedema.
C A woman presented with features of threatened
C Minimum duration between onset of symptoms
abortion at 32 weeks of pregnancy. True about
and death is seen in
antibiotic usage
PPH
Metronidazole if asymptomatic but significant
bacterial vaginosis, antibiotics if asymptomatic but C A primi patient in obstructed labor with negleted
significant bacteremia and antibiotics for pre-term shoulder presentation, showing signs of exhaustion
premature rupture of membranes. and dead fetus. Managed by
C A woman presents with leakage of fluid per Cesarean section
vaginum and meconium stained liquor of 34 weeks C Vaginal delivery recommended in
of gestation. Most likely organism causing infection First twin vertex and second twin breech.
Listeria monocytogenes.
C Interventions recommended in active management ALL INDIA 2008
of third stage of labor
Administration of uterotonic within 1 minute of C First sign of puberty in girls
delivery, controlled cord traction and gentle Breast budding (Thelarche).
massage of uterus. C True about calender method (rhythm method)
C Interventions recommended to prevent mother to Associated with no costs, safe period can also be
child transmission of HIV observed using temperature rhythm or mucous
HAART, elective cesarean section and intrapartum method and ectopic pregnancy is a reported
zidovudine and vitamin A prophylaxis to mother. complication of calender method.
C Least failure in sterilization occurs with
ALL INDIA 2009 Unipolar cautery
C Creamy fishy odor is characteristically seen in (Note: Also most complications).
Gardnella C Method associated with maximum chance of re-
C The recommended non-surgical treatment of stress canalization during surgery for reversal of tubal
incontinence ligation
Pelvic floor muscle exercises. Inshmus-isthmic anastomosis.
208 AIIMS and All India PGMEE—Review Questions

C Treatment showing best treatment for premens- C Dietary supplement recommended for a pregnant
trual syndrome lady on Heparin
SSRI Calcium
C TOC of a fluctuant non-tender swelling of the C True about breast infection during lactation
introitus of a woman is Can lead to abscess formation for which I and D
Marsupialisation may be required.
(Hint: Bartholin’s cyst). C Congenital infection associated with minimal
C Salpingitis or endosalpingitis is best confirmed by teratogenic risk to the fetus is
Hysteroscopy + laparoscopy HIV
C True about androgen insensitivity syndrome C A pregnant mother presents with history of
XY genotype, short vagina present and absent delivery of a previous child with CAH. The best
ovaries. management protocol for current pregnancy is
C Virus associated with cervical Ca-HPV. To start dexamethasone as soon as pregnancy is
C True about squamous cell carcinoma of cervix confirmed.
Common at squamocolumnar junction, post coital
bleeding is a common symptom and HPV 16 and 18 ALL INDIA 2007
are associated with high risk of carcinogenesis. C Shape of Nulli parous cervix is
C Ideal treatment for 55 years old woman with simple Circular
hyperplasia of endometrium with atypia C Fibroid uterus may present with
Simple hysterectomy Pelvic mass, infertility and polymenorrhoea.
C Condition diagnosed by chorionic villous biopsy C Most common gestational trophoblastic disease
Down’s syndrome, phenylketonuria and sickle cell following hydatidiform mole is
anemia. Invasive mole
C True regarding Doppler studies of the umbilical C Most common ovarian cyst to undergo torsion is
circulation Bengin cystic teratoma
Maternal smoking leads to increase in S/D ratio, C Non-contraceptive benefits of OCP seen in
normal pregnancy shows a characteristic decrease RA, endometriosis and endometrial carcinoma.
in S/D ratio with advancing gestational age and C True about IUD
changes in flow velocity wave forms of the
LNG releasing IUD has effective life of 5 years, IUD
umbilical artery may be important in clinical
can be used for emergency contraception within
management of high risk pregnancies.
5 days and the pregnancy rate of lippes loop and
C Known causes of recurrent abortions are CuT 200 are similar.
SLE, Rh incompatability and syphilis.
C IUCD with life span of 10 years
Induction at term is indicated in
CuT 380A
C
HTN, DM and renal disease
C An antihormonal substance used to induce
C Contraindication of vaginal delivery after previous ovulation
CS
Clomiphene citrate
Previous classical CS, no vaginal delivery in the
C Aspiration of sperms from testis done in
past and puerperal infection in previous pregnancy.
TESA
C Risk factors for placenta accreta includes
Maximum cardiac output during pregnancy is seen
Previous LSCS scar, previous curettage and previous
C
in
myomectomy.
C Tocolytic agents are Immediate postpartum period
Salbutamol, isoxsuprine and ritodrine. C Dexamethasone injection is given in a lady of 32
C Drugs useful in management of PPH weeks pregnancy to prevent
Ergometrine, misoprostol and oxytocin. RDS
C DOC for severe pre-eclampsia C Bishops score includes
Labetalol Dilatation, effacement and station of presenting part.
Obstetrics and Gynecology 209

C On TVS, shape of cervix indicating imminent C Most common cause of tubal block in India
preterm labor Tuberculosis
U C MOA of OCP are
C Doppler finding in IUGR associated with worst Inhibition of ovulation, prevention of fertilization
prognosis and interference with implantation of fertilized
Reversal of diastolic flow ovum.
C External cephalic version is C/I in C True about LNG releasing intrauterine system
PIH Can be used as hormone replacement therapy,
C Caudal regression syndrome is seen in babies of useful for treatment of endometrial hyperplasia
mothers having and irregular uterine bleeding can be problem
Gestational diabetes initially.
C Large baby is born. Likely complication in preg- C Mechanisms accounting for reduced risk of upper
nancy associated with it is genital fact infections in users of progestin releasing
Gestational diabetes IUDs
C Hypothyroidism in pregnancy is associated with Reduced retrograde menstruation, thickened cervical
PIH, prematurity and recurrent abortions. mucus and decidual changes in the endometrium.
C Antiepileptic associated with congenital malforma- C Emergency contraception prevents pregnancy by
tion when used in pregnant woman are Inhibition of ovulation, prevention of fertilization
Phenytion, carbamazepine and valproate. and interference with implantation of fertilized
C TOC for ovarian cyst in a postpartum period ovum.
Immediate removal C Prenatal diagnosis of 16 weeks of pregnancy can be
C Highest transmission of hepatitis B from mother to performed using
fetus occurs if the mother is infected during Amniotic fluid, chorionic villi and maternal blood.
IIIrd trimester C The best way of diagnosing Trisomy 21 during 2nd
trimester of pregnancy
ALL INDIA 2006 Amniocentesis
(Note: IOC)
C Hormone raised in PCOD
Tuberculosis flares up most commonly in a
Luteinizing hormone
C
pregnant woman in
C True about PCOD
Puerperium
Increased LH, can cause infertility and may be
associated with abnormal glucose tolerance test. C Tumors commonly known to increase in pregnancy
C Treatment for virilizing adrenal hyperplasia Pituitary adenoma, meningioma and neurofibroma.
Cortisone C Vitamin deficiency most commonly seen in a
C Among genital tract malignancy, risk of metastasis pregnant mother on phenytion therapy for epilepsy
for ovary is least in Folic acid
Carcinoma cervix. C A primigravida presents to casuality at 32 weeks
C IOC in a 55 years postmenopausal woman presenting gestation with acute pain abdomen for 2 hours,
with postmemopausal bleeding vaginal bleeding and decreased fetal movements
Fractional curettage should be managed by
C Surgery with highest incidence of ureteric injury Immediate cesarean section
Wertheim’s hysterectomy C Complications during pregnancy increasing the
C Microscopic findings in a case of bacterial vaginosis risk of PPH
in 40 years old woman presenting with complaints Hydramnios, macrosomia and twin pregnancy.
of profuse vaginal discharge but no discharge from C Indicators of impending uterine rupture during
cervix on the speculum examination are labor includes
Presence of clue cells, abundance of gram variable Fetal distress, fresh bleeding per vaginum and
coccobacilli and absence of lactobacilli. passage of meconium.
210 AIIMS and All India PGMEE—Review Questions

ALL INDIA 2005 C Anti-D prophylaxis should be given in


Medical abortion for 63 days of pregnancy, amnio-
C Aspermia term is used to describe centesis at 16 weeks and manual removal of
Absence of semen placenta.
C Drugs decreasing hot flushes in menopausal C The consequences of Rh incompatability are not
women are serious during first pregnancy because
Isoflavones, tibolone and androgens. Antibodies are not able to cross placenta.
C Drugs used in treatment of ectopic pregnancy C Most sensitive test for the detection of iron deple-
Actinomycin-D, mifepristone and methotrexate. tion in pregnancy
(RU-486)
Serum ferritin
C The highest incidence of gestational trophoblastic
C A 37 years multipara construction laboruer has a
disease is in
blood picture showing hypochromic anisocytosis.
Asia It is suggestive of
Most common cause of secondary amenorrhoa in
Combined iron and folic acid deficiency.
C
Indien
C Drugs that may be used in pregnancy associated
Endometrial TB
hypertension
C Patients of rectovaginal fistula should be initially
Methyldopa, hydaralazine and nifedipine.
treated with
Indications for antiphospholipid antibody testing are
Colostomy
C

C Most common pure germ cell tumor of the ovary is Early onset severe pre-eclampsia, unexplained
cerebrovascular accidents and three or more
Dysgerminoma
consecutive first trimester pregnancy losses.
C In a case of dysgerminoma of ovary, tumor markers
Risk factors for UTI in pregnancy
likely to be raised
C

Serum LDH Diabetes, sickle cell anemia and vesicoureteral


reflux.
ALL INDIA 2004 C Perinatal infections with highest risk of fetal infec-
tion in the first trimester
C USG fetal growth parameters are Rubella
Biparietal diameter, femur length and head C Antenatal steroids to induce lung maturity in the
circumference. fetus may be given to a pregnant mother of 32
C A couple have a 2 months old baby suffering from weeks gestation presenting with preterm labor in
Down’s syndrome. Karyotype of wife shows the following conditions
translocation variety of Down’s syndrome. PIH, DM and prolonged rupture of membranes for
Investigations advising to the parents before next more than 24 hours.
pregnancy
C Drugs effective for cervical ripening during pregnancy
Karyotyping
PGE2, oxytocin and misoprostol.
C Tests on maternal serum most useful in
Vaginal pH of 4 may be seen in
distinguishing between open neural tube defects
C

and ventral wall defects in a fetus Candidal vaginitis


Pseudocholinesterase C Most appropriate method for collecting urine for
C Screening by using MSAFP helps to detect culture in case of VVF is
Duodenal atresia, omphalocele and neural tube Foley’s catheter
defects. C A case of obstructed labor delivered by cesarean
C Antiprogesterone mifepristone (RU-486) is effective section, complains of cyclical passage of menstrual
for inducing abortion if the duration of pregnancy is blood in urine. Most likely site of fistula
63 days Vesico-uterine
C A case of pregnancy of 35 weeks with hydramnios C Diagnosis of a 25 years old obese women presenting
and marked respiratory distress is best treated by with oligomenorrhoea, infertility and hirsutism
Amniocentesis Polycystic ovaries.
Obstetrics and Gynecology 211

C A 50 years old woman presents with bilateral solid C Cut-off value of cervical length of 24 weeks of
ovarian tumors, ascites and upper GI endoscopy- gestation for prediction of preterm delivery
ulcerative growth in pyloric region of stomach. 2.5 cm
Diagnosis C Pure gonadal dysgenesis is diagnosed by presence of
Carcinoma stomach with Krukenberg’s tumor. Bilateral streak gonads
C Choice of adjuvant treatment for endometrial C Most common cause of first trimester abortion
carcinoma stage IA, grade I is Chromosomal abnormalities.
No treatment C A hypertensive pregnant woman of 34 weeks
comes with history of abdominal pain, bleeding per
vaginum, and loss of fetal movements. O/E-Uterus
ALL INDIA 2003 is contracted with increased uterine tone. FHS
C Risk factors known for development of endometrial absent. Diagnosis
carcinoma Abruptio placenta.
Use of HRT, family history and obesity. C A 20 years old woman presents with history of
C A case of GTN belongs to high risk group if disease sharp pain in the lower abdomen for 2–3 days every
develops after months approximately 2 weeks before the menses.
Full term pregnancy. Diagnosis
C A primigravida at 37 weeks of gestation reported to Mittelschmerz
labor room with central placenta previa with heavy C A 55 years old woman has recurrent urinary
bleeding per vaginum. O/E -fetal heart rate normal. retention after hysterectomy done for a huge
Best managed by fibroid. Cause
Cesarean section Injury to the hypogastric plexi.
C Hemodynamically stable nulliparous patient with C A 45 years old woman presents with bilateral
ectopic pregnancy has adnexal mass of 2.5 × 3 cm ovarian mass, ascites and omental caking on CT
and β-hCG-1500 mIu/ml. TOC scan. Diagnosis
Medical management. Malignant epithelial ovarian tumor.
C A 55 years old woman presenting with post-
ALL INDIA 2002
menopausal bleeding for 3 months has a 1 × 1 cm
nodule on the anterior lip of cervix. Next best C Character of vagina in normal pregnancy
investigation Increased number of lactobacilli.
Punch biopsy. C Associated with increased risk of thromboembolism
C Drugs used for management of PPH in normal pregnancy
Misoprostol, oxytocin and prostaglandin. Increased production of clotting factors by liver.
C Laparotomy performed in a case of ovarian tumor C Cephal-hematoma
revealed unilateral ovarian tumor with ascites Does not vary in tension with crying.
positive for malignant cells and positive pelvic C A pregnant woman developed idiopathic choles-
lymph nodes. Stage is tatic jaundice. Conditions associated
IIc Intense itching, markedly increased levels of alkaline
C Observed in normal pregnancy phosphatase and SGOT, SGPT less than 60 IU.
C A 30 years old woman presents with bilateral deep
Fall in serum iron concentration, increase in serum
vein thrombosis of legs, past history of recurrent
iron binding capacity and increase in blood oxygen
fetal loss and one attack of pulmonary embolism
carrying capacity.
and prolonged APT on coagulation testing
C B lynch suture is applied on Anti-phospholipid antibody syndrome.
Uterus. (Mn:But) C Contraindication in management during labor in a
C Pyometra is a complication associated with 20 years old pregnant woman with mitral stenosis
Carcinoma cervix, carcinoma endometrium and class II and rheumatic heart disease
pelvic radiotherapy. Methergin at delivery of anterior shoulder.
212 AIIMS and All India PGMEE—Review Questions

C Absolute contraindication for treatment of C Semen analysis of a 25 years old infertile male-
thyrotoxicosis in pregnancy of 6 months duration sperm count 12 million/ml, pH 7.5, volume 2 ml, no
I131 therapy agglutination seen. Morphology 60% normal and
C Associated with breech presentation at normal full 60% motile sperms. Diagnosis
term pregnancy Oligospermia
Fetal malformation, uterine anomaly and cornual C Sample for testing LH and FSH in a case of PCOD are
implantation of placenta. best taken on the following days of menstrual cycle
C Seen in the infant of a diabetic mother 8–10 days
Polycythemia C A 20 years old young girl presents with history of
C Always an indication of cesarean section rapidly developing hirsutism, amenorrhoea with
Untreated stage of Ib carcinoma cervix, active change in voice. Hormone tested in blood
primary genital herpes and Type IV placenta previa Testosterone
(major previa). C A 30 years old mother of two children is suffering
C True for episiotomies from amenorrhoea for last 12 months with history
Can be either midline or mediolateral, involvement of failure of lactation following 2nd delivery but
of anal sphincter is classified as 3rd, 4th degree. asymptomatic thereafter, skull X-ray showing
Perineal tear and midline episiotomies bleed less, empty sella. Diagnosis is
are easier to repair and heal more quickly. Sheehan’s syndrome
C In instrumental vaginal delivery C A 35 years old woman with children aged 5 and 6
Forceps may be used if ventouse fails. years has history of amenorrhoea and galactorrhoea.
C Karyotype of a 2 years old girl with Down’s Blood tests- Increased prolaction. The CT of head
syndrome has 21/21 translocation in karyotyping. may reveal
If father is a balanced translocation carrier, risk of Pituitary adenoma.
recurrence in subsequent pregnancies C A 45 years old woman presents with history of
100% polymenorrhoea for last 6 months. First line of
C The short retroviral regime administration in the management
peripartum period decreases the risk of vertical Oral contraceptives for 3 cycles.
transmission by C Indicated in menorrhagia
50% NSAIDS, tranexamic acid and norethisterone.
C Complete failure of Müllerian duct fusion results in C Most common genital proplapse
Uterus didelphys Cystocele
C Most common cause of female pseudohermaphro- C A 30 years old patient examined for infertility by
ditism is hysterosalpingography, reveals bead like fallopion
CAH tube and clubbing of ampulla. Most likely cause is
C A woman presents with hot flush after stopping of Mycobacterium tuberculosis.
menstruation. It can be relieved by C TOC for Chlamydial infection in a 25 years old
Ethinyl estradiol patient presenting with vaginal discharge
C Contraceptive LNG-IUD has cumulative pregnancy Azithromycin and contact tracing.
rate of 5 years of C May predispose to endometrial carcinoma
0.5 Unopposed estrogen, radiation and tamoxifen therapy.
14

Pediatrics

AIIMS NOVEMBER 2015 C Characteristic of Langerhans cell histiocytosis


Birbeck’s granule
C For last 9 days, an infant presents with 15–20 stools
(Note: Tennis racket appearance)
per day. Along with zinc, most rational treatment
option for the patient C A 10 years old boy presents with oliguria, frothy
ORS alone urine and on urine examination-no RBC/WBC, no
pus cells, 3+ proteinuria. Most likely diagnosis
C A 3.5 kg male infant born at term after uncomplicated
pregnancy and delivery developed respiratory Minimal change disease.
distress shortly after birth and requires mechanical C A 1 month old infant presents with conjugated
ventilation. Chest-X-ray-normal cardiac silhouette bilirubinemia and intrahepatic cholestasis. On liver
but diffuse ground glass appearance to the lung biopsy and staining with PAS red coloured
fields. Surfactant therapy fails and hypoxemia granules were seen inside the hepatocytes. Likely
worsens. Routine culture and Echo finding are cause
negative. Similar history in a term female sibling α-1 anti-trypsin deficiency.
dying at 1 month of age. Diagnosis C Brown fat is present in
Neonatal pulmonary alveolar proteinosis. Around adrenal gland, interscapular region and
C A 2 year old child is brought to emergency at 3 am around blood vessels.
with fever, barking cough, stridor during crying, (Note: Also around kidneys, nape of neck and
drinking normal, RR. 36/min, and temp 39.6 degree axillary region.)
Celsius. Next step is
C A child comes with coarse facial features, big
Dexamethsone tongue, hepatosplenomegaly and mucous discharge
(Note: Diagnosis- Mild croup/laryngotracheo- from nose. Diagnosis
bronchitis.) Hurler’s syndrome.
C Criteria for acute malnutrition are all except
C An infant at 7 months of age presents with history
Height for age of vomiting and failure to thrive. Patient improved
C Milk is deficient in with IV glucose but returned with the same
Iron and vitamin C complaints after one months. High blood glutamine
C Undernutrition is defined as and uracil levels were found. Likely enzyme defect
Weight for height < –2.5 standard deviation. Ornithine transcarbamoylase.

AIIMS MAY 2015 AIIMS NOVEMBER 2014


C Treatment given to a child suffering from diarrhea C A baby born by vaginal delivery has respiratory
Oral rehydration solution. distress. On examination, he has hyperinflated left

213
214 AIIMS and All India PGMEE—Review Questions

upper lobe and right mediastinal shift. The most hours later the baby showed hypotonia, lethargy,
likely diagnosis constricted pupils and two episodes of seizures.
Congenital lobar emphysema. The staging of HIE is
C Most likely cause of fluid discharge from umbilicus 2
on straining/crying C Asymmetric Moro’s reflex at birth is indicative of
Urachal fistula Erb’s palsy
C A newborn has not passed meconeum for the first C A very preterm baby on 30 ml/kg of enteral feeding
48 hours. Next investigation done developed sudden severe abdominal distension
Lower GI contrast study. with visible bowel loops on day 6 of life. The baby
C Ominous sign in a 10 days old newborn also showed temperature instability and lethargy.
Conjugated hyperbilirubinemia. X-ray of the abdomen showed portal venous gas.
The staging of NEC is
C A 4 years old child presents with persistent cough
following trachoesophageal fistula repair. The 2b
cough is dry, barky and occasionally associated C Babies with least risk of developing hypoglycemia-
with expiratory wheezing. The most likely diagnosis Appropriate for gestational age babies.
Tracheomalacia C The most common fungal infection in the neonates
C A 10 months old child weighing 5 kg and 68 cm in transmitted by caregiver’s hands is
height presents with cough. Her respiratory rate is Candida parapsilosis
48/min. There is no chest indrawing and grunt. C A child presents with deficient bone mineralization,
Most likely diagnosis low serum calcium, high serum phosphorus,
No pneumonia-only cough and cold. decreased urinary excretion of calcium and
phosphorus and elevated levels of alkaline
C An 8 years old child treated for nephrotic syndrome
phosphatase. The most likely diagnosis
developed diarrhea and acute kidney injury with
serum creatinine 4.5 mg/dl. His baseline creatinine Renal glomerular rickets.
was 0.9 mg/dl. Causes for elevation in serum C Bull neck is seen in severe cases of
creatinine Diphtheria
Renal vein thrombosis, furosemide induced C In a rural clinic, a 3 years old girl child who is
hypovolemia and diarrhea induced oliguria. emaciated is brought by her mother. Her Hb level
was 5 gm/dl. The girl has edema over her knees and
C Vasculitis not occurring in adults
ankles with discrete rash on her knees, ankles and
Kawasaki disease elbows. Most likely worm infested
C A 4 years old girl presents with severe headache, Hook worm
with fever and vomiting. Gram stain of the CSF
C A 10 years old child presents with edema, oliguria
showed Gram-positive rods. The most likely etiology
and frothy urine for the first time. Urine examina-
Listeria monocytogenes. tion showed +++ proteinuria, no RBC/WBC and no
casts. Serum albumin was 2.5 gm/dl and serum
AIIMS MAY 2014 creatinine was 0.5 mg/dl. The most likely diagnosis is
C A 32 weeks, 1400 gram neonate is born to a primigra- Minimal change disease
vida. The baby did not require resuscitation and C A 6 years old girl presents with fever for the past 5
showed stable vitals and shifted to NICU. Feeding days, generalized erythematous rash, strawberry
of the neonate by tongue and cervical lymphadenopathy. The most
Start IV feeding with minimal enteral feeding. likely diagnosis is
C Components of Kangaroo mother care Kawasaki disease
Skin to skin contact, exclusive breastfeeding and C Enzyme replacement therapy is available for
early discharge and follow-up. Gaucher’s disease
C A 32 weeks baby is born to a mother with eclampsia, C A child presents with albinism. He should be
who was given IV magnesium sulphate. The baby evaluated for
was resuscitated and transferred to the NICU. 12 Eye consultation
Pediatrics 215

C A 7 years old boy presents with a right sided C To establish the diagnosis of H type tracheo-
hemangioma and left sided focal seizures. The most esophageal fistula, what is required
likely diagnosis Tracheo-bronchoscopy.
Sturge Weber syndrome. C A 3 days old baby is admitted with intraventricular
C A 1.5 years old girl is brought with complaints of hemorrhage. Baby develops abdominal distension.
excessive enlargement of head, intolerance to feeds The X-ray abdomen showed pneumatosis portalis.
and severe malnourishment. MRI imaging was Stage of necrotizing enterocolitis
suggestive of a medulloblastoma causing obstruc- 2b
tive hydrocephalus. Example of irrational manage- C Children with germline retinoblastoma are more
ment likely to develop other primary malignancy
Radiotherapy 35–40 gray was given to the whole Osteosarcoma of lower limbs.
craniospinal axis. C A 4 years old girl presents with abdominal lump.
Bone scan is needed in
AIIMS NOVEMBER 2013 Neuroblastoma
C An 8 years old boy presents with increasing muscle AIIMS MAY 2013
weakness, bulky calves, muscle tightening and
serum creatine kinase levels increasing with the C Major Jones’ criteria for the diagnosis of rheumatic
age. Most likely diagnosis fever
Dystrophin deficiency Chorea, carditis and subcutaneous nodules.
(Note: Duchenne muscular dystrophy) (Note: Other major criteria are migratory polyarth-
C A 2 days old premature neonate develops GTCS. ritis and erythemia marginatum)
Investigation done to diagnose the pathology C A 2 days old neonate in the NICU develops seizures.
Next best investigation
Transcranial ultrasound.
C A pediatrician in a district hospital with specialized Transcranial ultrasound
neonatal care unit calls an ophthalmologist for C Characteristics of Rett syndrome
consultation for Increased incidence of mental retardation, seizures
A baby born at 28 weeks of gestation. and abnormal dendritic morphology in cortical
pyramidal cells.
C An Afroamerican kid of 6 years of age presented with
A case of meningomyelocele was posted for surgery.
abdominal pain, chronic hemolysis and abnormal
C
Till the patient is waiting for the surgery, covering
RBC shape on peripheral smear. Most likely cause
of the sac is protected by a gauze soaked in
Point mutation
Normal saline
(Hint: Sickle cell anemia)
C A 4 years old child presents to the emergency with
Drugs given in treatment of cyanotic spells in a
respiratory difficulty and noisy breathing. On
C
patient of TOF
examination, X-ray shows thumbs sign. The most
Phenylephrine, propranolol and sodium bicarbonate. probable diagnosis is
C Syndromes associated with uniparental disomy are Epiglottitis
Angelman’s syndrome, Prader-Willi syndrome and C A child presents with fever and sore throat, on
Russell-Silver syndrome. examination-respiratory rate was 36/min, tempe-
C A newborn has frothing of mouth. Cyanosis is rature 39°C, saturation on pulse oximetry 96%. The
present on day one. Most likely diagnosis child had barking cough and stridor only on crying.
Esophageal atresia Otherwise, the child was hydrated, able to drink
C A 6 weeks old baby presents with cough and cold and consolable. Next step
for the last 3 days, RR is 48/min, febrile, wheezing Single dose dexamethasone.
but no chest retraction. True statements are C A 6 years old child with steroid dependent nephrotic
Treat wheeze, treat fever and antibiotics not required. syndrome has developed corticosteroid toxicity
C Efficacy of phototherapy is affected by and posterior subscapular cataracts. Best alterna-
Type of light used, spectral irradiation by incident tive for the treatment
light and initial concentration of bilirubin. Levamisole
216 AIIMS and All India PGMEE—Review Questions

C A slow growing highly vascular tumor in adults C A pre-term 32 weeks newborn baby with respira-
affecting cerebellum, brain stem and spinal cord tory rate of 86/min, with grunting. On examination
Hemangioblastoma No nasal flaring, chest behind in movement than
C A 3 years old child has hepatosplenomegaly, on abdomen, minimal intercostal retraction and no
examination of the bone marrow, large cells are xiphisternal retraction. The Silverman score for the
seen with crumpled paper appearance. Which of neonate-4.
the following must have accumulated in these cells C A child presents with seizure, oval hypopigmented,
Glucocerebrosidose macules on the trunk and sub-normal IQ. Diagnosis is
C Following suggest a developmental delay in Tuberous sclerosis
milestones C A child presents with seborrheic dermatitis, sinusitis
Absence of pincer grasp at 9 months, not climbing and chronically draining ears. On examination
stairs up and down at 2 ½ years and not able to sit at child has failure to thrive, hepatosplenomegaly and
9 months. exophthalmos. Diagnosis is
C Babies can breathe while suckling due to Histiocytosis X
Higher position of larynx. C In a child CSF examination is not used in the
C A 2 years old boy presents with a lump in right diagnosis of
side of the abdomen. Ultrasound showed it to be Hodgkin’s lymphoma.
solid mass. On examination, his right arm and leg C The developmental age of a child who knows her
were found to be longer. The most likely diagnosis full name and gender, can eat without spilling and
is can dress herself without supervision is
Wilm’s tumor 4 years
C Most common organism causing neonatal sepsis C Most common organism causing neonatal sepsis
Klebsiella Klebsiella

AIIMS NOVEMBER 2012 AIIMS MAY 2012


C A pre-term infant with poor respiration at birth C Side effects of growth hormone (supplements)
starts throwing seizures at 10 hours after birth. therapy
Anti-epileptic of choice Slipped capital femoral epiphysis, gynaecomastia
Phenobarbitone and pseudotumor cerebri.
C True about trisomy 13 C Mitochondrial inheritance is transmitted
Bilateral microphthalmia and rocker bottom feet. From mother to all progeny, from father to none.
C A child with hypoglycemia is not able to utilize C Management of choice for Bell’s stage 1 NEC
glucose from glycogenolysis or gluconeogenesis. (necrotizing enterocolitis) in a neonate
Enzyme deficient is Conservative management with IV fluids and
Glucose 6 phosphatase antibiotics.
C A neurosurgeon spots a child with uncontrollable C A 10 years boy presents with seizure disorder with
laughing and precocious puberty, gets his MRI a vascular plaque along the ophthalmic and
done and diagnosed. Diagnosis is maxillary division of the trigeminal nerve present
Hypothalamic hamartoma since birth and unchanged in morphology.
C Signs of PDA in a preterm baby Diagnosis likely
Apnea, tachycardia and necrotizing enterocolitis. Sturge-Weber syndrome.
C Which of the following in the natural course of C Pathology of edema in nephrotic syndrome
disease has no reversal of shunt Sodium water retention > reduced plasma protein.
Tetralogy of Fallot C Finding in a newborn suggesting RDS (respiratory
C True about respiratory distress syndrome distress syndrome)
Air bronchogram seen on chest X-ray. Air bronchogram seen.
Pediatrics 217

AIIMS NOVEMBER 2011 C A 5 years old tall boy present with pubic hair
development, increased pigmentation of genitalia,
C NESTROFT test is a screening test for phallic enlargement and BP 130/90 mm Hg.
β thalassemia Measurement of hormone diagnostic is
C A 5 years old male child presenting with episodic Increased 11 deoxycorticol
anemia and jaundice since birth. Differential C Feature of juvenile idiopathic arthritis
diagnosis includes Rheumatoid nodule, uveitis and spikes of high
G6PD deficiency, PNH, hereditary spherocytosis. fever (Mn: FUN).
C True about newborn assessment C A 4 weeks old female child presents with severe dehy-
APGAR at 1 minute indicates respiratory resusci- dration, hyperkalemia, hyponatremia and normal
tation, fetus can rapidly washout CO 2 through genitalia. Measurement of hormone diagnostic
placenta and anaerobic metabolism causes acidosis.
Aldosterone
C Diagnosis of a 5 years old child presenting with peri-
C Hypoplasia of limbs and scarring is caused due to
vascular IgA deposition, neutrophilic collection,
Varicella
erythematous rash on the lower limbs and non-
blanching purpura is C A pregnant lady (non-compliant to spiramycin)
HSP had no other complaints than mild cervical lympha-
denopathy in first trimester. Child was born with
C Condition worsened by prostaglandin E infusion
hydrocephalus and intracerebral calcification
Obstructed TAPVC
Toxoplasmosis
C A 3.5 kg male infant born at term after uncomplica-
AIIMS MAY 2011
ted pregnancy and delivery developed respiratory
C True about a variant of dent disease in a male child distress shortly after birth and requires mechanical
with Fanconi syndrome with nephrocalcinosis ventilation. Chest-X-ray-normal cardiac silhouette
Hypercalciuria, proteinuria and rickets. but diffuse ground glass appearance to the lung
C Most common microorganisms associated with fields. Surfactant therapy fails and hypoxemia
cystic fibrosis worsens. Routine culture and echo finding are
Pseudomonas aeruginosa (non-mucoid). negative. Similar history in a term female sibling
C True about cystic fibrosis dying at 1 month of age. Diagnosis
AR, mutation in cystic fibrosis transport regulator Neonatal pulmonary alveolar proteinosis.
C A child presents with steroid resistant nephrotic C A child presents with respiratory distress, failure to
syndrome secondary to FSGS, not responsive to thrive, sweat chloride levels 35 and 40 mEq/L. Next
methylprednisolone. Next given best test for diagnosis of cystic fibrosis
Oral cyclosporine Nasal transmembrane potential difference.
C A previously healthy 6 weeks old female is found C True regarding a study under Australian collabo-
unresponsive in her crib. On examination- well rative trial on steroids use in neonates
developed and well nourished with normal blood Corticosteriods to children caused behavioural
pressure and appearance of the genitalia but with worsening.
increased pigmentation of her skin. Blood glucose C Prognostic factors in international prognostic index
level is 30 mg/dl. Diagnosis for lymphomas
Familial glucocorticoid deficiency. LDH, number of extra-lymphatic sites involved and
C Diagnosis of a 2 months old infant presenting with stage of disease
failure to thrive, recurrent emesis, hepatospleno- C A neonate discharged 2 days back healthy
megaly, adrenal insufficiency and adrenal calcifi- presented with severe respiratory distress and
cation noted radiologically shock on 7 days of life. Diagnosis
Wolman’s disease Hypoplastic left heart syndrome.
C True about congenital rubella C Ductus dependent blood flow is required in
IgG persists for more than 6 months, IgM antibody congental heart diseases
present at birth and most common anomalies are Hypoplastic left heart syndrome, pulmonary
hearing and cardiac defects. stenosis and TGA with intact ventricular septum.
218 AIIMS and All India PGMEE—Review Questions

AIIMS NOVEMBER 2010 C True about Eisenmenger syndrome


Dilatation of central pulmonary artery, peripheral
C A 3 days old neonate vomits everything he feeds, pruning of pulmonary arteries and pulmonary
presents with distended abdomen, diarrhea, veins are not distended.
Benedict’s test positive for reducing substance. The C Signs of impending Eisenmenger syndrome are
substance is urine is
Single S2, Loud P2 and Grahmsteel murmur.
Galactose
Most sensitive indicator of intravascular volume
Factors favouring the diagnosis of RDS in newborn
C
C
depletion in infant
Air bronchogram in chest X-ray
C Child with PDA will have Heart rate
Bounding pulses, necrotizing enterocolitis and C Alert 6 months old child presents with vomiting,
pulmonary hemorrhage (Mn: PNB). diarrhea, RR–45/min, HR–130/min SBP–85 mm Hg
C A child is able to breath and suckle at the same time and capillary refill time 4 secs. Diagnosis is
because of Early compensated hypovolemic shock.
High placed larynx. C True about congenital toxoplasmosis
C A neonate has ABO incompatibality. Peripheral Diagnosed by detection of IgM in cord blood, IgA is
smear will show more sensitive than IgM for detection and dye test
Microspherocytes is gold standard for IgG.
C Drug responsible for a newborn developing C Best indicator of growth monitoring in children
respiratory distress in neonatal ward Rate of increase in height and weight.
Opioids
Ponderal index is
AIIMS NOVEMBER 2009
C
Weight (in gm)/height (in cm)3 × 100.
C How will nurse prepare N/5 normal saline from C The poor prognostic factor associated with ALL in
10% dextrose and normal saline children is
80 ml 10% D with 20 ml NS. Testicular involvement
C A 32 weeks neonate presents with RR 86/min, C Vitamin deficiency responsible for neonatal
grunting present, no nasal flaring, abdomen behind seizure
in movement than chest, minimum intercostal Pyridoxine
retraction and no xiphisternal retraction. Silverman C Chromosomal anomalies more than 20% is associated
scoring is with
4 Omphalocele
Neonatal behavioural response scale in a neonate
Most common sequelae due to periventricular
C
C
seen crying with eyes closed and moving all his limbs
leukomalacia
Scale 6
Spastic diplegia
C A 30 weeks gestation mother delivered 1.2 kg baby
with moderate respiratory distress, RR-70/min, C Treatment of Kostmann’s syndrome
grunting and chest retraction present. Next step G-CSF (Mn: GK)
Nasal CPAP C Fetal alcohol syndrome is characterized by
C An 8 years old child presents with BP 180/100 mm/ Microcephally, low intelligence and septal defects
Hg, urea 90, S. creatinine 5. Urine finding-15–20 pus of heart.
cells, 1–2 RBC, protein + and no significant past C Vigorous baby is defined by
history of similar complaint. Diagnosis HR, Respiration and Tone of muscles (Mn: HRT).
Chronic interstial nephritis with VUR C True about ASO titre
ASO can be increased in school children, may be
AIIMS MAY 2010 negative in PSGN, and may be elevated in 20% cases
C True about cow’s milk of carditis.
Cow milk has less carbohydrate than mother’s C Most common cause of sepsis in India within
milk, has more K +, Na + ion than infant formula 2 months
feeds and has more protein than breast milk. Coagulase positive Staph. aureus > E. Coli.
Pediatrics 219

AIIMS MAY 2009 AIIMS NOVEMBER 2007


C Causes of bilateral medullary nephrocalcinosis in 3 C A girl child presented with fever, cough, dyspnea
months old infant are with X-ray showing right lower lobe patchy
Bartter’s syndrome, hyperoxaluria and prolonged consolidation. After 8 weeks of treatment
use of frusemide. symptoms improved but X-ray showed more dense
C A 3 months old child (male) with normal genitalia consolidation involving whole of right lower lobe.
presents with severe dehydration, hyperkalemia Next investigation done is
and hyponatremia. Measurement of hormone Bronchoscopy
diagnostic is C Vaccine C/I in child with h/o convulsions
17 hydroxyprogestrone. DPT
C A child present with nonblanching macules and C Included in National Immunization Programme
papules on lower extremities, mild abdominal pain OPV, measles and TT.
and IgA deposition on skin bispsy. Diagnosis C Next immunization plan for 18 months old child
HSP receiving one dose of DPT and OPV at 2 months of age
BCG, 2nd dose of DPT and OPV.
C 2 kg birth weight, 30 weeks baby to 18 years old
AIIMS NOVEMBER 2008
primigravida died after 48 hours with autopsy
C Drug used for treatment of refractory histocytosis showing bilateral enlarged kidney with multiple
Cladirabine radially arranged cysts. Associated finding
C A 10 years old child is always restless, in attentive Hepatic cyst and fibrosis
to study and always wants to play outside. Advice C True about poliomyelitis
Behaviour therapy IM injection and increased muscle activity increases
C A 2 years old child is brought to emergency at 3 am the risk of paralytic polio.
with fever, barking cough, stridor during crying, C Essential criteria for TOF includes
drinking normal, RR. 36/min, and temp 39.6°C. RVH, infundibular stenosis and overriding of aorta.
Next step is C Features of Down’s syndrome
Dexamethasone Increased free β-hCG levels, absent nasal bone, and
abnormal ductus venous flow velocity.
AIIMS MAY 2008 C Deep white matter lesion with bilateral deep bright
thalamic appearance is suggestive of
C Most common inherited childhood tumor
Krabbe’s disease
Retinoblastoma
C An 8 years old male child presents with non-
Patient presents with absent thumb, radial
blanching rashes over the shin and swelling of knee
C
deviation of wrist, bowing of forearm and
joint with hematuria +++ and protein+. Microscopic
thrombocytopenia. Investigations done are
analysis of renal biopsy shows
Bone marrow examination, platelet count and
Mesangial deposits of IgA.
karyotyping.
C Macrocephaly is seen in
Canavan’s disease AIIMS MAY 2007
C Turner’s syndrome is maximally associated with C Myopathy is seen in
Coarctation of aorta Cushing’s syndrome, oncogenic osteomalacia and
C Most common cause of seizure in newborn nutritional osteomalacia.
Hypoxia induced ischemic encephalopathy. C Reflex present in child at birth
C An infant with cleft lip, cleft palate, polydactyly, Crossed extensor reflex, asymetric tonic neck reflex
microcephaly with holoprosencephaly, ectodermal and Moro’s reflex.
scalp defect is suffering from C Persistence of Moro’s reflex is abnormal beyond
Trisomy 13 6 months of age
220 AIIMS and All India PGMEE—Review Questions

C Diagnosis of a 3 years old boy with normal develop- AIIMS NOVEMBER 2005
mental milestones with delayed speech and
difficulty in communication and concentration and C Recurrent respiratory tract infections may occur
not making friends also in
Autism TGA, VSD and total anomalous venous return.
C Most common cause of renal artery stenosis in C A female child has recently learned to eat with
children in India spoon without spilling, to dress and undress herself
with supervision and to understand that she is a
Takayasu aortoarteritis
girl. These skills are first mastered between the ages
Newborn baby presenting with profuse blee-
of 2 and 3 years.
C
ding from unbilical stump after birth. Diagnosis
C A 30 years old lady delivered a healthy baby at 37
is
weeks of gestation. She was a known case of chronic
Factor XIII deficiency HepB infection. She was positive for HBsAg but
C Most common cause of neonatal sepsis is hospital in negative for HBeAg. Most appropriate treatment
India Both active and passive immunization soon after birth.
Klebsiella C Switch over from fetal to adult hemoglobin synthesis
C A term neonate with unconjugated hyperbilirubi- begins at what gestation
nemia of 18 mg/dl on 20th day. Common causes 36 weeks
are C A 6 months old 3.2 kg weight boy presents with
Hypothyroidism, breast milk jaundice and G6PD recurrent vomiting , polyuria, S. urea–60 mg/dl,
def. creatinine 0.7 mg/dl, calcium–12.8 mg/dl,
C DOC for rheumatric fever prophylaxis is penicillin phosphate 3 mg/dl, pH–7.45, bicarbonate 25 mEq/
allergic patient L and PTH 140 pg/ml (Normal <60 pg/ml), daily
Erythromycin urinary calcium excretion reduced, U/S-bilateral
C Common to both acute and chronic malnutrition nephrocalcinosis. Diagnosis
Weight for age Mutation of the calcium sensing receptor.
C A 5 years old boy presents with overnight petechial C Most appropriate treatment of a child presenting
spots 2 weeks back he had h/o abdominal pain and with diarrhea, peripheral circulatory failure,
no hepatosplenomegaly. Diagnosis arterial pH 7.0, PCO2 15 mm Hg, PO2 76 mm Hg
Idiopathic thrombocytopenic purpura. Bolus of ringers lactate.
C A 5 years old boy gets electrocuted while playing in
C A 6 years old child with IQ 50 can do
a park. The child is apnoeic, ventilated with bag and
Identify colors mask. There are burns on each hand. Next step
Check pulses
AIIMS NOVEMBER 2006
C Mechanism of heat production in a neonate are AIIMS MAY 2005
Cutaneous vasoconstriction, universal flexion like a
C True about TOF
fetus and breakdown of brown fat with adrenaline
Ejection systolic murmur in second intercostal
secretion.
space, single second heart sound, and normal
C Single gene defect causing multiple unrelated
jugular venous pressure.
problems
C Earliest manifestation of Cushing’s syndrome
Pleiotropism
Loss of diurnal variation.
(Note: Opposite is Epistasis).
C FA present in breast milk important for growth C Conditions observed in marasmus
Docosahexaenoic acid. Extreme weakness, low insulin levels and muscle
C Constant in Schwartz formula for calculation of wasting.
creatinine clearance in a child depends upon C Characteristic radiological feature of transient
Mass, age and method of estimation of creatinine tachypnea of newborn is
(Mn: MAC). Prominent horizontal fissure
Pediatrics 221

C Deficiency of-keto acid decarboxylase leading to C Drug avoided in an infant 6 months old with TOF
block in metabolism of branched chain amino acid presenting with cyanotic spell initiated by crying
is observed in Isoprenaline
Maple syrup urine disease (Note: it causes peripheral pooling of blood).
C Dengue shock syndrome is characterized by C Liver biopsy of a neonate presenting with jaundice
Hepatomegaly, pleural effusion and thrombocyto- shows giant cell/neonatal hepatitis. Diagnosis
penia. Alpha-1 antitrypsin deficiency.
C Epiglottitis in a 24 years old child most commonly C Dietry management of a 3 years old boy with bilateral
due to infection with renal calculi secondary to idiopathic hypercalciuria
H. influenzae type b Increased water intake, low sodium diet and
C Cardiac defects characterized by ductus dependent avoidance of meat proteins.
blood flow C A 2 months old infant presenting with failure to
TGA with intact septum, interrupted aortic arch thrive, polyuria, medullary nephrocalcinosis
and hypoplastic left heart syndrome. affecting both kidneys and investigation showing-
C Principal mechanism in phototherapy reducing blood pH-7.48, bicarbonate 25 mEq/L, potassium 2
serum bilirubin mEq/L, sodium 126 mEq/L and chloride 88 mEq/
Structural isomerization (Mn: SI > PI > PO). L. Diagnosis
Structural isomerization > photoisomerisation > Bartter’s syndrome
photo-oxidation. C A 10 years old boy with nephrotic syndrome and
C Syndrome best associated with congenital heart steroid dependent for last 5 years now develops
disease markedly cushingoid with BP of 120/86 mmHg and
Holt-Oram syndrome small sub-capsular cataracts. TOC
C Formula for plasma osmolality is Cyclophosphamide
2[Na+] + glucose/18 + BUN/2.8 C A 12 years old boy has bilateral gynaecomastia,
height 148 cm, weight 58 kg, sexual maturity rating
C Leading cause of death in U-5 children in developing
stage 2. Cause
countries
Pubertal gynaecomastia.
Acute lower respiratory tract infections.
C A 9 years old child presents with growth retarda-
AIIMS NOVEMBER 2004 tion, propensity to hypoglycemia, short stature,
micropenis, increased fat and high pitched voice
C TOC for primary grade V VUR involving both and bone age of 6 years on skeletal survey. Diagnosis
kidneys in a 6 months old boy Growth hormone deficiency
Antibiotic prophylaxis C Oral rehydration mixture contains glucose and
C A 7 years old boy with left renal mass had bone pain sodium because both of them
due to bone metastatic deposits. The most likely Facilitate the transport of each other from the
renal tumor is clear cell sarocoma. intestinal mucosa to the blood.
C An 8 days old breastfed baby presents with C Features of TTP
vomiting poor feeding and loose stools, O/E—HR More common in females, petechiae, ecchymosis
190/min, BP 50/30 mm Hg, RR-72/min, capilary and bleeding and increased megakaryocytes in
refill time 4 sec, Hb-15 gm/dl, Na+ –120 Meq/L, K– bone marrow.
6.8 mEq/L, bicarbonate 15 mEq/L, urea–30mg/dl C Lead pipe appearance of colon on barium anema is
and creatinine 0.6 mg/dl. Diagnosis seen in
Acute tubular necrosis. Crohn’s involvement of the colon.
C A male infant presents with distension of abdomen C A 2 years old boy presents with vitamin D resistant
shortly after birth and less meconinm passed. Full rickets, S. calcium–9 mg/dl, phosphate-2.4 mg/dl,
thickness rectal biopsy is likely to show alkaline phosphatase–104/IU, normal intact PTH
Lack of ganglion cells and bicarbonate 2 mEq/L. Diagnosis
(Note: Hirschsprung disease) Hypophosphatemic rickets.
222 AIIMS and All India PGMEE—Review Questions

AIIMS MAY 2004 alkaline phosphatase 400 U/L, urine analysis


showing trace proteinuria with hyaline casts, no red
C Management of a young boy presenting with recurrent and white cells U/S-bilateral small kidneys and
gross hematuria, IgA nephropathy on renal biopsy MCU-normal. Diagnosis
and urinary protein excretion of 130 mg/day Nephronophthisis
Corticosteroids C Portal hypertension in children in India is commonly
(Note: Berger’s disease) due to
C Most appropriate technique for obtaining a urine EHPVO
specimen for culture in an 8 months old girl C Asphyxial injury in a term baby is characterized by
Suprapubic aspiration seizures, altered sensorium and difficulty in cleaning
C An 8 years old asymptomatic boy during routine oral secretions.
urine checkup (mid-stream clean catch void) is C Mother to child transmission of HIV prevented by
found to have E. coli 1,00,000 cc/ml on urine culture. Zidovudive to mother and baby, stopping breast
Management most appropriate feeding and elective cesarean section.
Nothing is required C Most common cause of diarrhea in a 6 months old
(Note: Asymptomatic bacteriuria a needs us treatment) infant
C A 2 years old child presents with discharge, seborrheic Rotavirus
dermatitis, polyuria and hepatosplenomegaly. C Young child presents with pyoderma and becomes
Diagnosis toxic and develop respiratory distress. Chest X-ray
Langerhan’s cell histiocytosis. showing-patchy areas of consolidation and
C Injection glucagon is effective for management of multiple bilateral thin walled air containing cysts.
persistent hypoglycemia in Etiology suggestive of
Nesidioblastosis, infant of diabetic mother and Staphylococcus aureus
large for date baby (Mn: NIL). (Hint: Pneumonia with pneumatoceles).
C Patency of ductus arteriosus in a neonate is C A 2 years old girl child presents with features of
maintained by hand wringing stereotype movements, impaired
Prostaglodin E1 language and communication development, breath
C Congenital hypertrophic pyloric stenosis normally holding spells, poor social skills and deceleration of
presents around 4 weeks after birth. head growth after 6 months of age. Diagnosis
C Weak giants are produced by Rett’s syndrome
Pituitary adenomas
C Vesicoureteric reflux is more common in
Newborn females AIIMS MAY 2003
C Bag and mask ventilation is C/I in C Characteristic features of kwashiorkar
Diaphragmatic hernia Edema, fatty liver and hypoalbuminemia.
C Cotrimoxazole was given for otitis media in a C Features of rickets
3 months old male infant, few days after which she Bow legs, cranitabes and pot belly.
developed extensive pilling of the skin, but no C In neonatal screening programme for detection of
mucosal lesions and baby not toxic. Diagnosis congenital hypothyroidism, blood sample for TSH
Staphylococcal scalded skin syndrome. estimation is taken from
C Microbe present in maternal genital tract responsi- Cord blood at time of birth.
ble for meningitis at 7 days of birth in a neonate C A neonate baby with normal APGAR score at birth
Steptococcus agalactiae presents with excessive frothing and choking on
attempted feeds. IOC is
AIIMS NOVEMBER 2003 X-ray chest and abdomen with red rubber catheter
C A 12 years old boy presents with nocturnal passed perorally into esophagus.
enuresis, short stature, normal BP, S. Urea–112 mg/ C Cyanosis produced in first year of life in
dl, creatinine–6 mg/dl, Na–119 mEq/L, K + –4 Double outlet RV, hypoplastic left heart syndrome
mEq/L, calcium–7 mg/dl, phosphate–6 mg/dl, and truncus arteriosus.
Pediatrics 223

C IOC for one year old infant presenting with poor C Retrolental fibroplasia is associated with
urinary stream since birth Low birth weight (ROP).
Voiding cystourethrography. C A full term baby, exclusively breast fed with ade-
C First clinical intervention in a neonate with quate hydration, normal systemic examination and
congenital diaphragmatic hernia weight same as birth weight at the end of 1 week was
Insert a nasogastric tube. passing golden yellow stools. Advice to the mother
C Investigations for diagnosis of HIV infection in a Reassure her that nothing is abnormal.
2 months old child C Features seen in child after birth whose mother is
DNA-PCR, P24 antigen assay and viral culture. exposed to Diethylstilbestrol during pregnancy
C In pediatric advanced life support, intraosseous Clear cell Ca, malformation of vagina and uterus
access for drug/fluid administration is recommended and vaginal adenosis.
for pediatric age of C A child died soon after birth. O/E-Hepatospleno-
< 6 years age megaly and edema all over the body. Diagnosis
C Newborns at increased risk of hypoglycemia are α thalassemia
Birth asphyxia, maternal diabetes and RDS. C TOC of a child presenting with fever for 2 days,
C A 10 years old child presenting with headache, altered sensorium and purpuric rashes and BP
vomiting, gait instability, diplopia, O/E-papillo- 90/60 mm Hg-IV pencillin
edema and gait ataxia. Diagnosis (Hint: Meningococcal meningitis).
Midline posterior fossa tumor
C A neonate presents with encephalitis without any
(Hint: ICT is increased). skin lesions most probable causative organism is
C A 3 years old child presents with history of bronchial HSV II
asthma, progressive shortness of breath for 1 day,
C An infant presents with hypotonia and hyporeflexia.
O/E-blue, gasping and unresponsive. First done is
During his intrauterine period there was polyhy-
Administer 100% oxygen by mask.
dramnios and decreased fetal movements. Diagnosis
C The metabolic disturbance in CHPS
Spinal muscular atrophy.
Hypochloremic alkalosis.
C Associated with proximal muscle weakness
(Note: Hirschprung disease)
Duchenne muscular dystrophy, polymyositis and
C Failure to pass meconium within 48 hours of birth
spinomuscular atrophy.
in a newborn with no obvious external abnormality
Congenital aganglionosis.
C Ductus dependent blood flow is necessary in ALL INDIA 2012
congenital diseases C A 1 year old girl is brought with the fever for 24 hours
Hypoplasia of left heart, obliterated aortic arch and and on and off coughing, passing foul smelling
transposition of great vessels with intact ventricular bulky stools and 4 attacks of bronchitis in the past.
septum. Diagnosis suggested
Transient tachypnea of newborn is commonly seen in
Cystic fibrosis
C
Elective cesarean section.
Regarding parenteral nutrition of newborns, 20%
Posterior iliac horns are seen in
C
C
intralipid has following advantages over 10%
Nail patella syndrome
intralipid solution
C A 10 years old boy presents with polyuria, poly-
dypsia, Na+–154, K–4.5, HCO–3–22, S. osmolality–295 Reduced phospholipids, increased calorie density/ml
and blood urea-50, urine specific gravity–1.005. of fluid and better triglyceride clearance.
Diagnosis C Most commonly identified fetal tumor
Diabetes insipidus Sacrococcygeal teratoma.
C Treatment of a child with pauci-immune crescentic GN C A patient of meningomyelocele posted for surgery
Prednisolone + cyclophosphamide. and the covering of sac is protected by a gauze piece
C Poor prognostic indicator of ALL is soaked in
Hypodiploidy Normal saline
224 AIIMS and All India PGMEE—Review Questions

C Least chance of infective endocarditis is seen with C Decreased hormone level of LH, FSH and testoste-
(among the options) rone in a child presenting with pubertal delay is
Small ASD suggestive of
C True about jaundice in neonates- Kallmann’s syndrome
Can be seen after ventouse delivery. C Most common tumor of fetus is
C A term infant born to a diabetic mother is lethargic Sacrococcygeal teratoma
few hours after birth and blood glucose 30 mg%. C Most common bacterial cause of diarrhea in children
done next in India
10% dextrose IV ETEC
A mentally challenged child presents with dysphagia,
Wide neural forminea is seen in
C
C
ophisthotonic spasms, choreoathetoid movements
Neurofibromatosis
and self mutilative behavior with positive family
history. Investigation suggested C True regarding jaundice in newborn
Serum uric acid Physiological jaundice starts after 48 hrs, Ventouse
C Fever for 24 hours, history of 3 episodes of chest delivery may be associated, breast milk jaundice
infection and constipation with passage of large peaks on day 7.
foul smelling bulky stools in a one-year-old child is C A 7 years old child presenting with ambiguous
suggestive of genitalia, normal height, weight and BP, bifid labia
Cystic fibrosis. with 2 separate perineal openings, 2.5 cm long
C Neonate borne out of non-consanguineous marriage phallus and no palpable gonads in the inguinal
had erosion and blisters in the area of contact with region and present Müllerian structures. Diagnosis
history of similar disease in elder slibling dying after Simple virilisation of primary CAH.
2 weeks. It is suggestive of epidemolysis bullosa. C In parentral nutrition 10% intralipid infusion, as
C Developmental age of a child using 4–6 words compared to 20% intralipid influsion will have
including proper nouns but communication is Less energy, less hypertriglyceridemia.
mainly nonverbal
15 months
ALL INDIA 2011
C Blood sugar of a gestational age neonate with birth
weight 4.0 kg, gestational age of delivery 40 weeks C Initial feeding method of choice in a premature
and lethargy is 30 mg/dl. Treatment is baby, born at 33 weeks of gestation with birth
Intravenous dextrose 10%. weight 1500 gm and stable vitals
C Hormone increased in Prader-Willi syndrome Orogastric tube feeding
Ghrelin C Most important prognostic factor in neonatal
C Mediastinal lymphadenopathy in a young boy congenital diaphragmatic hernia
presenting with dyspnea suggests Pulmonary HTN
T cell lymphoblastic ALL (Note: Least important prognostic factor - Delay in
C Long term sequelae associated with radiotherapy in emergent surgery).
children for treatment of CNS tumors are C A 38 weeks, 2.2 kg birth weight neonate presents
Endocrine deficits, musculoskeletal hypoplasia and with intolerance to feed/abdominal distension on
reduced IQ, learning and behavior difficulties. second day, negative sepsis screening and 72%
C Child presenting with skin infections, reduced platelets PCV. Best management
and reduced IgM is most probably suffering from
Partial exchange transfusion.
Wiskott-Aldrich syndrome.
C Earliest indicator of pathological GERD in infants
C Cause of seizure of a 5 years old child of nephrotic
syndrome on treatment with tacrolimus and Respiratory symptoms
prednisolone presenting with edema and seizure, C Most likely cause of infant of diabetic mother with
urea–98 mg/dl, Na+–135mEq/L albumin –1.5 mg/ weight 3.8 kg presenting with seizures after 16
dL, Ca+–7.5 mg/dl hours of birth
Tacrolimus toxicity Hypoglycemia
Pediatrics 225

C Additional component in pentalogy of Fallot phosphate 9 mg/dl and serum intact PTH of 30 pg/
ASD ml. Diagnosis
C X-ray chest feature differentiating between ASD Hypoparathyroidism
and VSD C A non-immunized child presents with recurrent
Enlarged left atrium bouts of severe cough followed by audible whoop.
C A previously healthy infant presents with recurrent Best specimen to isolate the organism and confirm
episode of abdominal pain, history of passing diagnosis
altered stool after episodes of pain but no history of Nasopharyngeal swab.
of vomiting or bleeding per recturm. Diagnosis C A previously healthy eight years old boy after
Intussuception completing 8 days out of 10 days course of cefaclor
C A 10 years old child presents with hemetemesis, for upper respiratory tract infection now presents
malena, mild splenomegaly but no obvious jaundice with high grade fever, arthralgia, pruritic erythe-
or ascites. Diagnosis matous lesion and lymphadenopathy. Diagnosis
Serum sickness like ilness (SSLI) > type II HS
EHPVO
reaction.
C A child presents with fever, mild respiratory
C Right sided isomerism is associated with
distress, started on oral antibiotics, showing initial
improvement but deteriorating again with fever, Asplenia
wheeze and exaggerated breathlessness. X-ray C A 5 years old child presents with burn, burn area
chest-Hyperlucency and PFT- obstructive pattern. equal to the size of his palm is equal to 1% body
Diagnosis surface area.
Bronchiolitis obliterans
C Most common tumor associated with NF-1 in a child ALL INDIA 2010
Juvenile myelomonocytic leukemia. C Health status of a child under 5 years of age is
C Most common cause of meningoencephalitis in affected by
children Infections, low birth weight and malnutrition.
Enterovirus C Investigation of choice for a neonate presenting
C Complete surgical removal of craniopharyngioma with fever, lethargy, abdominal distension, vomi-
leads to multiple endocrinopathies in a six years old ting, constipation and provisionally diagnosed as
child. Hormone replaced first volvulus neonatarum with suspected perforation
Plain X-ray.
Hydrocortisone
C A 10 years old boy presenting with seizure has BP
C One year old female child presents with short
in upper extremity 200/140 mm Hg and non-
stature, leathargy, constipation, palpaple goitre,
palpable femoral pulses. Diagnosis
low T4 and elevated TSH. Diagnosis
Coarctation of aorta.
Thyroid dyshormonogenesis.
C A 6 years old asymptomatic girl have persistent
C A 8 days male infant presents with vomiting, hypertension, no significant history, normal
lethargy, dehydration, features of shock, hyperpig- urinary examination. Diagnosis is
mentation of genital skin, normal external genitalia, Renal parenchymal disease.
unremarkable abdominal examination, blood
C Best management of a child presenting with
sodium 123 mEq/L, blood potassium 7 meq/L and
intermittent episodes of left sided flank pain, USG -
hypoglycemia. Diagnosis
large hydronephrosis with dilated renal pelvis and
Congenital adrenal hyperplasia. cortical thinning with normal ureter and kidney
C Common causes of ambiguous genitalia in a female differential function 19%
child Pyeloplasty
Fetal placental aromatase def., congenital adrenal C Diagnosis of a child presenting with hepatomegaly
hyperplasia and WNT-4 gene mutation. and hypoglycemia and no improvement in blood
C A 2 weeks old male baby presents with history of sugar even after epinephrine administration
seizures, serum calcium 5.5 mg/dl, serum von Gierke’s disease.
226 AIIMS and All India PGMEE—Review Questions

C Major risk factors associated with a greater risk of C True about childhood polycystic kidney disease
developing epilepsy after febrile seizures Pulmomany hypoplasia may be seen, renal cysts
Complex febrile seizure, developmental abnormali- are present at birth and congenital hepatic fibrosis
ties and positive family history of epilepsy (Mn: CDEF). may be seen.
C A child presents with short episodes of vacant stare C A 7 years old girl is brought with complaints of
several times a day. The vacant episode begins generalized swelling of the body. Urinary
abruptly and child remains unresponsive during examination reveals grade 3 proteinuria and
the episode. There is no associated history of aura or presence of hyaline and fatty casts. History of
pastical confusion. Diagnosis hematuria absent. True is
Absence seizures No IgG or C3 deposits on renal biopsy.
C Indicated in juvenile myoclonic epilepsy (JME) C True about neuroblastoma
Topiramate, Valproate and Zonisamide. Most common extracranial solid tumor in childhood
C A child presents with history of seizures and mental > 50% present with metastasis at time of diagnosis
retardation and multiple hypopigmented macules and often encase aorta and its branches at time of
on clinical examination. Diagnosis diagnosis.
Tuberous sclerosis. C Marker for neural tube defects
C Most common microbial agent associated with Increased acetylcholinesterase.
recurrent meningitis due to CSF leaks
Pneumococci ALL INDIA 2008
C A two years old child with a long history of
purulent nasal discharge and fever, now presents C True about Gomez classification
with conjunctival congestion, edema, fever 102°F/ Prognostic value for hospitalized children, based
103°F, WBC 12,000/mm3, negative culture for eye on 50th centile Boston standards and between 75
discharge, and opacification of ethmoidal sinus on and 89 % implies mild malnutrition.
X-ray. Next investigation C The most common cause of short stature
CT scan Constitutional
C A boy presents with weakness in lower limbs, calf C A very low birth weight preterm baby is on
hypertrophy, positive Gower’s sign and elevated ventilator for respiratory distress. Baby presents
CPK value of 10,000. Diagnosis is with clinical features of necrotizing enterocolitis
Duchenne muscular dystrophy. with perforation. Appropriate management is
C Primary metabolic bone disorder is scurvy is Peritoneal drainage
Decreased osteoid matrix formation. C Prebiotics is beneficial in
Necrotizing enterocolitis
ALL INDIA 2009 C Most commonly associated with coarctation of
C The most common fetal response to acute hypoxia aorta
Bradycardia Bicuspid aortic valve
C Vaccine with maximum efficacy after single dose C Right sided aortic arch is most strongly associated
Measles with
C Nutritional assessment methods indicating inade- Truncus arteriosus
quate nutrition are C True statement about PDA
Increased 1–4 years mortality rate, low birth weight It is a common heart lesion in rubella, hypoxia and
and decreased weight for height. immaturity are important in maintaining the
C Treatment options of hyperleukocytosis in a child patency and treatment is closure of defect by
with acute myeloid leukemia ligation and division of ductus.
Allopurinol, alkalinization and IV fluids. C A child presents with severe respiratory distress 2
C Prophylaxis of a neonate born to HBsAg positive days after birth. On examination- scaphloid
mother includes abdomen, decreased breath sound on left side. He
Both vaccines and immunoglobulin. was managed by prompt endotracheal intubation.
Pediatrics 227

After ET tube placement maximal cardiac impulse C Syndromes associated with NHL
shifted further to right. Next step in management Down’s syndrome, Patau syndrome and Klinefelter’s
Confirm position of endotracheal tube (not by syndrome (Mn: DPK).
X-ray) > Remove tube and reattempt intubation > C Good prognostic factors for childhood ALL
nasogastric tube insertion and bowel decompression. Female sex, hyperdiploidy and t(12, 21) translocation.
C A 6 years old child presenting with recurrent episodes C Minor criteria for diagnosis of RF according to
of gross haematuria for 2 years is suffering from modified Jone’s criteria
IgA nephropathy Fever
C True about Kawasaki disease (Note: Other minor criteria are arthralgia, increased
Associated with coronary artery aneurysm in up to ESR, increased creative protein, prolonged PR
25% of untreated cases. interval in ECG)
C True about 21 hydroxylase deficiency C Cardiomyopathy is a feature of
Most common cause of CAH in children, affected Duchenne’s muscular dystrophy, Friedrich’s ataxia
females present with ambiguous genitalia, and and Pompe disease.
affected males present with precocious puberty. C Most common mode of treatment for 1 year old
C Most common type of seizure in neonate child with asthma
Subtle Inhaled short acting β-2 agonist.
C Most common CNS involvement in group of intra- C Leukotrine receptor antagonist indicated in
uterine congenital infections-CMV and Toxoplamosis bronchial asthma
(most common symptomatic CNS involvement in Zafirlukast
HSV and Rubella) C Diagnosis of a child presenting with 3 days history
C True about cerebellar astrocytoma in pediatric age of upper respiratory tract infection with stridor
group Laryngotracheobronchitis.
Usually low grade tumors, have a good prognosis and C A newborn presents with treatment resistant CHF,
more commonly seen in first and second decades. bulging anterior fontanelles with bruit on
C Cyst in a child located and associated with vertebral auscultation and transfontanella USG finding of
defects hypoechoic midline mass with dilated lateral
Neuroenteric cyst ventricles. Diagnosis
C Treatment for nocturnal enuresis with least relapse Vein of Galen malformation.
rate C Most common microorganism causing meningitis
Bell alarm systems in a 1 year old child
C Resistant Plasmodium falciparum malaria in pediatric Pneumococcus
age group treated by C Jitteriness can be distinguished from seizures by
Clindamycin Abnormality of gaze, autonomic disturbance and
sensitivity to stimulus.
ALL INDIA 2007 C Diagnosis of a child presenting with microcephaly,
blue eyes, fair skins mental retardation and positive
C Age of a baby who has developed mouthing but has ferric chloride test
not developed stranger anxiety, likes and dislikes Phenyl ketonuria
for food
(Note: Melanin is synthesized from tyrosine is
5 months synthesized from phenylalanine. Phenylalanine
C Milestone developing first hydroxylase deficiency in phenylketonuria affects
Mirror play 6 months, crawling…., creeping…., this reactions).
Pincer graps. C Darkening of urine on standing is associated with
C Best indicator of long term nutritional status Alkaptonuria.
Height for age C Investigation for a premature baby of 34 weeks deve-
C Most common tumor in children is loping bullons lesion on skin and periostitis on X-ray
Leukemia VDRL for mother and baby.
228 AIIMS and All India PGMEE—Review Questions

C Drug treatment for a child presenting with drowsi- C The earliest indicator of response after starting iron
ness, decreased deep tendon reflexes, seizures, in young girl with iron deficiency
history of constipation and lines on gums Increased reticulocyte count.
EDTA C Late onset hemorrhagic disease of newborn is
C Cat eye syndrome characterized by
Partial trisomy 22 Onset at 4–12 weeks of age, intracranial hemorrhage
C Autosomal dominant conditions are can occur and IM vitamin K prophylaxis at birth has
Marfan’s syndrome, osteogenesis imperfecta and a protective role.
Ehlers-Danlos syndrome. C True for TOF
C In an AR disorder, one parent is normal, other is RVH, VSD and PS
carrier and child is also affected (Note: Overriding of aorta also occurs in TOF)
Uniparental disomy C Blalock and Taussing shunt is done between
C Father to none and mother to all, inheritance is Aorta to pulmonary artery.
Mitochondrial inheritance. C Left atrium in enlarged in
Aortopulmonary window, PDA and VSD.
C 1 months old boy presents with failure to thrive,
ALL INDIA 2006
features of congestive failure and femoral pulses
C A normally developing 10 months old child should feable as compared to branchial pulses. Diagnosis
be able to do Coarctation of aorta
Stand alone, play pick a boo and pick up a pellet C The most important determinant of prognosis in
with thumb and index finger. Wilm’s tumor
C Features of prematurity in a neonate Histology
No creases on sole, abundant lanugo and empty C Malignant tumor of childhood metastasizing to
scrotum. bones most often is
C Principal mode of heat exchange in an infant Neuroblastoma
incubator C A child with a small head, minor anomalies of the
Conduction, evaporation and radiation. face including thin upper lip, growth delay and
C Characteristics of Autism developmental disability can have
Delayed language development, repetitive behavior Chromosomal syndrome, Mendelian syndrome
and severe deficit in social interaction. and teratogenic syndrome.
C Features of Down’s syndrome C An affected male infant born to normal parents
Hypothyroidism, Brushfield’s spots and VSD. could be an example of
C Specific neonate malformation in maternal insulin AR disorder, polygenic disorder and vertically
dependent DM transmitted disorder.
In a family, the father has widely spaced eyes,
Caudal regression
C
increased facial hair and deafness. One of three
Potassium requirement of child
children has similar feature. The mother is normal.
C
1–2 mEq/kg Pattern of inheritance
C Sodium content of ReSoMal Autosomal dominant
45 mmol/L
(Note: ReSoMal is rehydrated solution for malnouri-
shed child) ALL INDIA 2005
C The most common etiology for acute bronchiolitis C Bart’s hydrops fetalis is lethal because
in infancy Hb Bart’s cannot release oxygen to fetal tissues.
Respiratory syncytial virus. C The coagulation profile in a 13 years old girl with
C True about Mumps menorrhagia having von Willebrands disease is
Meningoencephalitis can precede parotitis. Isolated prolonged aPTT and normal PT.
Pediatrics 229

C The most common leukocytoclastic vasculitis ALL INDIA 2004


affecting children
HSP C A two months old child able to
C Diagnosis of β-thalassemia is established by Sustain head level with the body when placed in
Hb electrophoresis. ventral suspension.
C The most common type of TAPVC C Webbing of neck, increased carrying angle, low
Supracardiac posterior hair line and short fourth metacarpal are
characteristic of
C Intestinal enzyme generally deficient in children
following an attack of severe infectious enteritis is Turner syndrome
Lactase C Ataxia telangiectasia is characterised by
C Most common renal cystic disease in infants IgA deficiency, chromosomal breakage and chronic
Unilateral renal dysplasia. sinopulmonary disease.
C Child with recurrent UTI is most likely to show C A 10 years old boy presents with fracture femur, Hb
Vesicoureteric reflux. 11.5 gm/dl, ESR 18 mm 1st hr, serum calcium 12.8
mg/dl, serum phosphorus 2.3 mg/dl, alkaline,
C The most common cause of renal scarring in a 3
phosphate 2.8 KA units and blood urea 32 mg/dl.
years old child is
Diagnosis
Vesicoureteral reflux induced pyelonephritis.
Hyperparathyroidism
C In a child, non-functioning kidney is best diagnosed
by C A 9 months old boy of Sindhi parents present with
c/o progressive lethargy irritability and pallor since
DTPA renogram
6 months of age, Hb-3.8 gm/dl, MCV-58 fl, MCH-
The most common presentation of a child with
19.4 pg/dl, normal osmotic fragility, and expansion
C
Wilm’s tumor
of erythroid marrow on X-ray skull. Diagnosis
As asymptomatic abdominal mass.
Iron deficiency anemia
C The most common malignant neoplasm of infancy
C A 5 years old child present with history of fever off
Neuroblastoma
and on for past 2 weeks, and petechial spots all over
C Common cause of congenital hydrocephalus the body and increasing pallor for past 1 month,
Aqueductal stenosis. O/E-splenomegaly of 2 cm below costal margin.
C In unconjugated hyperbilirubinemia, risk of Diagnosis
kernicterus increases with the use of Acute leukemia
Sulphonamide C A five days old, full term male infant was severely
C The approach to a neonate presenting with vaginal cyanotic at birth. Prostagladin E was administered
bleeding on day 4 of life is initially and later balloned atrial septostomy was
No specific therapy done which showed improvement in oxygenation.
C Blood specimen for neonatal thyroid screening is Diagnosis
obtained on Transposition of great vessels.
Cord blood C Coer en sabot shape of the heart is seen in
C Therapies required in a 4 hours old infant with TOF
severe birth asphyxia C An infant develops cough and fever with X-ray
Calcium gluconate, glucose and normal saline. suggestive of bronchopneumonia. Causative
C Drug used for fetal therapy of CAH viruses are
Dexamethasone Parainfluenza virus, influenza virus A and RSV.
C The karyotype of a patient with androgen insensi- C In neonatal cholestasis, if serum GGT is more than
tivity syndrome 600 IU/L, diagnosis is
46 XY Biliary atresia
C The protective effects of breast milk is associated C The neonatal kidney achieves concentrating ability
with equivalent to adult kidney by
IgA antibodies. One year of age.
230 AIIMS and All India PGMEE—Review Questions

C Neural tube defects are C Therapy of 5 years old boy passing 18 loose stools in
Encephalocele, myelomeningocele and anencephaly. 24 hours and vomiting twice in last 4 hours when he
C Features of absence seizure is irritable but drinking fluid is
Usually seen in childhood, 3 Hz spike wave in EEG Oral rehydration therapy
and precipitation by hyperventilation. C A 12 years old boy with no major medical illness
C A 1 year old child presenting with 2 week history of complains of breaking out with blocky area of
fever, vomiting and altered sensorium with cranial erythema that are pruritic over skin of his arm, leg
CT scan revealing basal exudates and hydrocephalus. and trunk everytime within an hour of eating sea
Etiological agent responsible foods. Diagnosis
Mycobacterium tuberculosis. Localised anaphylaxis
C Enzyme replacement therapy is available for C A 45 days old infant presents with icterus, followed
Gaucher’s disease 2 days after with symptoms and signs of acute liver
C Duchenne’s muscular dystrophy is a disease of failure, HBsAg positive. The mother is HBsAg
Sarcolemmal proteins. carrier. The mother’s hepatitis B serological profile
(Note: Dystrophin) is
C Bacterial meningitis is children (2 months–12 years HBsAg and HBeAg positive.
of age) is usually due to C A 5 years old asymptomatic boy, immunized with
Pneumococcus, Meningococcus and H. influenza 3 doses of recombinant Hep B vaccine at age of 1
type b. year, is detected to be HBsAg positive on 2 separate
C Essential features of ADHD occasions during a screening programme for
hepatitis B. His mother was treated for chronic
Hyperactivity, impulsivity and lack of concentration.
hepatitis B infection around the same time. Next
C A 20 years old woman had premature rupture of done
membrane and delivered a male child who became
Obtain anti-HBsAg antibody levels.
lethargic and apnoeic on the first day of birth and
went into shock. The mother had a previous history C A 18 years old boy presents with abdominal pain,
of abortion 1 year back on vaginal swab culture fever and bloody diarrhea for 18 months, 100 cm
growth of β-hemolytic colonies on blood agar was height, 14.5 kg weight, stool culture negative for
found. Gram staining shows Gram-positive. known enteropathogens, sigmoidoscopy finding
Organism is normal, mantoux test negative. During the same
period, child had an episode of renal colic and
Streptococcus agalactiae
passed urinary gravel. Diagnosis
C The most common agent associated with neonatal
Crohn’s disease
bacterial meningitis
C Young girl from Bihar presents with three episodes
Streptococcus agalactiae.
of massive hematemesis and malena, no history of
C β-lactamase producing ampicillin resistant strain of jaundice, O/E-large spleen, mild ascites, non-
H. influenzae has been isolated from CSF of a 2 years palpable liver, USG-portal vein not visualized, LFT
old boy suffering from meningitis. Appropriate normal and endoscopy revealing esophageal
antimicrobial of choice here is varices. Diagnosis
3rd generation cephalosporin > trimethoprim- Portal HTN due to extrahepatic obstruction.
sulphamethaxazole combination.
C True about pyuria in children
Infection can occur without pyuria, pyuria may be
ALL INDIA 2003 present without UTI, and isolated pyria is neither
C Transient myeloproliferative disorder of newborn confirmatory nor diagnostic for UTI.
is associated with C Most common cause of abdominal mass in neonates
Down’s syndrome Multicystic dysplastic kidneys.
C Features of Noonan syndrome C A 10 years old girl presents with swelling of one
AD, crypto-orchidism and hypertrophic cardiomyo- knee joint. Differential diagnosis are
pathy. JRA, TB and vilonodular synovitis.
Pediatrics 231

C A 1 month old baby presents with frequent C Six months old girl presents with recurrent UTI,
vomiting, failure to thrives moderate dehydration, USG- bilateral hydronephrosis, MCU-bilateral
blood Na+–122 mEq/L and blood K+–6.1 mEq/L. grade IV VUR. TOC
Diagnosis Ureteric reimplantation
21 hydroxylase deficiency. C The most common cause of ambiguous genitalia in
C A 2 months old baby presents with history of a newborn
jaundice, turmeric colored urine, pale stool since 21 hydroxylase deficiency
birth, O/E-liver span of 10 cm, firm in consistency, C Primary site of hematopoiesis in the fetus before
spleen of 3 cm. Most specific investigation for mid pregnancy
establishing diagnosis
Liver
Peroperative cholangiogram.
C Complications in the newborn of a diabetic
mother
ALL INDIA 2002 Hyperbilirubinemia, hypocalcemia and hypomag-
C Sign warranting further evaluation of develop- nesemia.
mental status in a healthy 12 weeks old infant C Administration of glucose solution is prescribed for
Doest not vocalize Child of a diabetic mother, history of hypoglycemia
C 6.7 kg weight 2 years old boy present with history and neonates.
of vomiting and diarrhea for last 8 days. O/E skin C Closure of PDA in a premature infant can be
pinch over anterior abdominal wall goes back stimulated by
quickly to its original position. Interpretation Prostaglandin inhibitors
Skin pinch cannot be evaluated in this child. C A 3 years old boy presents with fever, dysuria, gross
(Note: because child is severely malnourished) hematuria, O/E-Prominent suprapubic area dull
C An infant presents with history of seizures and skin on percussion, urinalysis-red blood cells but no
rashes, metabolic acidosis and increased blood proteinuria. Diagnosis
ketone levels. Diagnosis Posterior urethral valves
Multiple carboxylase deficiency. C True about β-thalassemia major
C True about RDS Microcytic hypochromic anemia, splenomegaly
Usually occurs in neonates born before 34 weeks of and target cells on peripheral smear.
gestation, is more common in babies born to C Diagnosis of congenital CMV infection in a neonate
diabetic mothers and leads to cyanosis. established by
C Common manifestations of congenital rubella Urine culture of CMV, intranuclear inclusion bodies
Deafness, mental retardation and PDA. in hepatocytes and CMV viral DNA in blood by PCR.
C A 10 years old boy presents with fever, bilateral C Type of Hb used in diagnosis of β-thalassemia trait
cervical lymphadenopathy, prior history of HbA2
sore throat, no hepatomegely, peripheral blood C Characteristic feature of juvenile myoclonic epilepsy
smear showing >20% lymphoplamacytoid cells. Myoclonic seizures frequently occur in morning.
Diagnosis
C A 15 years old girl presents with history of recurrent
Infectious mononucleosis. epistaxis, hematuria, hematochezia history of
C Most common genetic cause of liver disease in profuse bleeding from the umbilicat stump at
children birth. Inv. Normal PT, aPTT, TT and fibrinogen
α 1 antitrypsin deficiency. levels. Platelet count as well as platelet function test
C Childhood tumors most frequently metastasizing normal. Urea clot lysis test positive. Clotting factor
to the bone likely to be deficient
Neuroblastoma Factor XIII
232 AIIMS and All India PGMEE—Review Questions

15

Dermatology

AIIMS NOVEMBER 2015 C Figure showing multiple round papules were


found on dorsum of hand, flexure surface of
C A young patient of neurocysticercosis was taking forearm and shaft of penis. Most likely diagnosis
medications for seizure for last 1 month. He has Lichen nitidus
developed generalized peeling of skin (except palm C Figure showing a patient with bulla involving more
and soles). Most probably it is a case of than 30% body surface area and erosion of the lips
TEN for the past 7 days. Most probable cause
C A lady presents with history of burning sensation Drugs
on eating spicy food. On examination of buccal C Figure shows a lady with bluish lesion over left side
mucosa, bilateral white lacy streaks are seen. There of forehead and left eye. Diagnosis
is no history of tobacco abuse but amalgamated Nevus of Ota
third molar is present. Diagnosis C A 26 years old man from Bihar presents with nodules
Lichen planus over face, and hypopigmented normoaesthetic
C Type of oral candidiasis not presenting with white macules but no nerve thickening over back of neck.
patch There is history of prolonged fever in the childhood.
Chronic atrophic candidiasis. Most likely diagnosis
(Note: Red lesion is seen in chronic atrophic Post-kala-azar dermal leishmaniasis.
candidiasis.)
AIIMS NOVEMBER 2014
AIIMS MAY 2015
C True about lepromatous leprosy
C Figure showing multiple pustules in a pregnant
woman with diabetes. Drug that can be used ENL seen in > 50% of cases.
Cyclosporine C Nose is most commonly involved in stage of
C A patient presented with itchy tense blisters on the syphilis
skin and atheromatous plaque. Most likely diagnosis Congenital syphilis
Bullous pemphigoid. C A 20 years old man presents with a hyperpigmented
C Figure showing a lady with red macular lesions on lesion on the back. Test helpful for the diagnosis
central part of face around the cheek and chin with Potassium hydroxide mount.
flushing on sun exposure and increasing with C A 15 years old girl presents with a 9 months history
emotional disturbance of several comedones, cystic and nodular lesions
Acne rosacea. over face with intercommunicating sinuses. The
C Figure showing hypopigmented lesion on chin and best treatment for her would be
neck on one side with leukotrichia on the depigmen- Isotretinoin
ted area. Most likely diagnosis is C A 25 years old girl presents with papules, erythema
Segmental vitiligo. and telangiectasia over the face and history of

232
Dermatology 233

flushing and burning sensation on exposure to sun C A patient presented with thinning of nails
and on any emotional disturbance. The most likely and onycholysis. Other findings observed in
diagnosis him
Rosacea Basal degeneration > Violaceous papules
C Not a variant of lichen planus (Note: Lichen planus).
Lichen scrofulosorum C A patient presents with focal alopecia areata.
C Nikolsky’s sign is seen in Associated with
Pemphigus vulgaris Nail pitting, atopy and exclamatory mark.
An elderly patient presents with itchy tense blisters
A man presents with rashes on face and also
C
C
on normal looking skin as well as on urticarial
complains of decreased mental function. He is also
plaques. The most likely diagnosis
having few macular lesions on his skin. On CT scan,
Bullous pemphigoid
intracranial calcification was seen. His wife is
C A male patient presents with recurrent erythema- normal. His 10 years old daughter is also normal
tous plaque at the same site on glans penis, which but his 6 years old son is also having similar skin
sometimes forms a bulla and heals with hyperpig-
lesions. Most likely diagnosis
mentation. The most likely diagnosis
Autosomal dominant inheritance
Fixed drug reaction
(Note: Tuberous sclerosis)
AIIMS MAY 2014
AIIMS MAY 2013
C Olympian brow and rhagades are seen in
C A middle aged man presents with multiple painful
Congenital syphilis
blisters on a erythematous base along the T3
C A 60 years old gentleman presents with painful, dermatome on the trunk. Most likely etiological
grouped vesicles over erythematous plaques in T3 agent
dermatome region of trunk. Likely causative Varicella Zoster virus.
organism
C A young 8 years old boy presents with multiple
Varicella zoster discrete, shiny, pin head papules on dorsal aspect
C A 19 years old boy presents with several comedones, of hand, forearms and penis. Most likely diagno-
papules and pustules on face and trunk. Drug of sis
choice Lichen nitidus
Topical retinoic acid + oral doxycycline. C A 20 years old girl presents with a relatively
C A 37 years woman presents with multiple, linear, painless ulcer of 3 cm on the labia majora with
itchy wheals, with itching for 30 minutes at the site. raised margins. Most likely organism
The most likely diagnosis Treponema pallidum infection.
Dermatographic urticaria. C The Ridley- Jopling classification for leprosy is
C A young 8 years old boy presents with multiple based on
discrete, shiny, pin head papules on dorsal aspect of Histopathological, clinical, bacteriological and
hand, forearm and shaft of penis. Most likely immunological.
diagnosis C A male patient presents with sudden onset painful
Lichen nitidus. tense blisters with urticarial plaque. Best investiga-
C A 30 years old man presents with flaccid bullae on tion
an erythematous base and erosions over the oral Direct immunofluorescence.
mucous membrane. The blisters developed painful C A young male patient presents with painful ulcers
erosions on rupture. The immunofluorescent on the mouth and glans penis with blurred vision
examination of skin biopsy shows and history of recurrent epididymitis. Most
Fishnet IgG deposition in epidermis probable diagnosis
(Note: Pemphigus vulgaris) Behçet syndrome.
234 AIIMS and All India PGMEE—Review Questions

AIIMS NOVEMBER 2012 coalesced to form large sheet. The surface of the
macule showed fine scaling. Similar episode one
C A patient presents with oral ulcer and flaccid skin year ago subsiding with treatment. Investigation
bullae that are slow to heal. The lesion is that will confirm the diagnosis
Suprabasal KOH mount
C A young man presents with asymptomatic macules C Associated with sun exposure
and erythematous painless lesion over glans with
Actinic keratosis
generalized lymphadenopathy. Treatment of
choice
Benzathine penicillin. AIIMS NOVEMBER 2011
C A man has recurrent urticaria following exercise
and coming out in sun, diagnosis is C Diagnosis of a patient presenting with history of
Cholinergic urticaria recurrent oral ulcers (ulcers are small with a
yellow floor surrounded by an erythematous halo
on the lips) and multiple tender nodules on the
AIIMS MAY 2012 shin
C An infant has papulovesicular lesions on palms, Behçet’s syndrome
soles, face and trunk. The diagnosis would be C The only definitive indication of systemic corticoste-
Scabies roids in psoriasis
C A young boy had itchy, excoriated papules on the Impetigo herpatiformis
forehead and the exposed parts of the arms and legs (Note: It is pustular psoriasis in pregnancy)
for three years. The disease was most severe in the C Intercellular IgG deposition in epidermis in
rainy season improved completely in the winter. Pemphigus
The most likely diagnosis is C Granular IgA deposition at dermal papilla in
Insect bite hypersensitivity. Dermatitis herpetiformis
C A 25 years old sexually active wife of a long distance
truck driver presented with copious vaginal
discharge of 2 days duration. Syndromic manage- AIIMS MAY 2011
ment is
C Neonatal fat necrosis resembles
Azithromycin + metronidazole + fluconazole.
Poststeroidal panniculitis.
A lady developed pigmentation on the bridge of the
Fine reticular pigmentation with palmar pit are
C
C
nose and cheeks on exposure to sunlight. The most
seen in
likely diagnosis is
Dowling degos disease
Chloasma
C Child with erythematous non-blanching bosselated
C Characteristic feature of borderline leprosy
lesion on right side of face. Treatment is
Inverted saucer shaped lesions.
Flash light pumped dye laser.
C Multiple hypoaesthetic, hypopigmented macules
on right lateral forearm with numerous acid fast C True about incontinent pigmenti
bacilli is indicative of X linked dominant, primary skin abnormality and
Borderline leprosy avascularity of peripheral retina.
C A 60 years old male patient presents with discolora- C Child presents with linear verrucous plaques on the
tion, thickening and tunneling of 2 fingernails and trunk with vacuolization of keratinocytes in
1 toe nail. Investigation done stratum spinosum and stratum granulosum.
KOH mount Diagnosis is
C A 26 years old man presented with multiple, small Verrucous epidermal nevus.
hypopigmented macules on the upper chest and C True about acrodermatitis enteropathica
back for last three months. The macules were Low serum zinc levels, symptoms improve with
circular, arranged around follicles and many had zinc supplementation and autosomal recessive.
Dermatology 235

AIIMS NOVEMBER 2010 AIIMS NOVEMBER 2008


C Chemical peeling is done by C Drug causing hypopigmented lesion at centre of
Trichloracetic acid, carbolic acid and kojic acid. forehead of a girl
C A farmer has a single warty lesion on leg. Most Paratertiary butylphenol.
likely lesion is C Seen in Reiter’s syndrome
Tuberculosis verrucosa cutis. Oral ulcers, keratoderma blennorrhagica and
C A teenage girl with moderate acne complains of circinate balanitis.
irregular menses. Drug of choice is C A person suffering from B27 associated reactive
arthritis, urethritis and conjunctivitis. Most likely
Cyproterone acetate
organism in this case is
Ureaplasma urealyticum.
AIIMS MAY 2010 C In congenital dystrophic variety of epidermolysis
bullosa, mutation is seen in the gene coding for
C Primary skin diseases are
Collagen type 7.
Bowmen’s disease, psoriasis and lichen planus.
C A 26 years old male with fever and malaise for
2 weeks, arthritis of ankle joint and tender erythe- AIIMS MAY 2008
matous nodules over the shin. Diagnosis is C Auspitz sign is seen in
Erythema nodosum Plaque psoriasis
C Most common cause of plant induced dermatitis in
AIIMS NOVEMBER 2009 India
Parthenium
C Young boy with boggy swelling of scalp with multiple
discharging sinuses with cervical lymphadenopathy
with easily pluckable hair. Next investigation is AIIMS NOVEMBER 2007
KOH mount C Cicatricial alopecia is caused by
(Note: Tinea capitis) Lichen planus, DLE and pseudopalade.
C Flaccid bullae lesion with oral mucosal lesion. C True about vitiligo are
Immunofluorescence finding is Genetic predisposition present, narrow range UV-
Fishnet IgG in epidermis. B is very effective and topical steroid is very useful
(Note: Pemphigus vulgaris) for localized lesion.
C Sexually active male comes with complaints of C A 20 years old lady with diffuse hair loss for 1
recurrent ulcers over glans which heals with month, with past history of enteric fever, before 4
hyperpigmentation. It is a case of months, likely cause is
Fixed drug eruption Telogen effluvium
C Skin manifestations of dermatomyositis
Gottron’s patches, mechanic’s hands and periungual AIIMS MAY 2007
telangiectasia. C Tuberculides are seen in
(Note: Salmon’s patch is seen in still’s disease) Lichen scrofulosorum
C A 3 years old child presents with eczematous
AIIMS MAY 2009 dermatitis on extensor surfaces with family history
C Patch test is done to document of bronchial asthma. Likely diagnosis is
Delayed type hypersensitivity reaction. Atopic dermatitis
C Deposition of IgA in dermal papilla shown by
Dermatitis herpatiformis. AIIMS MAY 2006
C Intraepidermal IgG deposition is seen in C Chancre redux is a clinical feature of
Pemphigous Early relapsing syphilis.
236 AIIMS and All India PGMEE—Review Questions

C Anagen phase of hair is along the basement membrane. Most likely


Phase of activity and growth diagnosis
C Max Joseph’s space is a feature of Bullous pemphigoid
Lichen planus
C Ivermectin is given for AIIMS MAY 2004
Scabies C Multiple itchy erythematous wheals all over the
C Drugs used in Type II lepra reaction body for 2 days in young boy with no respiratory
Chloroquine, corticosteroids and thalidomide. problem is best treated by
C The main cytokine involved in ENL reaction Antihistamines
TNF alpha C Six weeks after starting multibacillary multidrug
C Investigations used for diagnosis of leprosy therapy in a borderline leprosy patient, he develops
FNAC, slit skin smear and skin biopsy pain in the nerves and redness and swelling of skin
C Diagnostic test for air borne contract dermatitis lesions. This is managed by
Patch test Analgesics, rest to the affected limb and systemic
corticosteroids.
AIIMS NOVEMBER 2005
C Itchy, excoriated papules on the exposed parts of a
25 years old man which worsens in rainy season
C Exclamation mark hair is a feature of and completely improves in winter is a case of
Alopecia areata Insert bite hypersensitivity.
C Pautrier’s microabscess is a feature of C Drug treatment for severe air borne contact
Mycosis fungoides dermatitis in an old man with uncontrolled HTN
C Adenoma sebaceous is a feature of and DM is
Tuberous sclerosis Azathioprine

AIIMS MAY 2005 AIIMS NOVEMBER 2003


C An adult male presents with joint pains invol- C A 30 years old adult man presents with large
ving both the knees and ankles, redness of eye spreading exuberant ulcer with bright red
and skin lesion 4 weeks after burning micturi- granulation tissue over glans penis but no
tion and urethral discharge. It is most probable lymphadenopathy. Causative agent
a case of Calymmatobacterium granulomatis.
Reiter’s syndrome C A 20 years old man presents with severe itching and
C Drug of choice for dermatitis herpetiformis white scaly lesion in the groin for past month.
Dapsone Likely organism
C Pinch purpura diagnostic of Trichophyton rubrum
Primary systemic amyloidosis. C There is gradually progressive plaque on the
buttock of a 12 years old child for last 2 years. 15 cm
diameter annular shaped plaque had scarring at
AIIMS NOVEMBER 2004 centre and crusting and induration at periphery.
C A 40 years old lady presents with erythema and Diagnosis is
swelling of periorbital region and papules and Lupus vulgaris
plaques on the dorsolateral aspect of forearms and C A man presented with scaly circular multiple small
knuckles with ragged cuticles for last 6 months but hypopigmented macules on upper chest and back
no muscle weakness. Diagnosis is recurring after one year. The macules were
Dermatomyositis arranged around follicles and many had coalesced
C A 40 years old gentleman presents with multiple to form large sheets. Appropriate confirmatory
blisters over the trunk and extremities. Direct investigation for diagnosis
immunofluorescence finding- Linear IgG deposits KOH mounts of scales.
Dermatology 237

C A 20 years old girl develops small itchy wheals after C Erythematous scaly lesions on external aspect of
physical exertions, walking in the sun, eating hot elbows and knee of a patient can be clinically
spicy food and when angry. She is suffering from diagnosis by
Cholinergic urticaria. Auspitz sign
C Multiple hypoesthetic erythematous large plaques C Actinic keratosis is seen in
with elevated margins on trunk and extremities in Squamous cell carcinoma
a 40 years old male with bilateral enlargement of C Wood’s lamp light is used in diagnosis of
both ulnar and lateral popliteal nerves Tinea capitis
Borderline lepromatous leprosy. C Drug of choice for multiple nodulocystic lesions on
the face of patient
AIIMS MAY 2003 Retinoid

C Drug indicated in pityriasis vesicolor


ALL INDIA 2012
Ketoconazole
C A sexually active adult male present with single C Total skin electron irradiation is used for the
painless ulcer on external genitalia. Investigation treatment of
required in this case Mycoses fungoides
Scrapings from ulcer for dark field microscopy. C One child presents with itchy vesicles, tense bullae,
C Podophyllum resin in indicated in linear IgA deposition in basement membrane zone
on direct immunofluorescence. Diagnosis
Condyloma acuminata
Chronic bullous disease of childhood.
C Oral erosions, flaccid bullae in the skin and
A neonate born out of non-consanguinous marriage
intraepidermal on histopathological examination
C
shows eroded skin and formation of blisters
in 24 years old girl indicates
whenever handled with history of one sibling
Pemphigus vulgaris
having died with same presentation. Diagnosis
Epidermolysis bullosa
AIIMS NOVEMBER 2002 C A young boy presents with erythematous plaques
C Multiple painful grouped vesicles on the D10 derma- and collarate of scales mainly on the trunk.
tome in a 40 years old male patient can be diagnosed as Diagnosis
Herpes zoster Pityriasis rosea
C Multiple grouped papulovesicle on both elbows, (Note: HHV7)
knees, buttocks and upper back with severe itching C A patient presents with erythematous scaly lesion
in 28 years old man. Diagnosis is on trunk with white plaques in oral mucosa or
Dermatitis herpetiformis. cavity and same erosive lesions in perianal area.
Diagnosis
C Multiple itchy papules on genitalia and fingers of a
child with similar finding in his sibling indicates the Secondary syphilis
diagnosis of
ALL INDIA 2011
Scabies
C Layer of epidermis not developed in very low birth C Lines of Blaschko represents
weight neonate within 7days Lines of development.
Stratum cornium C Pseudoisomorphic phenomenon is seen in
C Scabies is an example of Plane warts
Water washed disease (Note: also in eczema and Molluscum contagiosum)
C Cicatrising alopecia with perifollicular blue grey
patches is most commonly associated with
AIIMS MAY 2002
Whitish lesion in buccal mucosa.
C Treatment of pustural psoriasis among the following C Erythema nodosum is seen in
Retinoid Pregnancy, tuberculosis and SLE.
238 AIIMS and All India PGMEE—Review Questions

C A young female patient presents with a history of ALL INDIA 2008


fever and nodular lesion over shin. Histopathology
reveals foamy histiocytes with neutrophilic C Patchy loss of hair on scalp, eyebrows and beard.
infiltration. There is no evidence of vasculitis. Most He also gives history of rapid graying of hair in a
probable diagnosis is few areas. Diagnosis is
Erythema nodosum. Alopecia areata
C A young man presents with maculopapular rash 2 C A 40 years old female developed persistent oral
weeks after healing of a painless ulcer in genital ulcers followed by multiple flaccid bullae on trunk
area. Cause is and extremities. Microscopic examination of skin
Treponema pallidum biopsy immunofluorescence showed intercellular
(Note: Secondary syphilis) IgG deposit in the epidermis and suprabasal split
with acantholytic cells. It is a case of
Pemphigus vulgaris
ALL INDIA 2010
C Heterosexual male with multiple non-indurated
C A young lady presenting with white lacy lesions in painful ulcers with undermined edges and
oral cavity and proximal nail fold extending onto enlarged lymph nodes 5 days after exposure. It is
the nail bed. Most likely diagnosis is probably a case of
Lichen planus Chancroid
C An otherwise healthy male patient presents with a C Drug of choice for type 2 lepra reaction
creamy curd like white patch on tongue. It is a case of Steroids
Candidiasis
C A 17 years old girl is undergoing drug treatment for ALL INDIA 2007
last 2 years for acne. Now presenting with blue
black pigmentation of nails. Drug given was C True for pityriasis rosea
Minocycline Self limiting
C Treatment of erythematous skin rash with multiple
pus lakes in a pregnant women is ALL INDIA 2006
Corticosteroids/prednisolone.
C A patient presents with seven irregular
(Note: Pustular psoriasis)
hyperpigmented macules on the trunk and
C Stain used to study fungal morphology in tissue multiple small hyperpigmented macules in the
sections axillae and groins since early childhood.
PAS Investigation that may support the diagnosis
Slit lamp examination of the eye.
ALL INDIA 2009 C Pterygium of nail found in
C Scarring alopecia is seen in Lichen planus
Lichen planus C An unmarried girl presents with multiple nodular,
C Primary skin diseases are cystic, pustular and comadonic lesions on face,
Icthiosis, psoriasis and lichen planus. upper back and shoulder for 2 years. Drug of choice
C Tuberculide characterized by involvement of sweat Isotretinoin
gland, and hair follicles with non-caseating C A farmer presents with itchy erythematous papular
epitheloid granuloma lesions on exposed area of body which are severe in
Lichen scrofulosorum. summer and improves in winter. Diagnosed by
C A child with fever presents with multiple tender Patch test
erythematous skin lesions. On microscopic C An infant presents with itchy erythematous
examination, neutrophilic and histiocytic papules and exudative lesions on scalp, face, groins
infiltration in the dermis are seen in skin lesions. and axillae for last one month with vesicular lesions
Diagnosis is on palms. She is suffering from
Sweet syndrome Scabies
Dermatology 239

C Parakeratosis most commonly occurs in ALL INDIA 2003


Actinic keratoses
C A 40 years old man develops persistent oral ulcers
with multiple flaccid bullae on trunk and
ALL INDIA 2005 extremities. Direct immunofluorescence finding-
C A teenage boy presents with asymptomatic, Intercellular IgG deposits in the epidermis. The
multiple, erythematous, annular lesions with most probable diagnosis
collarate of scales on the trunk. He may be suffering Pemphigus vulgaris
from C Acantholysis is characteristic of
Pityriasis rosea Pemphigus vulgaris
C Drugs effective in treatment of pityriasis versicolor C Multiple asymptomatic oval and circular faintly
Clotrimazole, ketoconazole and selenium sulphide. hypopigmenterd macules with fine scaling on face
C Ill defined, hypopigmented slightly atrophic of 5 years old male child indicates
macule on the face of a 8 years old boy from Bihar Pityriasis alba
since last 6 months indicates towards diagnosis of C Treatment for tinea capitis on scalp of ten years old
Indeterminate leprosy male child
C Erythematous papulopustular lesion with Oral griseofulvin
erythema and telangiectasia on the convexities of C Diagnostic test for an itchy annular plaque on face
face for last 2 years in a 40 years old woman points of a patient
towards diagnosis of Potassium hydroxide mount.
Rosacea C Itchy, exudative lesions on face, palm and soles of
C Itchy annular scaly plaque in both groins of a an infant with similar complaint in sibling is best
factory worker which continue to extend at the treated with
periphery despite steroid ointment application is a Topical permethin
case of C Most common cause of recurrent genital ulceration
Tinea cruris in a sexually active male
C The only definite indication of systemic corticoste- Herpes genitalis
roid in pustular psoriasis C The syndromic management of urethral discharge
Extensive lesions mainly includes treatment of
Neisseria gonorrhea and Chlamydia trachomatis.
ALL INDIA 2004 C A psoriatic patient develops generalized pustules
all over the body after steroid withdrawal. Cause is
C A 30 years old male had severely itchy papulo- Pustular psoriasis
vesicular lesions on extremities, knees, elbow and
C Wickham’s stria is a feature of
buttocks for one year. Direct immune fluorescence
Lichen planus
finding
Duration of griseofulvin in finger nail dermato-
IgA deposition at dermoepidermal junction.
C
physis treatment
C Multiple hyperpigmented macules over the trunk
of a 5 years old boy, on rubbing with rounded end 3 months
of a pen, urticarial wheal is developed confined to C Maculoerythematous rash develops in a patient
border of lesion. Diagnosis after 3 days of fever and lasts for 48 hours. It is a case
Urticaria pigmentosa of
C A 20 years old man presents with repeated episodes Roseola infantum
of flexural eczema, contact urticaria, recurrent skin C Exfolliative dermatitis can be due to
infections, severe abdominal cramps and diarrhea Psoriasis, pityriasis rubra and drug hypersensitivity.
upon taking sea foods, it is a case of C Genital elephantiasis is caused by
Atopic dermatitis Lymphogranuloma venerum.
240 AIIMS and All India PGMEE—Review Questions

16

Anesthesia

AIIMS NOVEMBER 2015 with spontaneous respiration. The best anesthetic


circuit for him
C Colour coding for the medical oxygen cylinder Mapleson A
Black body with white shoulder. C Strategies to improve oxygenation, when used with
C Snap felt just before entering epidural space by appropriate ventilators
Piercing ligamentum flavum. Prone ventilation, high frequency oxygenation
C Craniovertebral joint contains ventilation and extracorporeal membrane oxygena-
Atlas, axis and basi-occiput. tion.
C Most reliable site to measure core temperature C Anesthetic most commonly used for day care
during general anaesthesia surgery
Distal esophagus Propofol
C A child underwent eye surgery under GA with
AIIMS MAY 2015 propofol and succinylcholine. After 8 hours, the
C Anesthetic agent which is painful on administering child started walking and developed muscle pain.
intravenously The most likely cause of muscle pain
Propofol Succinylcholine
C A coronary artery disease patient presents with HR- C The terms Bethune and Brethren are associated
48/min, BP-80/40, drug that can not be given with
Dexmedetomidine > Ketamine. Transesophageal echocardiography.
(Note: Dexmedetomidine lowers BP whereas keta-
mine worsens ischaemia by increasing workload). AIIMS MAY 2014
C Drugs used in day care surgery C A patient of head injury is intubated and ventilated.
Remifentanyl, propofol and midazolam. The ideal mode of ventilation in him would be
C Muscle relaxant that can be given in case of CMV
increased bilirubin and increased creatinine C The appropriate size of LMA for an average adult
Atracurium patient weighing 50 kg is
C In a patient with thrombocytopenia, target platelet 4.0
count after platelet transfusion to perform an C At supra MAC concentration, anesthetics lead to
invasive procedure shift of EEG wave from
50,000 β to δ waves
C The plane of surgical anesthesia during ether
AIIMS NOVEMBER 2014 anesthesia is defined as
C A 25 years old male is undergoing an incision From onset of regular respiration to cessation of
drainage of an abscess under general anesthesia spontaneous breathing.

240
Anesthesia 241

C Drug not affecting absorption and secretion of C A patient is posted for the surgery but has raised
cerebrospinal fluid intracranial tension. Anesthetics preferred
Nitrous oxide Sevoflurane
C Intravenous anesthetic agent contraindicated in C Suxamethonium is available as a clear, colorless
epileptic patients posted for general anesthesia liquid. The shelf life of suxamethonium is
Ketamine 2 years
C Midazolam causes
Anterograde amnesia, tachyphylaxis during high AIIMS NOVEMBER 2012
dose infusion and decreased cardiovascular effects C An infant with respiratory distress was intubated.
as compared to propofol. The fastest and accurate method to confirm intubation
Capnography
AIIMS NOVEMBER 2013 C EEG in anesthesia is useful in
C Contraindicated in an epileptic patient posted for Depth of general anesthesia.
general anesthesia C Kinemyography is used for
Ketamine Monitoring of neuromuscular function.
C Drug causing malignant hyperthermia C True about lidocaine
Suxamethonium It acts on sodium channels in both active and
C Tubocurare affects which muscle first inactive state, it is given IV in cardiac arrhythmias
Head and neck and extensive first pass metabolism.
C According to AHA 2010 guidelines, drug not used C Most potent agent among the following
in CPR Nitrous oxide
Atropine C Modified Allen’s test is used for checking the
C On doing laparoscopic cholecystectomy patient proper arterial supply at the
developed wheezing. Used in the treatment Wrist
Deepen the plane of anesthesia. C A patient with hypertension, under control by
C Sensory block for lower segment cesarean section is medication falls under which grade
given at the level of ASA 2
T4 C Dose of which muscle relaxant is calculated on the
C Contraindications for neuraxial block basis of total body weight of an obese person rather
Platelet count < 50,000, patient on clopidogrel and than its ideal body weight
local infection. Atracurium
C Effective strategies to decrease the risk of post- C After endotracheal intubation in a 2 years old child
puncture dural headache are posted for craniotomy, after 2 minutes, the bellows
Use of small bore needle, use of atraumatic needle of the anesthesia machine were found to be
and supplementation of fluids. collapsing. Next best thing to do next
C Induction of inhalational agent is faster with Increase the rate of flows.
Combination with nitrous oxide.
AIIMS MAY 2012
AIIMS MAY 2013
C In a pregnant patient, there is decreased requirement
C A woman is posted for elective cholecystectomy. Her of the spinal anesthetic agent because of the
preoperative evaluation and airway was normal. In Decreased volume of subarachnoid space,
the operating room, monitors were attached and engorgement of epidural veins and increased
antibiotics was given. Suddenly she became pulseless sensitivity of the nerves to anesthetic agent.
and unresponsive. Next step in the management C Seen in Scholine apnea
Start chest compressions. It is due to succinylcholine, can be inherited and
C True about hypothermia in anesthesia patients usually do not die of scholine apnea if
Can be prevented by administration of warm fluids. properly managed.
242 AIIMS and All India PGMEE—Review Questions

C A 40 years old lady undergoes incisional hernia C A patient complains of severe pain after thoracotomy.
surgery under general anesthesia and complains of Treatment
awareness during her surgery. Monitoring technique IV fentanyl
to prevent such awareness C A 70 kg weighted old athlete posted for surgery.
Bispectral index monitoring He was given succinylcholine due to unavailability
C A patient presented with blunt trauma to the of vecuronium, in intermittent dosing (640 mg
emergency with heart rate of 150/min, BP 80/50 total). During recovery patient was not able to
and is posted for emergency laparotomy. Anesthetic respire spontaneously and move limbs. It is due to
agent of choice Phase 2 blockade produced by succinylcholine.
Ketamine C Middle aged man had severe maxillofacial trauma
C A 30 years old man with no past medical history is due to road traffic accident. After 1 hour, pulse is
injured in a bomb blast, is suspected of splenic 120/minute, BP–100/70 mm Hg, SpO2–80% with
injury. Ideal anesthetic agent of choice for emergency oxygen. Immediate management is
laparotomy Orotracheal intubation
Etomidate C Inhalational agent preferred in patient with liver
C The most important constituent in soda lime for compromise for surgery for mitral stenosis
reabsorption of CO2 in a closed circuit is Xenon
Calcium hydroxide
C Maximum dose of lignocaine with adrenaline for
AIIMS MAY 2010
local blocks in ophthalmic surgeries is
7 mg/kg C Volatility of anesthetic agent is directly proportional
C A 10 years old child undergoing squint surgery to lowering the flow in portal vein. Portal vein flow
suddenly developed increased heart rate, is maximally reduced by
arrhythmia, high fever, metabolic and respiratory Halothane
acidosis on arterial blood gases and elevation of end C Local anesthesic causing methemoglobinemia
tidal CO2. First agent of choice Prilocaine > Benzocaine
Dantrolene
AIIMS NOVEMBER 2009
AIIMS NOVEMBER 2011
C Anesthesia of choice in a child with bladder
C With reference to the optimal management of exostrophy and chronic renal failure
patients with sepsis in ICU, all of the following Atracurium
interventions are evidence based except C Local anesthetic first used clinically was
Intensive blood glucose monitoring and prevention Cocaine
of hyperglycemia improves survival in critically ill C A 55 years old lady a known patient of thyrotoxicosis
patients. in control posted for APR. There was sudden drop
in BP and end tidal CO2 decreased from 40 to 10 mm
AIIMS MAY 2011 Hg during surgery. There was mill wheel murmur.
Diagnosis is
C Cisatracurium is preferred over atracurium because
Air embolism.
of
No histamine release.
AIIMS MAY 2009
C Laudonosine is a metabolite of
Atracurium C Dead space is increased by
Anticholinergic drugs, standing and hyperextension
of neck.
AIIMS NOVEMBER 2010 C Most common complication of coeliac plexus block
C Anesthetic agent causing adrenal suppression Hypotension
Etomidate (Note: Lower limb vasodilatation)
Anesthesia 243

C Induction agent producing cardiac stability C Most common nerve used for monitoring during
Etomidate anesthesia
C True about CPAP Ulnar nerve.
Initiate early in preterm with respiratory distress, (Note: and muscle is adductor pollicis)
initiate with > 50–60% FiO 2 and improved lung C Merit of nasotracheal intubation is
compliance and oxygenation. Good oral hygiene
C Flat capnogram seen with
Accidental extubation, dissociation of anesthetic AIIMS NOVEMBER 2005
tube and mechanical ventilation facling.
(Note: also seen in esophageal intubation and C Bradycardia is common after injection of
severe bronchospasm) Succinylcholine
C True about ketamine
Direct myocardial depressant, may induce cardiac
AIIMS MAY 2008 dysarrhythmias in patients receiving TCA and
C Technique used to maintain proper oxygen flow to emergence phenomena are more likely if anticholi-
patient—Proportionater between N2 and O2 control nergic premedication is used.
valve, different pin index for nitrogen and oxygen C Cardiovascular monitoring techniques are
and calibrated oxygen concentration analysis. Central venous pressure monitoring, pulmonany
C Most common cause of postoperative renal failure artery catheterization and transesophageal echo-
Decreased renal perfusion. cardiography.
C Shortest acting ND muscle relaxant C Confirmation of placement of double lumen tube
for pulmonary surgery by
Gentacurium > Mivacurium > Rapacuronium
Bronchoscopy
C During perioperative period, myocardial ischemia
AIIMS NOVEMBER 2007 can be detected by most sensitive and practical
C True about thiopentone investigation
2D transesophageal echocardiography.
Sodium carbonate is added to improve its
solubility, cerebroprotective and contraindicatd in C Most common cause of hypoxia during one lung
porphyria. ventilation
C Cerebral oxygen consumption is increased by- Increased shunt fraction.
Ketamine. C Anesthesia of technique of choice in LSCS in a
C Mivacurium, true are women with coarctation of aorta
Increasing the dose produces rapid onset of action, General anesthesia
bronchospasm and flushing. C Induction agent of choice for corrective surgery for
cyanotic heart disease in 5 years old child
Ketamine
AIIMS MAY 2007 C Pressure tracing that identifies placement of Swan-
Ganz catheter in the pulmonary artery
C Muscle relaxant of choice in hepatic and renal
Pulmonary artery pressure tracing has dicrotic
failure
notch from closure of pulmonary valve.
Cisatracurium > Atracurium
C The outcome following resuscitation of a cardiac
C Side effect of oxygen therapy are arrest is worsened if during resuscitation patient is
Absorption atelectasis, decreased vital capacity and given
endothelial damage. 5% Dextrose
C Seen in malignant hyperthermia C A 6 months old child is suffering from PDA with
Hypertension, hyperkalemia and metabolic congestive cardiac failure. Inhalational induction
acidosis. agent of choice for ligation of ductus arteriosus
(Note: and respiratory acidosis) Sevoflurane
244 AIIMS and All India PGMEE—Review Questions

AIIMS MAY 2005 C True about treatment of prolonged suxamethonium


apnea due to plasma cholinesterase deficiency after
C Epidural anesthesia with lignocaine with adrena- a single dose of suxamethonium
line was given for hernia surgery but patient Reversal with incremental doses of neostigmine,
developed hypotension and respiratory depression transfusion of FFP and plasmapheresis.
within 3 minute after block. Most common cause
Drug has entered the subarachnoid space.
C Drugs prescribed for treatment of beta-blocker AIIMS MAY 2004
induced excessive bradycardia and decrease in C Mechanism of neuromuscular blocking action of
cardiac output curare is
Calcium chloride, dopamine and glucagon. Competitive inhibition.
C Na nitroprusside infusion may result in major C Local anesthetic producing allergic reaction
vascular surgery Benzocaine
Cardiac complications. C The inhalational agent of choice for maintenance of
C Most common cause of morbidity and mortality in anesthesia in a 70 years old male patient undergoing
major vascular surgery surgery that may last for 4–6 hours
Cardiac complications. Desflurane
C Anesthetic drugs not suitable in acute porphyria for C Local anesthetic not used for intravenous regional
intravenous induction anesthesia for left trigger finger in an adult lady
Thiopentaone sodium. Bupivacaine
C Most common rhythm disturbance during early C Muscle relaxant of choice in a patient with a history of
postoperative period hypersensitivity to neostigmine for LSCS under GA
Tachycardia Atracurium
C The physiological dead space is increased in
Emphysema, positive pressure ventilation and
AIIMS NOVEMBER 2003
upright position.
C Device providing fixed performance oxygen therapy C Amide linked LA are
Venturi mask Etidocaine, lidocaine and prilocaine
C Highly cardiotoxic local anesthetic C Inhalational agent contraindicated in a patient with
Bupivacaine history of epilepsy
Enflurane
AIIMS NOVEMBER 2004 C Laryngeal mask airway is used for
Maintenance of the airway.
C A 52 years old male patient has triple vessel
C Anesthetic circuits suitable for both control assisted
coronary artery disease with poor left ventricular
ventilation.
function. Maintenance of anesthesia for coronary
artery bypass graft surgery Mapelson B and C, Mapelson D, and Mapleson E.
IV opioids C 100% oxygen is given at the end of anesthesia after
discontinuing nitrous oxide and removing endo-
C Oxygenator producing least damage to blood
tracheal tube to prevent
elements
Membrame oxygenator Diffusion hypoxia
C Trendelenburg position produces decrease in C Management of hypotension following spinal
Compliance, functional residual capacity and vital subarachnoid block
capacity. Infusion of 1L RL before the block, vasopressor drug
C Drugs eliminated by kidney like methoxamine and inotropic drug dopamine.
Pancuronium, pipecuronium and vecuronium. C Shortest acting IV analgesic
C Significant anesthetic problem in the morbidly Remifentanil
obese patient C Combination of drugs for day care surgeries
Decreased cardiac output relative to total body mass. Popofol, fentanyl and isoflurane
Anesthesia 245

C Maneuver performed during laryngoscopy and C Succinylcholine is contraindicated in


endotracheal intubation Cerebral stroke, closed head injury and tetanus.
Flexion of neck, extension of head at atlantoccipital C A child is developing progressive weakness of both
joint and lift of epiglottis by tip in a straight blade legs with history of a brother dying at the age of
laryngoscope. 15 years. Anesthetic drugs not given to him for
C Dysrhythmia, conduction abnormality and cardiac examination of eye under anesthesia
arrest in a paraplegic patient following succinyl- Succinylcholine
choline administration is due to C True about vecuronium
Hyperkalemia Has short duration of action, has high lipophilic
C Muscle relaxant used for endotracheal intubation activity and dose adjustment is not required in
that may lead to cardiac arrest in a young patient usual doses in kidney disease.
who had extensive soft tissue and muscle injury is C LA topically used
Succinylcholine Cocaine, dibucaine and lignocaine
C Pin index system in anesthesia machine prevents C Cerebroprotective anesthetic agent given prior to
Incorrect gas cylinder attachment. deep hypothermic circulatory arrest for surgery for
C Pin index number of N2O aortic arch aneurysm
3, 5 Thiopental sodium.
C The muscle relaxant of choice for a neonate for C In volume cycled ventilation, inspiratory flow rate
porto-enterostomy for biliary atresia is usually set as
Atracurium 60–100 l/min.
C Anesthetic drug not given in a 2 months old infant
undergoing surgery for biliary atresia AIIMS MAY 2002
Halothane
C True about diagnosis of air embolism by transeso-
Factors decreasing MAC of an inhalatimal drug
phagel echocardiography
C
are
Very sensitive investigation, continuous monitoring
Anemia, hyponatremia and hypothermia.
required to detect venous embolism and interferes
C Visual analog scale used to measure with Doppler when used together.
Pain intensity
C True about neonatal circumcision ALL INDIA 2012
GA administered to neonate as they also feel pain as
adults. C Total cerebral metabolic failure occurs at cerebral
blood flow of
10 ml/100mg/min
AIIMS NOVEMBER 2002 C Affected during anesthesia
C Inhalational anesthetic drug with rapid induction Visual evoked response, somatosensory evoked
of anesthesia response from median nerve and SER from
Desflurane posterior tibial nerve.
C A 5 years old child is undergoing squint surgery C Depth of anesthesia measured by
with uneventful induction and as surgeon grasps Bispectral index
the medial rectus after conjunctival incision, C True about total intravenous anesthesia
anesthetic looks at cardiac monitor It reduces cerebral metabolic rate.
To check oculocardiac reflex. C Contraindicated in renal failure
C Muscle relaxant causing increase in intracranial Pethidine
pressure C Agent causing pain on injection
Suxamethonium Rocuronium
C Inhaled gas that decreases pulmonary artery C Primary mechanism by which thiopentone offers
pressure in adults and infants cerebroprotection
Nitric oxide. Decreased brain metabolism.
246 AIIMS and All India PGMEE—Review Questions

C Following cannulation of right subclavian vein, C Fire breaks out during laser vocal cord surgery.
patient developed respiratory distress, BP dropped What should be done?
to 100/55 and heart rate increased to 140/min, Pouring sterile water, removing endotracheal tube
diminished air entry towards right side of chest and and treatment with steroid and antibiotics.
hyperresonant note on percussion. Diagnosis is
Tension pneumothorax. ALL INDIA 2010
C Most potent cerebral vasodilator
Hypercarbia C A patient with bilirubin value of 8 mg/dl and
serum creatinine 1.9 mg/dl is planned for surgery.
C A 45 years old male patient with history of smoking
Muscle relaxant of choice
is scheduled for surgery. True are
Atracurium
Nicotine has effects on carotid and aortic bodies and
can increase sympathetic tone, dose requirement of C True about neuromuscular blockade produced by
muscle relaxants are increased in smokers and succinylcholine
smoking causes decreased surfactant levels. No fade on train of four stimulation, no post-tetanic
C True about anesthetic machine facilitation and train of four ratio > 0.7.
Temperature of desflurane vaporizer chamber is 39°C. C A 25 years old overweight female patient given
fentanyl-pancuronium anesthesia for surgery.
After surgery and extubation she was observed to
ALL INDIA 2011
have limited movement of upper body and chest
C Definite airways are wall in recovery room. She was conscious and alert
Nasotracheal tube, orotracheal tube and cricothy- but limited voluntary respiratory effort. HR and BP
roidotomy. normal. Cause is
C Anesthetic agents contraindicated in patients with Incomplete reversal of pancuronium.
hypertension C A speedy intubation was performed in a 27 years
Ketamine old female patient with acute abdominal pain and
C Epileptogenic anesthetic agent is posted for laparotomy, but after the intubation,
Enflurane > Sevoflurane breath sounds were observed to be decreased on left
C Anesthetic agent with vasoconstrictor is contrain- side and a high end tidal CO2 was recorded. Likely
dicated in diagnosis is
Finger block Endobronchial intubation.
(Note: i.e. end artery block)
C Anesthetic agent to be avoided in sickle cell disease ALL INDIA 2009
IV regional anesthesia. C Sodium bicarbonate when given with local
(Note: also avoided in Raynaud‘s disease and anesthesia has following effect
scleroderma) Increases speed and quality of anesthesia.
C Spinal anesthesia given at the level (Note: also increases duration)
L2-4 C Anesthetic drug injected for paravertebral block is
C Anesthesia resident was giving spinal anesthesia likely to diffuse to
with patient suddenly undergoing aphonia and Epidural space, intercostal space and superior and
loss of consciousness. Cause inferior paravertebral spaces.
Vasovagal attack C Sign of successful stellate ganglionic block
C Index evaluating intraoperative awareness of Nasal stuffiness, Guttman’s sign and Horner’s
patient is syndrome.
Bispectral index
C Rise in end tidal CO2 due to thyroid surgery can be
AIIMS NOVEMBER 2008
due to
Malignant hyperthermia, thyroid storm and C Absortion of local anesthetic is maximum in route
neuroleptic malignant syndrome. IV > Tracheal > Intrapleural > Intercostal
Anesthesia 247

C Rating of postoperative pain in children by C Dissociative anesthesia is produced by


CHEOPS scale includes Ketamine
Cry, touch and torso.
C Use of nitrous oxide is contraindicated in surgeries ALL INDIA 2006
Cochlear implant, microlaryngeal surgery and
vitreoretinal surgery. C Indication for endotracheal intubation
(Note: Closed space surgeries) Pulmonary toilet, maintenance of patent air way
and to provide positive pressure ventilation.
ALL INDIA 2008 C Muscle relaxant of choice in renal failure
Atracurium
C Train of four fade is a characteristic feature of C Muscle relaxant with maximum duration of action
Non-depolarizing block. Doxacurium
True about propofol
Antibiotics not used with d-tubocurarine
C
C
Suitable for day care surgery, commercial prepara-
Doxacurium
tions contain egg and it does not trigger malignant
hyperthermia. (Note: Prolonged apnea)
C Not true about inhalational anesthetic agent C Local anesthetic belonging to ester group
Sevoflurane is more than isoflurane. Procaine
C Inhalational agent that is induction agent of choice
in children
ALL INDIA 2007
Sevoflurane
C Amide linked local anesthetics are C Anesthetic agent sensitizing the myocardium to
Lignocaine, bupivacaine and dibucaine. catecholamines
C A young boy undergoing eye surgery under day Halothane
care anethesia with succinylcholine and propofol C True about etomidate
and after 8 hours he starts walking and develops
Intravenous anesthetic, causes pain at site of
muscle pain. Likely cause is
injection and inhibits cortisol synthesis.
Succinylcholine
C Anesthetic agent triggering malignant hyperthermia
C An elderly man on ventilator receiving atracurium
Halothane, isoflurane and suxamethonium.
for last 3 days develops epileptic fits due to
(Note: ether also)
Accumulation of laudanosine.
C IV induction agent most suitable for daycare
C Opoid not given intrathecally
surgery
Remifentanil
Propofol
C Central neuraxial (spinal and epidural) anesthesia
C Drugs used to provide induced hypotension during
is contraindicated in
surgery
Platelet < 80,000, patient on oral anticoagulants and
Sodium nitroprusside, hydralazine and esmolol.
raised ICT.
C Postoperative pain relief recommended for 2
C Drugs used intraoperatively to control heart rate
months old infant undergoing a major surgery
Verapamil, esmolol and procainamide.
IV narcotic in lower dosage.
C A nonventilated preterm baby in incubator is under
observation. Best way to monitor the baby’s
breathing and detect apnoea ALL INDIA 2005
Impedence pulmonometry. C Fastest acting inhalational agent
C True about xenon anesthesia Desflurane > Cyclopropane > N2O > Sevoflurane
Nonexplosive, minimal cardiovascular side effects C Disadvantages of anesthetic ether
and low blood gas solubility. Slow induction, cautery cannot be used and irritant
C EEG activity in anesthesia is increased by nature of ether increases salivary and bronchial
N2O, ketamine and early hypoxia. secretions.
248 AIIMS and All India PGMEE—Review Questions

C Anesthetic agent increasing intracranial pressure C Induction agent causing adrenal cortex suppression
Sevoflurane Etomidate
C Drugs used in postoperative nausea and vomiting C During rapid sequence induction of anesthesia
following squint surgery in children Pre-oxygenation is mandatory.
Dexamethasone, ondansetron and propofol. C Patient recovers spontaneously and no reversal is
C The laryngeal mask airway of a patient in conditions required in depolarizing muscle relaxant in
In difficult intubation, in cardiopulmonary resusci- balanced anesthesia
tation and in child undergoing routine eye surgery. Atracurium
C The circuit used most commonly for ventilating a C The most appropriate anesthetic among the
spontaneously breathing infant during anesthesia following options for tendon lengthening operation
Jackson Rees modification. of 5 years old boy patient of Duchenne muscular
C Modes of ventilation that can be used for weaning dystrophy
off patients from mechanical ventilation Induction with IV suxamethonium and N 2O and
Assist control ventilation, pressure support venti- oxyen for maintenance.
lation and synchronized intermittent mandatory
C A 25 years old male patient undergoing incision
ventilation
and drainage of abscess and GA with spontaneous
C LSCS can be carried out under technique of anesthe- respiration. Most appropriate anesthetic circuit
sia is
General anesthesia, spinal anesthesia and combined
Mapleson A
spinal epidural anesthesia.

ALL INDIA 2004 ALL INDIA 2002


C A woman develops carpopedal spasm following C Anesthetic induction agent of choice in day care
LSCS for prolonged labour under subarachnoid facility for MTP in early pregnancy
block with lignocaine as anesthetic agent. Cause is Propofol
Hypocalcemia C In retrobulbar anesthetic block the last muscle to be
C Treatments given to a patient of PSVT with rendered akinetic is
hypotension under GA Superior oblique
Carotid sinus massage, adenosine 3–12 mg IV and C Neuromuscular blocking drug with shortest
DC cardioversion. duration of action
C Persistent numbness and paresthesia in the right
Suxamethonium
forearm and hand on 2nd postoperative day of 25
yrs old male patient who underwent debridement C Fluorinated anesthetic agent that corrodes metal in
and reduction of fractured both bone right forearm vaporizers and breathing systems
under axillary block following RTA. Causes of Halothane
neurological dysfunction could be C Inhalational anesthetic agent with minimum blood
Tourniquet pressure, tight cast or dressing and gas solubility coefficient
crushing injury to the hand and lacerated nerves. Desflurane
C Color code of oxygen cylinder C Anesthetic drug contraindicated in a patient with
Black cylinders and white shoulders. raised intracranial pressure
Inter scalene approach to brachial plexus block
Ketamine
C
provides optimal surgical anesthesia in the area of
distribution of nerves C Common side effects seen with fentanyl
Musculocutaneous, radial and median nerve. Chest wall rigidity
C Neuraxial block is absolutely contraindicated in C Allodynia means
Coagulopathy, patient refusal and severe hypovo- Perception of an ordinarily non-noxious stimulus
lemia. as severe pain.
Radiology 249

17

Radiology

AIIMS MAY 2015 C A middle aged lady presents with severe bone
pains at multiple sites. Plain radiograph reveals
C Investigation of choice in a child presenting with multiple lytic lesions in pelvis, ribs and femur,
features suggestive of biliary atresia fracture of the clavicles and subperiosteal reabsorp-
Hepatic scintography. tion of metacarpals over the radial aspect. The most
C Investigation of choice for acute appendicitis in likely diagnosis
children is Hyperparathyroidism
USG C The investigation of choice for acute appendicitis in
C A patient presented with tachyarrhythmia and was children is
implanted inflatable cardioverter defibrillator. Ultrasonography
Now he presents with shock. Investigation of C The investigation of choice in a patient with acute
choice to exclude displacement of ICD is head injury
Plain radiograph NCCT head
C Investigation of choice for stress fracture is C The gold standard for diagnosis of bronchiectasis is
MRI HRCT chest
C Expansion of subarachnoid space occurs on myelo- C The neuroimaging signs of tuberous sclerosis
graphy in includes
Indradural extramedullary tumours. Giant cell astrocytoma, subependymal nodules and
C Investigation of choice for fetal heart activity at white matter lesions.
6 weeks
USG AIIMS MAY 2014
C Non-invasive investigation of choice for myocardial C Puff of smoke appearance on cerebral angiography
viability is seen in
FDG PET scan Moyamoya disease.
C Most sensitive test for detecting fetal cardiac activity C A 7 years old patient presents with severe headache,
in 6 week pregnancy paralysis of upward gaze, loss of light perception
USG for fetal cardiac activity. and accommodation, nystagmus and failure of
convergence. CT scan showed homogenous hyper-
dense lesion above the sella and in the posterior part
AIIMS NOVEMBER 2014
of the third ventricle. MRI showed that the lesions
C A child presents with a history of injury to left fore- were homogenous and isointense on T1 weighted
finger tip with a glass piece. On examination, the imaging, and isointense on T2 weighted imaging
finger is swollen and tender. Best investigation to deter- with intense contrast enhancement. The most likely
mine the glass foreign body in the patient’s finger diagnosis
Plain radiograph Germinoma

249
250 AIIMS and All India PGMEE—Review Questions

AIIMS NOVEMBER 2013 elevated right hemidiaphragm is seen. Possible


diagnosis
C A 70 years old lady on treatment with alendronate Acute cholycystitis.
for 7 years for osteoporosis. She complains of pain C Stereotactic surgery is used for
in right thigh. Next done
Brain tumours
X-ray
C Symptomatic spinal injury without any radiological
evidence most commonly found in AIIMS MAY 2012
Children C Bracket calcification on skull X-ray is seen in
C Radiologically, increased pulmonary blood flow is Lipoma of corpus callosum.
indicated by C A child was taken for CECT chest and contrast was
Descending pulmonary artery diameter > 16 mm, injected; child had swelling, which gradually
Kerley B lines and more than 6 blood vessels in increased. There is numbness. There is pain on passive
outer 1/3rd. extension of fingers. He is not allowing you to touch
C A patient of RTA with chest and limbs injury has the arm but pulse was present. What next to be done
low SpO2. M mode ultrasound of right upper chest Immediate fasciotomy
shows stratosphere sign. Diagnosis is C Stereotactic radiotherapy is used in
Pneumothorax In operable stage 1 lung tumour.
C A 12 years old boy referred from endocrinology C Non-iodine containing contrast is
department complained of pain in hip joint and Gd DTPA
limp. Tests useful are C Maximum radiation exposure occurs in
X-ray pelvis, USG pelvis and CT pelvis. CT scan
C Most reliable test for diagnosing spinal TB is
AIIMS MAY 2013 CT guided biopsy.
C Stage of neurocysticercosis does not show edema
on CT scan AIIMS NOVEMBER 2011
Calcified nodular stage. C Investigation of choice for meningeal carcinomatosis
C An 8 years old child presents with left sided flank in CNS
pain and mental retardation. On USG- a hyperechoic Gd enhanced MRI.
lesion in the right kidney and multiple lesions in the
C Epidermoids can be differentiated from arachnoid
liver are seen. CT examination of abdomen showed
cyst in MRI by
–30 to –50 HU density of these lesions. Most
Restricted diffusion.
probably it is a case of
Tuberous sclerosis. C Investigation of choice for Zenker’s diverticulum
C A patient presents with cough and fever. On X-ray Barium swallow.
examination, a homogenous opacity silhouetting C Correlating with USG findings of pyloric stenosis
the right heart border is seen. Part of the lung Accurate 95%, segment length > 16 mm, thickness >
involved 4 mm.
Medial segment of right middle lobe. C Indicated in diagnosis of spondylolisthesis
MRI, CT scan and X-ray lumbar spine lateral view.
C Diagnosis of a child presenting with raised ICT with
AIIMS NOVEMBER 2012
lesion around foramen of Monro and multiple
C True about principle of MRCP periventricular calcific foci on CT scan
Use of heavily T2 weighted images without contrast. Subependymal giant cell astrocytoma.
C Phase of cell cycle most sensitive to radiation C Cavernous angioma on MRI is characterized by
G2M Reticulated popcorn like configuration.
C A middle aged patient presents with complaint of C Investigation of choice in a 20 years old patient
right hypochondrial pain. On plain chest X-ray, presenting with proptosis, pain in eye, chemosis,
Radiology 251

conjunctival congestion, and extraocular muscle C USG feature of thyroid nodule suggestive of
palsy after 4 days of trauma to the eye malignancy
Intracranial digital subtraction angiography. Hypoechogenicity, non-homogenous and microcal-
cification.
AIIMS MAY 2011 C Investigation of choice for a lesion of temporal bone
CT scan
C Most commonly dissected artery following arterio-
graphy by femoral route
Inferior mesenteric artery AIIMS NOVEMBER 2009
C Used in diagnosis of protein losing enteropathy C Structure causing posterior impression on barium
Tc albumin, Tc dextran and In transferrin. swallow is
C Pure β emitters are Aberrant right subclavian artery.
Yttrium 90, strontium 90 and phosphorous 32. C Rib notching is seen in
C Central dot sign is seen in Aortic disruption, Blalock Taussig shunt and
Caroli’s disease (Mn: C for C) pulmonary atresia with large VSD.
C Radiological features of LVHF are C A child presents with respiratory distress. Investi-
Increased flow in upper lobe veins, Kerley B lines gation of choice for suspected vascular ring
and cardiomegaly. MRI
C Radiation induced necrosis can be diagnosed by
AIIMS NOVEMBER 2010 PET
C Element absolete in radiotherapy
C Radiation exposure is least among the options
Radium 226
Micturating cystourethrogram.
C Most sensitive imaging for breast carcinoma-DCIS is
MRI AIIMS MAY 2009
C A 20 years female patient with 6th cranial nerve C True regarding scurvy
palsy on T2 weighted MRI shows hyperintense Bowing of legs
lesion in cavernous sinus showing homogenous
C Stereotactic radiosurgery uses
contrast enhancement
Proton, linear accelerator and gamma knife.
Schwannoma
C C1 and C2 can be best visualized by
Odontoid view
AIIMS MAY 2010 C CT or Hounsefield number depends upon
C A male patient was brought unconscious to the Mass density
hospital with external injuries. CT brain shows no C Radiological finding of renal papillary necrosis on
midline shift, but basal cistern were compressed excretory urogram are
with multiple small hemorrhages. Diagnosis is Tracks and horns from calyces, ring shadow and
Diffuse axonal injuries egg in cup appearance.
C A newborn presents with CHF with bulging anterior C Cerebral blood flow in an asphyxiated child is best
fontanellae and brui on auscultation. Transfontanellar measured by
USG shows a hypoechoic midline mass with dilated NIRS
lateral ventricles. Most likely it is a case of
Vein of Galen malformation.
AIIMS NOVEMBER 2008
C A 48 years old woman comes with bilateral pro-
gressive weakness of both lower limbs, spasticity C Gold standard investigation for recurrent GIST is
and mild impairment of respiratory movements. PET CT
MRI shows an intradural mid-dorsal midline C Earliest detectable congenital malformation by
enhancing lesion. Diagnosis is USG is
Meningioma Anencephaly
252 AIIMS and All India PGMEE—Review Questions

C Regarding CT true are C Basal ganglia calcification is seen in


Decrease in mA decreases radiation dose exposure Metformin, dehydration and renal failure.
significantly in pediatric chest, quality of radiation C Isotope used in RAIU
generated depends upon voltage and radiation I123
dose exposure is directly related to the time of
exposure.
AIIMS NOVEMBER 2006

AIIMS MAY 2008 C Bone scan of a patient with multiple myeloma shows
Cold spots.
C Absolute contraindication of MRI C Most sensitive radiological indicator for aneuploidy
Pacemaker is
C Most chemoresistant tumors among the following is Nuchal translucency.
Malignant fibrous histiocytoma.
C PACS in medical imaging stands for AIIMS MAY 2006
Picture archiving and communication system.
C Snowman appearance on X-ray is seen in C Differential diagnosis of spinal cord edema on MRI
TAPVC. Myelomalacia
C If the right cardiac silhoute is obliterated, it means C Mammography feature suggestive of malignancy
the pathology involves- Right middle lobe. Areas of spiculated microcalcification.
C Egg on side appearance is seen in C Extra axial dural based enhancing tumour on MRI
Uncorrected TGA. in a female patient of 40 yrs age presenting with
C Investigation of choice for acoustic neuroma recurrent headaches. Diagnosis is
MR with contrast. Meningioma
C Sign of increased ICT C Plethoric lung fields are seen in
Erosion of dorsum sella, sutural diastasis and ASD, VSD and TAPVC.
copper beaten appearance (Mn: CDE)
C Hair on end appearance is seen in AIIMS NOVEMBER 2005
Thallasemia C Investigation of choice for posterior urethra is
Voiding cystogram.
AIIMS NOVEMBER 2007 C Intensity modulated radiotherapy is used in
C Floating water lily sign is seen in Prostate cancer.
Hydatid cyst C Radioactive isotopes used for brachytherapy
C Hamptom’s hump is seen in chest X-ray in Cobalt 60, iodine 125 and iridium 192.
Pulmonary embolism. C Solitary hypoechoic lesion of the liver without
septate or debris is most likely to be
AIIMS MAY 2007 Simple cyst.

C Least radiosensitive tumor is AIIMS MAY 2005


Osteosarcoma
C Investigation not done in case of pheochromocy- C Investigation of choice for recent onset severe pain
toma is in the right hip in 40 years old lady on long term
FNAC steroid therapy
C Most common hormone deficiency seen after MRI
intracranial radiation therapy C Heberden’s nodes are found in
GH DIP joint in osteoarthritis
C Spongy appearance with central sunburst calcifi- C Investigation of choice for detection and characteri-
cation is seen in zation of ILD is
Serous cyst adenoma. HRCT
Radiology 253

C Most common cause of mixed cystic and solid supra- C Therapeutic mode commonly employed in intra-
sellar mass seen on cranial MR scan of 8 years old child operative radiotherapy
Craniopharyngioma Electron
C Most common cause of sclerotic skeletal metastasis C Radiation therapy to hypoxic tissue may be poten-
in a female tiated by
Breast carcinorna Metronidazoole
C Most radiosensitive tumor among following C Stereotactic radiosurgery is a form of
Ewing’s tumour (Melt like snow) Radiotherapy
C Investigation of choice for differentiating recurrence C Investigation of choice for extra-adrenal pheochro-
of brain tumor from radiation induced necrosis mocytoma
PET Scan MRI
C Radioluscent stones are
Allopurinol, orotic acid and xanthine.
AIIMS NOVEMBER 2004
C Rays used for treatment of deep seated tumors
C Earliest X-ray feature of childhood leukemia X-rays and gamma rays.
Radiolucent transverse metaphysical bands. C Vessel evaluated first in angiography for massive
C Continuous sheet of wire mesh of copper to shield recurrent hemoptysis in a patient of tuberculosis
the EEG cabins to avoid picking of noise from Bronchial artery
external electromagnetic disturbances is called as C Single 2 cm space occupying lesion of mixed
Faraday cage echogenicity in right lobe of liver on USG is found
C Attenuation value of zero HU (Hounsfield units) in in a 20 years old man. Next investigation is
CT scan corresponds to Hepatic scintigraphy (SPECT)
Water C X-ray is produced when
C Piezoelectric crystal technique is used in Electron beam strikes the anode.
Ultrasonography C HRCT, a special CT technique greater details
C Most common organ involved in bronchogenic utilizes
carcinoma Bone algorithm for image reinstruction.
Adrenals C Differential diagnosis of anterior mediastinal mass
Lymphoma, teratoma and thymoma.
AIIMS MAY 2004 C Most radiosensitive phase of cell cycle
C Investigation of choice for evaluation of blunt G2
abdominal trauma C Investigation (non-invasive) for definitive diagnosis
CT scan before preparing for epilepsy surgery of a boy
presenting with 10–12 partial complex seizure
C Features of malignant gastric ulcer on barium meal are
episodes per day despite 4 drug antiepileptic
Location on the greater curvature, Carman’s
regime with history of repeated high grade fever in
meniscus sign and radiating folds which do not
childhood and MRI normal
reach the edge of the ulcer.
Video EEG with ictal 99mTc-HMPAO Brain SPECT.
C The investigation of choice for imaging of urinary
C Dense persistent nephrogram may be seen in
tract tuberculosis
Acute ureteral obstruction, dehydration and
CT scan
systemic hypertension.
C Radiation dose for pain relief in bone metastasis
C The most recent advance in non-invasive cardiac
30 Gy in 10 fractions.
out put monitoring is use of
Electrical impedance cardiograph technology.
AIIMS NOVEMBER 2003
C Radiopaque shadow with air fluid level in chest
C Maximum permissible dose for radiotherapy in along with hemivertebra of 6th thoracic vertebra on
pregnancy plain X-ray in a newborn most likely cause is
0.5 rad Congenital diaphragmatic hernia.
254 AIIMS and All India PGMEE—Review Questions

AIIMS MAY 2002 C Hyperechoic hepatic metastases on USG are seen in


Mucinous cyst adenoma.
C X-ray features in mitral stenosis are
C A patient presents with raised ICT and ataxia and
Double atrial shadow, lifting up of left bronchus MRI-well demarcated cystic lesion with a mural
and posterior displacement of esophagus on nodule in the cerebellum. Diagnosis
barium swallow.
Hemangioblastoma
C A patient suffering from AIDS presents with
A 35 years male patient with history of asbestos
history of dyspnea and non-productive cough.
C
exposure presents with chest pain, X-ray-solitary
Chest X-ray shows bilateral hilar opacities but no
pulmonary nodule in right lower zone, CECT-
pleural effusion or lymphadenopathy
enhancing nodule adjoining the right lower costal
Pneumocystis carini pneumonia.
pleura with comet tail sign and adjacent pleural
C The best next investigation in a patient with solitary thickening. Diagnosis
pulmonary nodule on X-ray
Round atelectasis
CT Scan
C The pathophysiological phenomenon that occurs
C Egg shell calcification is seen in
during atheromatous plaque formation and is used
Silicosis for screening of asymptomatic coronary plaques on
CT scan is
ALL INDIA 2012 Calcium deposition in the atheromatous plaque.
A 40 years female patient with recurrent headache,
Basal exudates, infarcts and hydrocephalus on CT
C
C
MRI-extra axial dural based enhancing lesion.
are seen in
Diagnosis
Tubercular meningitis
Meningioma
C A 16 years old child presents with recurrent
C Gold standard test for measuring bone density
headache and blurred vision, and well demarcated
DEXA
supraseller, calcified, cystic and enhancing mass on
CT scan. Diagnosis C True about MRI
Craniopharyngioma Contraindicated in patient with pacemakers, useful
for localizing small lesions in the brain and useful
C Wide neural foramina is associated with
for evaluating bone marrow.
Neurofibromatosis
C Long term effects of radiotherapy for CNS tumors
C Gold standard method for detection of renal in children
scarring in patients with recurrent urinary tract Reduced IQ and learning, endocrinal dysfunctions
infection is and musculoskeletal problems.
99 Tc DMSA scan.
C A 45 years female patient presents with recurrent
abdominal pain, jaundice, USG-cluster of cysts with ALL INDIA 2011
lobulated margins in the head of pancreas, MRI- C True about contrasts in radiography
multicystic mass with a bunch of grapes appearance Ionic monomers have 3 iodine atoms per 2 particles
and grossly dilated pancreatic duct. Diagnosis in solution.
Intraductal papillary mucinous neoplasm (IPMN). C Contrast agent preferred in a patient with decreased
C Best view for detecting minimal pneumoperitoneum renal function to avoid contrast nephropathy
Left lateral decubitus with horizontal beam. Low osmolar contrast.
C Radioprotector C Consolidation of part of lung obliterating the aortic
Amifostine knuckle on X-ray chest
C A patient who underwent surgery and radio- Left upper lobe (posterior part).
therapy for a 4 cm tumor in the left parietal lobe and C Objective sign of pulmonary plethora in X-ray chest
2 months after, he presented with headache and is
vomiting. Best characterized by Diameter of descending right pulmonary artery >
18 FDG PET scan 16 mm.
Radiology 255

C Best investigation for bone metastasis is C Prunning of pulmonary artery is seen in-
Bone scan. Pulmonary hypertension.
C CT scan is least accurate in diagnosis of C True regarding ossified posterior longitudinal
Gallstones ligament (OPLL)
C Dose of radiation during whole body exposure that MRI best for diagnosis, low signal intensity on all
leads to hematological syndrome is MR sequences and gradient echo MR sequence may
2 Grey overestimate the canal stenosis.
C Test of choice for reversible myocardial ischemia
ALL INDIA 2010 Thallium scan.
C Walls of CT scanner room are coated with C Preoperative test necessary in Down’s syndrome
patient before surgery
Lead
C The major difference between X-ray and light is Echocardiography
Energy C Amifostine protects
C Most ionizing radiation is Salivary glands, kidney and GIT
Alpha
C Background radiation is
ALL INDIA 2008
Radiation present constantly from natural sources.
C Which best estimates the amount of radiation C Photoelectric effect can be best described as an
delivered to an organ in the radiation field Interaction between low energy incident photons
Absorbed dose. inner shell electron.
C True about stochastic effects of radiation C Allergic reaction to radiologic contrast agents
Probability of effect is a function of dose. are
C Egg on side appearance is seen in Anaphylactic reaction
TGA C Most radiosensitive phase of cell cycle is
C True about CT scan features of adrenal adenoma G2M
Calcification is rare, low attenuation and regular C Used to measure GFR
margins. 99m
Tc DTPA
C A patient presents with acute renal failure and
C Flaring of anterior ends of ribs is characteristically
anuria with normal USG. Investigation regarding
seen in
renal function that gives best information is
DTPA scan. Rickets
C A dense renogram is obtained by C Inferior rib notching is seen in
Rapid bolus injection of dye. Coarctation of aorta
C Earliest radiological sign of pulmonary hypertension
in chest X-ray is
ALL INDIA 2009
Cephalization of pulmonary vascularity.
C Focal and diffuse thickening of gallbladder wall C Earliest sign of left atrial enlargement is
with high amplitude reflections and comet tail
Posterior displacement of esophagus.
artefacts on USG suggests the diagnosis of
Most common feature of aortitis on chest X-ray
Adenomyositis
C

C The sensitivity of mammography is low in young Calcification of ascending aorta.


females because C Investigation of choice for detection and characte-
Young breast have dense tissue. rization of interstitial lung disease is
C Containing fat on mammography HRCT
Post-traumatic cysts, hamartoma and galactocele. C Diagnostic features of ileocecal tuberculosis on
C MRI features of mesial temporal sclerosis barium follow through
Atrophy of mammillary body, atrophy of fornix Pulled up contracted angle, widening of ileocecal
and atrophy of hippocampus. angle and strictures involving terminal ileum.
256 AIIMS and All India PGMEE—Review Questions

ALL INDIA 2006 C Technique in radiotherapy to counteract the effect


of tumor motion due to breaking is called
C Maximum ionization potential in Gating
Helium ion C Initial decay rate of a radio substance with n = 6 × 1023
C Radioisotopes used as systemic radionuclide radioactive atoms as t1/2 20s and disintegration
P32, Sr89 and Samarium 153. constant = 0.01/sec is 6 × 1021.
C P32 emits C The gold standard for diagnosis of osteoporosis
Beta-rays Dual energy X-ray absorptiometry.
C Used in treatment of differentiated thyroid cancer C The most sensitive investigation for diagnosing
I131 ureteric stones in a patient with acute colic
C Non-ionizing radiation used by NCCT of abdomen
MRI, thermography and ultrasonography. C Ultrasound marker with increased risk for Down’s
C Imaging technique that gives maximum radiation syndrome in fetus
exposure to the patient Nuchal edema
CT scan C Most preferred route for cerebral angiography
C Radiotherapy for Wilms’ tumor after surgery is done Trans femoral.
Within 10 days C Tumor showing calcification on CT scan
C Radiosensitive tumor Meningioma
Dysgerminoma
C Rib notching on chest radiograph caused by ALL INDIA 2004
Coarctation of aorta.
The investigation of choice for vestibular
Objective sign of pulmonary plethora in a chest
C
C
schwannoma is
radiograph
Gadolinium enhanced MRI.
Diameter of descending right pulmonary artery >
16 mm. C Radiological signs of scurvy are
C The investigation of choice for evaluation of an Frenkel’s line, pelican spur and zone of demarcation
aneurysm is near epiphysis.
Arteriography C Widened duodenal C loop with irregular mucosal
pattern on barium meal is suggestive of
C The imaging test most sensitive for early renal
tuberculosis Carcinoma head of pancreas.
Intravenous urography C Most penetration beam among following is
C Assessment of ventricular function is most accurately 18 MV photon.
done by C Radiation tolerance of whole liver
Echocardiography 40 Gy
C The most important sign of renal artery stenosis on C Postoperative radiotherapy is used in
angiogram is Colon cancer, head and neck cancer and soft tissue
Presence of collaterals sarcoma.
C The MRI shows lesion in multiple sclerosis in C CT scan feature of acute pancreatitis
White matter Ill defined outline of pancreas, enlargement of
pancreas, and poor contrast enhancement.
Classic CT finding of acute subdural hematoma
ALL INDIA 2005
C
Crescent shaped hyperdense lesion.
C In nuclear medicine, gamma camera is used for C The first investigation in a case of suspected
Measuring the radioactivity. subarachnoid hemorrhage is
C Most useful radiopharmaceutical for bone imaging is NCCT
99m
Tc MDP. C Radioisotopes are used in
C Radioisotopes used as permanent implant Mass spectroscopy, RIA and sequencing of nucleic
Gold 198, Iodine 125 and Palladium 103. aid.
Radiology 257

ALL INDIA 2003 ALL INDIA 2002


C Epiphyseal dysgenesis is found in C Most penetrating power in- Gamma rays.
Hypothyroidism C Contrast for proper radiographic image in a heavy
C Fraying and cupping of metaphyses of long bones built person
in a child occur in Increased milliampere
Hypophosphatasia, metaphyseal dysplasia and C Sudden onset stridor and respiratory difficulty in a
rickets. 2 years old boy with decreased breath sound and
C HRCT of the chest is investigation of choice for wheeze on right side and right opaque hemithorax
Interstitial lung diseases. on chest X-ray. Most likely it is a case of
C Recognized X-ray feature of rheumatoid arthritis Foreign body aspiration.
Bone erosions C Acute respiratory distress with hyperinflation of
C Extensive pleural thickening and calcification of unilateral lung in chest X-ray in a 2 years old child
diaphragmatic pleura are features of is suggestive of
Asbestosis Foreign body aspiration.
C Biconvex hyperdense lesion displacing gray-white C Solid well circumscribed hypoechoic renal mass on
matter interface on CT scan of a head injury patient USG abdomen in a 3 years old boy is suggestive of
suggests Wilm’s tumor.
Extradural hematoma C Multiple air fluid levels on X-ray abdomen in a
C Imaging investigation of choice for posterior fosse newborn presenting with intestinal obstruction and
tumor is constipation. Differential diagnosis are
MRI Duodenal atresia, Ladd’s bands and ileal atresia.
C X-ray of dorsolumbar spine of 8 years old boy with C True about loculated pleural effusion
back pain and mild fever shows solitary collapsed They form obtuse angles against the mediastinum
dorsal vertebra with preserved disc space but no when viewed in profile, have unsharp margins
associated soft tissue shadow. It is most likely a case when viewed enface and do not conform to
of segmental distribution.
Histiocytosis C X-ray feature of Pindborg’s tumor
C Hyperthyroidism can be treated by Driven snow appearance.
I131 C Craniospinal irradiation is used in treatment of
C Radioisotope commonly used in external beam Medulloblastoma
radiotherapy in the treatment of cancer patients C Prophyloacetic cranial irradiation is indicated in
Cobalt 60 Small cell carcinoma of lung, ALL and NHL.
258 AIIMS and All India PGMEE—Review Questions

18

Psychiatry

AIIMS NOVEMBER 2015 C A young patient on antidepressant presents with


altered sensorium and hypotension. ECG reveals
C A 30 years old lady presents with mania for 2 weeks. wide QRS complex and right axis deviation. Next
Her urine pregnancy test is positive. Drug of choice best given
in this condition Sodium bicarbonate
Haloperidol (Note: Diagnosis-TCA poisoning with metabolic
C Most common cause of suicide in India according to acidosis)
national crime bureau 2014
Hanging
C Cognitive triad of Beck’s for depression includes AIIMS MAY 2015
Helplessness, hopelessness and worthlessness. C Features found in catatonia
C A 20 years old male patient of schizophrenia is on Ambitendency, akinesia and ambivalence.
risperidone for the last 2 months. There is no family (Note: Akathisia is motor restlessness and side
history. Treatment is continued for effect of antipsychotic drugs)
2 years C Overwhelming sense of responsibility and
C A child with pervasive developmental disorder uncertainty is the feature of
have Generalized anxiety disorder
Impaired communication, impaired social interaction C A woman has repeated thoughts of contamination.
and stereotypy. She knows that it is irrational and absurd and tries
(Note: Impaired cognition excludes the diagnosis of to resist against it but unable to failure to resist leads
pervasive developmental disorders like autism, to marked distress. Treatment of choice
Asperger syndrome, Rett syndrome, childhood Exposure and response prevention.
disintegrative disorder (CDD) and pervasive C A patient on haloperidol develops dystonia. Drug
developmental disorder not otherwise specified that can be given
(PDD-NOS)
Promethazine
C A male patient of bipolar disorder is well controlled
on medication but symptoms of mania appear as he C A patient has involuntary excessive blinking and
tapers down the drug. Treatment compliance can be grunting. This has started affecting his social life.
improved by Drug appropriate for him
Psychoreduction Risperidone
(Note: Psychoreduction is the education offered to (Note: Haloperidol is the drug of choice for
individuals with a mental health condition and Tourette syndrome)
their families to help empower them and deal with C Avoidant shy histrionic personality is type
their condition in an optimal way. Type C > Type B.

258
Psychiatry 259

C A cigarette smoker thinks that smoking will not C A child is not eating vegetables. His mother starts
cause him cancer as he smokes less number of ciga- giving a chocolate each time he finishes vegetables
rettes and eats well and exercises regularly. This is in the diet. The condition is called
Self exemption Operant conditioning
C A 16 years old girl presents with recurrent abdomi-
AIIMS NOVEMBER 2014 nal pain but biochemical assays and ultrasound
C Loss of memory caused by neuronal degeneration abdomen is normal. After waking up from sleep,
and not associated with normal aging process she suddenly complains of loss of vision of bilateral
Amnestic syndrome. eyes. Ophthalmologist finds nothing on examination.
Most likely diagnosis
C A patient with a history of alcohol consumption of
250 ml everyday for the past 12 years comes to the Malingering
hospital and is diagnosed as alcohol dependent C Cognitive remediation is used for
syndrome. Drug avoided in the patient Cognitive restructuring
Disulfiram
C A 20 years old man with schizophrenia was given AIIMS MAY 2013
10 mg haloperidol. 3 days later, he was brought back
by the father complaining of the patient looking C In Paget’s theory of cognitive development,
upwards and the eyeballs rolling up. Clinical exami- concepts like out of sight is out of the following
nation was otherwise normal. Most likely cause stage
Acute dystonic reaction. Sensorimotor stage.
C The treatment of choice for PTSD is C An alcoholic brought to the emergency after 2
Cognitive behavior therapy. days of abstinence complained of nausea,
C True about autism vomiting and dizziness and developing seizures
Poor eye contact, language delay and dermatoglyphic on 2nd day progressing to the GTCS. Drug of
abnormality. choice
Chlordiazepoxide > Diazepam
AIIMS MAY 2014 C Drug used as an anticraving agent in patients of
alcohol dependence
C True about blackouts
Acamprosate
Remote memory is relatively intact during the
A child was assessed with wechsler intelligence
blackout, it is a discrete episode of anterograde
C
scale and his intelligence was found to be in the
amnesia and associated with alcohol intoxication.
average range. His IQ is most likely to be
C According to ICD 10 revision, for establishing a
diagnosis of mania, the symptoms should persist 90
for at least C While performing a mental status examination
1 week (MSE), the awareness or understanding of a patient
of his/her illness is called
C The evidence based psychological therapy of choice
for depression is Insight
Cognitive behavior therapy.
AIIMS NOVEMBER 2012
AIIMS NOVEMBER 2013 C A man started taking alcohol at 20 years, presently
C A chronic alcoholic patient, who has not consumed taking 3 quarters daily over 30 years, now complains
alcohol for last 2 days, is brought to the emergency that he gets kick in 1 quarter, diagnosis is
by his wife with complaints of nausea, vomiting Reverse tolerance.
and dizziness. On second day he develops seizures C A child does not eat vegetables. His mother starts
that progressed to GTCS. Drug of choice for this giving him a chocolate every time he finishes
condition vegetables in the diet. The condition is
Chlordiazepoxide Operant conditioning
260 AIIMS and All India PGMEE—Review Questions

C The awareness regarding the disease in psychiatric AIIMS MAY 2011


patient in mental status examination
Insight C Cognitive model of depression was given by
C A middle aged man complains of lack of sleep Beck
during the night time. To know the duration of the C Best therapy to teach daily life skill to a mentally
time he is truly asleep or awake can be ascertained by challenging child is
Actigraphy Contingency
C A 14 years old boy is not able to get good grades on C Reflex hallucination is a morbid variety of
9th standard exam. But he is very sharp and Synanesthesia
intelligent. Best test to diagnose the problem (Note: In cannabis intoxication)
Specific learning disability test. C Healthy thinking includes
C True about frontotemporal dementia Continuity, constancy and organization.
Stereotypic behavior, age less than 65 years and
affective symptoms. AIIMS NOVEMBER 2010
Diagnosis a boy having diarrhea, rhinorrhoea,
AIIMS MAY 2012
C
sweating and lacrimation is
C The diagnostic feature that differentiates PTSD Heroin withdrawal.
from other disorders that occur following a C Middle aged lady complains of sudden onset
stressful incident is breathlessness, anxiety, palpitation and feeling of
Re-experiencing and avoidance of trauma. impending doom. Physical examination is normal.
C Disorder of forms of thought Diagnosis is
Derailment, tangentiality and loosening of associa- Panic attack
tion. C True about late onset schizophrenia
C Why brain is considered responsive? Onset after 45 years
Its functions are profoundly affected by changes in C A 20 years old boy complains of hearing of voices,
external environment. aggressive behavior for 2 days. He has fever for 2
days. When asked to his family, they says that he
AIIMS NOVEMBER 2011 has been muttering to self and gesticulating.
C Type D personality is a risk factor for There is no history of psychiatric illness. Diagnosis
is
Coronary artery disease.
Acute psychosis
(Note: Negative affectivity and social inhibition)
C Anterograde amnesia is seen in
C Specific somatoform disorders are
Head injury
Chronic fatigue syndrome, IBS and somatization
disorder.
C Counter transference is AIIMS MAY 2010
Doctor’s feeling towards patient. C A 3 years old girl has normal developmental
(Note: Transference is patient’s feeling towards milestones except delay in speech. She has difficulty
doctor) in concentration, communication and relating to
C Somatic passivity seen in others and does not make friends but plays with
Paranoid schizophrenia. herself. It is a case of
C Type 2 bipolar disorder Autism
Hypomania and depression. C Drug of choice in obsessive compulsive disorder is
C Panic attack is associated with disturbance in Fluoxetine
neurotransmitters C Naltrexone is used in opoid dependence to
Serotonin, cholecystokinin/pentograstin and Prevent relapse
GABA. C Most common substance abuse in India is
(Note: also NA, endogenous opioids and orexin) Tobacco
Psychiatry 261

C A young girl presenting with repeated episodes of memoty deficit, headache, convulsions, abnormal
excessive eating followed by purging by laxatives. movements, forgetfulness, 4 attacks during day and
Diagnosis is 2 attacks at night but CT scan is normal. It is a case of
Bulimia nervosa Epilepsy
C Increased suicidal tendency is associated with
AIIMS NOVEMBER 2009 Serotonin
C Stimulation of cranial nerve leading to elevation of
mood AIIMS NOVEMBER 2007
Vagus nerve C Perceptual interpretation of a real object is
A 20 years old girl presents with complaints of
Illusion
C
nausea, vomiting and pain in the legs. Physical
C Type A personality is characterized by
examination and laboratory finding normal.
Hostility, time pressure and competitiveness.
Probably it is a case of
Somatoform pain disorder. C Schizophrenia, true are
C Substance dependence is due to 3rd person auditory hallucination, inappropriate
Personality, peer pressure and family history of emotions and formal thought disorder.
substance abuse. C Psychodynamic model of disease explains the
C Included in personality trait psychopathologic cause of all mental illness to be
Sensation seeking, neuroticism and openness to Unconscious conflict
experience. C Learning involves
Modeling, exposure and response prevention.
AIIMS MAY 2009
AIIMS NOVEMBER 2006
C Anticraving agents for alcohol dependence are
Acamprosate, naltrexone and topiramate. C The drug for long term maintenance in opoid
C True about hallucination addiction
It occurs in the absence of perceptual stimulus. Methadone (Mn: M for M)
C A man taking 20 cigarettes per day, started
coughing, his family suggesting quitting cigarettes. AIIMS MAY 2006
He is ready to quit but thinks quitting will make him C A patient on haloperidol develops hyperpyrexia,
irritable. The best health planning model followed is rigidity, mutism, akinesia, sweating , tachycardia
Precontemplation and preparation. and increased BP and increased WBC count and
increased creatine phosphokinase with no history
AIIMS NOVEMBER 2008 of any other drug intake or any signs of infection
C Chemical increasing on electroconvulsive therapy Neuroleptic malignant syndrome .
Brain derived neurotrophic factor. C A patient on haloperidol develops tongue protrusion,
C Type 2 schizophrenia is characterized by stiffness, abnormal posture of limbs and trunk,
Negative symptoms, poor response to treatment oculogyric crisis with no loss of consciousness which
and CT scan abnormal. improved within a few minute of diphenyldramine
C According to Disability Act 1995, seventh disability HCI injection. He suffered from
Acute dystonia.
is usually referred to as
Mental illness C An adult person complaints of fear of leaving home,
fear of travelling alone and fear of being in a crowd
and develops anxiety and palpitation in these
AIIMS MAY 2008
situations and avoids public transport. He is
C Atypical antipsychotics are suffering from
Clozapine, olanzapine and risperidone. Agoraphobia.
C A 25 years old girl presents with 6 months history of (Note: Fear of open space, public space and crowded
altered sensorium, involuntary movements, space from where there is no easy escape)
262 AIIMS and All India PGMEE—Review Questions

C HIAA is a metabolite of writing and making frequent spelling mistakes but


Serotonin mathematical ability and social adjustments are
(Note: HIAA is 5-hydroxy indole acetic acid) good. He is suffering from
Specific learning disablility.
AIIMS NOVEMBER 2005 C Habits of day to day life can be removed by
Negative conditioning.
C Treatment of choice for a patient feeling uncomfor-
table in using lift, travelling and being in crowded
places AIIMS MAY 2004
Exposure C An engineering student presents with gradual
C Difference between obsession and delusion onset of suspiciousness, muttering, and smiling
The idea is regarded as senseless by patient in without reason, decreased socialization, violent
obsession. outbursts and lack of interest in studies for
C Dementia of Alzheimer’s disease is associated 8 months. He has blunt effect, thought broadcast,
with relatively preserved cognition, impaired judgment
Delusion, depressive symptoms and apraxia and and insight. He is suffering from Schizophrenia.
aphasia. C A 34 years old housewife presents with 3 months
C A person who laughs one minute and cries the next history of feeling low, lack of interest in activities,
with no clear stimulus have multiple bodyaches, worthlessness, altered
Labile effect. appetite and sleep with early morning awakening.
Drugs prescribed
AIIMS MAY 2005 Antidepressants
C A schizophrenic patient put on haloperidol
Opioid peptides are
presents with restlessness, fidgety, irritability and
C
Leu enkephalin, met enkephalin and beta endor- cannot sit at one place. Treatment of choice
phin Addition of beta-blocker.
C Inability to enjoy previously pleasurable activities (Note: Akathisia)
in a 25 years old lady with depressed mood for last
C A middle aged man with past history of mania
6 months is termed as
presents with depressed mood, anhedonia,
Anhedonia profound psychomotor retardation for last one
C A 2 years history of irritability, low mood, lack of month. Treatment
interest in surroundings and general dissatisfaction Antidepressants and mood stabilizers.
in middle aged man but no significant alteration in
sleep or appetite favours the diagnosis of
Dysthymia AIIMS NOVEMBER 2003
C Adulterants of heroin are C A schizophrenic patient on antipsychotic medication
Chalk powder, fructose and quinine. develops purposeless involuntary facial and limb
C Maintenance of drug for opioid dependence movements, constant chewing and puffing of
alternative to methadone is cheeks. Drugs likely to be involved
Buprenorphine Fluphenazine, haloperidol and loxapine.
(Note: Tardive dyskinesia)
A college student had a fight with the neighbouring
AIIMS NOVEMBER 2004
C
boy. On the next day while out, he started feeling
C Features of obsessive compulsive disorder that two man in police uniforms were observing his
Compulsive checking behavior, preoccupation movements. He was frightened, and thought police
with rule and perfectionism interfering with were after him and would arrest him. It is a case of
performance. Delusion of persecution.
C A 14 years old boy passes his examination with C A 40 years old man presents with abdominal pain
poor grades with difficulty in expressing himself in and headache with history of six abdominal
Psychiatry 263

surgeries. He seems to maintain sick role and seeks C Differential diagnosis of premenstrual tension
attention from the nurses. Extensive examinations includes
and investigations reveal no physical or psycho- Psychiatric depressive disorder, panic disorder and
pathological illness. He is suffering from generalized anxiety disorder.
Factitious disorder.
AIIMS MAY 2002
AIIMS MAY 2003 C An old man for last 10 years suspects his neighbours
and feels that whenever he passes, they plan against
C Drug of choice in ADHD
him behind his back. He also feels his wife has been
Methylphenidate
replaced by a double and calls police for help. He is
C Yawning is a feature of well groomed and alert. He is suffering from
Opioid withdrawal Paranoid schizophrenia.
C False perception without any external stimulus is C A patient is brought with 6 months history of odd
called behavior and he talks to self and sometimes mutters
Hallucination to himself loudly. There is family history of a
C Schneider’s first rank symptom relative disappearing some years back. He is
Voice commenting on actions. suffering from
C Elderly housewife lost her husband (by myocardial Schizophrenia
infarction) with home she stayed alone for decade. C Three years after an encounter with earthquake, a
A week after, she heard his voice talking to her but woman has nightmare about the episode and she
say nothing. Then she often heard his voice conver- also gets up in the night and feels terrified. She is
sing with her and also discussed her daily matters most likely suffering from
with him. But this provoked sadness and anxiety Post-traumatic stress disorder.
when preoccupied with his thought. Drug prescri-
bed ALL INDIA 2012
Haloperidol
C True about clozapine
If WBC < 3000/mm3, clozapine should be disconti-
AIIMS NOVEMBER 2002 nued, action of clozapine is more on D1 receptors
C Lack of insight is a feature of than D2 and combination of clozapine and
Mania, reactive psychosis and schizophrenia. carbamazepine should be better avoided.
C A 16 years old girl with marked fluctuations in her (Note: Clozapine induced agranulocytosis is
mood and pervasive pattern of unstable idiosyncratic and not dose dependent)
interpersonal relationship presents with several C Criteria for substance dependence
attempts of wrist slashing in the past in response to Repeated unsuccessful attempts to quit the
trivial fights at home. She is suffering substance, characteristic withdrawal symptoms;
Border line personality disorder. substance taken for relieving withdrawal and
C A 65 years old male present with marked substance taken in larger amount and for longer
forgetfullness, visual hallucination, deterioration in period than intended.
condition for last one year and mini-mental status C Serotonin dopamine antagonists are
examination score of 10. Diagnosis Zotepine, risperidone and sertindole.
Dementia C Paranoid psychosis observed with cocaine abuse
C A woman at 40 years of age presents with can be explained by
complaints of ache and pain from all over her body Intoxication
for 4 years. She has insomnia and has lost her C Best cognitive behavior therapy technique for
appetite she has lack of interest in work and poor treatment of 35 years old female patient diagnosed
socialization. She denies feeling of sadness. with OCD and she washing her hands several times
Diagnosis a day
Major depression Response prevention
264 AIIMS and All India PGMEE—Review Questions

C Poor scholastic performance is associated with C A chronic smoker taking 20 cigarettes per day has
Anxiety, ADHD and specific learning disability. developed chronic cough. His family suggested
C Delusion seen in quitting cigarettes. He is ready to quit but thinks
Dementia, depression and schizophrenia. quitting will make him irritable. His stage of
C Repetitive transcranial magnetic stimulation behavior change
(rTMS) is USFDA approved for the treatment of Contemplation and cost analysis.
Depressive disorder. C True about imbecile
C True about narcotic drugs and psychotropic Impaired self care, condition usually congenital or
substances (NDPS) act acquired at an early age and intellectual capacity
equivalent to a child of 3–7 years of age.
It provides for severe punishment for drug users
and peddlars alike.
ALL INDIA 2010
C Bizarre interpretation of an object is most appro-
priately known as C Autistic disorder is characterized by
Illusion Lack of social interaction, stereotypic movements
C General adaptation syndrome (GAS) is seen in and delayed development of speech.
Stressful situations. C Cognitive errors are
C A 16 years male patient suffering from drug abuse Catastrophic thinking, arbitrary inference and
has his perception of sensory modalities crossed overgeneralization.
over (he complains of sound can be seen and sights C In behavior therapy, behavior is increased by
can be heard). Drug responsible Reward, operant conditioning and negative
LSD reinforcement.
C Parts of cognitive behavior change technique
ALL INDIA 2011 Pre-contemplation, contemplation and action.
C A 60 years old man had undergone cardiac bypass
C A 40 years old patient presents with history of surgery 2 days back. Now he has started forgetting
depressed mood, loss of appetite, insomnia and no things and not able to recall names and phone
interest in surrounding for past 1 year. These numbers of relatives. It is a case of
symptoms followed soon after a business loss 1 year Cognitive dysfunction.
back. True about her management
(Note: Impaired memory and intellect)
Antidepressant treatment based on the side effect C Alcoholic paranoia is associated with
profile of the drugs.
Fixed delusions
C Antidepressant drug used in nocturnal enuresis
(Note: Alcohol induced psyclotic disorder)
Imipramine
C An old man is brought by his wife. He thinks that he
(Note: TOC-Bell and alarm, DOC-Desmopressin) has has committed sins all through his life. He is
C Most common cause of premature death in very much depressed and has considered
schizophrenia committing suicide. He had also attached sessions
Suicide with a spiritual guru. He does not want to hear
C A patient with acute psychosis who is on anything on the contrary. Treatment
haloperidol 20 mg/day for last 3 days, has an Antipsychotic and antidepressant.
episode characterized by tongue protrusion, C A 30 years old man for 2 months suspects his wife is
oculogyric crisis, torticollis, and abnormal posture having an affair with his boss. He thinks his friend
of limbs and trunk without loss of consciousness for is also involved from abroad and provides
20 minutes before presenting to the emergency. But technology support. He thinks people talk ill about
improved soon after diphenylhydramine. Diagnosis him. His friends tried to convince but he is not
is convinced at all. Otherwise he is normal, does not
Acute dystonia. have any thought disorder or any inappropriate
C Drugs used in treatment of alcohol dependence behavior
Acamprosate, naltrexone and disulfiram. Persitent delusional disorder.
Psychiatry 265

C A 28 years old girl presented with sadness, C Conditions included in diagnosis of bipolar
palpitation, loss of appetite and insomnia. But there disorder
is no complaint of hopelessness, suicidal thoughts Mania alone, mania and depression and mania and
and there is no past history of any precipitating anxiety.
event. She is doing her office job normally and her C Most common symptom of alcohol withdrawal is
social life is normal and she is doing well in other Tremor
areas of life
Generalized anxiety disorder.
ALL INDIA 2006
C A patient presents with self harm with suicidal
intent. Conditions requiring an immediate C Loosening of association is an example of
specialist assessment Formal thought disorder.
Formal thought disorder, social isolation and C Good prognosis for schizophrenia
chronic severe physical illness. Acute onset, late onset and married.
C Intense nihilism, somatization and agitation in old
age are the hallmark symptom of
ALL INDIA 2009
Involutional melancholia.
C As per ICD/DSM criteria, patients of which disease (Note: Form of severe depression)
are eligible for disability benefit as per national trust C Mutism and akinesis in a person appearing awake
act and alert is a condition known as
Mental retardation. Stupor
C Lithium is used in treatment of C Bright light treatment is most effective for
Major depression, vascular headache and Seasonal affective disorder.
neutropenia. (Note: Mood disorders with seasonal pattern)
C FDA approved indications of modafinil are C Rivastigmine and donepezil are used in manage-
Obstructive sleep apnoea syndrome, shift work ment of
syndrome and narcolepsy. Dementia
C Pavlov’s experiment is an example of
ALL INDIA 2008 Classical conditioning.
C Dementia precox term was given by
Kraeplin ALL INDIA 2005
C Clinical features of post-traumatic stress disorder C Preservation is
Flashback, hyperarousal and emotional num- Persistent and inappropriate repetition of the same
bing. thoughts
C A 40 years old father of son suffering from leukemia C Difference between hysteria and hypochondriasis
diagnosed 2 months back presents with lethargy, Symptoms do not normally reflect understandable
headache, low mood and sleep deprivation. Social physiological or pathological mechanism in
interaction is good but has absented himself from hysteria.
work. It is most probably C A 20 years old boy complains of feeling changed
Adjustment disorder from inside. He describes himself as feeling strange
C A 30 years man presents to OPD with erectile as if he is different from self. He is tense and anxious
dysfunction. Basic screening tests are normal. Next but unable to locate the change in himself. It is a case
step of
Oral sildenafil citrate. Depersonalization
C A lady has history of repetitive, irresistible thoughts
of contamination with dirt associated with
ALL INDIA 2007
repetitive hand washing. She reports these thoughts
C Delusion is a disorder of to be her own and dispersing. Treatment of choice
Thought Exposure and response prevention.
266 AIIMS and All India PGMEE—Review Questions

C Clinical approach for an 18 years old student ALL INDIA 2003


complaining of lack of interest in studies for last
6 months. He has frequent fights with parents and C Impulse control disorders are
has frequent headache Pyromania, trichotillomania and kleptomania.
Rule out depression. C Characteristic of delirium tremens is
Confusion with autonomic hyperactivity and
ALL INDIA 2004
tremors.
C An alcoholic who stopped drinking alcohol for last
C Signs of organic brain damage are evident on 3 days was brought with complaints of irrelevant
Bender Gestalt test. talking and has disorientation to time, place and
C A 25 years old nurse broke up with her boyfriend, person and visual hallucination. No history of head
lost interest in her hobbies of painting and dancing, injury. It is a case of
became convinced she would not be able to work Delirium tremens
again with feeling of worthlessness and attempted C A 40 years old housewife presents with complaints
suicide. It is most likely a case of of headache for last 6 months. Nothing was found
Adjustment disorder. abnormal despite several consultations and
C A housewife complains that her nose was longer investigation. She is still insistant that there is
than usual, and feels her husband did not like something wrong in her head and seeks another
her because of that deformity. It was not possible consultation. She is suffering from
to convince her that there was no deformity. She Hypochondriasis
has C A young adult man presents with increased alcohol
Delusion intake, sexual indulgency irritability, lack of sleep
and not feeling fatigued even on prolonged exercise
C An alcohol addict for ten years presents with
for last 3 weeks. He is suffering from
history of no alcohol intake for last 3 days, coarse
tremors, visual hallucination and disorientation to Mania
time. Drug of choice C Principle used in behavior therapy to change mala-
daptive behaviours using response as reinforce is
Chlordiazepoxide > Diazepam
Operant conditioning.
C A 30 years old married woman from a poor family
C A young boy feels that the dirt has hung onto him
strongly believes that her rich neighbour boy is in
whenever he passes through the dirty street causing
deep love with her despite his denial and resistance
distress and anxiety leading to preoccupation with
from her family. She is able to maintain her daily
dirt and contamination and social withdrawal.
routine. She is suffering from
Insight is present. He is suffering from
Delusional disorder Obsessive compulsive disorder.
C Non-REM sleep associated with C A 50 years old man presents with back pain, lack of
Night terrors interest in recreational activities, low mood,
C A 16 years old boy does not attend school because lethargy, decreased sleeps and appetite for two
of fear of being harmed by classmates. He thinks his months. No other physical or somatic abnormality
classmate laugh at and talk about him. He is scared was found. He is prescribed
of going out of his home. He is suffering from Sertraline
Schizophrenia (Note: SSRI)
C A 40 years old female executive director is C Features of hallucination are
convinced that management has denied her It occurs in the absence of perceptual stimulus,
promotion by preparing false reports about her independent of the will of observer and sensory
competence and has forged her signature to convict organs are not involved.
her. She even files a complaint in the police but C Symbol resembling adopted children
attends to her work and manages the household. [?] [?]
She is most likely suffering from C IQ of 15 years old boy with mental age of 9 is
Persistent delusional disorder. 60
Psychiatry 267

ALL INDIA 2002 with thrashing movements of his limbs. There is no


apparent precipitating event or he does not recall
C Features of hallucination these events. He is suffering from
It is a vivid sensory perception, occurs in inner sub- Temporal lobe epilepsy
jective space and occurs in absence of perceptual C A young lady presents with repeated episodes of
stimulus. overeating followed by purging after use of
C Suggestive of organic cause of behavioral symp- laxative. Diagnosis is
toms Bulemia nervosa
Prominent visual hallucinations. C An 11 years old boy is all the time so restless, the
C Delusion is present in whole class is unable to concentrate. He is hardly
Depressing, mania and delirium. ever in his seat and roams around the class. Does
C A boy presents with short lasting episodic beha- not play quietly. Diagnosis is
vioral changes including agitation, dream like state ADHD
268 AIIMS and All India PGMEE—Review Questions

19

Orthopedics

AIIMS NOVEMBER 2015 C Jersey finger


Flexor digitorum profundus
C Hyperextension of MCP joint and flexion of DIP
(Note: FDP tendon is injured at its point of attach-
joint of little and ring fingers occur due to
ment to the distal phalanx).
Ulnar nerve
C Judget view is for
(Note: Claw hand)
Pelvis
C Meragia paresthetica is due to the involvement of
C A 45 years old man jumps from height from a
Lateral cutanous nerve of thigh building. One witness told the doctor that patient
C True about osteoblast- jumped from the height due to fire in the building
Regulated by BMP and landed on the ground on feet. Finding in
Derived from osteoprogenitor cells keeping with the details
Have neuropeptide receptors Ring fracture in skull with lumbar spine injury.
(Note: Plasma membrane showing multiple folds C Surgery useful for removal of intervertebral disc
are characteristic of osteoclasts.) Laminotomy, laminectomy and hemilaminectomy.
C Axillary nerve can be injured in- (Note: Laminoplasty is used for lumbar canal stenosis)
Fracture surgical neck of humerus C Irreparable rotator cuff injury is treated by
Shoulder dislocation Tendon transfer.
Intramuscular injections.
AIIMS NOVEMBER 2014

AIIMS MAY 2015 C Bone dysplasia is strictly because of


Faulty development
C Tractions used in lower limb are
C The maximum change in bone mineral density in
Gallow’s, Bryant and Perkin’s. hemiplegic patients after 1 year is seen in
(Note: Dunlop’s traction is used in supracondylar Humerus of the paretic side.
fracture of humerus in children)
C A 2 months old baby is lifted by her mother by
C Most active part of bone holding the wrist of the child. The child started
Periosteum crying and was not moving her elbow. The most
C Shenton line is in likely diagnosis
Hip Pulled elbow
(Note: Imaginary line drawn along the inferior C Feature of Colles’ fracture
border of superior pubic ramus and along the Radial displacement, dorsal displacement and
infero-medial border of the neck of femur. supination.

268
Orthopedics 269

C A 10 years old child presents with a cortex based C Causes of clubfoot in neonates
lesion surrounded by reactive sclerosis in the CTEV, spina bifida and arthrogryposis multiplex
middle of shaft of tibia. The most likely diagnosis congenita.
Fibrous cortical defect. C A 40 years old man presented with acute onset pain
C For pronation to occur, which 2 joints must have and swelling of left great toe. On X-ray, punched
their axis of rotation in parallel out lytic lesion seen on phalanx with sclerotic
Talonavicular and calcaneocuboid joints. margins and overhanging bony edges. Most likely
C The H-reflex is important for assessing which of the diagnosis
following Gout
S1 radiculopathy. C Both bone and disc spaces are destroyed in
C Sunray appearance in osteosarcoma is due to Tuberculosis
Periosteal reaction. C True about osteoporosis
C An elderly post-menopausal lady presented with Calcitonin decreases pain, bisphosphonates are
swelling in her right great toe. X-ray of the foot work horse for treatment and PTU is used in severe
showed punched out erosions of the great toe, with osteoporosis.
overhanging margins and adjacent soft tissue
masses. Most likely diagnosis AIIMS MAY 2013
Gouty arthritis.
C An elderly lady is on treatment with alendronate
for 7 years. She now presents with complaints of
AIIMS MAY 2014 pain in the thigh. The best investigation for her
C The earliest sign of cord compression in tuberculosis X-ray
of thoracic spine C An elderly lady falls and is diagnosed with Colle’s
Extensor plantar fracture. After proper treatment, she now
C A 5 years old boy presents with pain and swelling complains of stiffness and severe pain in the wrist
in diaphysis of tibia along with fever and raised ESR. with cold sensation and cyanosis of the fingers.
Most likely diagnosis X-ray of the hand shows spotty deossification. Most
Ewing sarcoma likely diagnosis
C The signs of malignant transformation in osteochon- Sudeck’s dystrophy.
droma are C A 22 years old man suffered a left knee injury while
Pain, increase in size and increase in thickness of playing football. On examination, there was
cartilage cap. anterior laxity in full extension but it was normal at
90° flexion. Most likely injured part is
Posterolateral bundle of ACL.
AIIMS NOVEMBER 2013
C After trauma, lower limb is noted to be shortened
C A 24 years old college student injures his right knee with abduction and internal rotation. Type of hip
while playing hockey. The patient presents with dislocation present
instability of knee joint in full extension without Central
instability at 90° of flexion. The structure most C A person was found lying in the right lateral position
commonly damaged by the police. He had injuries on his right face, hand
Posterolateral bundle of anterior cruciate ligament. and on the right knee. Which nerve injury can
C True about supracondylar fracture of humerus explain the injuries caused in the patient
Nerve injury related manifestations are transitory. Femoral nerve
C Gallow’s traction is applied in C A child presents with a fracture in his leg. He was
Fracture shaft of femur. given Gallow’s traction after being admitted. He is
C A 7 years old child presents with fever and tibial suffering from
swelling exhibits periosteal reaction on X-ray and Fracture shaft femur.
raised ESR and TLC. Next done C True about supracondylar fracture in children
MRI The neurological complications are transitory.
270 AIIMS and All India PGMEE—Review Questions

AIIMS NOVEMBER 2012 C A middle aged lady presents with complains of


lower back pain. On examination there is weakness
C A young male patient becomes unconscious suddenly, of extension of right great toe with no sensory
awakens and finds both of his arms adducted and impairment. An MRI of the spine reveals prolapsed
internally rotated. Most probable diagnosis intervertebral disc at the level of
Posterior dislocation. L4–L5
C Ankle sprain due to forced inversion of a plantar C A 8 years old child presents with fever, pain and
flexed foot is due to injury to swelling in mid thigh, lamellated appearance and
Anterior talofibular ligament. Codmen’s triangle on X-ray, histopathology
C An elderly lady is on treatment with alendronate showing small round cell tumour positive for MIC
for 7 years and now presents with pain in the thigh. 2. Diagnosis is
Next done Ewing’s sarcoma.
X-ray C An old lady fell in the bathroom and could not move
C A young man gets involved in a fistcuffs, injures his and had leg in externally rotated position, with
middle finger and there is slight flexion of DIP joint, tenderness in triangle of scarpa and limb movement
X-ray normal. Management not possible due to pain. On X-ray no hip fracture
Splint the finger in hyperextension. was seen. Next done
C A 12 years old boy presents with a central expansile MRI
cystic lesion without any periosteal reaction in C Gallow’s traction used for
proximal humerus on X-ray. What can be done for
Fracture shaft of femur
his treatment
C After alcohol binge at night, patient comes to the
Curettage, intralesional steroids and intralesional
emergency with unable to move his hand due
sclerosing agent.
sleeping on the arm chair next done
C Dennis stability concept is based on
Knuckle bender splint.
3 column concept
C A person is able to abduct his arm, internally
C A pedestrian is hit by a moving vehicle on lateral
rotate it, place the back of hand on the lumbosacral
side of the knee. X-ray is done and a fracture line is
joint, but is not able to lift it from the back. Cause
seen running through the intercondylar eminence.
is
Structures most likely to be injured
Subscapularis tendon tear.
Anterior cruciate ligament.
C A 7 years old child presents with fever, swelling of
tibia, raised ESR and TLC and periosteal reaction in AIIMS NOVEMBER 2011
plain X-ray. Next investigation
C Markers of bone formation are
MRI
Alkaline phosphatase, type 1 procollagen and
A 2 years old child presents with rickets and limb
osteocalcin.
C
deformity. He is treated with vitamin D and
C Provocative tests performed in a 56 years old female
calcium. Best time for limb deformity surgery
presenting with nocturnal pain in the right thumb,
When bone specific alkaline phosphatase is
index and middle fingers for past 3 months
normal.
Phalen’s test, Tinel’s sign and tourniquet test.
C Ideal management for a 65 years old alcoholic
AIIMS MAY 2012 suffering from diabetes presenting with flexion
C A newborn child presents with inverted foot and deformity at the right little finger over the
the dorsum of the foot cannot touch the anterior metacarpophalangeal joint of around 15°
tibia. It is a case of Observation
CTEV C A cricketer holds a catch and then presents with
C Dial test is positive in knee injury in road traffic pain at the base of right thumb. He should be
accident in injury of examined to specifically rule out
Posterolateral corner injury. Ulnar collateral ligament.
Orthopedics 271

C The first centre of ossification appear during C A person is able to abduct his arm, internally rotate
At the end of 2nd month of pregnancy. it, place the back of hand on lumbosacral joint, but
C A middle aged lady presenting with complaints of is not able to lift it from back. Etiology is
lower back pain. On examination Subscapularis tendon tear.
Weakness of extension of great toe with no sensory
impairment. MRI reveals prolapsed IV disc at level- AIIMS MAY 2010
L4–L5. C Posterior glenohumeral instability can be tested by
(Note: Extensor hallucis longus is supplied by L5) Jerk test
C Meralgia paresthetica is due to involvement of
AIIMS MAY 2011 Lateral cutaneous nerve of thigh (Mn: Mera is late)
C Characteristic radiological feature of fibrous
C Non-neoplastic lesions stimulating bone tumor are dysplasia is
Fibrous dysplasia, bone island and bone infarct. Ground glass appearance.
C In bounce home test of knee joint, end feel is descri- C A 66 years old man presents with pain and swelling
bed as of right knee. Ahlback grade 2 osteoarthritic
Bony, spongy and firm. changes were found on investigation. Further
C According to Enneking system, true regarding management is
active benign tumor is Conservative
Intracapsular, thick rim of reactive bone and C A 65 years old man with back pain since 3 months,
extended curettage is treatment. raised ESR, marked stiffness and mild restriction of
C Variant of giant cell tumour is chest movements and syndesmophytes in
vertebrae on X-ray. Diagnosis is
Chondroblastoma
Ankylosing spondylitis.
C Percutaneous vertebroplasty is indicated in C Synovial fluid-true are
Metastasis, osteoporosis and hemangioma. Follows non-Newtonian fluid kinetics, contains
C True about high tibial osteotomy hyaluronic acid and variable viscosity.
Deformity recurs after a long time, done through C A lady presents with right knee swelling with
cancellous bone and done in case of unicompart- CPPD crystals on aspiration. Next investigation
mental disease. among the choice is
TSH
AIIMS NOVEMBER 2010 C Major mineral of bone is
Hydroxyapatite
C A patient presents with injury to left knee after road
C Pulsatile tumor is
traffic accident. Dial test was positive. Cause is
Osteoclastoma > osteosarcoma.
Posterolateral coroner ligament injury.
C Gallow’s traction is used for fracture AIIMS NOVEMBER 2009
Shaft femur
C Commonest site in bone involvement in hematoge- C Seen in pseudogout
nous osteomyelitis Large joints affected, chondrocalcinosis and
deposition of calcium pyrophosphate.
Metaphysis
C Young girl presenting with swelling of right thigh
Associated with poor prognosis in Ewing’s sarcoma
with history of trauma 2 months back. Now she
C
Fever presents with swelling of midshaft of femur and
C A 35 years old lady with chronic backache with D12 low grade fever. ESR is mildly raised. Laminated
collapse but intervertebral disc space maintained. periosteal reaction on X-ray. Next investigation is
Differential diagnosis are MRI
Multiple myeloma, osteoporosis and metastasis. C Treatment of choice for non-united fracture of lower
C A newborn child presents with inverted foot and 1/4th tibia with multiple discharging sinuses and
dorsum of foot cannot touch the tibia. Probably it is various puckered scar with 4 cm shortening of leg
CTEV Ilizarov’s fixation.
272 AIIMS and All India PGMEE—Review Questions

AIIMS MAY 2009 AIIMS NOVEMBER 2007


C Tarsal tunnel syndrome is associated with C Surgical staging of bone tumor is by
Rheumatoid arthritis. Enneking
C Osteosclerotic bone metastasis is found most C Kanavel’s sign is seen in
commonly in Tenosynovitis
Prostate carcinoma. C Diagnostic features of osteomalacia
C Earliest site of bone involvement in hematogenous Increased alkaline phosphatase, proximal myopathy
osteomyelitis and looser’s zone.
Metaphysis
C A 7 years old boy with sudden onset hip pain and AIIMS MAY 2007
hip held in abduction.Normal hemogram and
raised ESR. Next step C Young child presenting with a lesion in upper tibia.
USG guided aspiration of hip. X-ray shows radiolucent area with Codman’s
C True about crescent fracture is triangle and sunray appearance. Diagnosis is
Fracture of iliac bone with sacroiliac disruption. Osteosarcoma
C Kocher Lagenbeck approach for emergency aceta- C X-ray of young man showing heterotropic calcifica-
bular fixation is done in tion around bilateral knee joints. Next investigation is
Open fracture and recurrent dislocation inspite of Serum phosphate/serum alkaline phosphatase.
closed reduction and traction. C Bohler’s angle is decreased in fracture of
Calcaneus
AIIMS NOVEMBER 2008 C Deep heat therapy are
C Velpeau bandage and sling and swathe splints are Short wave diathermy, ultrasound therapy and
used in microwave therapy (Mn: Deep Sumi)
Shoulder dislocation C Most common site of osteosarcoma is
C Joints involved in rheumatoid arthritis according to Lower end of femur.
1987 modified ARA criteria are C True about congenital torticollis
Knee, ankle and metatarsophalangeal. Spontaneous resolution in most cases, 2/3rd cases
C Watson Jones ’ operation is done for have palpable neck mass at birth and uncorrected
Hip replacement cases develop plagiocephaly.
C Risser localizer cast is used in C Rickets in infancy is characterized by
Idiopathic scoliosis Craniotabes, rachitic rosary and wide open
fontanelles.
C A person with multiple injuries develops fever,
restlessness, tachycardia, tachypnea and perium- C In patients with osteoarthritis of knee joint, atrophy
bilical rash. Diagnosis is occurs most commonly in
Fat embolism Quadriceps only
C Most common cause of insertional tendonitis of C A 60 years old male with bony abnormality at upper
tendoachilles is tibia associated with sensorineural hearing loss.
Serum alkaline phosphatase increased but serum
Overuse
calcium and phosphate normal. Skeletal survey
AIIMS MAY 2008 shows ivory vertebrae and cotton wool spots in X-
ray skull. Diagnosis is
C Best prognosis after nerve repair Paget’s disease
Radial
C Seen in osteopetrosis
Compression of cranial nerve, osteomyelitis of AIIMS NOVEMBER 2006
mandible and pancytopenia. C Pigmented villonodular synovitis most commonly
C Material used in vertebroplasty affects
PMMA Knee
Orthopedics 273

C Associated with CV junction anomalies C Pain and tenderness over lateral condyle of
Basilar invagination, odontoid dysgenesis and humerus with painful dorsiflexion of wrist indi-
ankyosing spondylitis. (Mn: ABO) cates
C Bankart’s lesion involves part of glenoid labrum Tennis elbow
Antreior part (Mn: Bankart’s has an) C Diagnosis of one year old child presenting with
multiple fractures seen in various stages of healing
is
AIIMS MAY 2006
Batterred baby syndrome.
C Transient synovitis/toxic synovitis of hip is charac-
terized by
AIIMS MAY 2005
May follow upper respiratory infection, ESR and
WBC counts are usually normal and USG hip shows C Pathognomonic feature of rheumatoid arthritis
widening of joint space. Extensor pollicis brevis and abductor pollicis
C The malunion of supracondylar fracture of longus.
humerus leads most commonly to C Bone tumor of epiphysis of long bone
Cubitus varus Chondroblastoma
C Skeletal traction can be given by C Level of tendon sheath contraction in trigger finger
Bohler’s stirrup, Kirschner’s wire and Steinman’s is at
pin. MCP joint
C Recurrent dislocation of shoulder occurs with
Bankart’s lesion, capsular laxity and Hill- Sach’s AIIMS NOVEMBER 2004
lesion.
C Sudden increase in pain in osteochondroma caused C Middle aged alcoholic male patient presents with
due to pain in dorsal spine tenderness at dorsolumbar
Bursitis, fracture and sarcomatous change. junction and destruction of 12th dorsal vertebra
with loss of disc space between D12-L1 vertebra in
C Complications of supracondylar fracture of humerus
X-ray. Diagnosis
in children are
Pott’s spine
Malunion, myositis ossificans and compartment
C Painful swelling over the left shoulder in a 15 years
syndrome.
old boy with osteolytic area and stippled calcifica-
C The differential diagnosis of giant cell tumor in tion over proximal humeral epiphysis on X-ray and
small bones of the hands or feet includes immature fibrous matrix with scattered giant cells
Aneurysmal bone cyst, hyperparathyroidism and on biopsy may be diagnosed as
osteosarcoma. Chondroblastoma
C X-ray of leg of a 15 years old boy reveals evidence of C Slight flexion of DIP joint in middle finger injury
aggressive bone tumor with both bone destruction during fist cuff with normal X-ray finding in a 25
and soft tissue mass and biopsy further revealing years old male patient can be best managed by
bone cancer with neural differentiation. Most likely Splint the finger in hyperextension.
diagnosis is C A 2 years old child holding her father’s hand
Ewing’s sarcoma. slipped and fell but did not let go of her father’s
hand, continued to cry and holding the forearm in
pronated position refused to move the affected
AIIMS NOVEMBER 2005 extremity. Appropriate management
C Lab finding of 50 years old patient presenting Supinate the forearm.
with multiple pathological fractures are serum C A 50 years old man sustained posterior dislocation
Ca-11.5, phosphate 2.5 mg/dl and alkaline phos- of left hip in an accident, reduced after 3 days. He
phates 940 IU/dl. The most probable diagnosis complains of pain in hip (left) after 6 months but X-
is ray of pelvis normal. Next investigation
Hyperparathyroidism MRI of hip
274 AIIMS and All India PGMEE—Review Questions

AIIMS MAY 2004 C Stability of joint is maintained by


Deltoid ligament, lateral ligament and shape of
C A man presents with acute onset of low backache superior talar articular surface.
radiating to the right lower limb. Examination of C Prognosis of union of simple and undisplaced
right leg reveals- SLRT < 400, weakness of EHL, fracture of shaft of tibia of right leg with above knee
sensory loss in first web space and brisk knee jerk. plaster cast with fractured limb affected by
Diagnosis is poliomyelitis four years age
Prolapsed intervertebral disc L4–5. Fracture will unite normally.
C A woman on regular treatment for pemphigus C Best investigation for prognosis of recovery of the
vulgaris presents with pain in right hip and knee. paralyzed arm of a pole vaulter who fell during pole
Tenderness in scarpa’s triangle and limitation of vaulting
adduction and internal rotation of right hip joint but
Electromyography.
no limb length discrepancy was found on
Differential diagnosis of a patient with a history of
examination. Diagnosis is
C
burning pain localized to plantar aspect of foot
Avascular necrosis of femoral head.
C Following a seizure attack, a patient presents with Tarsal-tunnel syndrome.
pain in right shoulder and right upper limb was C Cause of avascular necrosis of head of femur in an
adducted and internally rotated as found on elderly woman with fracture neck of femur may be
examination. Most likely diagnosis is Damage to retinacular branches of circumflex
Posterior dislocation of shoulder. femoral arteries.
C Following excision of right radial head, a patient
develops inability to extend the fingers and thumb AIIMS MAY 2003
of that side but no sensory deficit. Most likely nerve C Direct impact on bone produces
injured is
Transverse fracture
Posterior interosseous nerve.
C A 12 years old presents with low grade fever and
C Three bony point relationship in elbow is preserved pain in the leg for many weeks with X-ray revealing
in a 10 years old boy presenting with cubitus varus mass in diaphyseal region of femur with overlying
deformity and history of trauma 3 months back in cortical erosion and soft tissue extension and biopsy
case of of lesion showing numerous small round cells, rich
Malunited supracondylar fracture of humerus. in PAS positive diastase sensitive granules.
C Osteoblastic metastasis is most commonly seen Diagnosis is
in Ewing’s sarcoma
Prostate cancer C Whip lash injury is caused due to
C A 30 years old male patient presents with slowly Acute hyperextension of the spine.
progressive swelling in middle 1/3rd of right tibia C Bony lesion showing osteoblastic activity on bone
with X-ray finding showing multiple sharply scan
demarcated radioluscent lesion separated by areas Paget’s disease, osteoid osteoma and fibrous
of dense and sclerotic bone and biopsy finding dysplasia.
revealing island of epithelial cells in a fibrous
stroma. Diagnosis is AIIMS NOVEMBER 2002
Adamantinoma
C Medial meniscus of knee joint is injured more than
lateral meniscus because of
AIIMS NOVEMBER 2003 Less mobility
C Microfracture surgery is done for C Component of crush syndrome
Osteochondral defect of femur. Massive crushing of muscles, myohemoglobinuria
C Most common component of lateral collateral and acute tubular necrosis.
ligament complex offered in an ankle sprain is C An adult man uses hammer for repair work since
Anterior component morning and by evening feels pain in lateral side of
Orthopedics 275

elbow and also that hammer was becoming heavier C After lifting something heavy from ground, a
and heavier. Muscles affected are patient complains of back pain radiating to lateral
ECRL and ECRB leg and great toe of lower limb. Diagnosis
C In jaw dislocation displacement of articular disc L4–L5 disc prolapse.
beyond articular tubercle of TM joint resulting from C Complications of fracture neck of femur
spasm of excessive contraction of Avascular necrosis, non-union and shortening (Mn:
Lateral pterygoid ANS)
C The classical flexion and rotation deformity at hip C Following an accident a person is not able to abduct
and knee joints in poliomyelitis are due to his shoulder and flexion movement of elbow joint is
contracture of also not possible. Other movements are normal. Site
Tensor fascia lata. of injury
C Most sensitive test in an acutely injured knee of an Upper trunk.
athlete is C A patient presents with recurrent posterior
Lachmann test dislocation of shoulder. On radiography, most
C A 10 years old boy presents with pain in right arm probable site for Hill Sach’s lesion
near the shoulder with X-ray finding showing Anteromedial
expansile lytic lesion in upper third of humerus. C Avascular necrosis in fractured proximal fragment
Diagnosis is may be of scaphoid bone is due to
Simple bone cyst. Retrograde blood flow in scaphoid.
Activity difficult for a patient with anterior cruciate
A young girl with rapid increase in height for last
C
C
deficient knee joint
one year is having difficulty in sitting cross legged
Walk down hill and squatting and complains of knee going into
axilla every time she squats. Diagnosis
AIIMS MAY 2002 Slipped capital femoral epiphysis.
C Most common site of adamantinoma of long bone C A patient after RTA presents with flexion and exter-
nal deformity of the left hip, shortening of affected
Tibia
limb by 7 cm, mass noted in left gluteal region moving
C Chondroblastoma occurs in with the movement of the femur. X-ray finding
Epiphysis Acetabular roof fracture with central dislocation.
C X-ray finding of a 10 years old boy with pain in
C A 30 years old female patient presents with pain
upper end of humerus reveals lesion of metaphysis
and tenderness in index finger just under the nail,
with breech of overlying cortex. Most likely it is a
inability to wash her hands with cold water but no
case of
history of trauma or injury. Further finding
Aneurysmal bone cyst.
Ridging of nail, discoloration and pin head tenderness
C Twisting injury of knee in flexed position results in
(Hint: Glomus tumour).
injury to
Tubercular osteomyelitis may be
Anterior cruciate ligament, capsular tear and
C

meniscal tear Paucibacillary and hematogenous.


C Chondrocalcinosis is seen in C Most common cause of bone disease in India
Ochronosis especially in females
C Most commonly injured structure in torsion of knee Nutritional deficiency.
Medial meniscus C Included as a yellow flag sign for low back pain
Systemic steroids, social isolation and high
functional limitation at/after 4 weeks.
ALL INDIA 2012
C Pain due to post-amputation neuroma is best
C Dermatome of little finger of hand treated by
C8 Ultrasound therapy.
276 AIIMS and All India PGMEE—Review Questions

C A 40 years old male patient presents with weakness C Structure first fixed during reimplantaion of an
and pain in lower limbs, history of paralysis of both amputated digit is
lower limbs during childhood. Diagnosis Bone
Post polio syndrome.
C True about SACH foot
Shoes can be worn out, central wooden keel present ALL INDIA 2010
and base of lower limb prosthesis C Lift off test
(Note: Solid ankle cushion heel foot). Subscapularis
C True about menisci
ALL INDIA 2011 Lateral menisci covers more tibial articular surface,
medial meniscus is more commonly injured and
C Main blood supply to head and neck of femur
menisci are predominantly made up of
comes from
Type 1 collagen
Medial circumflex femoral artery.
C Sixty years old lady is presenting with long C Median nerve lesion at the wrist causes
standing history of pain and swelling in her right Thenar atrophy, weakness of 1st and 2nd lumbricals
knee and pain is significantly interfering with her and weakness of FPB.
activities of daiy living. Radiological evaluation C True about articular changes in aging
shows grade 3 changes of osteoarthritis. Manage- Synthesis of proteoglycans is decreased, total
ment of such a patient is proteoglycan content is decreased and total water
Total knee replacement. content of cartilage is decreased.
C Atheletic teenage girl complaining of anterior knee C Metal on metal articulation should be avoided in
pain on climbing stairs and on getting up after Young female.
prolonged sitting. Most likely diagnosis is C Breathlessness and chest pain on second post-
Chondromalacia patellae. operative day of hip replacement with right ventri-
C A 50 years old lady sprained her ankle 2 moths back cular dilatation and tricuspid regurgitation on
from which she made a steady recovery. 2 months echocardiography. Diagnosis is
after the injury she gradually developed severe Pulmonary embolism.
pain in her right ankle with significant limitation of
C Characteristic triad of Klippel-Feil syndrome
ankle movement. Clinical examination reveals
includes
edema and shiny skin. Likely diagnosis is
Short neck, low hairline and restricted neck move-
CRPS 1
ment.
C Most common nerve involved in supracondylar
fracture of humerus is C Progression of congenital scoliosis is least likely in
vertebral anomaly
Anterior interosseous nerve.
C A six years old boy presents with painful limp, Block vertebra
tenderness in femoral triangle and some limitation C A patient involved in road traffic accident presents
of hip movements with normal X-ray. Next course with quadriparesis, sphincter disturbance,
of action is sensory level up to the upper border of sternum
MRI scan and respiratory rate of 35/minute. Level of lesion
C Blount’s disease is characterized by is
Genu valgum, genu recurvatum and internal tibial C4–C5
torsion. C True about synovial cell sarcoma
C True about radial nerve palsy Occurs more often at extra articular sites, usually
Loss of nerve supply to ECRB, EPB and loss of seen in patients less than 50 years of age and knee
sensation over first dorsal web space. and foot are common sites involved.
C Agent decreasing bone resorption in osteoporosis C Brown tumor is seen in
Alendronate, etidronate and strontium. Hyperparathyroidism
Orthopedics 277

ALL INDIA 2009 C A patient with HIV is on therapy with protease


inhibitor. He presents with limitation of abduction
C In gout, synovial fluid aspiration shows and internal rotation of the hip. The diagnosis most
Monosodium urate crystals. likely is
C Recommended treatment of chronic low back pain Avascular necrosis of femoral head.
NSAID, exercise and epidural steroid injection. C A 56 years old type 2 diabetic presents with
C Recurrent dislocations are least common in swelling of left ankle with effusion but no pain. X-
Ankle ray shows severe osteopenia with bone destruction,
C The primary pathology in atheletic pubalgia is extensive osteophytosis and loose bodies.
Abdominal muscle strain. Management includes
C Most common cause of death after total hip Rest and splinting, aspiration and compression
replacement bandage and ankle arthrodesis.
Thromboembolism C Pollicization can be best described as
C Myodesis is employed in amputations for Thumb reconstruction.
Trauma, tumor and children. C A 16 years old male presenting with extensive
heterotrophic ossification over the neck, back and
C Nerves involved in entrapment neuropathy
shoulders and decreased chest movements with
Median nerve, ulnar nerve and lateral cutaneous history of progressive immobility since the age of
nerve of thigh. 3 years. True are
C Muscle most commonly affected by congenital Predisposed to pneumonia, have short hallux and
absence is increased expression of BMP4 gene.
Pectoralis major
C T 10 protocol for treatment of osteosarcoma includes ALL INDIA 2007
High dose methotrexate, BCD (bleomycin, cyclo-
phosphamide, and doxorubicin) and vincristine. C Tube cast applied in fracture around
C Milkman’s fracture is a type of Knee
Claw hand involves
Pseudofracture
C
Ulnar nerve
ALL INDIA 2008 C Most common force involved in fracture of spine is
Flexion.
C Autologous bone graft is taken from C Mechanism of violence in burst fracture of spine
Iliac crest Compression violence.
C True about tubercular osteomyelitis C Bone tumor in epiphysis
Secondary osteomyelitis, sequestrum uncommon Osteoclastoma
and inflammation minimal.
C Investigation required in osteosarcoma
Coronary ligaments of the knee are
MRI femur, CT scan and bone scan.
C
Ligaments connecting menisci to tibia.
C Pulsating tumor of bone
Patient gets up from sitting position, movement in
Osteosarcoma
C
knee joint
C Marker of new bone formation
Medial rotation of femur on a fixed tibia.
Alkaline phosphatase
C In acute knee injuries swelling and hemarthrosis
with muscle spasm. Most sensitive test to detect
ALL INDIA 2006
anterior cruciate ligament injury is
Lachman’s test C The ideal treatment of bilateral idiopathic clubfoot
C The blood supply of anterior cruciate ligament is in a neonate is
primarily derived from Manipulation by mother
Middle genicular artery. C True about developmental dysplasia of hip
C Menisectomy is a more suitable option than More common in female, oligohydromnios is associa-
meniscal repair in ted with higher risk and hourglass appearance of
Meniscal tear in inner zone. capsule may prevent a successful closed reduction.
278 AIIMS and All India PGMEE—Review Questions

C True about brachial plexus injury C Clinical features of Klippel-Feil syndrome


Abductors and external rotators of the shoulder are Gross limitation of neck movements, low hairline
paralyzed in Erb’s palsy, ipsilateral Horner syndrome and bilateral neck webbing.
may be present in Klumpke’s palsy and histamine
test in useful to differentiate between preganglionic ALL INDIA 2004
and post-ganglionic lesions.
C Rugger jersy spine is seen in C An army recruit, smoker, 6 months into training
Renal osteodystrophy. complains of pain at posteromedial aspect of both
legs with acute point tenderness and aggravation of
C Conditions presenting as an eccentric osteolytic lesion
pain on physical activity. It is most probably a case
Aneurysmal bone cyst, fibrous cortical defect and
of
giant cell tumor.
C True about exostosis Stress fracture
Present at growing end of bone, covered by cartilagi- C Cartilaginous tissue forms over the fractured bone
nous cap and malignant transformation may occur. ends with a cavity in between containing clear fluid
C Brown tumors are found in in some old fractures called as
Hyperparathyroidism Pseudoarthrosis
C Radioresistant malignant tumor C Aseptic loosening in cemented total hip replace-
Osteosarcoma ment due to hypersensitivity response to
C Expansile lytic osseous metastates are characteristic of High density polythene debris.
Renal cell cancer. C In olecranon fracture, excision of proximal fragments
is indicated in
ALL INDIA 2005 Old ununited fractures, non-articular fractures and
elderly patient.
C Sudden breathlessness two days after fracture
femur in RTA in a 30 years old patient is caused by C Parts of vertebra commonly affected in tuberculosis
Fat embolism of spine are
C Difficulty in walking and pain in both hips after 2 Body, lamina and pedicle.
years of steroid treatment indicated after renal C Complications of malunited Colle’s fracture
transplant in 44 years old male patient. Cause Carpal instability, carpal tunnel syndrome and
Avascular necrosis reflex sympathetic dystrophy.
C Investigation of choice for suspected Perthes’ C The management of fat embolism includes
disease Oxygen, heparinization and LMW dextran.
MRI C Treatment of osteogenic sarcoma of lower end of
C Subtrochanteric fractures of femur can be treated by femur
Skeletal traction on Thomas splint, condylar blade Chemotherapy + limb salvages surgery + chemo-
plate and Ender’s nail. therapy.
C Commonly involved sites in pelvic fracture
Acetabula, alae of ileum and pubic rami. ALL INDIA 2003
C True about fracture atlas
C Commonest cause for neuralgic pain in foot
Jefferson fracture is most common type, CT scan
Compression of communication between lateral
done for diagnosis and atlanto occipital fusion may
and medial plantar nerves.
sometimes be required.
C Lateral X-ray cervical spine of a young woman with C Avascular necrosis is seen in fracture of
mild quadriparesis after accident shows C5–C6 Femur neck, scaphoid and talus.
fracture dislocation. Best line of management C Kinbock’s disease is due to avascular necrosis of
Cervical traction with instrument fixation. Lunate bone
C The most common sequelae of tubercular C Classical expansile lytic lesion in transverse process
spondylitis in an adolescent patient is of vertebra is found in
Bony ankylosis Aneurysmal bone cyst.
Orthopedics 279

C Right lower limb of a male patient following RTA C Inversion injury at ankle can cause
was flexed, adducted, internally rotated and short. Fracture tip of lateral malleolus, fracture base of 5th
Diagnosis metatarsal and fracture of sustentaculum tali.
Posterior dislocation of hip. C A 40 years old laborer suddenly develops acute
C Sciatic nerve injury may occur in lower back pain with right leg pain and weakness of
Posterior dislocation of hip joint. dorsiflexion of right great toe. True is
C Osteomalacia is associated with The appearance of foot drop indicates early surgical
Increase in osteoid maturation time. intervention.
C Most sensitive test in suspected old tear of anterior C Most common cause of acute osteomyelitis
cruciate ligament is Staphylococcus aureus.
Lachman test C Endosteal scalloping and punctate calcification in
C In actinomycosis of spine, abscess usually erodes expansile lesion in the centre of femoral metaphysis
Towards the skin. in a 45 years old male patient favours the diagnosis
of
Chondrosarcoma
ALL INDIA 2002
C A child presents with predisposition to fracture,
C Carpal tunnel syndrome is due to compression of anemia and hepatosplenomegaly despite diffusely
Median nerve increased radiographic density of bones
C Most common nerve involved in fracture of surgical Osteopetrosis.
neck of humerus C A 10 years old child presents with scoliosis, hairy
Axillary nerve tuft in skin of back and neurological deficit with X-
C Associated with supracondylar fracture of ray AP view showing multiple vertebral anomalies
humerus and vertical bony spur overlying lumbar spine
Rare after 15 years of age, cubitus varus deformity Diagnosis is
commonly results following malunion and Diastematomyelia
extension type fracture is more common than the C Unexplained hypotension in head injury patient
flexion type. warrants evaluation of
C Deformity produced due to malunion developed in Thoracic spine
fracture of lateral condyle of femur C Complete transaction of spinal cord at C7 level
Genu valgum. produces
C Patellar tendon bearing POP cast is indicated in Hypotension, anesthesia below level of lesion and
Fracture tibia. areflexia below level of lesion.

You might also like